{"question": "Treatment of nasopharyngeal carcinoma stage T1 is", "exp": "Radiotherapy is the treatment of choice for nasopharyngeal cancer. Stage I and II are treated by radiotherapy alone while stage III and IV require concomitant radiation and chemotherapy or radiation followed by chemotherapy. External beam radiation of 6000-7000 cGy can be delivered by the linear accelerator to the primary and both sides of the neck. More advanced techniques of radiotherapy such as three-dimensional conformal radiotherapy and intensity modulated radiotherapy (IM) are now being used more and more. They allow higher dose delivery to the tumour with reduced damage to the adjacent normal structures such as spinal cord, brainstem and parotid glands. IM has also been used for a recurrent disease where conventional radiotherapy produces more serious side effects such as transverse myelitis (Ref: Diseases of Ear, Nose and Throat, P.L Dhingra, 7th edition, page 285)", "cop": 1, "opa": "Radiation therapy", "opb": "Surgery", "opc": "Local resection", "opd": "Chemotherapy", "subject_name": "ENT", "topic_name": "Pharynx", "id": "fe8b42bf-531c-43f2-89af-7268dafc4c9b", "choice_type": "single"} {"question": "An electrode of a cochlear implant is placed in", "exp": "Pas of the implant: External pa- Microphone, Speech processor, and Transmitter coilInternal pa- Receiver stimulator, Electrode arrayThe electrode array is inseed into the scala tympani by widening the round window (Cochleostomy)Ref: Hazarika; 3rd ed; Pg: 87", "cop": 3, "opa": "Horizontal semicircular canal", "opb": "Scala media", "opc": "Scala tympani", "opd": "Scala vestibule", "subject_name": "ENT", "topic_name": "Ear", "id": "8c5fb106-37f5-4283-ad51-9de4040418ea", "choice_type": "single"} {"question": "60 years old man with foul breath, Regurgitates food eaten 3 days ago, the likely diagnosis", "exp": null, "cop": 2, "opa": "Loss of tone of upper esophageal sphincter", "opb": "Zenker's Diverticulum", "opc": "Achalasia cardia", "opd": "Carcinoma esophagus", "subject_name": "ENT", "topic_name": null, "id": "8fb62140-9920-4c0b-ac9c-524492439015", "choice_type": "single"} {"question": "\"Facial palsy' in Herpes Zoster infection is seen in", "exp": "(A) Ramsay Hunt syndrome # Herpes zoster oticus (HZ oticus) is a viral infection of the inner, middle, and external ear. HZ oticus manifests as severe otalgia and associated cutaneous vesicular eruption, usually of the external canal and pinna.> When associated with facial paralysis, the infection is called Ramsay Hunt syndrome.> Pathophysiology: Reactivation of the varicella-zoster virus (VZV) along the distribution of the sensory nerves innervating the ear, which usually includes the geniculate ganglion, is responsible for HZ oticus. Associated symptoms such as hearing loss and vertigo are thought to occur as a result of transmission of the virus via direct proximity of cranial nerve (CN) VIII to CN VII at the cerebellopontine angle or via vasa vasorum that travel from CN VII to other nearby cranial nerves.> If diagnosis of Ramsay Hunt syndrome is not established by physical examination alone, consider a head CT scan to investigate other etiologies of facial paralysis.> Early administration (<72 h) of acyclovir showed an increased rate of facial nerve function recovery and prevented further nerve degeneration. Evidence is accumulating that VZV may be responsible for many cases of Bell palsy that go unrecognized because of a lack of cutaneous findings (zoster sine herpete).> Accordingly, the clinician should entertain more liberal use of antivirals such as acyclovir, valacyclovir, and famciclovir. Systemic corticosteroids are used to relieve acute pain, decrease vertigo, and limit the occurrence of postherpetic neuralgia, although evidence proving benefit attributed specifically to steroids is still limited.", "cop": 1, "opa": "Ramsay Hunt syndrome", "opb": "Meckerson Rosenthal syndrome", "opc": "Sturge Weber syndrome", "opd": "Guillian Barre syndrome", "subject_name": "ENT", "topic_name": "Miscellaneous (E.N.T.)", "id": "cc037632-ed75-4263-8ed6-58ec35cb7acd", "choice_type": "single"} {"question": "Not used in the treatment of Juvenile Larynegeal papillomatosis", "exp": null, "cop": 2, "opa": "INF α", "opb": "INF β", "opc": "Bevacizumab", "opd": "Cedofovir", "subject_name": "ENT", "topic_name": null, "id": "74f810fb-59d2-417d-87d0-3590eba9c585", "choice_type": "single"} {"question": "Caldwell view is done for", "exp": "Caldwell view is the occipito frontal view. The frontal sinuses are seen clearly in this view.", "cop": 4, "opa": "Sphenoid sinus", "opb": "Maxillary sinus", "opc": "Ethmoid sinus", "opd": "Frontal sinus", "subject_name": "ENT", "topic_name": null, "id": "54a81705-0c24-43b2-a212-bd0c49f3ac76", "choice_type": "single"} {"question": "Nasopharyngeal angiofibroma of stage II is treated with", "exp": null, "cop": 4, "opa": "Radiotherapy", "opb": "Chemotherapy", "opc": "Concurrent chemo-Radiation", "opd": "Surgery", "subject_name": "ENT", "topic_name": null, "id": "4a739fdf-0f61-4e45-8087-0dd308d58c5c", "choice_type": "single"} {"question": "Superior margin of infants larynx as at the level of", "exp": null, "cop": 2, "opa": "Cervical spine C1", "opb": "Cervical spine C2", "opc": "Cervical spine C4", "opd": "Cervical spine C6", "subject_name": "ENT", "topic_name": null, "id": "80a3b6ca-d241-40e7-bcbb-848ccc79fb7e", "choice_type": "single"} {"question": "Preferred treatment in a 60years old patient with Maxillary carcinoma involving anterolateral pa of maxilla is", "exp": "Treatment of maxillary carcinomas: Early cases with Stage I and II squamous cell carcinomas are treated with surgery or radiation with equal results. T3 and T4 lesions are treated by combined modalities of radiation and surgery. Radiation in such cases may be given preoperatively or postoperatively Ref: Dhingra 7e pg 233.", "cop": 3, "opa": "Radiotherapy only", "opb": "Total/extended Maxillectomy followed by radiotherapy", "opc": "Radiotherapy followed by total/extened maxillectomy", "opd": "Total/extended maxillectomy alone", "subject_name": "ENT", "topic_name": "Nose and paranasal sinuses", "id": "b293f3d7-ae14-4e09-97fd-71568f827539", "choice_type": "single"} {"question": "Tobey Ayer test is positive in", "exp": "Tobey Ayer test is positive in lateral sinus thrombosis. Tobey-Ayer test: This is to record CSF pressure by the manometer and to see the effect of manual compression of one or both jugular veins. Compression of a vein on the thrombosed side produces no effect while compression of a vein on the healthy side produces a rapid rise in CSF pressure which will be equal to bilateral compression of jugular veins. (Ref: Textbook of diseases of ENT, PL Dhingra, 7th edition, pg no. 93)", "cop": 1, "opa": "Lateral sinus thrombosis", "opb": "Petrositis", "opc": "Cerebral abscess", "opd": "Subarachnoid hemorrhage", "subject_name": "ENT", "topic_name": "Ear", "id": "d55b5232-8314-4cd3-89a5-b583039c8600", "choice_type": "single"} {"question": "Cartilaginous part of the external auditory canal is", "exp": "Lateral one-third (8mm) of the total external auditory canal (24mm) is cartilaginous.", "cop": 2, "opa": "Medial 1/3", "opb": "Lateral 1/3", "opc": "Medial 2/3", "opd": "Lateral 2/3", "subject_name": "ENT", "topic_name": null, "id": "ee0f5bb0-9f1e-454b-9c34-1e75e50fe304", "choice_type": "single"} {"question": "External auditory canal exostosis occurs due to", "exp": "Exostoses are multiple and bilateral, often presenting as smooth, sessile, bony swellings in the deeper pa of the meatus near the tympanic membrane.They often arise from the compact bone. Exostoses is often seen in persons exposed to the entry of cold water in the meatus as in swimmers and divers. Reference: Diseases of Ear, Nose and Throat, PL Dhingra, 7th edition, pg no. 118", "cop": 4, "opa": "Repeated instrumentation", "opb": "Recurrent otitis externa", "opc": "Wide external auditory meatus", "opd": "Recurrent prolonged cold water exposure", "subject_name": "ENT", "topic_name": "Ear", "id": "e5763c20-89b5-4d15-a4ff-3e20a23e6659", "choice_type": "single"} {"question": "VEMP detects lesion of", "exp": "The vestibular evoked myogenic potential (VEMP) are sho latency electromyograms that are evoked by acoustic stimuli of high intensity and recorded from surface electrodes over the tonically contracted sternocleidomastoid muscle. The origin of VEMP is in the saccule. The response pathway consists of Saccule; Inferior Vestibular Nerve, Lateral Vestibular Nucleus, Lateral Vestibulospinal Tract and Sternocleidomastoid muscle.The test provides diagnostic information about saccular and/or inferior vestibular nerve function. An intact middle ear is required for the response quality.Ref: Hazarika; 3rd edition", "cop": 3, "opa": "Cochlear nerve", "opb": "Superior vestibular nerve", "opc": "Inferior vestibular nerve", "opd": "Inflammatory lesions", "subject_name": "ENT", "topic_name": "Ear", "id": "a08f4fee-3851-4e64-8e38-9033ac872cf3", "choice_type": "single"} {"question": "CSOM with Picket fence fever is seen in", "exp": "Sigmoid sinus thrombosis/lateral sinus thrombosis is an inflammation of inner wall of lateral venous sinus with formation of an intra sinus thrombus It occurs as a complication of chronic suppuration of middle ear or acute coalescent mastoiditis, and cholesteatoma Picket fence fever temperature never comes back to normal baseline. Remittent fever. Complications of CSOM - Intra temporal Intracranial Mastoiditis Meningitis - m/c Petrositis Brain abscess Facial paralysis Extradural abscess Labyrinthitis Subdural abscess Lateral sinus thrombophlebitis Otitic hydrocephalus", "cop": 2, "opa": "Meningitis", "opb": "Sigmoid Sinus Thrombosis", "opc": "Brain Abscess", "opd": "Extradural Abscess", "subject_name": "ENT", "topic_name": "Complications of Chronic Otitis Media", "id": "19de9e2f-472f-40ab-8530-767da4bb3910", "choice_type": "single"} {"question": "Bed of tonsil is formed by", "exp": "The tonsillar bed is formed by the following: Pharyngobasilar fascia Superior constrictor muscle Buccopharyngeal fascia Styloglossus muscle Glossopharyngeal nerve Ref: PL Dhingra, Diseases of Ear, Nose & Throat, 7th edition, pg no. 291", "cop": 1, "opa": "Superior constrictor", "opb": "Middle constrictor", "opc": "Inferior constrictor", "opd": "Platysma", "subject_name": "ENT", "topic_name": "Pharynx", "id": "7337977a-c286-4394-b472-f4681001a787", "choice_type": "single"} {"question": "The most common benign tumor of the salivary gland is", "exp": "PLEOMORPHIC ADENOMA It is the most common benign tumour of salivary glands. It can arise from the parotid, submandibular or other minor salivary glands. In the parotid it usually arises from its tail. Pleomorphic adenomas are slow-growing tumours. They are usually seen in the third or fouh decade, with propensity for females. They are called \"mixed tumours\" because both epithelial and mesenchymal elements are seen in histology. The stroma of the tumour may be mucoid, fibroid, vascular, myxochondroid or chondroid and its propoion to the epithelial element may vary. Though tumour is encapsulated, it sends pseudopods into the surrounding gland which are left behind if the tumour is simply shelled out. It is therefore essential that surgical excision of the tumour should include normal gland tissue around it. In the parotid, it amounts to superficial parotidectomy. Ref:- Dhingra; pg num:-234", "cop": 3, "opa": "Mucoepidermoid tumor", "opb": "Wahin's tumor", "opc": "Pleomorphic adenoma", "opd": "Acinic cell tumor", "subject_name": "ENT", "topic_name": "Oral cavity & Oesophagus", "id": "64eb2434-0b3a-4c33-8b26-babb6eeb712c", "choice_type": "single"} {"question": "Type of tympanogram seen in thick tympanic membrane condition is", "exp": "TypeConditionType ANormal tympanogramType AsCompliance is lower at or near ambient air pressure. Seen in fixation of ossicles, e.g. otosclerosis or malleus fixationType ADHigh compliance at or near ambient pressure.Seen in the ossicular discontinuity or thin and lax tympanic membraneType BA flat or dome-shaped graph. No change in compliance with pressure changes. Seen in middle ear fluid or thick tympanic membraneType CMaximum compliance occurs with negative pressure in excess of 100 mm H2O. Seen in a retracted tympanic membrane and may show some fluid in the middle earRef: Dhingra; 6th edition; Chapter 4; Page no: 24", "cop": 2, "opa": "Type A", "opb": "Type B", "opc": "Type C", "opd": "Type D", "subject_name": "ENT", "topic_name": "Ear", "id": "ef74c674-c96e-4fb0-8871-5bfe38fabe16", "choice_type": "single"} {"question": "Commonest postoperative complication of tonsillectomy is", "exp": "Commonest postoperative complication of tonsillectomy is haemorrhage Haemorrhage may be primary, secondary and reactionary haemorrhage Primary haemorrhage occurs at the time of operation Reactionary haemorrhage occurs within a period of 24 hrs Secondary haemorrhage occurs 5-10th postoperative day Re: Textbook of Ear, Nose and Throat, Dhingra, 6th Edition; Pg no: 430", "cop": 2, "opa": "Palatal palsy", "opb": "Haemorrhage", "opc": "Injury to uvula", "opd": "Infection", "subject_name": "ENT", "topic_name": "Diagnostic and operative ENT", "id": "64f8c4bd-6cf6-47f5-ad43-57c21c62f668", "choice_type": "single"} {"question": "Not a surgical landmark for parotid surgery is", "exp": "surgical landmark of parotid surgery are : Tragal pointer Digastric posterior belly Styloid process tympansmastoid suture ref : hazarika 4th ed", "cop": 1, "opa": "Inferior belly of omohyoid", "opb": "Tragal pointer", "opc": "Digastric posterior belly", "opd": "Dissecting from peripheral branches", "subject_name": "ENT", "topic_name": "All India exam", "id": "64a6e069-0cba-4a64-a9a4-f090c35bcc32", "choice_type": "single"} {"question": "Peritonsillar abscess is most commonly caused by", "exp": "Peritonsillar abscess ( Quinsy) is the collection of pus in the peritonsillar space which lies between the capsule of tonsil and superior constrictor muscle. A peritonsillar abscess usually follows acute tonsillitis. First crystal Magna is infected and sealed off. Culture of pus from the abscess May reveal beta-hemolytic streptococci, S.aureus, or anaerobic organisms. (Ref: Diseases of ENT, PL Dhingra, 7th edition, pg no.298)", "cop": 3, "opa": "Streptococcus pneumonia", "opb": "Staphylococcus aureus", "opc": "Beta hemolytic streptococcus", "opd": "Hemophilus influenza", "subject_name": "ENT", "topic_name": "Pharynx", "id": "f26b3e4d-831f-4366-87ab-2436ad8d71f9", "choice_type": "single"} {"question": "Following operations are done in case of otosclerosisa) Stapedectomyb) Fenestration c) Stapedotomyd) Sacculotomye) Mastoidectomy", "exp": null, "cop": 3, "opa": "ab", "opb": "bc", "opc": "abc", "opd": "bcd", "subject_name": "ENT", "topic_name": null, "id": "f5ca08af-bc08-46d3-bbbe-f19c589820c3", "choice_type": "single"} {"question": "A young patient with a headache, epiphora, bilateral nasal obstruction but no fever. Diagnosis is", "exp": "CLINICAL FEATURES 1. Age and sex. Tumour is seen almost exclusively in males in the age group of 10-20 years. Rarely, it may be seen in older people and females. 2. Profuse, recurrent and spontaneous epistaxis. This is the most common presentation. The patient may be markedly anaemic due to repeated blood loss. 3. Progressive nasal obstruction and denasal speech. It is due to the mass in the postnasal space. 4. Conductive hearing loss and otitis media with effusion. It occurs due to obstruction of the eustachian tube. 5. Mass in the nasopharynx. Tumour is sessile, lobulated or smooth and obstructs one or both choanae. It is pink or purplish in colour. Consistency is firm but digital palpation should never be done until at the time of operation. 6. Other clinical features like broadening of the nasal bridge, proptosis, swelling of the cheek, infratemporal fossa or involvement of IInd, IIIrd, IVth and VIth cranial nerves will depend on the extent of the tumour (Ref: Diseases of Ear, Nose and Throat, P.L Dhingra, 7th edition, page 279 )", "cop": 2, "opa": "Nasal polyp", "opb": "Juvenile angiofibroma", "opc": "Nasal carcinoma", "opd": "Rhinoscleroma", "subject_name": "ENT", "topic_name": "Pharynx", "id": "0301e367-c3b0-450c-b514-24f163f4b979", "choice_type": "single"} {"question": "Odiosoft rhino is a method used for", "exp": "Odiosoft rhino is an equipment used to measure the sound produced by nasal airflow. Measurement of nasal air flow sounds helps us to indirectly assess the presence/absence of nasal block as well as accurately identifying the site of the block. Ref:", "cop": 3, "opa": "Detecting softness of nasal cailages", "opb": "Detecting nasal elasticity", "opc": "Assessing nasal block", "opd": "Detecting nasal mucosal secretory function", "subject_name": "ENT", "topic_name": "All India exam", "id": "69ffe5c6-fd54-4dc5-9605-da86fec807d6", "choice_type": "single"} {"question": "Cawheel sign is seen in", "exp": "Cawheel appearance is seen in ASOM. Leash of blood vessels appears along the handle of malleus and at the periphery of tympanic membrane impaing it a ca-wheel appearance. Later, whole of tympanic membrane including pars flaccida becomes uniformly red. Seen in stage of Presuppuration Reference: Dhingra; Chapter 10; Page no: 62", "cop": 1, "opa": "ASOM", "opb": "CSOM", "opc": "Serous otitis media", "opd": "Tuberculous otitis media", "subject_name": "ENT", "topic_name": "Ear", "id": "99f54050-82a9-4bfb-8233-7056c56a7c19", "choice_type": "single"} {"question": "In sensorineural hearing loss, weber’s test is lateralized to", "exp": "In Sensorineural deafness: Lateralized to better ear.\nIn conductive deafness: Lateralized to worse ear.", "cop": 1, "opa": "Normal ear", "opb": "Defective ear", "opc": "Not lateralized", "opd": "May alternate", "subject_name": "ENT", "topic_name": null, "id": "1aef5237-d7b9-47e3-b15c-b3e4dd8aeed1", "choice_type": "single"} {"question": "Esthesio Neuroblastoma is tumour arising from", "exp": null, "cop": 3, "opa": "Spinal cord", "opb": "Piamater", "opc": "Olfactory nasal mucosa", "opd": "Arachnoid", "subject_name": "ENT", "topic_name": null, "id": "75489aca-d51a-4a46-a0e0-2bda26663250", "choice_type": "single"} {"question": "Most common presentation in nasopharyngeal carcinoma", "exp": "NASOPHARYNGEAL CANCER CLINICAL FEATURES:- Cervical nodal metastases:- This may be the only manifestation of nasopharyngeal cancer. A lump of nodes is found between the angle of jaw and the mastoid and some nodes along the spinal accessory in the posterior triangle of neck. Nodal metastases are seen in 75% of the patients, when first seen, about half of them with bilateral nodes. Ref:- Dhingra; pg num:-252", "cop": 1, "opa": "Cervical lymphadenopathy", "opb": "Nasal stuffiness", "opc": "Hoarseness of voice", "opd": "Epistaxis", "subject_name": "ENT", "topic_name": "Pharynx", "id": "10720cf5-2387-4fe0-ae22-e75c15f55edb", "choice_type": "single"} {"question": "Most common cause of acute epiglottitis is", "exp": "It is an acute inflammatory condition confined to supraglottic structures i.e, epiglottis, aryepiglottic folds and arytenoids. There is marked oedema of these structures which may obstruct the airway. Aetiology It is a serious condition and affects children of 2-7 years of age but can also affect adults. Haemophilus influenzae B is the most common organism responsible for this condition in children. (Ref: Textbook of diseases of ENT, PL Dhingra, 7th edition, pg no. 327)", "cop": 1, "opa": "Hemophilus influenza type B", "opb": "Pneumococcus", "opc": "Streptococcus", "opd": "Staphylococcus", "subject_name": "ENT", "topic_name": "Ear", "id": "9d0842b2-c2a6-418a-94ad-776a34b8d641", "choice_type": "single"} {"question": "Internal Acoustic Meatus connects Inner Ear to", "exp": null, "cop": 3, "opa": "Anterior Cranial fossa", "opb": "Middle Cranial fossa", "opc": "Posterior Cranial fossa", "opd": "Posterior and middle Cranial fossa", "subject_name": "ENT", "topic_name": null, "id": "a9e9c2fa-87e4-40cf-b384-72b9fa5b45da", "choice_type": "single"} {"question": "Absorption of one of the following can occur without being broken down", "exp": "Absorption of undigested protein can occur, especially in infants. This decreases with age. However, adults can still absorb undigested protein. The M cells (microfold cells) overlying the Payer's patches absorb antigens.", "cop": 1, "opa": "Protein", "opb": "Triglycerides, although this seen only in infants", "opc": "a-Dextrins", "opd": "Sucrose", "subject_name": "ENT", "topic_name": "All India exam", "id": "ce199622-3114-4bf6-b328-9c988bb847d1", "choice_type": "single"} {"question": "Keratosis obturans is", "exp": "Keratosis obturans is a collection of the pearly white mass of desquamated epithelial cells in the deep meatus. This, by its pressure effect, causes absorption of bone leading to widening of the meatus so much so that facial nerve may be exposed and paralyzed. Presenting symptoms may be a pain in the ear, hearing loss, tinnitus and sometimes ear discharge. (Ref: Textbook of diseases of ENT, PL Dhingra, 7th edition, pg no. 58)", "cop": 2, "opa": "Foreign body in external auditory canal", "opb": "Desquamated epithelial cells and Cholesterol", "opc": "Cholesterol crystals surrounded by calcium", "opd": "Wax in external auditory canal", "subject_name": "ENT", "topic_name": "Ear", "id": "65ec6bfb-eebb-424d-9b40-a10a3e873be0", "choice_type": "single"} {"question": "Weber's test lateralizing to the affected side means", "exp": "Weber's testLateralized to poorer ear- conductive hearing lossLateralized to better ear- SNHLNo lateralization - NormalRef: PL Dhingra, Diseases of Ear, Nose & Throat, 7th edition, pg no. 24", "cop": 1, "opa": "Conductive hearing loss", "opb": "Sensorineural hearing loss", "opc": "Mixed loss", "opd": "Brainstem damage", "subject_name": "ENT", "topic_name": "Ear", "id": "c91d97a7-be59-497d-b517-28f9f72329e2", "choice_type": "single"} {"question": "Gutzmann&;s pressure test is done for", "exp": "PUBERPHONIA (MUTATIONAL FALSETTO VOICE) Normally, childhood voice has a higher pitch. When the larynx matures at pubey, vocal cords lengthen and the voice changes to one of lower pitch. This is a feature exclusive to males. Failure of this change leads to persistence of childhood high-pitched voice and is called puberphonia. It is seen in boys who are emotionally immature, feel insecure and show excessive fixation to their mother. Psychologically, they shun to assume male responsibilities though their physical and sexual development is normal. Treatment:- Treatment is training the body to produce low-pitched voice. Pressing the thyroid prominence in a backward and downward direction relaxes the overstretched cords and low tone voice can be produced (Gutzmann's pressure test). The patient pressing on his larynx learns to produce low tone voice and then trains himself to produce syllables, words and numbers. Prognosis is good. Ref:- Dhingra; pg num:-314", "cop": 2, "opa": "Laryngomalacia", "opb": "Puberphonia", "opc": "Laryngeal polyp", "opd": "Vocal cord polyp", "subject_name": "ENT", "topic_name": "Larynx", "id": "e4abfe0e-fb2c-4b7c-8bce-ec529cc028ef", "choice_type": "single"} {"question": "Diagnosis in a ten year old boy with recurrent epistaxis and a unilateral nasal mass is", "exp": "Nasopharyngeal fibroma/Juvenile nasopharyngeal angiofibroma is a major cause for recurrent epistaxis in adolescent males. It is testosterone dependent. Benign tumour but locally invasive Ref - PL DHINGRA diseases of ear nose and throat, 6th edition", "cop": 3, "opa": "Antrochoanal polyp", "opb": "Hemangioma", "opc": "Angiofibroma", "opd": "Rhinolith", "subject_name": "ENT", "topic_name": "Nose and paranasal sinuses", "id": "e805ad44-6f84-4e4e-8d52-bdbe959caebe", "choice_type": "single"} {"question": "The usual location of glomus jugulare tumour is", "exp": "Glomus jugulare tumour arises from the dome of jugular bulb and invades hypotympanum and jugular foramen causing neurological signs of IX and XII cranial nerve involvement. Ref: Textbook of diseases of ENT, PL Dhingra, 7th edition, pg no. 121", "cop": 2, "opa": "Epitympanum", "opb": "Hypotypanum", "opc": "Mastoid tip cells", "opd": "Internal auditory meatus", "subject_name": "ENT", "topic_name": "Ear", "id": "e837fc9a-be3a-46f9-92e8-71a213b9696d", "choice_type": "single"} {"question": "Most common cause of parapharyngeal abscess is", "exp": "Infection of tonsillar fossa leads to the parapharyngeal abscess as a delayed complication of tonsillectomy. Infection of parapharyngeal space can occur from teeth, ear and other spaces like parotid and retropharyngeal spaces. In spite of it being the smallest space in relation to the pharynx, it is the most commonly infected space of the pharynx as infections from tonsil(most commonly), dental, parotid, retropharyngeal, preveebral and submandibular space can all lead to parapharyngeal abscess. (Ref: Scott Brown, 8th edition, VOL 3, Pg no. 551)", "cop": 1, "opa": "Removal of tonsil", "opb": "Haematogenous spread", "opc": "Penetrating trauma", "opd": "Blunt trauma", "subject_name": "ENT", "topic_name": "Pharynx", "id": "6ea64b1b-00c3-4fc9-b6ac-4abd683b59d3", "choice_type": "single"} {"question": "Incisura terminals are between", "exp": "There is no cailage between tragus and crus of the helix and an incision made in this area will not cut through the cailage and is used for the endaural approach in surgery of external auditory canal and mastoid. Reference: Dhingra 6th edition; page no 2", "cop": 1, "opa": "Tragus and crux of helix", "opb": "Ear lobule and antihelix", "opc": "Antihelix and external auditory meatus", "opd": "Tragus and ear lobule", "subject_name": "ENT", "topic_name": "Ear", "id": "57ea9963-bf50-4d55-b707-0319d9901dc7", "choice_type": "single"} {"question": "In a child with hyperophied adenoids, the voice abnormality that is seen is", "exp": "HYPONASALITY (RHINOLALIA CLAUSA) It is lack of nasal resonance for words which are resonated in the nasal cavity, e.g. m, n, ng. It is due to blockage of the nose or nasopharynx. Ref:- Dhingra; pg num:-315", "cop": 4, "opa": "Hot potato voice", "opb": "Staccato voice", "opc": "Rhinolalia apea", "opd": "Rhinolalia clausa", "subject_name": "ENT", "topic_name": "Larynx", "id": "16bdfb21-a03c-40de-82d7-8a30da51fc46", "choice_type": "single"} {"question": "Antral sign is seen in", "exp": "Anterior bowing of the posterior wall of maxillary sinus often called antral sign or Holman-Miller sign, which is pathognomic of angiofibroma. (Ref: Diseases of ear, nose and throat, PL Dhingra, 7th edition, page 281)", "cop": 1, "opa": "Juvenile angiofibroma", "opb": "Otosclerosis", "opc": "CSOM", "opd": "Sinusitis", "subject_name": "ENT", "topic_name": "Pharynx", "id": "a9f37ddf-9f92-4872-9cdc-94f20bd18e8f", "choice_type": "single"} {"question": "The cadaveric position of vocal cords is", "exp": "Positions of the vocal cord: 1. Median- cords touching the midline 2. Paramedian-1.5 mm from the midline 3. Intermediate (Cadaveric)- 3.5 mm from the midline 4. Slightly abducted- 7 mm 5. Fully abducted- 9 mm. Ref: Hazarika; 3rd ed; Pg 626", "cop": 3, "opa": "Midline", "opb": "1.5mm from midline", "opc": "3.5mm from midline", "opd": "7.5mm from midline", "subject_name": "ENT", "topic_name": "Larynx", "id": "02f6f3a3-8ac2-4528-b6aa-8ca02932579e", "choice_type": "single"} {"question": "Haplike's test is done for", "exp": null, "cop": 2, "opa": "Cochleas function", "opb": "Vestibular function", "opc": "Middle Ear function", "opd": "Not related to the Ear", "subject_name": "ENT", "topic_name": null, "id": "80ba9684-d040-4483-942d-2aea83e2f08f", "choice_type": "single"} {"question": "Recent studies suggest causative organism of Bells palsy is", "exp": "(Herpes virus): (105- Dhingra 5th edition; 185- P. Hazarika 3rd/ed)Bell's palsyViral theory - Most of the evidence support the viral aetiology due to herpes simplex herpes zoster or the EB virus (105-D)* Viral infection like Herpes zoster, poliomyelitis and infectious mononucleosis which proves that facial nerve is very prone for viral infections (185- Hazarika)* Herpes simplex virus (HSV-1) has been known to cause fascial paralysisBell's phenomenon - The patient will not be able to close the eye on the affected side. The eyeball will roll up on every attempt of closure", "cop": 1, "opa": "Herpes virus", "opb": "HIV virus", "opc": "Papilloma virus", "opd": "Rota virus", "subject_name": "ENT", "topic_name": "Facial Nerve And Its Disorders", "id": "45e1db40-1fca-4599-a65c-eec2a264ab4e", "choice_type": "single"} {"question": "In the right middle ear pathology, Weber&;s test will be", "exp": "Weber test: In this test, a vibrating tuning fork is placed in the middle of the forehead or the veex and the patient is asked in which ear the sound is heard. Normally, it is heard equally in both ears. It is lateralized to the worse ear in conductive deafness and to the better ear in sensorineural deafness. In weber test, sound travels directly to the cochlea bone. Lateralization of sound in weber test with a tuning fork of 512 Hz implies a conductive loss of 15-25 dB in ipsilateral ear or a sensorineural loss in the contralateral ear. Ref:- Dhingra; pg num:-22", "cop": 3, "opa": "Centralized", "opb": "Normal", "opc": "Lateralized to right side", "opd": "Lateralized to left side", "subject_name": "ENT", "topic_name": "Ear", "id": "816055af-7780-4536-a99b-2461d0e40282", "choice_type": "single"} {"question": "Decompression of endolymphatic sac is the treatment for", "exp": "(A) (Meniere's disease) (104- Dhingra 6th)Surgical Treatment of Meniere's Disease1. Conservative procedures2. Destructive procedures3. IntermittentA. Decompression of Endolymphatic sacB. Endolymphatic shunt operationC. Sacculotomy (fick's operation)D. Section of vestibular nerveE. Ultrasonic destruction of vestibular labyrinthLabyrinthectomyLow pressure pulse therapy (Meniett device therapy)Meniere's Disease TreatmentA. GENERAL MEASURESB. MANAGEMENT OF ACUTE ATTACK1. Reasurance2. Cessation of smoking3. Low salt diet4. Avoid excessive intake of water5. Avoid over- indulgence in coffee, tea and alcohol6. Avoid stress and bring a change in life cycle7. Avoid activities requiring good body balance1. Reassurance - to allay worry and anxiety2. Bed rest with head supported on pillows3. Vestibular sedatives to relieve vertigo* Dimenhydrinate (Dramamine)* Promethazine theoclate (Avomine)* Prochlorperazine (stemetil)* Diazepam given intravenously* Atropine4. VasodilatorsA. Inhalation of carbogen (5% C02 with 95% O2)B. Histamine drip - Histamine diphosphate 2.75 mg dissolved in 500 ml of glucoseC. MANAGEMENT OF CHRONIC PHASE1. Vestibular sedative- prochlorperazine2. Vasodilators- Nicotinic acid, Betahistine3. Diuretics - furosemides4. Propantheline bromide - alone or combination with vasodilators or vestibular sedatives5. Elimination of allergen- food or inhalant allergen6. Hormones - endocrinal disorders such as hypothyroidism- Medical treatment* Intratympanic Gentamicin therapy (chemical Labyrinthectomy)* MICRO WICK - made up of polyvinyl acetate", "cop": 1, "opa": "Menier's disease", "opb": "BPPV", "opc": "Labyrinthitis", "opd": "Otosclerosis", "subject_name": "ENT", "topic_name": "Ear", "id": "4a5b4e2a-2737-431a-af68-1563f15c589a", "choice_type": "single"} {"question": "Epistaxis in elderly patient is commonest in", "exp": "The major cause for epistaxis in an elderly person is hypeension. Ref: Dhingra 7e pg 198.", "cop": 4, "opa": "Nasapharyngeal carcinoma", "opb": "Foreign body", "opc": "Bleeding disorder", "opd": "Hypeension", "subject_name": "ENT", "topic_name": "Nose and paranasal sinuses", "id": "47bbff3a-24d5-4928-871f-1b2d6a16e250", "choice_type": "single"} {"question": "Lombard&;s test is used in diagnosis of", "exp": "Lombard&;s test: This test is based on \"Lombard&;s principle\". This principle says that one raises his / her voice when speaking in the noisy environment. While performing this test, the patient is allowed to read a book. Noise is introduced into the ear. The noise is gradually increased till the patient raises his / her voice or stops the process of reading. If there is no change in voice loudness level the patient does not have a functional hearing defect.", "cop": 4, "opa": "Conductive hearing loss", "opb": "Sensorineural hearing loss", "opc": "Mixed hearing loss", "opd": "Non-organic hearing loss", "subject_name": "ENT", "topic_name": "Ear", "id": "858c23c6-5418-4413-a65e-d88904ffe9cb", "choice_type": "single"} {"question": "In Bell's palsy, neurological lesion is seen in", "exp": "C/f of Bell's Palsy : Onset is sudden. Patient is unable to close his eye. Neurological lesion is seen in Upper and lower pa of one half of face On attempting to close the eye, eyeball turns up and out (Bell's phenomenon). Saliva dribbles from the angle of mouth. Face becomes asymmetrical Tears flow down from the eye (epiphora). Pain in the ear may precede or accompany the nerve paralysis. Some complain of noise intolerance (stapedial paralysis) or loss of taste (involvement of chorda tympani). Paralysis may be complete or incomplete. Bell's palsy is recurrent in 3-10% of patients.", "cop": 2, "opa": "Upper pa of face", "opb": "Upper and lower pa of one half of face", "opc": "Lower pa of one-half of face", "opd": "Upper and lower pa of opposite side of face", "subject_name": "ENT", "topic_name": "Facial Nerve and its disorders", "id": "623a7df4-f1cf-48b5-8831-78e73b2184a6", "choice_type": "single"} {"question": "Thornwaldt's abscess treatment does not include", "exp": "Treatment includes- Antibiotics to treat infection Marsupialization of cystic swelling Adequate removal of its lining membrane Ref: Textbook of ENT, Dhingra; 6th edition; Pg no: 245 ref img", "cop": 4, "opa": "Antibiotics", "opb": "Marsupialization", "opc": "Removal of lining", "opd": "Antihistaminics", "subject_name": "ENT", "topic_name": "Miscellaneous ENT", "id": "1c02c6ee-c748-46eb-8150-0d3b4c45dc4d", "choice_type": "single"} {"question": "Focal length of head mirror used in ENT OPD", "exp": "Head mirror: It is a concave mirror used to reflect light from the Bull's eye lamp onto the pa being examined. It has a focal length of approximately 25 cm. The examiner sees through the hole in the centre of the mirror. Diameter of the mirror is 89 mm (3 1/2 \") and that of the central hole is 19 mm (3/4\") Ref: Dhingra 7e pg 427.", "cop": 2, "opa": "9 Inch", "opb": "10 Inch", "opc": "11 Inch", "opd": "12 Inch", "subject_name": "ENT", "topic_name": "Ear", "id": "d1e38cda-65a5-484d-86cc-ae7c7d430af2", "choice_type": "single"} {"question": "Ohngren's line is seen in", "exp": "A theoretic plane passing between the medial canthus of the eye and the angle of the mandible; used as an arbitrary dividing line in classifying localized tumours of the maxillary sinus; tumours above the line invade vital structures early and have a poorer prognosis, whereas those below the line have a more ourable prognosis. Growths of the maxillary sinus(carcinoma of maxillary sinus) situated above this plane(suprastructural) have a poorer prognosis than those below it(intrastructural). (Ref: Textbook of diseases of ENT, PL Dhingra, 7th edition, pg no. 233)", "cop": 1, "opa": "Carcinoma maxilla", "opb": "Carcinoma ethmoid", "opc": "Carcinoma mandible", "opd": "Carcinoma Larynx", "subject_name": "ENT", "topic_name": "Nose and paranasal sinuses", "id": "f4180daa-0ace-45aa-a438-2e97ead740ae", "choice_type": "single"} {"question": "During Acute tonsillitis referred pain from tonsil to the middle ear is along", "exp": "Tonsils and tonsillar fossa are supplied by the glossopharyngeal nerve Any irritation or pain can be referred to the ear along the tympanic branch of the glossopharyngeal (Jacobson's) nerve.", "cop": 1, "opa": "Glossopharyngeal nerve", "opb": "Facial nerve", "opc": "Trigeminal nerve", "opd": "Vagus nerve", "subject_name": "ENT", "topic_name": "NEET 2018", "id": "0d1b5bcf-b364-4e3b-ba06-17972cff3c40", "choice_type": "single"} {"question": "The best time of hearing assessment in an infant is", "exp": "The American Academy of Pediatrics (AAP), Joint Committee on Infant hearing (2007), has recommended that all newborn infants be screened for hearing impairment either as a neonate or before 1 month of age and that those infants who fail newborn screening should have an audiological examination to verify hearing loss before age of 3 months.Ref: PL Dhingra, Diseases of Ear, Nose & Throat, 7th edition, pg no. 132", "cop": 1, "opa": "1st month of life", "opb": "3-6 months", "opc": "6-9 months", "opd": "9-12 months", "subject_name": "ENT", "topic_name": "Ear", "id": "1bae52cb-e4f0-47bd-b654-b7256c0342e7", "choice_type": "single"} {"question": "Loudness recruitment phenomenon is associated with", "exp": "Ans. d (Cochlear deafness) (Ref. Dhingra 3rd ed., 4, 49; 29, 148)The rapid growth of loudness in ears with sensorineural hearing loss is known as loudness recruitment. Its presence helps signify that the hearing loss is caused by pathology within the cochlea. Recruitment is a symptom of cochlear pathology associated with hair cell damage, particularly to outer hair cells.ASSESSMENT OF HEARING LOSS1. Clinical testsa. Finger friction testb. Watch testc. Speech testd. Tuning fork tests2. Audiometric testsa. Pure tone audiometry (measures threshold of hearing by air and bone conduction and thus the degree and type of hearing loss)b. Speech audiometry (measures ability of patient to hear and understand speech)c. Bekesy audiometry (differentiates cochlear from retrocochlear and organic from functional hearing loss)d. Impedance audiometry, i.e., tympanometry and acoustic reflex measurements (particularly useful in children).3. Special testsa. Recruitment (seen typically in lesions of cochlea, e.g., Meniere's disease, presbycusis)b. Short increment sensitivity index (SISI test)c. Threshold tone decay (Carhart's) test (a decay more than 25dB is diagnostic of a retrocochlear lesion)d. Evoked response audiometry (e.g., BERA)e. Otoacoustic emissionsf. Central auditory testsAcoustic Reflex# Loud sound > 70 dB; causes bilateral contraction of stapedial muscles which can be detected by tympanometry.# Reflex mediated by VII and VIII nerves.# It is protective reflex against loud sound.# Normally, stapedial reflex is elicited at 70-100 dB SL. If patient claims total deafness but the reflex can be elicited, it indicates NOHL (non-organic hearing loss). Hyperacusis in Bell's palsy is due to the paralysis of the stapedius muscle.# Uses:1. Objective method to detect hearing in infants and children.2. To find malingering3. To detect cochlear pathology. Presence of Stapedial reflex at lower frequencies, instead of 70 dB indicates Recruitment and cochlear pathology.4. To detect VIII and VII nerve lesions.", "cop": 4, "opa": "Retrocochlear deafness", "opb": "Conductive deafness", "opc": "Sensorineural deafness", "opd": "Cochlear deafness", "subject_name": "ENT", "topic_name": "Hearing Loss", "id": "abfab05a-0f08-419f-ae94-582589acfa86", "choice_type": "single"} {"question": "A chronic smoker with history of hoarseness of voice found to have fixation of vocal cords on examination. The most likely treatment he will require is", "exp": "Fixation of vocal cords on examination is T3 lesion.Total laryngectomy. The entire larynx including the hyoid bone, pre-epiglottic space, strap muscles and one or more rings of trachea are removed. Pharyngeal wallis repaired and lower tracheal stump sutured to the skin for breathing. Laryngectomy may be combined with block dissection for nodal metastasis.Total laryngectomy is indicated in the following conditions:(i) T3 lesions (i.e. with cord fixed)(ii) All T4 lesions(iii) Invasion of thyroid or cricoid cailage(iv) Bilateral arytenoid cailage involvement(v) Lesions of posterior commissure(vi) Failure after radiotherapy or conservation surgery(vii) Transglottic cancers, i.e. tumours involving supraglottis and glottis acrossDhingra 6e pg: 310", "cop": 2, "opa": "LASER", "opb": "Subtotal laryngectomy", "opc": "Stripping of the vocal cord", "opd": "Radiotherapy", "subject_name": "ENT", "topic_name": "Pharynx", "id": "d0dfc865-7f5c-4003-85b0-7c8d71b1d3dc", "choice_type": "single"} {"question": "Bell's palsy is", "exp": "B. i.e. (Ipsilateral LMN paralysis) (4- Dhingra 4th) (104 - Maqbool 11th)* BELL'S PALSY - Idiopathic infranuclear lesion of the facial nerve more common in diabetic (angiopathy) and pregnant women (retension of fluid)* BELL'S PHENOMENON - up and out rolling of the eyeball during forceful closure**Clinical presentation - LMN paralysis** patients is unable to close eye, saliva dribbles from the angle of mouth, face become asymmetrical, Tears flow down from the eye (epithora) stapedial reflex or loss of taste* Epiphora in bell's palsy is due to - lacrimal pump failure*** Melkersson's syndrome - Triad - Facial paralysis, swelling of lips, and fissured tongue* Recurrent facial palsy - causes - Melkersson's syndrome DM, sarcoidosis and tumoursPoor prognostic indicators - advanced age, hyperacusis, severe initial painBilateal facial paralysis - seen in GB syndrome sarcoidosis, sickle cell disease, acute leukemia, bulbar palsy, leprosy and sother systemic disorders.Supra nuclear (central) palsv primarily involves the lower part of face and emotional responses may be intact (eg- the patients not be able to show you his teeth but will smile to a Joke. Nuclear palsv affects all ipsilateral muscles of facial expression resulting in paralysis of the ipsilateral side (eg. mouth is pulled to the normal side and may droop on the affected side facial crease are effaced, eyelid may not close).", "cop": 2, "opa": "Hemiparesis with contralateral facial nerve palsy", "opb": "Ipsilatera lower motor meuron lesion", "opc": "Ipsilateral upper motor neuron lesion", "opd": "Contralateral upper motor neuron lesions", "subject_name": "ENT", "topic_name": "Facial Nerve And Its Disorders", "id": "c414bcae-26b0-411a-938f-b9bb16b7bb1b", "choice_type": "single"} {"question": "Infection of CNS spreads to the inner ear through", "exp": "The scale tympani is closed by a secondary tympanic membrane. It is also connected with subarachnoid space through aqueduct of the cochlea", "cop": 1, "opa": "Cochlear aqueduct", "opb": "Endolymphatic sac", "opc": "Vestibular aqueduct", "opd": "Hyrtl's fissure", "subject_name": "ENT", "topic_name": null, "id": "6c3f9fc3-a922-436a-a87a-6017b524c6d2", "choice_type": "single"} {"question": "In carcinoma of lower lip secondaries are seen in", "exp": "Carcinoma Lip: Mostly, it is squamous cell carcinoma, often seen in males in the age group of 40-70 years. Lower lip is more often involved. Site of predilection is between the midline and commissure of the lip. Lesion is of exophytic or ulcerative type. Lymph node metastases develop late. Submental and submandibular nodes are the first to be involved; other deep cervical nodes may also get involved later. Treatment: It is surgical excision with adequate safety margin of healthy tissue and plastic repair of the defect. Lymph node metastases require block dissection. Radiotherapy also gives good results in early cases. Ref:- Dhingra; pg num:-226", "cop": 1, "opa": "Upper cervical LN", "opb": "Axillary LN", "opc": "Supraclavicular LN", "opd": "Mediastinal LN", "subject_name": "ENT", "topic_name": "Oral cavity & Oesophagus", "id": "9a8b8201-90f3-48f9-9679-cb404e368a96", "choice_type": "single"} {"question": "CSF rhinorrhea is most commonly seen in fracture of", "exp": "CSF rhinorrhea occurs when there is a communication between the intracranial (intradural) and nasal cavities. CSF, rhinorrhea indicates a basal tear of dura mater along with a fracture involving the paranasal sinuses, frontal, ethmoid, or sphenoid bone. Cribriform plate of the ethmoid bone separates the anterior cranial fossa from the nasal cavity and is the most common site of fracture in CSF rhinorrhea because it is extremely thin. The cerebrospinal fluid has high glucose content, makes a dry hanky stiff, and if mixed with blood it classically produces a halo effect on the white bedclothes. Patients of CSF rhinorrhea are at increased risk of meningitis paicularly pneumococcal. The patient should be given prophylactic antibiotic (penicillin) followed by X-ray. Fractures of the middle 1/3rd of the face should be reduced and most of the times CSF rhinorrhea stops. Indications for anterior fossa explorations are the persistence of CSF rhinorrhea for> 10 days; the presence of a fracture involving the frontal or ethmoidal sinus; aerocele; an attack of meningitis that has been treated. Ref : Diseases of ENT by Dhingra 6th edition Pgno : 163-165", "cop": 1, "opa": "Cribriform plate", "opb": "Temporal bone", "opc": "Nasal bone", "opd": "Occipital bone", "subject_name": "ENT", "topic_name": "Nose and paranasal sinuses", "id": "af1d9004-2a98-455b-8db7-634b0a2c0605", "choice_type": "single"} {"question": "Treatment of choice in nasopharyngeal carcinoma.", "exp": "Radiotherapy: It is the treatment of choice for nasopharyngeal cancer. Stage I and II are treated by radiotherapy alone while Stage III and IV require concomitant radiation and chemotherapy or radiation followed by chemotherapy.", "cop": 2, "opa": "Chemotherapy", "opb": "Radiotherapy", "opc": "Chemo given with radiation", "opd": "Surgery", "subject_name": "ENT", "topic_name": "FMGE 2019", "id": "7e8c9869-3336-45be-8afb-d987c010d225", "choice_type": "single"} {"question": "Nerve involved in hyperacusis", "exp": "Paralysis of stapedius muscle which is supplied by 7th cranial nerve causes hyperacusis or phonophobia Ref: Dhingra 7e pg 510.", "cop": 1, "opa": "Facial nerve", "opb": "Glossopharyngeal", "opc": "Vagus", "opd": "Hypoglossal", "subject_name": "ENT", "topic_name": "Ear", "id": "5a58cf65-ebfe-42c4-8822-d51d4397e157", "choice_type": "single"} {"question": "Glomus tumour invading the veical pa of the carotid canal. It is", "exp": "FISCH classification is used for glomus tumours based on the extension into surrounding anatomic structures. Type A tumour - Tumour limited to the middle ear cleft (glomus tympanicum) Type B tumour - Tumour limited to the tympanomastoid area with no infralabyrinthine compament involvement Type C tumour - Tumour involving the infralabyrinthine compament of the temporal bone and extending into the petrous apex; Type C1 tumour - Tumor with limited involvement of the veical poion of the carotid canal; Type C2 tumour - Tumour invading the veical poion of the carotid canal; Type C3 tumour - Tumour invasion of the horizontal poion of the carotid canal Type D1 tumour - Tumour with an intracranial extension less than 2 cm in diameter; Type D2 tumour - Tumour with an intracranial extension greater than 2 cm in diameter", "cop": 3, "opa": "Type B", "opb": "Type C1", "opc": "Type C2", "opd": "Type C3", "subject_name": "ENT", "topic_name": "Ear", "id": "a7d66965-15fb-4922-84da-4d8cfa9e8e4f", "choice_type": "single"} {"question": "Heimlich&;s maneuver is used in", "exp": "Heimlich maneuverAbdominal thrusts also called the Heimlich maneuver, is a first aid procedure used to treat upper airway obstruction (or choking) by foreign objects. The Heimlich maneuver is named after Dr. Henry Heimlich, who first described it in 1974.Ref: PL Dhingra, Diseases of Ear, Nose & Throat, 7th edition, pg no. 367", "cop": 1, "opa": "Upper airway obstruction by foreign body", "opb": "BPPV", "opc": "Eustachian tube patency test", "opd": "Tympanic membrane integrity test", "subject_name": "ENT", "topic_name": "Larynx", "id": "1af897a3-13e5-4da8-8654-e7e596bc4597", "choice_type": "single"} {"question": "High tracheotomy is done in", "exp": "Only indication for high tracheostomy - Laryngeal carcinoma.", "cop": 2, "opa": "Vocal cord palsy", "opb": "Laryngeal carcinoma", "opc": "Subglottic stenosis", "opd": "Laryng omalacia", "subject_name": "ENT", "topic_name": null, "id": "51b2838e-3249-46a3-a7da-87559fd3b1f2", "choice_type": "single"} {"question": "In the Nodal \"N\" staging of laryngeal carcinoma, extranodal invasion is indicated by", "exp": null, "cop": 3, "opa": "N2C", "opb": "N3A", "opc": "N3B", "opd": "N3C", "subject_name": "ENT", "topic_name": null, "id": "0760745c-312f-4875-99f4-6d96a2800962", "choice_type": "single"} {"question": "The preferred treatment of verrucouse carcinoma of the larynx is", "exp": "Verrucous Carcinoma 1) Verrcous carcinoma makes up only 1-2% of laryngeal carcinomas. 2) The larynx is the second most common site of occurence in the head and neck after the oral cavity. 3) Most common site of involvement is vocal cord. 4) Grossly, verrucous carcinoma appears as a fungating, papillomatous, grayish white neoplasm. 5) Microscopically, it is well differentiated squamous cell carcinoma with minimal cytological atypia. 6) It has low metastatic potential . 7) Hoarseness is the most common presented symptom. Pain and dysphagia may occur but are less common. 8) Treatment of most verrucous tumors is primary surgery. Endoscopic laser surgery is appropriate as the tumor is less aggressive than usual squamous cell carcinoma. Ref:- Current otolaryngology; pg num:- 444", "cop": 1, "opa": "Endoscopic removal", "opb": "Electron beam therapy", "opc": "Pulmonary surgery", "opd": "Total laryngectomy", "subject_name": "ENT", "topic_name": "Larynx", "id": "d432e39a-53a3-4eda-a78c-0d989498ade9", "choice_type": "single"} {"question": "Pinna develops from", "exp": "Pinna\n\nIt develops from both 1st and 2nd brachial arches\nPeriod of development starts from 4–6 weeks and adult configuration is attained by 20th week\nFrom the", "cop": 3, "opa": "1st pharyngeal arch", "opb": "1st and 3rd pharyngeal arch", "opc": "1st and 2nd pharyngeal arch", "opd": "2nd pharyngeal arch", "subject_name": "ENT", "topic_name": null, "id": "041f6abe-f13f-43b7-8024-d77174640cb7", "choice_type": "single"} {"question": "Attenuation reflex is lost in case of", "exp": "It is based on the fact that a loud sound, 70-100 dB above the threshold of hearing of a paicular ear, causes bilateral contraction of the stapedial muscles which can be detected by tympanometry. Stapedial reflex is elicited with a sound of 70-100 dB SL Absence of Stapedial reflex when hearing is normal indicates lesion of the facial nerve, proximal to the nerve to stapedius. Stapedial reflex becomes absent when stapes is fixed.", "cop": 1, "opa": "Stapedial palsy", "opb": "Glomus tumor", "opc": "Internal ear pathology", "opd": "Malingerers", "subject_name": "ENT", "topic_name": null, "id": "d8e13281-1cf3-403a-a87d-4a06658abd77", "choice_type": "single"} {"question": "The most common laser used in laryngeal surgery is", "exp": "The LASER is an acronym for Light Amplification by Stimulated Emission of RadiationDepending upon the lasing medium, various types of lasers with differing wavelength can be created. The lasing medium can be solid (ruby, Nd: YAG or potassium titanyl phosphate); gas (CO2 or Helium-Neon) or liquid (pumped inorganic dye in a glass tube).CO2 laser: Wavelength 10,400 nm, invisible, requires an aiming beam of a helium-neon laser, cannot pass through flexible optical fibers and requires aiculated arm with a series of reflective mirrors to direct the beam to the target area.Clinically it has been used in laryngeal surgery to excise vocal nodules, polyps, cysts, granulomas or juvenile laryngeal papillomas.It cuts precisely and a spot size of 0.3 mm can be achieved; lesion is first delineated and then dissected with microlaryngeal instruments. Microflaps can be raised to treat Reinke's edema. It is also used for leukoplakia, T1 lesion of vocal cord or localized lesions of supraepiglottis and infraepiglottis. Transverse cordotomy and endoscopic paial or complete arytenoidectomy can also be done in bilateral abductor paralysis.CO2 does not penetrate into the deep tissue, hence there is decreased risk of scarring.Ref: Dhingra; 6th edition; Chapter 71; Page no: 357", "cop": 3, "opa": "Argon laser", "opb": "Nd YAG laser", "opc": "CO2 laser", "opd": "KTP laser", "subject_name": "ENT", "topic_name": "Ear", "id": "2044e3f8-ad26-4186-9752-ffc50dcb8b4f", "choice_type": "single"} {"question": "Most common malignancy arising in thyroglossal cyst is", "exp": "Thvroglossal Cyst Contains clear to cloudy fluid with sparse cellularity. Cystic background contains few if any epithelial cells. Some inflammatory cells and crystals might be present. Rarely malignant transformation occurs in about 1% of the cases. Most common malignancy arising is papillary carcinoma followed by squamous cell carcinoma.", "cop": 2, "opa": "Follicular", "opb": "Papillary", "opc": "Medullary", "opd": "Anaplastic", "subject_name": "ENT", "topic_name": null, "id": "46353a17-e87a-43f6-b1af-35c81a534fb4", "choice_type": "single"} {"question": "Not a content of sphenopalatine fossa", "exp": null, "cop": 3, "opa": "Maxillary Artery", "opb": "Maxillary nerve", "opc": "Mandibular nerve", "opd": "Pterygopalatina ganglion", "subject_name": "ENT", "topic_name": null, "id": "8a0445e3-11dc-4ee7-b806-6d5b35049c2c", "choice_type": "single"} {"question": "Muller's manoeuvre is used to", "exp": "Muller's manoeuvre is used to find level and degree of obstruction in sleep-disordered breathing with flexible nasopharyngoscope Ref: Textbook of ENT, Dhingra; 6th edition; Pg no: 449 ref img", "cop": 3, "opa": "To find out opening of mouth", "opb": "To remove laryngeal foreign body", "opc": "To find degree of obstruction in sleep disordered breathing", "opd": "To remove foreign body from ear", "subject_name": "ENT", "topic_name": "Miscellaneous ENT", "id": "ead393a5-5cc5-47b5-9459-6de0085deac2", "choice_type": "single"} {"question": "Acute Otitis media in Children is most commonly due to", "exp": "(C) Streptococcus pneumoniae # Causative Organisms of ASOM:> Most common organisms in infants and young children are Streptococcus pneumoniae (30%), Haemophilus influenzae (20%) and Moraxella catarrhalis (12%).> Other organisms include Streptococcus pyogenes, Staphylococcus aureus & sometimes Pseudomonas aeruginosa.> In about 18-20%, no growth is seen.> Many of the strains of H. influenzae & Moraxella catarrhalis are ft-lactamase producing.", "cop": 3, "opa": "Morexiello catarrhalis", "opb": "H. influenza", "opc": "Streptococcus pneumoniae", "opd": "Staphylococcus aureus", "subject_name": "ENT", "topic_name": "Miscellaneous (E.N.T.)", "id": "4ca341ca-b23e-46cc-b3c3-c9c986d9db27", "choice_type": "single"} {"question": "Acoustic reflex is lost in case of", "exp": "ACOUSTIC REFLEX is the contraction of the stapedius muscle causing a change in impedance.The minimum sound intensity required to produce stapedial reflex is called stapedial reflex threshold(S). Reflexes are bilateral and can be measured on the ipsilateral and contralateral side. For ipsilateral the reflex is measured in the ear in which the tone is given while for contralateral the reflex is tested in the ear opposite to which the tone is given. Normally,the reflex occurs about 70-90 dB above the patient's threshold. ref:Hazarika 3rd edition,page no.63", "cop": 1, "opa": "Stapedial palsy", "opb": "Glomus tumour", "opc": "Internal ear pathology", "opd": "Malingering", "subject_name": "ENT", "topic_name": "Ear", "id": "0a091b83-1503-44a2-9ae6-e3cf6cffa099", "choice_type": "single"} {"question": "Menieres disease is characterized by the triad of", "exp": "Meniere's disease is a disorder of the inner ear that can affect hearing and balance to a varying degree. It is characterized by episodes of veigo and tinnitus and progressive hearing loss, usually in one ear. It is caused by lymphatic channel dilation, affecting the drainage of endolymph.", "cop": 3, "opa": "Deafness, nystagmus, tremors", "opb": "Deafness, veigo, nystagmus", "opc": "Deafness, tinnitus, veigo", "opd": "Deafness, tremors, tinnitus", "subject_name": "ENT", "topic_name": null, "id": "2911ee95-bc5f-4ebc-9f10-4cd14290a1db", "choice_type": "single"} {"question": "Infection of CNS spreads in inner ear through", "exp": "Ans. A Cochlear aqueductAs we know that cochlear aqueduct (Aqueduct of Cochlea) is a connection between scala tympani (containing perilymph) and the subarachnoid space (containing CSF). On occassions, paicularly in young children, the Cochlear aqueduct is large and open.Infection can spread to the inner ear from the infected CSF or vice versa, the cochlear aqueduct resulting in severe profound hearing loss (meningitic labyrinthitis).", "cop": 1, "opa": "Cochlear aqueduct", "opb": "Endolymphatic sac", "opc": "Vestibular aqueduct", "opd": "Hyle fissure", "subject_name": "ENT", "topic_name": null, "id": "9f7a8734-3f08-4cc1-ab85-0c7753664fe0", "choice_type": "single"} {"question": "Ideal site for myringotomy and grommet inseion in serous otitis media is", "exp": "In serous otitis media, a radial incision is made on the anteroinferior quadrant, in acute suppurative otitis media circumferential incision is made on the posteroinferior quadrant. Ref-ENT Dhingra textbook 6th edition, page no. 65.", "cop": 2, "opa": "Anterior superior quadrant", "opb": "Anterior inferior quadrant", "opc": "Posterio-superior", "opd": "Posterio-inferior", "subject_name": "ENT", "topic_name": "Diagnostic and operative ENT", "id": "8b59f2a8-d14e-460f-81d2-fa9f15282302", "choice_type": "single"} {"question": "Fracture of mid face does not lead to", "exp": "Proptosis is the forward protrusion or displacement of the eyeball. It does not occur in fracture of middle third of the face. It may occur in the fractures of the upper third of face especially fracture of the supraorbital ridge. Ref: Textbook of ENT; Dhingra; pg 181", "cop": 1, "opa": "Proptosis", "opb": "Lengthening of face", "opc": "Malocclusion of teeth", "opd": "Anesthesia of upper lip", "subject_name": "ENT", "topic_name": "Nose and paranasal sinuses", "id": "3a82f533-f89d-4650-ac93-b6ade10e01dc", "choice_type": "single"} {"question": "The length of the external auditory canal is", "exp": "Eustachian tube, also called auditory or pharyngotympanic tube, connects nasopharynx with the tympanic cavity. In an adult, it is about 36 mm long and runs downwards, forwards and medially from its tympanic end, forming an angle of 45deg with the horizontal. It is divided into two pas: bony, which is posterolateral, forms one-third (12 mm) of the total length and f ibrocailaginous, which is anteromedial, forms two-thirds (24 mm). The two pas meet at isthmus which is the narrowest pa of the tube. Ref- Dhingraa textbook for ENT 6tg edition.pg- 57.", "cop": 4, "opa": "15 mm", "opb": "10 mm", "opc": "24 mm", "opd": "36 mm", "subject_name": "ENT", "topic_name": "Ear", "id": "72ff6f44-a475-4f44-a37a-77f8a3d7ac46", "choice_type": "single"} {"question": "Hiatus semilunaris is present in", "exp": "Middle meatus: Uncinate process is a hook-like structure running in from anterosuperior to the posteroinferior direction in the middle meatus. It&;s postero superior border is sharp and runs parallel to the anterior border of bulla ethmoidalis. This gap between the two is called hiatus semilunaris .it is two-dimensional space of 1-2 mm width. (Ref: Diseases of Ear, Nose and Throat and head and neck surgery, Dhingra 7th edition, Pg no. 150)", "cop": 2, "opa": "superior meatus", "opb": "Middle meatus", "opc": "Inferior meatus", "opd": "Sphenoethmoidal recess", "subject_name": "ENT", "topic_name": "Nose and paranasal sinuses", "id": "4d230a88-42cb-498f-bae2-1699a666e077", "choice_type": "single"} {"question": "In Maxillary carcinoma of a 60 year old patient involving anterolateral pa of maxilla, the preferred treatment is", "exp": "Paranasal sinus cancer is uncommon and represents only 0.2 to 0.8% of all malignancies. Cancer of paranasal sinus constitutes 3% of all carcinomas of the aerodigestive tract. The majority of paranasal sinus malignancies (50-80%) originate within the maxillary sinus antrum. Malignancies rarely occur within the other sinuses and originate in the ethmoid, frontal, and sphenoid sinuses in 10%, 1% and 1% respectively. The cause of parasinus malignancy is unknown. However several risk factors have been associated and therefore it is seen more commonly in people working in hardwood furniture industry, nickel refining, leather work, and manufacturer of mustard gas. More than 80% of the malignant tumours are of squamous cell variety. Rest are adenocarcinoma, adenoid cystic carcinoma, melanoma, and various type of sarcomas. Workers of furniture industry develop adenocarcinoma of the Ethmoids and upper nasal cavity. While those engaged in Nickel refining get squamous cell and Anaplastic carcinoma. Clinical features for maxillary carcinoma It is seen more commonly in the 7th decade of life. Males are affected more commonly than females. Early features of maxillary sinus malignancy are nasal stuffiness, blood-stained nasal discharge, facial paraesthesias or pain and epiphora. These symptoms may be missed or simply treated as sinusitis. Late features will depend on the direction of spread and extent of growth. Medial spread to nasal cavity gives rise to nasal obstruction, discharge and epistaxis. It may also spread into anterior and posterior ethmoid sinuses and that is why most antral malignancies are antroethmoidal in nature. Anterior spread causes swelling of cheeks. Inferior spread leads to expansion of alveolus with dental pain, loosening of teeth, poor fitting dentures, ulceration of gingiva. Superior spread invades the orbit causing proptosis, diplopia, ocular pain and epiphora. Posterior spread is into pterygomaxillary fossa, pterygoid plate and the pterygoid muscles causing trismus. Lymphatic spread in maxillary carcinoma is rare and occurs only in the late stages. Most commonly involved lymph node is submandibular lymph node followed by jugular nodes. Treatment of maxillary carcinoma For squamous cell carcinoma, the treatment of choice is a combination of radiotherapy and surgery. Radiotherapy can be given before or after surgery. Very often, a full course of pre-operative telecobalt therapy is given, followed 4 - 6 weeks later by surgical excision of the growth by total or extended maxillectomy.", "cop": 3, "opa": "Radiotherapy only", "opb": "Total/extended Maxillectomy followed by radiotherapy", "opc": "Radiotherapy followed by total/extended maxillectomy", "opd": "Total/extended maxillectomy alone", "subject_name": "ENT", "topic_name": null, "id": "b08a2c08-fd32-4634-b6e4-15f7c9c9b22d", "choice_type": "single"} {"question": "Labyrinthine aery is a branch of", "exp": "BLOOD SUPPLY OF LABYRINTH The entire labyrinth receives its aerial supply through labyrinthine aery, which is a branch of anterior-inferior cerebellar aery but sometimes from the basilar aery. Ref:- Dhingra; pg num:-10,11", "cop": 3, "opa": "Internal carotid aery", "opb": "Posterior cerebellar aery", "opc": "Anteroinferior cerebellar aery", "opd": "Basilar aery", "subject_name": "ENT", "topic_name": "Ear", "id": "eed20946-c5e8-48a2-8f15-d5e53e050590", "choice_type": "single"} {"question": "Not a surgical landmark for parotid surgery", "exp": "surgical landmark of parotid surgery are : Tragal pointer Digastric posterior belly Styloid process tympansmastoid suture ref : hazarika 4th ed", "cop": 1, "opa": "Inferior belly of omohyoid", "opb": "Tragal pointer", "opc": "Digastric posterior belly", "opd": "Dissecting from peripheral branches", "subject_name": "ENT", "topic_name": "All India exam", "id": "81c80479-6277-4219-86f2-af454124cadc", "choice_type": "single"} {"question": "Apple jelly nodules on the nasal septum are found in case of", "exp": "Lupus vulgaris is a low-grade tuberculous infection commonly affecting nasal vestibule or the skin of nose and face. \nThe skin lesions manifest characteristically as brown, gelatinous nodules called \"apple -jelly\" nodules. \nIn the vestibule, it presents as chronic vestibulitis. \nPerforation may occur in the cartilaginous part of the nasal septum. Treatment is same as for tuberculosis.", "cop": 2, "opa": "Tuberculosis", "opb": "Lupus Vulgaris", "opc": "Syphilis", "opd": "Rhinoscleroma", "subject_name": "ENT", "topic_name": null, "id": "a65f7388-30bf-49f0-8264-29948e5b8e14", "choice_type": "single"} {"question": "Ideal hearing aid for patient with anotia is", "exp": "Anotia (\"no ear\") describes a rare, congenital deformity, that involves the complete absence of the pinna, the outer projected poion of the ear, and narrowing or absence of the ear canal. A bone-anchored hearing aid is an implantable device that allows the listener to hear by bone conduction without the inconvenience of a traditional bone conduction hearing aid. It's more comfoable, and the sound quality is better, paly because the device stays in place. Unlike a middle ear implant, which requires a normally functioning middle ear, this type of hearing aid is typically used by listeners who have conductive or mixed hearing loss. Bone conduction hearing aids have traditionally been used by people born without ear canals or with small, misshapen ears that can't suppo conventional hearing aids, or by people who have chronic middle ear disease that's aggravated by wearing a hearing aid or earmold in the ear canal (for example, people with draining ears).", "cop": 2, "opa": "Incanal", "opb": "Bone anchored hearing aid", "opc": "Vestibular", "opd": "Transcutaneous", "subject_name": "ENT", "topic_name": null, "id": "787a3117-4dee-46a2-9fb7-1ff555551f34", "choice_type": "single"} {"question": "Postitional veigo is", "exp": "The disease is caused by the disorder of posterior semicircular canal. Otoconial debris when settles in the cupula of the posterior semicircular canal in a critical head position causes displacement of cupula and veigo. (Ref: Diseases of EAR, NOSE AND THROAT by PL DHINGRA - 6th Edition)", "cop": 4, "opa": "Lateral", "opb": "Superior", "opc": "Inferior", "opd": "Posterior", "subject_name": "ENT", "topic_name": "Ear", "id": "5b19e0c0-55a9-44b6-bac1-7382edf82e86", "choice_type": "single"} {"question": "Gutzmann pressure in pubuophonia is given", "exp": null, "cop": 1, "opa": "Backwards and Downwards", "opb": "Backwards", "opc": "Backwards and up", "opd": "Downwards", "subject_name": "ENT", "topic_name": null, "id": "165a150a-0110-40c5-ba03-a199ea2190e8", "choice_type": "single"} {"question": "Most common cause of fungal sinusitis is", "exp": "Commonest cause of fungal sinusitis is Aspergillus fumigatusOther causes (Non-invasive form) - A.niger, A.flavus, CandidaNon-invasive fungal sinusitis presents as a fungal ball or allergic fungal rhinosinusitis (AFRS)Ref: Hazarika; 3rd ed; Pg 338", "cop": 1, "opa": "A. fumigatus", "opb": "A niger", "opc": "A flavus", "opd": "Candida", "subject_name": "ENT", "topic_name": "Nose and paranasal sinuses", "id": "426bbf91-8868-4449-aa60-3193e5f98916", "choice_type": "single"} {"question": "Irvin Moore sign is positive in", "exp": "Irwin-Moore sign:- Expression of cheesy material from the tonsil, on pressing anterior pillar seen in case of chronic tonsillitis. Ref:- Textbook of ENT, Hazarika; pg num:- 480", "cop": 3, "opa": "Adenoid hyperophy", "opb": "Acute tonsillitis", "opc": "Chronic tonsillitis", "opd": "Epiglottitis", "subject_name": "ENT", "topic_name": "Pharynx", "id": "c1bcdb12-b44e-41c3-aaae-086d1c25eec7", "choice_type": "single"} {"question": "Sensory supply of Larynk below vocal cord", "exp": "Sensory supply larynx :\n\nAbove VC = Internal branch of superor laryngeal nerve.\nBelow VC = Recurent laryngael nerve.", "cop": 3, "opa": "Extereal branch of superior laryngeal nerve", "opb": "Internal branch of Inferior laryngeal nerve", "opc": "Recurrent laryngeal narve", "opd": "Inferior pharyngeal nerve", "subject_name": "ENT", "topic_name": null, "id": "a4d36786-f8c2-4e88-8bae-11549a038150", "choice_type": "single"} {"question": "Topical Mitomycin C is useful in treatment of", "exp": "Dilation for laryngotracheal stenosis is typically done using serial dilation using various sizes of Jackson bronchoscopes. The various studies have showed that mechanical dilation used alone can have a recurrence rate of stenosis of greater than 90%. Thus, various adjunctive procedures have been developed in order to try to decrease the rate of restenosis. Mitomycin-C is an example of one of these adjunctive agents. Ref : Internet sources", "cop": 2, "opa": "Angiofibroma", "opb": "Tracheal stenosis", "opc": "Skull base Osteomyelitis", "opd": "Laryngeal carcinoma", "subject_name": "ENT", "topic_name": "All India exam", "id": "0ac1298a-8130-4312-835c-1fe8acfe81d8", "choice_type": "single"} {"question": "A 38 year old female complaining of decreased hearing in the right ear for the last two years. On testing with a 512 Hz tuning fork, the rinne's test without masking is negative on the right ear and positive on the left ear. With the weber's test, the tone is perceived as louder in the left ear. The most likely problem the patient has is", "exp": "Patient is presenting with right sided deafness.Interpretation of Rinne's test in this patientRight side negative means : -i) Right sided conductive deafness.orii) Right sided severe sensorineural hearing loss which gives false negative results.Left side positive test means : -Normal left earLeft sided sensorineural hearing loss (However this is not the case in this patient as the patient is presenting with right sided hearing loss).Interpretation of Weber test in this patientWeber is lateralized to left ear. That means : -Right sided sensorineural hearing loss.Left sided conductive deafness (However, this is not the case as patient is presenting with right sided hearing loss).Thus , this patient is suffering from severe right side sensorineural hearing loss in which the Rinne's test is false negative and weber is lateralized to left ear.", "cop": 2, "opa": "Right conductive hearing loss", "opb": "Right sensorineural hearing loss", "opc": "Left sensorineural hearing loss", "opd": "Left conductive hearing loss", "subject_name": "ENT", "topic_name": null, "id": "d9bb99e3-d945-4b79-abd5-eec99857e933", "choice_type": "single"} {"question": "Goldring's criteria is used to diagnose", "exp": null, "cop": 2, "opa": "Quinsy", "opb": "Ludwig's Angina", "opc": "Naropharyngeal carcinoma", "opd": "Chronic Tonsillitis", "subject_name": "ENT", "topic_name": null, "id": "3420d907-faf8-4253-a221-a074a45d4f11", "choice_type": "single"} {"question": "Transverse fracture of maxilla is", "exp": "Le Fo-I fracture or transverse fracture of maxilla runs above and parallel to the palate. It crosses the lower pa of nasal septum, maxillary antra and the pterygoid plates. Le Fo II fracture is a pyramidal fracture passing through the root of the nose, lacrimal bone, floor of the orbit, upper pa of the maxillary sinus and pterygoid plates. This fracture has some features common with zygomatico fractures. Le Fo III ( Craniofacial dysjunction)This is complete separation of facial bones from the cranial bones. Ref: Textbook of diseases of ENT, PL Dhingra, 7th edition, pg no. 207", "cop": 1, "opa": "Le Fo - 1", "opb": "Le Fo -2", "opc": "Le Fo-3", "opd": "Craniofacial disruption", "subject_name": "ENT", "topic_name": "Nose and paranasal sinuses", "id": "90695bd3-ea4c-43c8-a167-de19bf54aed8", "choice_type": "single"} {"question": "A suspected case of Dysphagia lusoria should be best investigated by", "exp": "Barium swallow, bronchoscopy, or angiographies are investigations to diagnose but angiography is diagnostic. Ref : ENT textbook by Dhingra 6th edition Pgno : 447", "cop": 3, "opa": "HRCT", "opb": "Spiral CT", "opc": "MRA", "opd": "DSA", "subject_name": "ENT", "topic_name": "All India exam", "id": "060b549c-53d3-4e21-b6ab-ca9aa9cac422", "choice_type": "single"} {"question": "Artery that does not supply Tonsil is", "exp": null, "cop": 4, "opa": "Lingual artery", "opb": "Facial artery", "opc": "Maxillary artery", "opd": "Temporal artery", "subject_name": "ENT", "topic_name": null, "id": "1d304a8a-6ade-4797-b1d1-543f59722a9e", "choice_type": "single"} {"question": "Otitic barotrauma results due to", "exp": "Otitic barotrauma is a nonsuppurative condition resulting from the failure of the eustachian tube to maintain middle ear pressure at ambient atmospheric level. The usual cause is rapid descent during an air flight, underwater diving or compression in a pressure chamber. Reference: Diseases of Ear, Nose and Throat, PL Dhingra, Shruthi Dhingra,7th edition, pg no.71", "cop": 2, "opa": "Ascent in air", "opb": "Descent in air", "opc": "Linear acceleration", "opd": "Sudden acceleration", "subject_name": "ENT", "topic_name": "Ear", "id": "83890495-5b7d-4e3f-9918-93013f067fe8", "choice_type": "single"} {"question": "Commonest extracranial complication of CSOM", "exp": "Pus may break through mastoid coex leading to subperiosteal abscess which may even burst on surface leading to a discharging fistula It occurs as a sequence of mastoiditis which is a most common complication Ref: Dhingra 7e pg 84.", "cop": 3, "opa": "Petrositis", "opb": "Labyrinthitis", "opc": "Subperiosteal abscess", "opd": "Facial nerve paralysis", "subject_name": "ENT", "topic_name": "Ear", "id": "a37e00f3-1f80-4841-8fcb-244e287e39a2", "choice_type": "single"} {"question": "Cauliflower ear is seen in", "exp": "Hematoma of the auricle*It is the collection of blood between the auricular cailage and its perichondrium. It usually occurs due to blunt trauma and often seen in boxers, wrestlers and rugby players, therefore it is also called Boxer's ear *If hematoma gets infected severe perichondritis may set inRef: PL Dhingra, Diseases of Ear, Nose & Throat, 7th edition, pg no. 52", "cop": 3, "opa": "Pseudomonas infection", "opb": "Aspergillus infection", "opc": "Post traumatic cases", "opd": "Tuberculosis", "subject_name": "ENT", "topic_name": "Ear", "id": "8f9f4435-d05f-4001-b1a1-3b352c39efe6", "choice_type": "single"} {"question": "Initial screening test for newborn hearing disorder is", "exp": "OAEs are used as a screening test of hearing in neonates and to test hearing in uncooperative or mentally challenged individuals after sedation. Other uses of OAE: They help to distinguish cochlear from retrocochlear hearing loss. OAEs are absent in cochlear lesions, e.g. ototoxic sensorineural hearing loss. They detect ototoxic effects earlier than pure tone audiometry. OAEs are also useful to diagnose retrocochlear pathology, especially auditory neuropathy. Auditory neuropathy is a neurologic disorder of CN VIII. Audiometric tests, e.g. SNHL for pure tones, impaired speech discrimination score, absent or abnormal auditory brainstem response, show a retrocochlear type of lesion but OAEs are normal. Note: OAEs are absent in 50% of normal individuals Lesions of the cochlea Middle ear disorders (as sound travelling in the reverse direction cannot be picked up) and When hearing loss exceeds 30 dB Ref: Dhingra; 6th Edition; pg no 28", "cop": 2, "opa": "ABR - Auditory Brainstem Response", "opb": "Otoacoustic Emissions", "opc": "Free Field Audiometry", "opd": "Visual reinforcement audiometry", "subject_name": "ENT", "topic_name": "Ear", "id": "8a566bd1-77cb-4b41-82d8-991ffe55f1af", "choice_type": "single"} {"question": "Treatment of choice in central safe perforation is", "exp": "Treatment of choice in Tubotympanic type of CSOM is tympanoplasty, which involves eradication of middle ear disease with ossiculoplasty or myringoplasty or both.Myringoplasty involves only repair of tympanic membrane perforation.Modified mastoidectomy is done along with tympanoplasty in selected cases.Ref: Hazarika; 3rd ed; Pg 154", "cop": 1, "opa": "Tympanoplasty", "opb": "Modified mastoidectomy", "opc": "Myringoplasty", "opd": "Conservative management", "subject_name": "ENT", "topic_name": "Ear", "id": "9b002bb5-1b63-43fe-b5f5-01f60ad1e107", "choice_type": "single"} {"question": "Thudichum speculum is used for visualizing", "exp": "Thudichum speculum or vienna type speculum is used to visualize anterior nasal cavity (examination is called anterior rhinoscopy). Posterior nares can be visualized during posterior rhinoscopy using posterior nasal mirror.", "cop": 3, "opa": "Posterior nasal cavity", "opb": "Posterior nares", "opc": "Larynx", "opd": "Anterior nasal cavity", "subject_name": "ENT", "topic_name": null, "id": "a90a4c4e-1c23-4a32-8fcd-45bb02db8519", "choice_type": "single"} {"question": "Pyramidal fracture of maxilla is", "exp": "Pyramidal or Le Fo 2 fracture of maxilla passes through the root of nose, lacrimal bone, the floor of the orbit, the upper pa of the maxillary sinus and pterygoid plates. Ref: Textbook of diseases of ENT, PL Dhingra, 7th edition, pg no. 207", "cop": 2, "opa": "Le Fo - 1", "opb": "Le Fo -2", "opc": "Le Fo-3", "opd": "Craniofacial disruption", "subject_name": "ENT", "topic_name": "Nose and paranasal sinuses", "id": "6602723f-8347-418c-9441-aaf7320e0879", "choice_type": "single"} {"question": "Treatment of choice for T2 carcinoma of nasopharynx is", "exp": "Radiotherapy is the treatment of choice for nasopharyngeal cancer. Stage I and II are treated by radiotherapy alone while stage III and IV require concomitant radiation and chemotherapy or radiation followed by chemotherapy. External beam radiation of 6000-7000 cGy can be delivered by the linear accelerator to the primary and both sides of the neck. More advanced techniques of radiotherapy such as three-dimensional conformal radiotherapy and intensity modulated radiotherapy (IM) are now being used more and more. (Ref: Diseases of Ear, Nose and Throat, P.L Dhingra, 7th edition, page 285)", "cop": 2, "opa": "Surgery", "opb": "Radiotherapy", "opc": "Chemotherapy", "opd": "Surgery and postoperative radiotherapy", "subject_name": "ENT", "topic_name": "Pharynx", "id": "01385b2f-23d0-4fce-844f-d5b847b16551", "choice_type": "single"} {"question": "Complication of trauma to danger area of the face", "exp": "The area of the upper lip and the low rent part of the nose is the danger area of the face. It is due to that this area is the common site of infection.\nThis area is drained by a facial vein which communicates with the cavernous sinus through the superior ophthalmic vein and pterygoid venous plexus through the emissary's vein,\nIn case of any infection of this area, it may spread to the cavernous sinus causing infection and/or thrombosis.", "cop": 3, "opa": "Visual loss", "opb": "Meningitis", "opc": "Cavernous sinus infection", "opd": "Loss of memory", "subject_name": "ENT", "topic_name": null, "id": "8e77c4cc-5ed8-4fe3-bfe1-0b99e6725464", "choice_type": "single"} {"question": "Sense of gravity is detected by", "exp": "Utricle and saccule are stimulated by displacement of the otolithic membrane during the head tilts.They are concerned with acceleration and deceleration. (Ref: Diseases of EAR, NOSE and THROAT by PL Dhingra; 6th edition; page no.18)", "cop": 4, "opa": "Horizontal semicircular canal", "opb": "Lateral semicircular canal", "opc": "Superior semicircular canal", "opd": "Utricle and saccule", "subject_name": "ENT", "topic_name": "Ear", "id": "bdbab6e1-6614-4aa2-8ccb-baa1de8f6dba", "choice_type": "single"} {"question": "Paracusis Willisii is seen in", "exp": "Paracusis Willisii. An otosclerotic patient hears better in noisy than in quiet surroundings. This is because a normal person will raise his voice in noisy surroundings.Ref: Dhingra; 6th edition; Chapter 13; Page no: 87", "cop": 1, "opa": "Otosclerosis", "opb": "Meniere's disease", "opc": "Glomus tumor", "opd": "Tubercular otitis media", "subject_name": "ENT", "topic_name": "Ear", "id": "3fba8f32-a009-4cab-ae7d-b9ac96df6cc4", "choice_type": "single"} {"question": "Crypta magna is seen in", "exp": "APPLIED ANATOMY OF PALATINE (FAUCIAL) TONSILS Medial surface of the tonsil is covered by nonkeratinizing stratified squamous epithelium which dips into the substance of tonsil in the form of crypts. Openings of 12-15 crypts can be seen on the medial surface of the tonsil. One of the crypts, situated near the upper pa of tonsil is very large and deep and is called crypta magna or intratonsillar cleft. It represents the ventral pa of second pharyngeal pouch. From the main crypts arise the secondary crypts, within the substance of tonsil. Crypts may be filled with cheesy material consisting of epithelial cells, bacteria and food debris which can be expressed by pressure over the anterior pillar. Ref:- Dhingra; pg num:-257", "cop": 4, "opa": "Nasopharyngeal tonsil", "opb": "Lingual tonsil", "opc": "Tubal tonsil", "opd": "Palatine tonsil", "subject_name": "ENT", "topic_name": "Pharynx", "id": "6eb0972f-1ade-4346-8d1e-d3cb175621ad", "choice_type": "single"} {"question": "Pott's puffy tumor is seen in", "exp": "Ans. is 'b' i.e., Paranasal sinus Complications of sinusitis* Complications in properly managed sinusitis are uncommon.* Complication of sinus infection may be1) Local:- Mucocele / mucopyocele, mucous retention cyst, osteomyelitis.2) Orbital :- Periorbital cellulitis, orbital cellulitis, orbital abscess, subperiosteal abscess, cavernous sinus thrombosis, superior orbital fissure syndrome, orbital apex syndrome, edema of eye lids, retrobular neuritis with impaired vision.3) Intracranial :- Intracranial abscess (Epidural, subdural, parenchymal), meningitis, seizure, sepsis, focal neurological deficit.4) Descending infections :- Pharyngitis, laryngitis, tonsillitis, tracheobronchitis, otitis media.5) Systemic :- Toxic shock syndrome (very rare)* Osteomyelitis is more common in frontal sinusitis. Osteomyelitis of frontal bone can cause subperiosteal abscess known as Pott's puffy tumor.* Orbital cellulitis is particular^ common in ethmoid sinusitis.* Cavernous sinus thrombosis and intracranial complications are more common with sphenoid sinusitis.", "cop": 2, "opa": "Vertebrae", "opb": "Paranasal sinus", "opc": "Bones forming ankle joint", "opd": "Neck", "subject_name": "ENT", "topic_name": "Nose and PNS", "id": "926c119e-27e2-4ff7-8934-e5af02b26c20", "choice_type": "single"} {"question": "Keratinizing squamous cell carcinoma of nasopharynx is", "exp": "WHO classification based on histopathology Present WHO terminology Former terminology Type I (25% Keratinizing carcinoma Squamous cell carcinoma Type II (12%) Nonkeratinizing differentiated carcinoma * Transitional cell carcinoma * Intermediate cell carcinoma * Lymphoepithelial carcinoma Type III (63%) Nonkeratinizing undifferentiated carcinoma * Anaplastic carcinoma * Clear cell carcinoma * Lymphoepithelial carcinoma (Schminke type is tumour with diffusely distributed malignant tissue) * Spindle cell carcinoma (Ref: Diseases of Ear, Nose and Throat, P.L Dhingra, 7th edition, page 283)", "cop": 1, "opa": "Type I", "opb": "Type II", "opc": "Type III", "opd": "Type IV", "subject_name": "ENT", "topic_name": "Pharynx", "id": "ad1acb17-a1a8-43da-a3d0-211cba9c2961", "choice_type": "single"} {"question": "The Eustachian tube is made up of", "exp": "Eustachian tube Anatomy Eustachian tube, also called auditory or pharyngotympanic tube, connects nasopharynx with the tympanic cavity. In an adult, it is about 36 mm long and runs downwards, forwards and medially from its tympanic end, forming an angle of 45deg with the horizontal. It is divided into two pas: bony, which is posterolateral, forms one-third (12 mm) of the total length and fibrocailaginous, which is anteromedial, forms two-thirds (24 mm). The two pas meet at isthmus which is the narrowest pa of the tube. The fibrocailaginous pa of the tube is made of a single piece of cailage folded upon itself in such a way that it forms the whole of medial lamina, roof and a pa of the lateral lamina; the rest of its lateral Lamina is made of fibrous membrane. The tympanic end of the tube is bony, measures 5 x 2 mm and is situated in the anterior wall of middle ear, a little above the level of floor. The pharyngeal end of the tube is slit-like, veically. The cailage at this end raises an elevation called torus tubarius, which is situated in the lateral wall of the nasopharynx, 1-1.25 cm behind the posterior end of inferior turbinate. Ref : ENT textbook by Dhingra 6th edition Pgno : 57,58", "cop": 3, "opa": "Hyaline cailage", "opb": "Fibrocailage", "opc": "Elastic cailage", "opd": "Fibro-elastic cailage", "subject_name": "ENT", "topic_name": "All India exam", "id": "9539a81d-5b13-470c-b385-8ac1882461c6", "choice_type": "single"} {"question": "Not a differential Diagnosis for grey white membrane on Tonsil", "exp": null, "cop": 2, "opa": "Candidiasis", "opb": "Ludwig's Angina", "opc": "Vincent's Angina", "opd": "Malignancy tonsil", "subject_name": "ENT", "topic_name": null, "id": "1c9ce032-cf70-4271-a044-b50a52d9516e", "choice_type": "single"} {"question": "Paralysis of 3rd, 4th & 6th nerves with involvement of ophthalmic division of 5th nerve, localizes the lesion to", "exp": "CAVERNOUS SINUS THROMBOSIS Aetiology Infection of paranasal sinuses, paicularly those of ethmoid and sphenoid and less commonly the frontal, and orbital complications from these sinus infections can cause thrombophlebitis of the cavernous sinus(es).The valveless nature of the veins connecting the cavernous sinus causes easy spread of infection. Clinical features Onset of cavernous sinus thrombophlebitis is abrupt with chills and rigors. Patient is acutely ill. Eyelids get swollen with chemosis and proptosis of eyeball. Cranial nerves III, IV and VI which are related to the sinus get involved individually and sequentially causing total ophthalmoplegia. Pupil becomes dilated and fixed, optic disc shows congestion and oedema with diminution of vision. Sensation in the distribution of V1 (ophthalmic division of CN V) is diminished. CSF is usually normal. Condition needs to be differentiated from orbital cellulitis. CT scan is useful for this. Treatment. It consists of i.v. antibiotics and attention to the focus of infection, drainage of infected ethmoid or sphenoid sinus. Blood culture should be taken before staing antibiotic therapy. Role of anticoagulants is not clear. Ref : ENT textbook by Dhingra 6th edition Pgno : 201", "cop": 1, "opa": "Cavernous Sinus", "opb": "Apex of orbit", "opc": "Brainstem", "opd": "Base of skull", "subject_name": "ENT", "topic_name": "All India exam", "id": "d94fb070-d3ec-46fe-b3fd-abc8020967fe", "choice_type": "single"} {"question": "In pure tone audiogram the symbol X is used to mark", "exp": "Ref:- Dhingra; pg num:-30", "cop": 4, "opa": "No change in air conduction in right ear", "opb": "Bone conduction in right ear", "opc": "Air conduction in right ear", "opd": "Air conduction in left ear", "subject_name": "ENT", "topic_name": "Ear", "id": "22246f82-e246-489d-8d58-53bea7276b22", "choice_type": "single"} {"question": "Prerequisite for Endoscopic sinus surgery", "exp": "Recent research on inflammatory sinus disease has implicated a central role for the ethmoid labyrinth, which may influence changes in the maxillary and frontal sinuses. CT can provide an excellent definition of the paranasal sinuses and paicularly the ethmoids, which is a prerequisite for endoscopic surgery. Direct coronal CT can readily demonstrate disease in the infundibulum, frontal recess and posterior ethmoids in the same orientation confronting the endoscopist and helps in the planning of ethmoidal surgery.", "cop": 2, "opa": "MRI of paranasal sinus", "opb": "CT of PNS", "opc": "Mucocilliary clearing testing", "opd": "Acoustic tests", "subject_name": "ENT", "topic_name": "Nose and paranasal sinuses", "id": "217a45d1-5c89-498e-9291-046a223502f8", "choice_type": "single"} {"question": "Not a supra hyoid muscle", "exp": null, "cop": 1, "opa": "Omohyoid", "opb": "Myloyiod", "opc": "Genihoyoid", "opd": "Digastrics", "subject_name": "ENT", "topic_name": null, "id": "974183cb-4861-4643-b0c4-41f5d7396357", "choice_type": "single"} {"question": "FISCH classification is used for", "exp": "FISCH classification is used for glomus tumours based on the extension into surrounding anatomic structures. Type A tumour - Tumour limited to the middle ear cleft (glomus tympanicum) Type B tumour - Tumour limited to the tympanomastoid area with no infralabyrinthine compament involvement Type C tumour - Tumour involving the infralabyrinthine compament of the temporal bone and extending into the petrous apex; Type C1 tumour - Tumor with limited involvement of the veical poion of the carotid canal; Type C2 tumour - Tumour invading the veical poion of the carotid canal; Type C3 tumour - Tumour invasion of the horizontal poion of the carotid canal Type D1 tumour - Tumour with an intracranial extension less than 2 cm in diameter; Type D2 tumour - Tumour with an intracranial extension greater than 2 cm in diameter", "cop": 4, "opa": "Juvenile nasopharyngeal angiofibroma", "opb": "Nasopharyngeal carcinoma", "opc": "Vestibular schwannoma", "opd": "Glomus tumor", "subject_name": "ENT", "topic_name": "Ear", "id": "ab73d8b8-f2dd-49d4-be1c-234acdff1782", "choice_type": "single"} {"question": "Membrane incised during hemilaryngectomy is", "exp": "Ans. is 'b' i.e., Cricothyroid Steps in hemlaryngectomyIndications and containdications* Ideal for bulky lesions of the membranous true vocal cord* Normal or slightly impaired vocal cord mobility* No involvement of the supraglottis* Tumor may extend as far as the apex of the ventricle* Subglottic extension of no more than 5 mm posteriorly (10 mm anteriorly)* The body of the arytenoid is a contraindication, although the vocal process is not* No more than one third of the contralateral vocal cord involved (consider supracricoid partial laryngectomy)Technical points* Administer perioperative antibotics* Re-encloscope the patient* Perform a preliminary tracheostomy* Make a horizontal skin incision over the thyroid ala.* Raise the flaps superiorly and inferiorly while avoiding the tracheostomy incision* Incise the perichondrium from the thyroid notch down the midline over the ericothyroid membrane.* Elevate the perichonadrium with the attached overlying strap muscles.* Mark the cartilage incisions while retaining a 3 mm wide posterior strip.* Out with a saginal saw* Use a headlight* Incise the perichondrium along the superior border of the thyroid ala. Incise across the petiole of the spiglotis. This will provide panoramic view of the larynx.* Incise the cricothyroid membrane along the superior aspect of the circoid cartilage.* Using a no. 15 blade, cut down to the true vocal cord and examine and cut through the one vocal cord either in the midline or in lesions that cross the anterior commissure. 2 to 3 mm behind the visible tumor.* Using hooks, open the thyroid alae like a book.* STOP. Carefully evaluate the lesion and determine the ares to be excised.* Excise the tumor by incising along the apex of the aryepiglotic fold down around the arytenoid or through the vocal process of the arytenoid, depending on the posterior extent of the tumor.* Orient and examing the surgical specimen and select sites for frozen section examination of the margins* Obtain hemostasis* Suture the contralateral cord to the anterior end of the remaining cartilage or to the external perichondrium* An epiglotopex must be performed in cases of extended vertical partial laryngectomy.* Insert a nasogastric tube for feeding* Close the perichondrium with probable suture* Inferiorly based or pedicled muscle flaps may be used reconstrict the glottis but are probably not necessary* A cricopharyngeal myotomy is not required* Close the wound in layers over a chain* Apply a neck dressing", "cop": 2, "opa": "Thyrohyoid", "opb": "Cricothyroid", "opc": "Aryepiglottic", "opd": "Infralaryngeal", "subject_name": "ENT", "topic_name": "Larynx", "id": "f3b5a1d9-4c50-4dd2-a847-a5d88cd821ba", "choice_type": "single"} {"question": "A patient has carcinoma on the right side of anterior 2/3rd of the tongue with lymph node of size 4cm in level 3 on the left side of the neck. Stage of the disease is", "exp": "Ref: Dhingra 7e pg 306.", "cop": 3, "opa": "N0", "opb": "N1", "opc": "N2", "opd": "N3", "subject_name": "ENT", "topic_name": "Ear", "id": "60f422f0-352a-4647-810f-ec0a3bf70741", "choice_type": "single"} {"question": "Pink reflex through intact tympanic membrane in active otosclerosis is known as", "exp": "Ans. is 'b' i.e., Schwartz sign * Schwartz sign is a pink reflex, seen through intact tympanic membrane, in the area of oval window. It indicates active otosclerosis usually during pregnancy.* Lyre's sign is splaying apart of internal and external carotid arteries on angiogram in cases of carotid body tumour of the neck.* Chvostek's sign seen in hypocalcaemia as after total thyroidectomy where parathyroids have also been removed. Tapping over the distribution of facial nerve produces a twitch.", "cop": 2, "opa": "Schwabach's sign", "opb": "Schwartz sign", "opc": "Lyre's sign", "opd": "Chvostek's sign", "subject_name": "ENT", "topic_name": "Ear", "id": "771d6038-e3ac-4461-9c92-b058fda0b433", "choice_type": "single"} {"question": "A 60 yr old lady presented with mild hearing loss & pulsatile tinnitus of right ear. Otoscopy revealed following findings. Most probable diagnosis is", "exp": "FINDINGS OF GLOMUS TUMOR: Otoscopy shows a red reflex through intact tympanic membrane. \"Rising sun\" appearance is seen when tumour arises from the floor of middle ear. Sometimes,tympanic membrane appears bluish and may be bulging. \"Pulsation sign\" (Brown sign) is positive, i.e. when ear canal pressure is raised with Siegel's speculum, tumour pulsates vigorously and then blanches; reverse happens with release of pressure All these findings suggest diagnosis of Glomus tumor", "cop": 2, "opa": "Acoustic neuroma", "opb": "Glomus tumor", "opc": "Squamous cell carcinoma", "opd": "Aural Polyp", "subject_name": "ENT", "topic_name": "Glomus", "id": "e4498791-b38f-49c4-b7bf-3287bdd4ab47", "choice_type": "single"} {"question": "Common age group for otosclerosis is", "exp": "Otosclerosis, more aptly called otospongiosis, is a primary disease of the bony labyrinth. In this, one or more foci of irregularly laid spongy bone replace pa of the normally dense enchondral layer of the bony otic capsule.Age of onset. Hearing loss usually stas between 20 and 30 years of age and is rare before 10 and after 40 years.Ref: Hazarika; 3rd ed; Pg 177", "cop": 3, "opa": "5-10 years", "opb": "10-20 years", "opc": "20-30 years", "opd": "30-45 years", "subject_name": "ENT", "topic_name": "Ear", "id": "346c0ae1-1462-458d-8819-9b4af82bc49a", "choice_type": "single"} {"question": "Rinne's positive means", "exp": "Ans: C (Normal hearing) Ref: Diseases of the Ear Nose and Throat by PL Dhingra. 5th edn .2010 page 26Explanation:Rinne's TestIt is a tuning fork test and is used to assess hearing status in a patientIn Rinne's test air conduction (AC) is compared with bone conduction (BC)Rinne's positive is AC>BC and is seen in normal hearing and SNHLRinne's negative is AC < BC and is seen in conductive hearing loss", "cop": 3, "opa": "Conductive hearing loss in ipsilateral ear", "opb": "Conductive hearing loss in opposite ear", "opc": "Normal hearing", "opd": "AC < BC", "subject_name": "ENT", "topic_name": "Ear", "id": "beebf41d-9955-4ce2-aa3e-9f0e3ee23421", "choice_type": "single"} {"question": "Unsafe perforation of tympanic membrane is", "exp": "Ans. b (Marginal).(Ref. Diseases of ENT, PL Dhingra, 2nd ed., 71, 3rd ed., 99)# Tubotympanic disease or central perforation remains localized to mucosa and that too mostly to anterioinferior part of middle ear cavity.# While atticoantral type or marginal or unsafe otitis media involves posteriosuperior part of middle ear cleft (attic, antrum, posterior tympanum and mastoid) and is associated with cholesteatoma, which due to its bone eroding property causes risk of serious complications and hence called unsafe or dangerous type.# Attic and posterosuperior marginal perforations are in dangerous type of CSOM and are often associated with cholesteatoma while central perforation is considered safe, as cholesteatoma is usually not associated with them.", "cop": 2, "opa": "Central", "opb": "Marginal", "opc": "Paracentral", "opd": "Multiple central", "subject_name": "ENT", "topic_name": "CSOM and its Complications", "id": "012edbaf-4cc6-4995-b1b5-b0a58c1c8cb0", "choice_type": "single"} {"question": "Most common fractured bone", "exp": "Facial trauma, also called maxillofacial trauma, is any physical trauma to the face.\n\nCommonly injured facial bones include the nasal bone, maxilla, and the mandible.\n\nThe mandible may be fractured at its symphysis, body, angle, ramus, and condyle.\n\nNasal fracture is the most common facial fracture and the third most common fracture of the skeleton overall.\nFacial fractures occur for a variety of reasons related to sports participation:\n\ncontact between players (eg, a head, fist, elbow); contact with equipment (eg, balls, pucks, handle bars); or contact with the environment, obstacles, or a playing surface (eg, wrestling mat, gymnastic equipment, goalposts, trees).\n\nFacial fractures may be associated with head and cervical spine injuries and it requires a significant amount of force.", "cop": 2, "opa": "Nasoethmoid bone", "opb": "Nasal bone", "opc": "Zygomatic bone", "opd": "Mandible", "subject_name": "ENT", "topic_name": null, "id": "98c7d34e-0311-41b5-a07f-88413263b0a5", "choice_type": "single"} {"question": "The Most common cause of vocal cord palsy is", "exp": "Thyroid surgery is the most common cause when both recurrent and external laryngeal nerves of one side may be involved. It may also occur in lesions of nucleus ambiguous or that of the vagus nerve proximal to the origin of superior laryngeal nerve. Thus, the lesion may lie in the medulla, posterior cranial fossa, jugular foramen or parapharyngeal space. Ref: PL Dhingra Textbook of Ear, Nose and Throat, Edition 6, page - 301", "cop": 1, "opa": "Total thyroidectomy", "opb": "Bronchogenic carcinoma", "opc": "Aneurysm of aoa", "opd": "Tuberucular lymph nodes", "subject_name": "ENT", "topic_name": "Larynx", "id": "39695e30-04d4-4c02-ac3e-f91f04fe3c56", "choice_type": "single"} {"question": "The causative agent for virulent external ear infection in a diabetic elderly is mostly by", "exp": "Pseudomonas aeruginosa is the agent which is responsible for virulent external ear infection in diabetics, mainly Necrotizing otitis externa. (Ref: Textbook of diseases of ENT, PL Dhingra, 7th edition, pg no. 55)", "cop": 3, "opa": "Staphylococcus", "opb": "Streptococcus", "opc": "Pseudomonas", "opd": "Hemophilus influenzae", "subject_name": "ENT", "topic_name": "Ear", "id": "13ce024b-7668-4eb8-a539-97c38c99cfbd", "choice_type": "single"} {"question": "The ideal time for Quinsy surgery after acute tonsillitis is", "exp": "Tonsillectomy is usually advised 6 weeks after the quinsy attack (Interval tonsillectomy)Hot tonsillectomy- Tonsillectomy during the active abscess stageIn Quinsy, pus collects lateral to the capsule of the tonsil, in the peritonsillar space containing loose areolar tissueRef: Hazarika; 3rd ed; Pg 487", "cop": 3, "opa": "2 weeks", "opb": "4 weeks", "opc": "6 weeks", "opd": "12 weeks", "subject_name": "ENT", "topic_name": "Pharynx", "id": "57dde80e-8e45-414a-9189-43d477a5a744", "choice_type": "single"} {"question": "Caloric test assesses the function of", "exp": "Ans. c (Lateral semicircular canal) (Ref. Bhargava 5th Ed, 36; Dhingra 4th ed., 43)CALORIE TEST (HALLPIKE TEST)# To test vestibular function by stimulating the Labyrinth by water of 30degC (cold) and 44degC (hot).# Position of patient: Head raised to 30deg from horizontal in supine position to bring horizontal semicircular canal in vertical position.# Inference: Nystagmus produced (COWS).WaterNystagmusCold (30degC)Opposite SideWarm (44degC)Same SideInterpretationa) Normal: Time taken from start of irrigation to end point of Nystagmus is taken and it is 80-120 seconds.b) Canal paresis:Duration of Nystagmus shortened, i.e., < 80 sec.Indicates depressed function of ipsilateral Labyrinth, vestibular nerve and vestibular nuclei.Seen in:1. Meniere's disease; 2. Acoustic neuroma; 3. Vestibular neuritis 4. Vestibular nerve section; 5. Postural vertigoDirectional preponderance: Duration of Nystagmus increases, i.e., > 120 sec.", "cop": 3, "opa": "Ventral semicircular canal", "opb": "Anterior semicircular canal", "opc": "Lateral semicircular canal", "opd": "Posterior semicircular canal", "subject_name": "ENT", "topic_name": "Assessment of Vestibular Function", "id": "79a89532-30fa-4f60-bb5e-69c0a3fab540", "choice_type": "single"} {"question": "Drug of choice for treatment of epiglottitis is", "exp": "Ampicillin or third generation cephalosporins are effective against H. influenzae and are given by parenteral route (i.m. or i.v.) without waiting for results of throat swab and blood culture. Ref: PL Dhingra 7th edition of Ear, Nose and Throat; Pg no 328", "cop": 1, "opa": "Ceftriaxone", "opb": "Sulphamethoxazone", "opc": "Chloramphenicol", "opd": "Clindamycin", "subject_name": "ENT", "topic_name": "Larynx", "id": "f44eae11-bfc0-4d31-9083-6b6f0e3123c9", "choice_type": "single"} {"question": "Tracheostomy is not indicated in", "exp": "Friends, it is quite obvious that pneumothorax can be a complication of tracheostomy (if not performed properly) rather than an indication.", "cop": 4, "opa": "Emphysema", "opb": "Bronchiectasis", "opc": "Atelectasis", "opd": "Pneumothorax", "subject_name": "ENT", "topic_name": null, "id": "0c3d4026-824f-4ae5-a143-d7e7211c66e9", "choice_type": "single"} {"question": "After tonsillectomy, secondary haemorrhage occurs on", "exp": "COMPLICATIONS OF TONSILLECTOMY Secondary haemorrhage:- Usually seen between the fifth to tenth postoperative day. It is the result of sepsis and premature separation of the membrane. Usually, it is heralded by bloodstained sputum but may be profuse. Simple measures like removal of clot, topical application of dilute adrenaline or hydrogen peroxide with pressure usually suffice. For profuse bleeding, general anaesthesia is given and bleeding vessel is electrocoagulated. Ligation of bleeding vessel is difficult due to friable tissue. Sometimes, approximation of pillars with mattress sutures may be required. External carotid ligation may also be required if bleeding is uncontrollable. Transfusion of blood or plasma, depending on blood loss, is given. Systemic antibiotics are given for control of infection. Ref:- Dhingra; pg num:-430", "cop": 2, "opa": "Within 24 hours", "opb": "5-10 postoperative day", "opc": "After 2 weeks", "opd": "After 1 month", "subject_name": "ENT", "topic_name": "Pharynx", "id": "a49c2808-a03f-4737-8b9a-dfe6af0005bc", "choice_type": "single"} {"question": "Most superior sinus in the face", "exp": "Frontal sinus: They are situated between the inner and outer tables of frontal bones, above and deep to the supraorbital margin. Maxillary sinus: It occupies the body of the maxilla. Ethmoid sinus: They are thinned walled air cavities int he lateral wall of the ethmoid bones. Spehnoid sinus: it occupies the body of the sphenoid. (Ref: Textbook of diseases of ENT, PL Dhingra, 7th edition, pg no. 209 and 210)", "cop": 1, "opa": "Frontal", "opb": "Ethmoid", "opc": "Maxillary", "opd": "Sphenoid", "subject_name": "ENT", "topic_name": "Nose and paranasal sinuses", "id": "1f74aa62-3c50-4a43-9214-06534c86a989", "choice_type": "single"} {"question": "M/C nerve injured in ligation of superior thyroid artery", "exp": null, "cop": 4, "opa": "Recurent laryngeal nerve", "opb": "Facial nerve", "opc": "Mandibular neve", "opd": "External laryngeal nerve", "subject_name": "ENT", "topic_name": null, "id": "a7290ce8-34a7-453b-bdcf-116fb60c687b", "choice_type": "single"} {"question": "Associated with objective tinnitus", "exp": "Objective tinnitus is less frequent. Vascular lesions, such as glomus tumours or carotid aery aneurysm cause swishing tinnitus synchronous with the pulse and can be temporarily abolished by pressure on the carotid aery. Ref: PL DHINGRA diseases of ear nose and throat,7th edition, page no 121", "cop": 4, "opa": "Meinere's disease", "opb": "Acoustic neuroma", "opc": "Ear wax", "opd": "Glomus tumor", "subject_name": "ENT", "topic_name": "Ear", "id": "d4c99a66-f43c-4f8a-b665-3bf69505978c", "choice_type": "single"} {"question": "low tracheostomy is incision below", "exp": null, "cop": 3, "opa": "2nd Tracheal ring", "opb": "3rd Tracheal ring", "opc": "4th Tracheal ring", "opd": "1st Tracheal ring", "subject_name": "ENT", "topic_name": null, "id": "9e28c892-0683-41cd-923f-5136e087db3f", "choice_type": "single"} {"question": "Acinic cell carcinoma of the salivary gland arise most often in the", "exp": "ACINIC CELL CARCINOMA It is a low-grade tumour which appears similar to a benign mixed tumour. It presents as a small, firm, movable and encapsulated tumour, sometimes bilateral. Metastases are rare. A conservative approach of superficial or total parotidectomy is adopted. Ref:- Dhingra; pg num:-235", "cop": 1, "opa": "Parotid salivary gland", "opb": "Submandibular salivary glands", "opc": "Minor salivry glands", "opd": "Sublinguial salivary glands", "subject_name": "ENT", "topic_name": "Oral cavity & Oesophagus", "id": "d9393ab7-3528-4102-bbde-3fd72bdb43ad", "choice_type": "single"} {"question": "Unilateral conductive hearing loss in a patient with history of head injury. On examination, tympanic membrane is normal and mobile. The cause for deafness could be", "exp": "Acquired causes of conductive hearing loss include: Perforation of tympanic membrane Fluid in the middle ear-acute otitis media, serous otitis media, hemotympanum Mass in the middle ear (benign/malignant) Disruption of ossicles-trauma to ossicular chain, CSOM, cholesteatoma Fixation of ossicles-otosclerosis, tympanosclerosis Eustachian tube blockage", "cop": 3, "opa": "Collection of fluid in the middle ear", "opb": "Otosclerosis", "opc": "Dislocation of the incudostapedial joint", "opd": "ASOM", "subject_name": "ENT", "topic_name": null, "id": "c6342ea9-db6d-43b8-a86f-89d39ba9f582", "choice_type": "single"} {"question": "A high tracheostomy may be indicated in", "exp": "A high tracheostomy may be indicated only in carcinoma of larynx because in such cases, total larynx anyway would ultimately be removed and a fresh tracheostomy made in a clean area lower down A high tracheostomy is done above the level of thyroid isthmus Re: Textbook of Ear, Nose and Throat, Dhingra, 6th Edition; Pg no: 316", "cop": 4, "opa": "Scleroma of the larynx", "opb": "Multiple papillomatosis of larynx", "opc": "Bilateral vocal cord paralysis", "opd": "Carcinoma of larynx", "subject_name": "ENT", "topic_name": "Diagnostic and operative ENT", "id": "70e15d4f-db6a-4dbb-8e79-ab7972836f40", "choice_type": "single"} {"question": "Cailaginous pa of external auditory canal is", "exp": "Lateral 1/3rd of the external auditory canal is cailaginous and medial 2/3rd is bony.Ref: Hazarika; 3rd ed; Pg 9", "cop": 2, "opa": "Medial 1/3", "opb": "Lateral 1/3", "opc": "Medial 2/3", "opd": "Lateral 2/3", "subject_name": "ENT", "topic_name": "Ear", "id": "064b5578-c0ce-4bff-bb4e-93df3062c35b", "choice_type": "single"} {"question": "Nerve arising by two roots that surround middle meningeal Artery", "exp": null, "cop": 4, "opa": "Facial Nerve", "opb": "Maxillary Nerve", "opc": "Jacobson's Nerve", "opd": "Auriculotemporal Nerve", "subject_name": "ENT", "topic_name": null, "id": "d109afb6-8455-4c13-bd5d-8f11d26614f6", "choice_type": "single"} {"question": "Mc Ewans triangle is located on", "exp": "MacEwen's triangle is bounded by temporal line,posterosuperior segment of bony external auditory canal and the line drawn as a tangent to the external canal. It is an impoant landmark to locate the mastoid antrum in mastoid surgery. Reference-Dhingra 6/e,p 6.", "cop": 1, "opa": "Mastoid antrum", "opb": "Inner ear", "opc": "Cochlea", "opd": "Saccule", "subject_name": "ENT", "topic_name": "Ear", "id": "b955f7bc-bc61-4662-9cbc-daa90b60986b", "choice_type": "single"} {"question": "Most common mode of treatment for laryngomalacia is", "exp": "Mostly, treatment is conservative. Tracheostomy may be required for some cases of severe respiratory obstruction. Supraglottoplasty is required in cases of severe laryngomalacia. Ref: PL Dhingra 7th edition of Ear, Nose and Throat; Pg no 333", "cop": 1, "opa": "Reassurance", "opb": "Medical", "opc": "Surgery", "opd": "Wait and Watch", "subject_name": "ENT", "topic_name": "Larynx", "id": "b693c60a-cdfa-4b0d-9a8e-ad8605982eed", "choice_type": "single"} {"question": "Otomycosis is most commonly caused by", "exp": "Fungal (otomycosis):- Caused by Aspergillus (most common) and candida (2nd most common)Rare causative fungus are Phycomycetes, Rhizopus, Actinomyces, PenicilliumRef: PL Dhingra, Diseases of Ear, Nose & Throat, 7th edition, pg no. 55", "cop": 1, "opa": "Aspergillus", "opb": "Candida Albicans", "opc": "Mucormycosis", "opd": "Histoplasmosis", "subject_name": "ENT", "topic_name": "Ear", "id": "aca336d1-ddf5-40c7-8458-7e3c94580969", "choice_type": "single"} {"question": "Nasopharyngeal carcinoma causes deafness by", "exp": "Due to the obstruction of the eustachian tube, there is a conductive hearing loss with serious or suppurative otitis media. Tinnitus and dizziness may occur. Presence of unilateral serous otitis media in an adult should raise suspicion of nasopharyngeal growth. Rarely, the tumour grows up the tube into the middle ear. (Ref: Diseases of Ear, Nose and Throat, P.L Dhingra, 7th edition, page 284)", "cop": 1, "opa": "Blocking the Eustachian tube", "opb": "Serous otitis media", "opc": "Temporal bone metastasis", "opd": "Radiation", "subject_name": "ENT", "topic_name": "Pharynx", "id": "3ef21e6d-a395-4a55-81e8-ed82418ce5b7", "choice_type": "single"} {"question": "Cochlear implants are used for", "exp": "(C) Moderate and severe hearing loss # Cochlear implant stimulate auditory system with electrical impulses, while hearing aid use surviving hair cells.# Indications of cochlear implant:> Postlingually deaf patient (hearing loss after development of language)> Bilateral severe sensorineural deafness who do not respond to amplified sound stimuli.> Inability to benefit from hearing aid.> In order to select the patient for cochlear implant following tests are done to evaluate the degree of deafness and to assess the quality of surviving neuron. Pure tone audiometry, Electrocochleography, Promontory stimulation", "cop": 3, "opa": "In refractory vertigo", "opb": "Sever tinnitus", "opc": "Moderate & severe hearing loss", "opd": "Loss of balance", "subject_name": "ENT", "topic_name": "Miscellaneous (E.N.T.)", "id": "0c9505a4-f6e2-4fc3-9032-569eccd7815f", "choice_type": "single"} {"question": "Treatment of leukoplakia is", "exp": "Management of Leukoplakia : (i) Many of the lesions will disappear spontaneously if the causative agent is removed.(ii) In lesions with higher potential for malignant change, a biopsy is taken to rule out malignancy.(iii) In suspicious small lesions, surgical excision or ablation with laser or cryotherapy can be done. (Ref: Textbook of diseases of ENT, PL Dhingra, 7th edition, pg no. 253)", "cop": 2, "opa": "Radiation", "opb": "Excision and Cryotherapy", "opc": "Topical chemotherapy", "opd": "Repositioning of ill fitting dentures", "subject_name": "ENT", "topic_name": "Oral cavity & Oesophagus", "id": "169c8fe5-b977-4083-b8f2-16d7ec3972cb", "choice_type": "single"} {"question": "The mucoperichondrial flap in septoplasty is made on", "exp": "The mucoperichondrial flap is elevated on one side over the quadrangular septal cailage.Indication for surgery in DNS- Symptomatic DNS- nasal obstruction, epistaxis, headache, recurrent sinusitis.In SMR, the mucoperichondrial flap is elevated on both the sides of the septal cailage.Incision- Freer`s incision in septoplasty, Killian`s in SMR.Ref: Hazarika; 3rd ed; Page no: 283", "cop": 2, "opa": "Alar cailage", "opb": "Septal cailage", "opc": "Maxilary spine", "opd": "Sphenoidal spine", "subject_name": "ENT", "topic_name": "Nose and paranasal sinuses", "id": "16648090-e7f1-40bf-ad0f-f244d2b2acb2", "choice_type": "single"} {"question": "Flick's operation and Cody Track's procedure is for", "exp": "Fick's operation for sacculotomy is puncturing the saccule with a needle through stapes footplate. Cody's tack procedure consists of placing a stainless steel track through the stapes footplate. The tack would cause periodic decompression of the saccule when it gets distended. Both these operations were claimed to have shown good results but they could not be reproduced by others and this abandoned. Cochleosacculotomy is another similar procedure in which, instead of the saccule, the cochlear duct is punctured and drained into perilymph(otic- periotic shunt)This is done as a conservative procedure of surgical treatment of Meniere's disease. Ref: Diseases of EAR, NOSE and THROAT by PL Dhingra; 7th Edition; page no.115", "cop": 2, "opa": "Otosclerosis", "opb": "Meniere's disease", "opc": "Atrophic rhinitis", "opd": "Benign positional paroxysmal veigo", "subject_name": "ENT", "topic_name": "Ear", "id": "8296de7b-2bd6-4ca3-b48f-2b7c2c5a731b", "choice_type": "single"} {"question": "Telefono is", "exp": "TELEFONO consists of repeated slapping of the sides of the head by open palms of the assailant this may cause rupture of ear drums. Ref : internet sources", "cop": 3, "opa": "Pulling of hair", "opb": "Beating on soles", "opc": "Beating on ears", "opd": "Beating on fingers", "subject_name": "ENT", "topic_name": "All India exam", "id": "83c1e444-a250-4033-9209-cd22f7b62fb8", "choice_type": "single"} {"question": "Saccule develops from", "exp": "ref : hazarika 4th ed", "cop": 4, "opa": "Saccus anterior", "opb": "saccus medius", "opc": "Pars superior", "opd": "Pars Inferior", "subject_name": "ENT", "topic_name": "All India exam", "id": "87ab8fb0-3d81-4c09-a019-aceb1fc77856", "choice_type": "single"} {"question": "Prussak&;s space is situated in", "exp": "Prussak&;s space: Prussak&;s space lies medial to pars flaccida, lateral to the neck of malleus and above the lateral process of malleus. Anteriorly, posteriorly and superiorly, it is bounded by lateral malleal ligament. Posteriorly, it also has a gap through which the space communicates with epitympanum. Ref:- Dhingra; pg num:-449", "cop": 2, "opa": "Ear canal", "opb": "Epitympanum", "opc": "Mesotympanum", "opd": "Hypotympanum", "subject_name": "ENT", "topic_name": "Ear", "id": "8c073bf8-a143-4ea9-b85e-730dce0f3b04", "choice_type": "single"} {"question": "Le Fo classification involves fracture of", "exp": "Le Fo classification involves the fractures of maxilla which includes transverse fracture, pyramidal fracture and craniofacial dysfunction. Ref: Textbook of diseases of ENT, PL Dhingra, 7th edition, pg no. 207", "cop": 1, "opa": "Maxilla", "opb": "Mandible", "opc": "Zygomatic arch", "opd": "Nasal bone", "subject_name": "ENT", "topic_name": "Nose and paranasal sinuses", "id": "5184dc42-d555-4a0c-9219-b8a9e41204ff", "choice_type": "single"} {"question": "Gold standard surgical procedure for prevention of cheonic aspiration is", "exp": null, "cop": 3, "opa": "Thyroplasty", "opb": "Tracheostoncy", "opc": "Tracheal diversion and permanent tracheostomy", "opd": "feeding gastrostomy", "subject_name": "ENT", "topic_name": null, "id": "d98dbaa1-fc4b-4966-bfd9-f8fa4ed7b91c", "choice_type": "single"} {"question": "Trench mouth is caused by", "exp": null, "cop": 2, "opa": "β - hemalytic streptococci", "opb": "Borrelia vincenti", "opc": "Epstein Barr virus", "opd": "Diphtheria", "subject_name": "ENT", "topic_name": null, "id": "3afe29f0-2f63-4d40-9f28-2068b3c8b68c", "choice_type": "single"} {"question": "Delta sign in CT scan is characteristic of", "exp": "Contrast-enhanced CT scan can show sinus thrombosis by typical delta sign. It is a triangular area with rim enhancement and the central low-density area is seen in posterior cranial fossa on axial cuts. MR imaging better delineates thrombus. \"Delta sign\" may also be seen on contrast-enhanced MRI. MR venography is useful to assess the progression or resolution of thrombus. Ref : Diseases of ENT by Dhingra 6th edition Pgno : 84,85", "cop": 3, "opa": "Petrositis", "opb": "Otogenic brain abscess", "opc": "Lateral sinus thrombophlebitis", "opd": "Otosclerosis", "subject_name": "ENT", "topic_name": "Ear", "id": "ee1e2269-76c4-4651-9b63-48b24359b45c", "choice_type": "single"} {"question": "Cricothyrotomy is converted to tracheostomy at an early stage to", "exp": "Answer: a) Avoid subglottic stenosis (DHINGRA 6th ED, P-320)Cricothyrotomy or Laryngotomy or Mini-tracheostomy:* This is a procedure for opening the airway through the cricothyroid membrane.* Patient's head and neck is extended, lower border of thyroid cartilage and cricoid ring are identified.* Skin in this area is incised vertically and then cricothyroid membrane cut with a transverse incision.* This space can be kept open with a small tracheostomy tube or by inserting the handle of knife and turning it at right angles if tube is not available.* It is essential to perform an orderly tracheostomy as soon as possible because perichondritis, subglottic oedema and laryngeal stenosis can follow prolonged laryngotomy", "cop": 1, "opa": "Avoid subglottic stenosis", "opb": "Avoid hypoxia", "opc": "Prevent damage to epiglottis and vocal cords", "opd": "Facilitate oxygenation", "subject_name": "ENT", "topic_name": "Larynx", "id": "26b75fe0-00c3-4967-a0a9-80291c780942", "choice_type": "single"} {"question": "Bell's palsy patient comes on day 3. Treatment given would be", "exp": "Steroids prevent synkinesis and crocodile tears and shoen recovery time of facial paralysis. Acyclovir reduces incidence of herpes zoster oticus. Ref: Dhingra 6th edition pg. 95", "cop": 3, "opa": "Intratympanic steroids", "opb": "Oral steroids and Vitamin B", "opc": "Oral steroids and Acyclovir", "opd": "Vitamin B and vasodilator", "subject_name": "ENT", "topic_name": "Ear", "id": "b70b0bbb-b89f-434f-a6db-ce8c9b070fd3", "choice_type": "single"} {"question": "Abscess related to sternocleidomastoid is", "exp": "Bezold abscess It can occur following acute coalescent mastoiditis when pus breaks through the thin medial side of the tip of the mastoid and presents as a swelling in the upper pa of neck. The abscess may (i) lie deep to sternocleidomastoid, pushing the muscle outwards, (ii) follow the posterior belly of digastric and present as a swelling between the tip of mastoid and angle of jaw, (iii) be present in upper pa of posterior triangle, (iv) reach the parapharyngeal space or (v) track down along the carotid vessels Ref: https://en.m.wikipedia.org/wiki/Bezold%27s_abscess", "cop": 1, "opa": "Bezold abscess", "opb": "Citelli's abscess", "opc": "Luc abscess", "opd": "Parapharyngeal abscess", "subject_name": "ENT", "topic_name": "Ear", "id": "895347d6-0c03-42fb-a529-1882946d852c", "choice_type": "single"} {"question": "Schwartz sign is seen in", "exp": "Ans. is 'a' i.e., Otosclerosis FINDINGS IN OTOSCLEROSISSymptoms of otosclerosiso Hearing loss Bilateral conductive deafness which is painless and progressive with insidious onset. In cochlear otosclerosis sensorineural hearing loss also occur along with conductive deafness,o Paracusis willissii An otosclerotic patient hears better in noisy than quiet surroundings.o Tinnitus More common in cochlear otosclerosis.o Speech Monotonous, well modulated soft speech,o Vertigo is uncommon.Signs in otosclerosiso Tympanic membrane is quite normal and mobile.o In 10% of cases flamingo - pink blush is seen through the tympanic membrane called as Schwartze sign,o Various tests show conductive hearing loss.Tuning fork tests in otosclerosiso As otosclerotic patients have conductive deafness, the tuning fork tests results will be as followsRinnes NegativeWebers Lateralized to the ear writh greater conductive loss.Absolute bone conduction (ABC) Normal (can be decreased in cochlear otosclerosis).4) Gelles testNo change in the hearing through bone conduction when air pressure of ear canal is increased by Siegle's speculum.Audiometry in otosclerosiso Audiometry is one of the important tools in evaluation of a patient of otosclerosis. Various audiometric tests are1) Pure tone audiometry# Shows loss of air conduction. more for lower frequencies with characteristic rising pattern. Bone conduction is normal. However in some cases, there is a dip in bone conduction curve which is maximum at 2000 Hz (2 KHz) and is called the Carhart's notch.Remembero Dip in bone conduction in otosclerosis (Carhart's notch)o Dip in noise induced hearing loss (Acoustic dip)- 2000 Hz- 4000 Hz2) Impedance audiometry# Impedance audiometry showsTympanometryo Patient with early disease may show type A tympanogram (because middle ear areation is not affected) Progressive stapes fixation results in classical As type tympanogram.Acoustic (stapedial reflex)o It is one of the earliest sign of otosclerosis and preceedes the development of airbone gap. In early stage, diphasic on-off pattern is seen in which there is a brief increase in compliance at the onset and at the termination of stimulus occurs. This is pathognomonic of otosclerosis. In later stage the reflex is absent.", "cop": 1, "opa": "Otosclerosis", "opb": "Serous otitis media", "opc": "CSOM", "opd": "ASOM", "subject_name": "ENT", "topic_name": "Otosclerosis", "id": "95349be0-02ab-43d1-ae25-8346c2e4f950", "choice_type": "single"} {"question": "Potato tumor is", "exp": "Rhinophyma (Potato tumor)Rhinophyma is a benign tumor of the tip of the nose due to hyperophy of sebaceous glandIt is caused by granulomatous infiltration and occurs as a complication of long-standing acne rosaceaAlcoholism is mistakenly attributed as a cause of this disease, but heavy alcohol consumption does aggravate the conditionThe usual presentation is due to cosmetic appearance or obstructionTreatment of choice is debulking of the tumor by carbon dioxide laserRef: PL Dhingra, Diseases of Ear, Nose & Throat, 7th edition, pg no. 161 - 162", "cop": 2, "opa": "Rhinosporidiosis", "opb": "Hyperophied sebaceous gland", "opc": "Nasopharyngeal angiofibroma", "opd": "Tubercular infection", "subject_name": "ENT", "topic_name": "Nose and paranasal sinuses", "id": "7e21eb14-069c-47c1-a953-4aed515fa969", "choice_type": "single"} {"question": "Ear is sensitive to", "exp": "External auditory canal 3000 Hz Tympanic membrane 800-1600 Hz Middle ear 800 Hz Ossicular chain 500-2000 Hz Thus the greatest sensitivity of sound transmission is between 500- 3000 Hz Ref: Dhingra 7e pg 17.", "cop": 1, "opa": "500-3500 HZ", "opb": "1000-3000 HZ", "opc": "300-5000 HZ", "opd": "5000-8000 HZ", "subject_name": "ENT", "topic_name": "Ear", "id": "f1894284-9677-429f-b636-b4b6551cf90c", "choice_type": "single"} {"question": "Traumatic epistaxis is not a cause in", "exp": "Trauma is not a course for posterior epistaxis. It can occur spontaneously, often due to hypeension or aeriosclerosis. Mostly seen after the age of 40 years Ref: Dhingra 7e pg 199.", "cop": 2, "opa": "Anterior epistaxis", "opb": "Posterior epistaxis", "opc": "Childhood", "opd": "Adulthood", "subject_name": "ENT", "topic_name": "Nose and paranasal sinuses", "id": "b97508a8-39c8-4e69-9fbe-f8c82cbc0b63", "choice_type": "single"} {"question": "Mucoepidermoid carcinoma of parotid arises from", "exp": "MUCOEPIDERMOID CARCINOMA It is slow-growing tumor but can invade the facial nerve. Histologically, there are areas of mucin-producing cells and the squamous cells, and hence the name. Greater the epidermoid element, more malignant is the behaviour of the tumour. The tumours have been fuher classified as low grade and high grade. Low-grade tumours have good prognosis. Low-grade tumours are more common in children. Low-grade tumours of the parotid are treated by super ficial or total parotidectomy, depending on the location of the tumour. Facial nerve is preserved. High-grade tumours being more aggressive are treated by total parotidectomy. Facial nerve may be sacrificed if invaded by the tumour. Some surgeons also combine radical neck dissection because of high incidence of microscopic spread of the tumour. Ref:- Dhingra; pg num:-235", "cop": 4, "opa": "Myoepithelial cells", "opb": "Acinus", "opc": "Excretory cells", "opd": "Mucus secreting and epidermal cells", "subject_name": "ENT", "topic_name": "Oral cavity & Oesophagus", "id": "c2f19be8-cfd7-47ad-bf40-4130c0f603e4", "choice_type": "single"} {"question": "Caloric test is done for", "exp": "Ans. (a) Semicircular canalRef. Dhingra's ENT 5th ed. / 48* Caloric test is used to assess vestibular function. The basis of this test is to induce nystagmus by thermal stimulation of the vestibular system.* Done by taking water of 2 temperatures, cold (30degC) and hot (44degC).* Position of the test: Supine then lift head at 30deg forward or patient sitting with bending backwards 60*.* This position is used in order to make lateral/ horizontal SCC in vertical position.* Cold water elicits nystagmus in opposite eye and warm water elicit nystagmus in same eye (Mn: COWS).* Interpretation:# Normal: Time taken from start of irrigation to end point of Nystagmus is taken and it is 80 seconds --120 seconds.# Canal paresis: Duration of Nystagmus shortened i.e. < 80 sec.* Indicates depressed function of ipsilateral Labyrinth, vestibular nerve and vestibular nuclei.* It is seen in:# Meniere's disease# Acoustic neuroma# Vestibular neuritis# Vestibular nerve section# Postural vertigo", "cop": 1, "opa": "Semicircular canal", "opb": "Macula", "opc": "Saccule", "opd": "Cochlea", "subject_name": "ENT", "topic_name": "Assessment of Vestibular Function", "id": "a5370ba7-4fd4-4514-b168-5cb78fed20c9", "choice_type": "single"} {"question": "Rose's position is extension at", "exp": null, "cop": 2, "opa": "C1 and C2 joint", "opb": "Atlanto - occipital joint", "opc": "C3 and C4", "opd": "C5 and C6", "subject_name": "ENT", "topic_name": null, "id": "920a5d74-b4f1-4559-b0f6-b5f9507c3eb8", "choice_type": "single"} {"question": "Not a Tests for Eustachian Tube", "exp": null, "cop": 1, "opa": "Glycerol Test", "opb": "Valsalva Maneuver", "opc": "Frenzel test", "opd": "Tonybee's test", "subject_name": "ENT", "topic_name": null, "id": "336a49ac-c041-4d98-a7f5-a7b56d09b98b", "choice_type": "single"} {"question": "The most common presentation of adult rhinosporidiosis is", "exp": "Most common presentation of adult rhinosporidiosis is the protruding polypoidal mass through the nares and multiple site involvement. In the early stages, the patient may complain of nasal discharge which is often blood tinged and nasal stuffiness. Sometimes, frank epistaxis is the only presenting complaint.", "cop": 2, "opa": "Anosmia", "opb": "Polypoid mass", "opc": "Pain", "opd": "Halitosis", "subject_name": "ENT", "topic_name": null, "id": "256ee527-bdea-4129-926e-f0aab163f2f6", "choice_type": "single"} {"question": "Paranasal sinuses present at birth are", "exp": "Paranasal sinuses develop as outpouchings from the mucous membrane of a lateral wall of the nose. \nThe growth of sinuses continues during childhood and into early adult life.\nRadiologically maxillary sinuses can be identified at 4-5 months, ethmoids at 1 year,  sphenoid at 4 years and frontals at 6 years.", "cop": 2, "opa": "Frontal and maxillary", "opb": "Ethmoid and maxillary", "opc": "Frontal and ethmoid", "opd": "Sphenoid and ethmoid", "subject_name": "ENT", "topic_name": null, "id": "a0bcad42-e6be-4972-a441-2fd8cf203c40", "choice_type": "single"} {"question": "Cochlear basal turn dysplasia is known as", "exp": "Congenital malformations of the inner ear Complete labyrinthine aplasia (Michel), Complete membranous labyrinthine dysplasia (Siebenmann-Bing), Cochleosaccular dysplasia (Scheibe), Cochlear basal turn dysplasia (Alexander) Alexander dysplasia : It affects only the basal turn of membranous cochlea. Thus only high frequencies are affected. Residual hearing is present in low frequencies and can be exploited by amplification with hearing aids. Ref : Diseases of ENT by Dhingra 6th edition Pgno : 115", "cop": 3, "opa": "Michel dysplasia", "opb": "Scheibe dysplasia", "opc": "Alexander dysplasia", "opd": "Bing Schiebenmann dysplasia", "subject_name": "ENT", "topic_name": "Ear", "id": "b3181617-ea9f-413a-b914-68819a80e503", "choice_type": "single"} {"question": "Earliest sign of laryngeal tuberculosis is", "exp": "Laryngeal examination of tuberculosis of the larynx Hyperaemia of the vocal cord in its whole extent or confined to posterior pa with impairment of adductor is the first sign. Swelling in the Interarytenoid region giving a manipulated appearance. Ulceration of vocal cord giving the mouse-nibbled appearance. Superficial ragged ulceration on the arytenoids and Interarytenoid region. Granulation tissue in the Interarytenoid region or vocal process of arytenoid. Pseudoedema of the epiglottis \" turban epiglottis \". Swelling of ventricular bands and aryepiglottic folds. The marked pallor of the surrounding mucosa. Ref: Textbook of diseases of ENT, PL Dhingra, 7th edition, pg no. 331", "cop": 1, "opa": "Erythema of vocal cords and loss of adduction", "opb": "Mouse nibbled appearance", "opc": "Ulceration of vocal cords", "opd": "Swelling in the interarytenoid region", "subject_name": "ENT", "topic_name": "Larynx", "id": "d1cfc8fe-6a2a-453b-a19a-ec5a79bf8cd7", "choice_type": "single"} {"question": "Earliest symptom of glomus tumour is", "exp": "The earliest symptoms are hearing loss and tinnitus. The earliest symptom of a glomus tumour is pulsatile tinnitus which is of swishing character and synchronous with pulse. Ref: Dhingra 7th edition pg. 121", "cop": 1, "opa": "Pulsatile tinnitus", "opb": "Deafness", "opc": "Headache", "opd": "Veigo", "subject_name": "ENT", "topic_name": "Ear", "id": "2970ddf6-f9c9-4850-9573-9ea325eee088", "choice_type": "single"} {"question": "Treatment of choice for glue ear is", "exp": "Ans. is 'c' i.e. Myringotomy with ventilation tube insertion * Treatment of choice for glue ear is - Myringotomy and aspiration of effusion with placement of ventilation tube. Glue ear or Otitis media with effusion (OME) (also k/a serous otitis media, secretory otitis media, Mucoid otitis media) OME is characterized by a non-purulent effusion of the middle ear that may be either mucoid or serous. Symptoms usually involve hearing loss or aural fullness but typically do not involve pain or fever. In children hearing loss is generally mild and is often detected only with an audiogram. The fluid is nearly sterile. The condition is commonly seen in school-going children. It is the leading cause of hearing loss in children. Pathogenesis Two main mechanisms are thought to be responsible. Eustachian tube dysfunction Increased secretory activity of middle ear mucosa Treatment Treatment may be medical or surgical Medical measures are controversial and involve Decongestants Antiallergic measures Antibiotics Middle ear aeration Valsalva manoeuvre politzerisation or eustachian tube catheterization chewing gum Surgical Myringotomy and aspiration of fluid Grommet or ventilation tube insertion Surgical t/t of causative factor - adenoidectomy, tonsillectomy, and/or wash out of maxillary antra.", "cop": 3, "opa": "Myringotomy with cold knife", "opb": "Myringotomy with diode laser", "opc": "Myringotomy with ventilation tube insertion", "opd": "Conservative treatment with analgesics & antibiotics", "subject_name": "ENT", "topic_name": "Disorders of Middle Ear (Otitis Media)", "id": "53ffa039-3244-46bf-a343-ef1b30b56027", "choice_type": "single"} {"question": "Cochlear implant done in", "exp": null, "cop": 2, "opa": "Scala media", "opb": "Scala tympani", "opc": "Scala Vestibuli", "opd": "Middle Ear", "subject_name": "ENT", "topic_name": null, "id": "dfad73aa-0dba-4a0d-92de-d5f0921c3bd2", "choice_type": "single"} {"question": "Laryngofissure is", "exp": "Laryngofissure Opening the larynx in midline. Ref:- Stedman Dictionary; pg num:- 937", "cop": 1, "opa": "Opening the larynx in midline", "opb": "Removal of arytenoids", "opc": "Making window in thyroid cailage", "opd": "Removal of epiglottis", "subject_name": "ENT", "topic_name": "Larynx", "id": "51a8e99e-a97e-4bb5-be4d-9780a854de60", "choice_type": "single"} {"question": "Electrode of cochlear implant is placed at", "exp": "Principles of cochlear implants The electrical impulses are sent from the processor to the transmitting coil which in turn sends the signal to the surgically implanted receiver/stimulator radiofrequency. The receiver/stimulator decodes the signal and transmits it to the electrode array. Current day implants are multichannel processors with the electrode having a linear array of electrode contacts used to deliver multiple channels of current to different places along the basilar membrane. The electrode array which has been placed in the scala tympani of the cochlea stimulates the spiral ganglion cells. The auditory nerve is thus stimulated and sends these electrical pulses to the brain which are finally interpreted as sound. Ref : Dhingra 7e pg 139.", "cop": 3, "opa": "Horizontal semicircular canal", "opb": "Scala media", "opc": "Scala tympani", "opd": "Scala Vestibuli", "subject_name": "ENT", "topic_name": "Ear", "id": "e00db127-fc1d-4b4a-9f53-bda1bc57f66c", "choice_type": "single"} {"question": "Patient following peanut consumption presented with laryngeal edema, stridor, hoarseness of voice and swelling of tongue. Most likely diagnosis is", "exp": "Allergic angloedema: Most common type and usually affects those with some kind of food allergy. It can also be caused by insect bites, contact with latex, and some medications, such as penicillin or aspirin. In severe cases the throat can swell, making it hard for the patient to breath.", "cop": 1, "opa": "Angioneurotic edema", "opb": "Pharyngeal abscess", "opc": "Foreign body larynx", "opd": "Foreign body bronchus", "subject_name": "ENT", "topic_name": null, "id": "010b02bc-380b-40e1-a30c-2eb54b95b0d4", "choice_type": "single"} {"question": "Not a premalignant lesion of oral cavity", "exp": null, "cop": 3, "opa": "Erythroplakia", "opb": "Syderophonic dysphagia", "opc": "Ranula", "opd": "Submucous fibrosis", "subject_name": "ENT", "topic_name": null, "id": "729e4d04-a6e2-4dde-9a95-e1f8a270ae76", "choice_type": "single"} {"question": "Stapedial reflex is mediated by", "exp": "Stapedial reflex: Afferent : Vestibulo-cochlear N. Efferent: Facial N. Note: A person who feigns total deafness and does not give any response on pure tone audiometry but shows a positive stapedial reflex is a malingerer. Ref: Dhingra; 6th Edition; pg no 24", "cop": 4, "opa": "V and VII nerves", "opb": "V and VIII nerves", "opc": "VII and VI nerves", "opd": "VII and VIII nerves", "subject_name": "ENT", "topic_name": "Ear", "id": "c7042187-dde8-4e4f-b9a9-2940bda9159c", "choice_type": "single"} {"question": "Craniofacial dissociation is seen in", "exp": "In Le Fo 3 fracture there is a complete separation of facial bones from the cranial bones. The fracture line passes through the root of nose, ethmofrontal junction, superior orbital fissure, lateral wall of the orbit, frontozygomatic and temporozygomatic sutures and the upper pa of pterygoid plates Ref: Textbook of diseases of ENT, PL Dhingra, 7th edition, pg no. 207", "cop": 3, "opa": "Le Fo 1 fracture", "opb": "Le Fo 2 fracture", "opc": "Le Fo 3 fracture", "opd": "Tripod fracture", "subject_name": "ENT", "topic_name": "Nose and paranasal sinuses", "id": "a850ef0c-e0af-43b6-9823-48b9003898a9", "choice_type": "single"} {"question": "Most common organism causing laryngotracheobronchitis is", "exp": "Para-influenza virus I and II is the most common organism causing laryngotracheobronchitis. Ref: Textbook of diseases of ENT, PL Dhingra, 7th edition, pg no. 328", "cop": 2, "opa": "RSV", "opb": "Para-influenza virus", "opc": "Influenza virus", "opd": "H. Influenza", "subject_name": "ENT", "topic_name": "Larynx", "id": "6448577a-661f-4085-acfe-ccef03c820fc", "choice_type": "single"} {"question": "Spontaneous veical nystagmus is seen in the lesion of", "exp": "Veical downbeat nystagmus indicates lesion at craniocervical region such as Arnold-Chiari malformation or degenerative lesion of the cerebellum. Veical upbeat nystagmus is seen in lesions at the junction of pons and medulla or pons and midbrain. Ref: Dhingra 7e pg 43.", "cop": 1, "opa": "Mid brain lesion", "opb": "Labyrinth", "opc": "Vestibule", "opd": "Cochlea", "subject_name": "ENT", "topic_name": "Ear", "id": "8df502a1-0b22-47df-af26-d8b73ec095f0", "choice_type": "single"} {"question": "Gelle&;s test is done in", "exp": "Gelle's test: It is also a test of bone conduction and examines the effect of increased air pressure in ear canal on the hearing. Normally, when air pressure is increased in the ear canal by Siegel's speculum, it pushes the tympanic membrane and ossicles inwards, raises the intralabyrinthine pressure and causes immobility of basilar membrane and decreased hearing, but no change in hearing is observed when ossicular chain is fixed or disconnected. Gelle's test is performed by placing a vibrating fork on the mastoid while changes in air pressure in the ear canal are brought about by Siegel's speculum. Gelle's test is positive in nor- mal persons and in those with sensorineural hearing loss. It is negative when ossicular chain is fixed or disconnected. It was a popular test to find out stapes fixation in otosclerosis but has now been superceded by tympanometry. Ref:- Dhingra; pg num:- 22,23", "cop": 1, "opa": "Otosclerosis", "opb": "Serous otitis media", "opc": "Traumatic deafness", "opd": "Senile deafness", "subject_name": "ENT", "topic_name": "Ear", "id": "b4de3125-ce37-4e6e-84fa-faf1f81e56d8", "choice_type": "single"} {"question": "Length of the external auditory canal is", "exp": "EXTERNAL AUDITORY CANAL\n\nIt starts at the external auditory meatus and ends at the tympanic membrane, where a bony structure (bony outer attic wall at tympanic membrane, where ECA ends) projects, called scutum.\nIt is almost S-shaped curved canal of about 24 mm length.\nIt has - outer cartilaginous part (2/3rd) and - inner bony part (l/3rd).\nIt is directed upwards, backward and medially in the outer part and downwards, forwards and medially in the inner part.\nThe periosteum is very firmly adherent in EAC and hence even a minor infection like a furuncle is extremely painful.", "cop": 3, "opa": "20 mm", "opb": "22 mm", "opc": "24 mm", "opd": "15 mm", "subject_name": "ENT", "topic_name": null, "id": "608efc59-d322-47ad-abc9-70b218b8442a", "choice_type": "single"} {"question": "Fluctuating, recurring, variable sensorineural deafness is seen in", "exp": "Sensorineural hearing loss is also seen in presbycusis, ototoxicity, Meniere&;s disease, acoustic neuroma, also been associated with diabetes, hypothyroidism, kidney disease, autoimmune disorders, multiple sclerosis and blood dyscrasias. Ref: PL DHINGRA diseases of ear nose and throat, 6th edition, page no 33", "cop": 3, "opa": "Serous otitis media", "opb": "Hemotympanum", "opc": "Perilabyrinthine fistula", "opd": "Labrinthine concussion", "subject_name": "ENT", "topic_name": "Ear", "id": "cdc688a2-b802-4972-b27d-d406d1bd25f2", "choice_type": "single"} {"question": "Ohgren's line passes from", "exp": "A theoretic line passing between the medial canthus of the eye and the angle of the mandible; used as an arbitrary dividing line in classifying localized tumours of the maxillary sinus; tumours above the line invade vital structures early and have a poorer prognosis, whereas those below the line have a more ourable prognosis. It is an imaginary plane drawn, extending between medial canthus of eye and angle of the mandible. (Ref: Textbook of diseases of ENT, PL Dhingra, 7th edition, pg no. 233)", "cop": 1, "opa": "Medial canthus to angle of mandible", "opb": "Lateral canthus to angle of mandible", "opc": "Medial canthus to mastoid process", "opd": "Lateral canthus to mastoid process", "subject_name": "ENT", "topic_name": "Nose and paranasal sinuses", "id": "247fece8-f3cf-4e63-bd23-a7b7818e5614", "choice_type": "single"} {"question": "A 2 years old male is brought to the hospital there is a compressible swelling at the root of nose, likely diagnosis is", "exp": "Ans. is 'a' i.e.meningoencephalocele It is herniation of the brain along with meninges through a congenital bony defectSwelling shows cough impulse*.It may be reducible.", "cop": 1, "opa": "Meningoencephalocele", "opb": "Lacrimal sac cyst", "opc": "Ethmoid sinus cyst", "opd": "Orbital extension arterial malformation", "subject_name": "ENT", "topic_name": "Congenital Anomalies of Nose and PNS", "id": "8531eb3d-7add-457a-92ba-c2bbddab5027", "choice_type": "single"} {"question": "Nerve damaged in radical mastoidectomy is", "exp": "Facial nerve is involved in radical mastoidectomy Facial paralysis is the complication Ref: Dhingra 7e pg 460", "cop": 1, "opa": "Facial nerve", "opb": "Cochlear nerve", "opc": "Vestibular nerve", "opd": "Mandibular nerve", "subject_name": "ENT", "topic_name": "Miscellaneous ENT", "id": "bc05ff3d-f2c6-4455-8805-a0e513d414a4", "choice_type": "single"} {"question": "Most common site of sinusitis in children", "exp": "Acute ethmoid sinusitis Aetiology Acute ethmoiditis is often associated with infection of other sinuses. Ethmoid sinuses are more often involved in infants and young children. (Ref: Diseases of ENT, PL Dhingra, 7th edition, of no. 216)", "cop": 2, "opa": "Maxillary sinus", "opb": "Ethmoidal sinus", "opc": "Frontal sinus", "opd": "Spenoid sinus", "subject_name": "ENT", "topic_name": "Nose and paranasal sinuses", "id": "4dd10665-c1cf-4614-a21a-4e9406aad286", "choice_type": "single"} {"question": "Most common malignancy arising in the thyroglossal cyst is", "exp": "Thyroglossal CystContains clear to a cloudy fluid with sparse cellularityThe cystic background contains few if any epithelial cellsSome inflammatory cells and crystals might be presentRarely malignant transformation occurs in about 1% of the casesMost common malignancy arising is papillary carcinoma followed by squamous cell carcinomaRef: PL Dhingra, Diseases of Ear, Nose & Throat, 7th edition, pg no. 378 - 379", "cop": 2, "opa": "Follicular", "opb": "Papillary", "opc": "Medullary", "opd": "Anaplastic", "subject_name": "ENT", "topic_name": "Larynx", "id": "45aba9b6-007f-45fd-9730-bd4ab1a50448", "choice_type": "single"} {"question": "The unilateral sensorineural hearing loss may occur in", "exp": "Mumps(epidemic parotitis) may result in a profound sensorineural hearing loss (90 dB or more), unilaterally (one ear) or bilaterally (both ears). Coronavirus and Rotavirus are associated with diarrhoea.", "cop": 4, "opa": "Coronavirus", "opb": "Pertussis", "opc": "Rotavirus", "opd": "Mumps", "subject_name": "ENT", "topic_name": null, "id": "6e6f80dc-b0f8-413d-8bb9-464ed809720c", "choice_type": "single"} {"question": "Early tonsillectomy is not done in", "exp": "INDICATIONS They are divided into: A. ABSOLUTE 1. Recurrent infections of throat. This is the most common indication. Recurrent infections are fuher defined as: (a) Seven or more episodes in 1 year, or (b) Five episodes per year for 2 years, or (c) Three episodes per year for 3 years, or (d) Two weeks or more of lost school or work in 1 year. 2. Peritonsillar abscess. In children, tonsillectomy is done 4-6 weeks after abscess has been treated. In adults, second attack of peritonsillar abscess forms the absolute indication. 3. Tonsillitis. It causes febrile seizures. 4. Hyperophy of tonsils. It causes: (a) airway obstruction (sleep apnoea), (b) difficulty in deglutition and (c) interference with speech. 5. Suspicion of malignancy. A unilaterally enlarged tonsil may be a lymphoma in children and an epidermoid carcinoma in adults. An excisional biopsy is done. B. RELATIVE 1. Diphtheria carriers, who do not respond to antibiotics. 2. Streptococcal carriers, who may be the source of infection to others. 3. Chronic tonsillitis with bad taste or halitosis which is unresponsive to medical treatment. 4. Recurrent streptococcal tonsillitis in a patient with valvular hea disease. Ref: Dhingra 7e pg 487.", "cop": 1, "opa": "Thyroid storm", "opb": "Suspected malignancy", "opc": "Peritonsillar abscess", "opd": "Rheumatic fever", "subject_name": "ENT", "topic_name": "Diagnostic and operative ENT", "id": "1fc1eb1a-a833-4691-a7af-89428aa66ef0", "choice_type": "single"} {"question": "Sensory supply of middle ear is provided by", "exp": "Tympanic plexus provide the nerve supply It is formed by Tympanic branch of glossopharyngeal nerve Sympathetic fibres from plexus around the internal carotid aery Reference-Dhingra 6/e, p 8.", "cop": 2, "opa": "Facial", "opb": "Glossopharyngeal", "opc": "Vagus", "opd": "Trigeminal", "subject_name": "ENT", "topic_name": "Ear", "id": "f3330e45-4d2f-401d-894f-31cbba4526a8", "choice_type": "single"} {"question": "Endolymphatic decompression is seen in", "exp": "Surgical management of Meniere&;s disease-Conservative Surgeries-1.Endolymphatic sac decompression2.Cochleostomy3.Sacculotomy4.Cervical sympathectomyDestructive surgeries-1.Vestibular nerve section2.Labyrinthectomy3.Laser destruction of inner earRef: Hazarika; 3rd ed; Pg 196", "cop": 3, "opa": "Tinnitus", "opb": "Acoustic neuroma", "opc": "Meniere disease", "opd": "Endolymphatic fistula", "subject_name": "ENT", "topic_name": "Ear", "id": "83c39f93-2f70-4bf6-b4c9-4669daae7467", "choice_type": "single"} {"question": "Most commonly used tuning fork in ear examination is", "exp": "512Hz is used normally Note: These tests are performed with tuning forks of different frequencies such as 128, 256, 512, 1024, 2048 and 4096 Hz, But for routine clinical practice, tuning fork of 512 Hz is ideal. Forks of lower frequencies produce a sense of bone vibration while those of higher frequencies have a shoer decay time and are thus not routinely preferred. Ref: Dhingra; 6th Edition; pg no 21", "cop": 3, "opa": "128 Hz", "opb": "256 Hz", "opc": "512 Hz", "opd": "1024 Hz", "subject_name": "ENT", "topic_name": "Ear", "id": "418a3875-0237-4742-82eb-70337557f49a", "choice_type": "single"} {"question": "Third window effect is seen in", "exp": "Ans. b. Dehiscent superior semicircular canal Dehiscence of bone overlying the superior semicircular canal can cause a constellation of vestibular and auditory symptoms and signs. These abnormalities can be understood in terms of the effect of the dehiscence in creating a third mobile window into the inner ear.", "cop": 2, "opa": "Perforated tympanum", "opb": "Dehiscent superior semicircular canal", "opc": "Round window", "opd": "Oval window", "subject_name": "ENT", "topic_name": null, "id": "9df812ce-2d74-402c-a9dc-e8374d29b5d5", "choice_type": "single"} {"question": "Otospongiosis is inherited as", "exp": "aetiology Anatomical basis Heredity: Autosomal dominant trait with the incomplete penentrance. Race: Whites are affected more than blacks. Sex: Females are affected twice as often as males. (Ref: Textbook of diseases of ENT, PL Dhingra, 7th edition, pg no.95)", "cop": 1, "opa": "Autosomal dominant", "opb": "Autosomal recessive", "opc": "X linked dominant", "opd": "X linked recessive", "subject_name": "ENT", "topic_name": "Ear", "id": "98b211c3-1ef9-402b-8545-eff32a9bceb7", "choice_type": "single"} {"question": "Commonest complication of CSOM is", "exp": "50% of brain abscesses in adults and 25% in children are otogenic in origin. In adults, an abscess usually follows CSOM with cholesteatoma. (Ref: Textbook of diseases of ENT, PL Dhingra, 7th edition, pg no. 90)", "cop": 3, "opa": "Subperiosteal abscess", "opb": "Mastoiditis", "opc": "Brain abscess", "opd": "Meningitis", "subject_name": "ENT", "topic_name": "Ear", "id": "97362526-055a-4e9c-b8f8-d31ce57a8ad6", "choice_type": "single"} {"question": "Lyre sign in seen in", "exp": "Lyre sign is an angiographic finding seen in carotid body tumour of the neck. There is splaying apa of internal and external carotid aeries on angiogram. Ref: Diseases of Ear, Nose and Throat by P.L. Dhingra, 9th edition, p460.", "cop": 4, "opa": "Grave's disease", "opb": "Meniere's disease", "opc": "Lateral sinus thrombosis", "opd": "Carotid body tumour", "subject_name": "ENT", "topic_name": "All India exam", "id": "864bab68-61df-4859-8db3-7ebf20c9961f", "choice_type": "single"} {"question": "Laryngocele arises from herniation through", "exp": "Ans:A.)Thyrohyoid Membrane Laryngocele It is an air-filled cystic swelling due to dilatation of the saccule. Laryngoceles can be classified into three types: Internal - A laryngocele is defined as internal if the dilatation of Sacculeof the ventricle lies within the limits of the thyroid cailage. External - If the laryngocele extends beyond the thyroid cailage and protrudes through the thyrohyoid membrane producing a lateral neck mass, it is considered external. Combined - In which both internal and external components are seen.", "cop": 1, "opa": "Thyrohyoid membrane", "opb": "Cricoepiglottic membrane", "opc": "Thyroid membrane", "opd": "Cricovocal membrane", "subject_name": "ENT", "topic_name": null, "id": "4b9fc57e-2f34-46f9-acd5-433eb3ec86c0", "choice_type": "single"} {"question": "The location of schatzki&;s ring is", "exp": "SCHATZKI'S RING It occurs at the junction of squamous and columnar epithelium at the lower end of oesophagus and has also been called lower oesophageal ring. Usually seen in patients above 50 years of age. Cause is unknown. Symptomatic patients complain of intermittent dysphagia and some may even present with bolus obstruction. It may be associated with hiatus hernia. Treatment is oesophageal dilatation. Ref:- Dhingra; pg num:-345", "cop": 3, "opa": "Lower end of pharynx", "opb": "Upper end of esophagus", "opc": "Lower end of esophagus", "opd": "Antrum of stomach", "subject_name": "ENT", "topic_name": "Oral cavity & Oesophagus", "id": "d343d41f-8785-43ed-87c6-8bf18c474486", "choice_type": "single"} {"question": "Type 1 thyroplasty is for", "exp": "Thyroplasty types Type I: It is medial displacement of vocal cord as in achieved in Teflon paste injection. Type II: It is lateral displacement of the vocal cord and is used to improve the airway. Type III: It is used to shoen the vocal cord. Relaxation of vocal cord lowers the pitch. This procedure is done in mutational falsetto or in those who have undergone gender transformation from female to male. Type IV: This procedure is used to lengthen the vocal cord and elevate the pitch. It coves the male character to a female and has been used in gender transformation. Ref: Textbook of diseases of ENT, PL Dhingra, 7th edition, pg no. 341", "cop": 3, "opa": "Vocal cord shoening", "opb": "Vocal cord lengthening", "opc": "Vocal cord medialization", "opd": "Vocal cord lateralization", "subject_name": "ENT", "topic_name": "Larynx", "id": "3ee95dcb-082f-42df-a56c-b7557b4519e6", "choice_type": "single"} {"question": "Not an example of velopharyngeal Insufficiency", "exp": null, "cop": 1, "opa": "Cleft lip", "opb": "Cleft palate", "opc": "Bifid uvula", "opd": "Palatal paralysis", "subject_name": "ENT", "topic_name": null, "id": "7a0c744e-a279-47ab-8592-3c79297b774d", "choice_type": "single"} {"question": "Lymphatic drainage of the oropharynx is mainly through", "exp": "Lymphatics from oropharynx drain into the upper jugular chain, particularly the jugulodigastric (tonsillar) node. Some parts drain to the posterior cervical group also. The base of the tongue may drain bilaterally.\nLymphatic drainage of nasopharynx - upper deep cervical nodes.", "cop": 3, "opa": "Superficial cervical lymph nodes", "opb": "Submandibular nodes", "opc": "Jugulodigastric nodes", "opd": "Jugulo-omohyoid nodes", "subject_name": "ENT", "topic_name": null, "id": "d8785385-854c-4a60-9720-1a85c2b83da8", "choice_type": "single"} {"question": "Parapharygeal space is also known as", "exp": "Parapharyngeal space is situated on the side of the pharynx. It is pyramidal in shape with the base at the base of the skull and apex at the hyoid bone. It contains carotid vessels, jugular vein, last 4 cranial nerves and cervical sympathetic chain. Ref: Diseases of ENT, PL Dhingra, 7th edition, pg no. 301", "cop": 3, "opa": "Retropharyngeal space", "opb": "Pyriform sinus", "opc": "Lateral pharyngeal space", "opd": "Pterygomaxillary space", "subject_name": "ENT", "topic_name": "Pharynx", "id": "4e5b932f-b4af-4520-a837-af06b82158dc", "choice_type": "single"} {"question": "Malignant otitis externa is caused by", "exp": "Malignant otitis externa is often caused by difficult to treat bacteria such as Pseudomonas aeruginosa. Only are cases of malignant otitis externa due to S.aureus, Proteus mirabilis and Aspergillus fumigates have been repoed.Ref: PL Dhingra, Diseases of Ear, Nose & Throat, 7th edition, pg no. 55", "cop": 2, "opa": "Hemophilus influenzae", "opb": "Pseudomonas aeroginosa", "opc": "Streptococcus pyogenes", "opd": "E. coli", "subject_name": "ENT", "topic_name": "Ear", "id": "066a6372-0254-4dea-8893-b54e78028d98", "choice_type": "single"} {"question": "Graft for myringoplasty", "exp": "Ans. is 'a' i.e., Temporalis fascia Graft material used for myringoplasty are : Temporalis fascia (most common) Tragal cailage Perichondrium from the tragus Vein", "cop": 1, "opa": "Temporalis fascia", "opb": "Iliacus fascia", "opc": "Coles fascia", "opd": "I hotibial band", "subject_name": "ENT", "topic_name": null, "id": "86f4c16d-2e9a-484d-a35e-ed8c615630b4", "choice_type": "single"} {"question": "Supranuclear lesion of facial nerve affects", "exp": "It is caused by cerebrovascular accidents (haemorrhage, thrombosis or embolism), tumour or an abscess. It causes paralysis of only the lower half of face on the contralateral side. Forehead movements are retained due to bilateral innervation of frontalis muscle. Involuntary emotional movements and the tone of facial muscles are also retained. Ref: Dhingra 7e pg 107.", "cop": 1, "opa": "Lower pa of face", "opb": "Upper pa of face", "opc": "Face", "opd": "Spares Face", "subject_name": "ENT", "topic_name": "Ear", "id": "8dcbd9d1-33ab-4e5c-9167-be70a888255c", "choice_type": "single"} {"question": "FESS means", "exp": "Ans. d (Functional endoscopic sinus surgery), (Ref. Scott Brown, Otolaryngology, 6th ed., 12, 29)Functional Endoscopic Sinus Surgery (FESS) is based on the philosophical concept that function can be restored to the paranasal sinuses and normal structures can be preserved through removal of minimal amounts of tissue. This concept embodies a new understanding of the pathophysiology and development of sinus disease and requires a more delicate, minirealist approach to sinus surgery than the extirpative procedures of the past. These techniques were developed by Walter Messerklinger of Graz, Austria. Uncinectomy is 1st step of FESS.", "cop": 4, "opa": "Factual endoscopic sinus surgery", "opb": "Functional endonasal sinus surgery", "opc": "Factual endonasal sinus surgery", "opd": "Functional endoscopic sinus surgery", "subject_name": "ENT", "topic_name": "Operative Surgery", "id": "b59aa3d0-1a64-4067-9632-1f32e4674d7d", "choice_type": "single"} {"question": "The nerve supply of middle ear", "exp": "Tympanic branch supplies innervation to the medial surface of the tympanic membrane,tympanic cavity,mastoid air cells and the bony eustachian tube. Reference- Dhingra 6/e,p 8.", "cop": 1, "opa": "Tympanic plexus", "opb": "Facial nerve", "opc": "Glossopharyngeal nerve", "opd": "Trigeminal nerve", "subject_name": "ENT", "topic_name": "Ear", "id": "d522a1aa-fa62-45e3-9368-fcd9e81238dd", "choice_type": "single"} {"question": "Fistula tests stimulates", "exp": null, "cop": 1, "opa": "Lateral semicircular canal", "opb": "Posterior semicircular canal", "opc": "Round window", "opd": "Anterior semicircular canal", "subject_name": "ENT", "topic_name": null, "id": "1de4c5e3-b923-4ce3-9a62-8024625a0f2e", "choice_type": "single"} {"question": "The size of the Fenestra vestibule on an average is", "exp": "The size of the Fenestra vestibule on an average is 3.25 mm long and 1.75 mm wide. Ref- Dhingra ENT Textbook 6th edition.", "cop": 2, "opa": "3.50 mm long and 2.50 mm wide", "opb": "3.25 mm long and 1.75 mm wide", "opc": "3.75 mm long and 1.50 mm wide", "opd": "4.00 mm long and 2.00 mm wide", "subject_name": "ENT", "topic_name": "Ear", "id": "fca5f376-2f62-4b1a-ac27-63b0075a5d94", "choice_type": "single"} {"question": "Treatment of middle ear papilloma is", "exp": "In carcinomas of middle ear radical mastoidectomy followed by radiotherapy is an alternative ton an en bloc removal of temporal bone Ref: PL DHINGRA diseases of ear nose and throat, 6th edition, page no 131", "cop": 3, "opa": "Myringotomy and simple excision", "opb": "Myringectomy and simple excision", "opc": "Tympanomastoidectomy", "opd": "Local infiltration with podophyllin", "subject_name": "ENT", "topic_name": "Ear", "id": "1c1609ba-d8a0-4dbc-8b93-5c1f44ce5d3e", "choice_type": "single"} {"question": "Prussak space is in", "exp": null, "cop": 1, "opa": "Epitympanum", "opb": "Mesotympanum", "opc": "Hypotympanum", "opd": "Inner Ear", "subject_name": "ENT", "topic_name": null, "id": "77c25799-730d-496b-88a9-7e68e88acaba", "choice_type": "single"} {"question": "Synkinesis is a sequel of", "exp": "Synkinesis or mass movement is a complication of peripheral facial nerve paralysis which occurs due to cross-innervation of fibres. Ref: Dhingra 6th edition pg. 98", "cop": 1, "opa": "Facial nerve paralysis", "opb": "Trigeminal nerve paralysis", "opc": "Superficial temporal nerve paralysis", "opd": "Greater petrosal nerve paralysis", "subject_name": "ENT", "topic_name": "Ear", "id": "7ca62da0-88b6-4cb3-83ed-18dd9a1003fc", "choice_type": "single"} {"question": "Length of external auditory canal is", "exp": "Ans. c (24 mm). (Ref. PL Dhingra Diseases of ENT 2nd/pg.3)EXTERNAL AUDITORY CANAL# It starts at the external auditory meatus and ends at the tympanic membrane, where a bony structure (bony outer attic wall at tympanic membrane, where ECA ends) projects, called scutum.# It is almost S shaped curved canal of about 24 mm length.# It has:- outer cartilaginous part (2/3rd) and- inner bony part (l/3rd).# It is directed upwards, backwards and medially in outer part and downwards, forwards and medially in inner part.# The periosteum is very firmly adherent in EAC and hence even a minor infection like a furuncle is extremely painful.", "cop": 3, "opa": "20 mm", "opb": "22 mm", "opc": "24 mm", "opd": "15 mm", "subject_name": "ENT", "topic_name": "Anatomy of External Ear & Pinna", "id": "6f5ef9c5-9d53-4717-b1f2-17ade0c43c8d", "choice_type": "single"} {"question": "Singular nerve is a", "exp": "A small subdivision of the vestibular nerve that carries information from the posterior semicircular canal to the brain. Singular neurectomy is a surgical procedure to divide the singular nerve, occasionally used in the treatment of benign paroxysmal positional veigo. Ref-Dhingraa textbook for ENT 6th edition.", "cop": 2, "opa": "Superior vestibular nerve supplying posterior semicircular canal", "opb": "Inferior vestibular nerve supplying posterior semicircular canal", "opc": "Superior vestibular nerve supplying anterior semicircular canal", "opd": "Interior vestibular nerve supplying anterior semicircular canal", "subject_name": "ENT", "topic_name": "Ear", "id": "96a7a7f2-7b09-4156-b653-80ce9434b15d", "choice_type": "single"} {"question": "Paracusis is seen in", "exp": "(Otosclerosis) (87-Dhingra 4th edition) (89- Maqbool 11th)Paracusis- Willisii - An otosclerotic patients hears better in noisy than quite surrounding. This is because a normal person will raise his voice in noisy surroundings, like in public transport, machine shops, engine rooms\"Schwartze sign \" - tympanic membrane shows flemingo-pink tinge in otosclerosis\"Carhart's notch \" -This is a pure tone audiometric findings is characteristic of otosclerosis (maximum at 2000 Hz)Clinical symptoms of Otosclerosis* Hearing loss conductive deafness* Paracusis - willisii* Tinnitus - More common in cochlear otosclerosis* Speech- monotonous* Vertigo- is uncommon* Presbycusis - sensorineural hearing loss associated with physiological aging process in the ear", "cop": 2, "opa": "CSOM", "opb": "Otosclerosis", "opc": "Meniere's disease", "opd": "ASOM", "subject_name": "ENT", "topic_name": "Ear", "id": "d2e9ccde-b397-4841-b5b5-7aca441d592c", "choice_type": "single"} {"question": "The ideal time for operation of quinsy after an attack of acute tonsillitis is", "exp": "Treatment The above conservative measures may cure peritonsillitis. If a frank abscess has formed, incision and drainage will be required. INCISION AND DRAINAGE: A peritonsillar abscess is opened at the point of a maximum bulge above the upper pole of the tonsil or just lateral to the point of junction of the anterior pillar with a line drawn through the base of uvula. INTERVAL TONSILLECTOMY: Tonsils are removed 6 weeks following an acute attack of quinsy. ABSCESS OR HOT TONSILLECTOMY: Some people prefer to do hot tonsillectomy instead of incision and drainage. (Ref: Diseases of Ear, Nose and Throat and head and neck surgery, 7th edition Pg no. 298)", "cop": 3, "opa": "2 weeks", "opb": "4 weeks", "opc": "6 weeks", "opd": "12 weeks", "subject_name": "ENT", "topic_name": "Pharynx", "id": "5e695609-529a-4660-9683-9bf9ac65b18d", "choice_type": "single"} {"question": "Most common cause of oroantral fistula", "exp": "A most impoant cause of oroantral fistula is dental extraction. It is a communication between the antrum and the oral cavity. Roots of the second premolar and upper molars are closely related to the antral cavity and their extraction may lead to fistula formation. It presents with regurgitation of food foul smelling discharge filling nose and inability to build positive or negative pressure in the mouth. Ref: Textbook of diseases of ENT; PL Dhingra; 7th edition, pg 208", "cop": 3, "opa": "TB", "opb": "Penetrating injury", "opc": "Tooth extraction", "opd": "Iatrogenic", "subject_name": "ENT", "topic_name": "Nose and paranasal sinuses", "id": "4f1024d9-0342-4cd9-b5ea-4f6d0d2d91ed", "choice_type": "single"} {"question": "Bing test is", "exp": "Bing test: It is a test of bone conduction Examines the effect of occlusion of ear canal on the hearing. A vibrating tuning fork is placed on the mastoid while the examiner alternately closes and opens the ear canal by pressing on the tragus inwards. A normal person or one with sensorineural hearing loss hear louder when the ear canal is occluded and softer when the canal is open (Bing positive). A patient with conductive hearing loss will appreciate no change (Bing negative). Ref: Dhingra; 6th Edition; pg no 22", "cop": 3, "opa": "Audiometric test", "opb": "Air conduction test", "opc": "Bone conduction test", "opd": "Special test", "subject_name": "ENT", "topic_name": "Ear", "id": "29bec1f7-7dd3-4935-a88b-53038b891680", "choice_type": "single"} {"question": "Paracusis Willisii is a feature of", "exp": "An otosclerotic patient hears better in noise than in quiet surroundings. This is called Paracusis Willisii. This is because a normal person will raise his voice in noisy surroundings Ref: Diseases of EAR, NOSE and THROAT by PL Dhingra; 7th Edition; page no 96", "cop": 2, "opa": "Tympanosclerosis", "opb": "Otosclerosis", "opc": "Meniere's disease", "opd": "Presbycusis", "subject_name": "ENT", "topic_name": "Ear", "id": "36c0eb4e-2438-4230-841a-21c6dc14b822", "choice_type": "single"} {"question": "In Bilateral palsy of recurrent larygeal nerve there is", "exp": "In BLL RLN palsy - VC is in median position, so speech preserved has stridor and dysphoea.", "cop": 3, "opa": "Complete loss of speech with steidor and dysphonia", "opb": "Comlete loss of speech but no difficulty in breathing", "opc": "Preserved speech with severe stridor and dyspnocr", "opd": "Pressure speech with no difficulty of breating.", "subject_name": "ENT", "topic_name": null, "id": "848d52f2-733f-4ed0-9c5b-8038b01ba69a", "choice_type": "single"} {"question": "Most definitive diagnosis of sinusitis is by", "exp": "Sinuscopy is a method of Endoscopic sinus observation or surgery using Nose telescope. It involves the evaluation of nasal and sinus passages using the endoscope called Sinuscope. The Sinuscope has a narrow tube with a built-in camera so that the physician can see the internal details of the sinuses. By observing the sinus, it is possible to diagnose the problems and treat properly.", "cop": 3, "opa": "X- ray PNS", "opb": "Proof puncture", "opc": "Sinuscopy", "opd": "Transillumination test", "subject_name": "ENT", "topic_name": "Nose and paranasal sinuses", "id": "33d6780f-f9b8-4528-bbd9-2b6c065ada10", "choice_type": "single"} {"question": "Elastic cailage is present in", "exp": "Elastic cailage is present in - Epiglottis. It is a yellow, leaf-like elastic cailage forming the anterior wall of the laryngeal inlet. It is attached to the body of the hyoid bone by the hyoepiglottic ligament, which divides into suprahyoid and infrahyoid epiglottis. Other elastic cailage structures are corniculate, cuneiform and tip of arytenoid near corniculate cailage. Thyroid, cricoid and most of the arytenoid cailage are by hyaline cailage. Ref: Textbook of diseases of ENT, PL Dhingra, 7th edition, pg no. 319", "cop": 2, "opa": "Thyroid cailage", "opb": "Epiglottis", "opc": "Cricoid", "opd": "Arytenoid cailage", "subject_name": "ENT", "topic_name": "Larynx", "id": "4917e382-2cf7-46b6-b0ea-5d4158f11417", "choice_type": "single"} {"question": "Most common site for the initiation of otosclerosis is", "exp": "In stapedial otosclerosis which is the most common variety, the lesion stas just in front of the oval window in an area called \"fissula ante fenestram\". Ref: Diseases of EAR, NOSE, THROAT by PL Dhingra; 7th Edition; page no.95", "cop": 3, "opa": "Food plate of stapes", "opb": "Margins of stapes", "opc": "Fissula ante fenestrum", "opd": "Fissula post fenestrum", "subject_name": "ENT", "topic_name": "Ear", "id": "138c15d2-fbb3-4b15-a15d-893e412dcf4f", "choice_type": "single"} {"question": "Investigation of choice for nasopharyngeal angiofibroma", "exp": "Computed tomography (CT) scan of the head with contrast enhancement is now the investigation of choice. It has replaced conventional radiographs. It shows the extent of a tumour, bony destruction or displacements. Anterior bowing of the posterior wall of maxillary sinus often called antral sign or Holman-Miller sign is pathognomic of angiofibroma (Ref: Diseases of Ear, Nose and Throat, P.L Dhingra, 7th edition, page 280)", "cop": 4, "opa": "X-ray", "opb": "MRI", "opc": "Plane-CT", "opd": "CT-contrast", "subject_name": "ENT", "topic_name": "Pharynx", "id": "576613a8-7dd6-475b-902d-688b2594b344", "choice_type": "single"} {"question": "Lombard test is used in diagnosis of", "exp": "Lombard's test depends on the fact that to the normal man the sound of his own voice is necessary to the proper regulation of its tone and intensity. The Barany noise apparatus is adjusted in the patient's sound ear,and its machinery staed in order to accustom him to its grating noise. He is given a book, and told to read aloud in his natural voice,and not to stop reading when the instrument is set in action.As soon as the noise begins,a man whose opposite ear is profoundly deaf will at once raise his voice . The malingerers on the other hand,claiming a one-sided deafness which is not real,will continue to read in an even tone or in a tone only slightly elevated. In this way,Lombard's test is used in the diagnosis of Non -Organic hearing loss. ref:LOGAN TURNER'S textbook,8th edition,page no.400", "cop": 3, "opa": "Conductive hearing loss", "opb": "Sensorineural hearing loss", "opc": "Non-organic hearing loss", "opd": "Mixed hearing loss", "subject_name": "ENT", "topic_name": "Ear", "id": "c997e5dc-266c-4ca1-8569-e2051c601aaf", "choice_type": "single"} {"question": "The glomus tumor invasion of jugular bulb is diagnosed by", "exp": "MRI - gives soft tissue extent of tumor; Magnetic Resonance Angiography (MRA) shows compression of the carotid aery whereas magnetic resonance venography shows invasion of jugular bulb by the tumor.", "cop": 4, "opa": "Carotid angiography", "opb": "Veebral venousvenography", "opc": "X-ray", "opd": "Jugular venography", "subject_name": "ENT", "topic_name": null, "id": "9c20eba0-1614-4dd2-8654-a1545b4338ce", "choice_type": "single"} {"question": "Kemicetine solution does not include", "exp": "Kemicetine solution has chloramphenicol,Estradiol,Propylene Glycol,Vitamine D2.", "cop": 1, "opa": "Chloroform", "opb": "Estradiol", "opc": "Propylene Glycol", "opd": "Vitamin D2", "subject_name": "ENT", "topic_name": null, "id": "b9ff2c4b-75d7-4fb4-a5e1-065fc2176d95", "choice_type": "single"} {"question": "Spontaneous vertical nystagmus is seen in the lesion of", "exp": "“Vertical nystagmus means vertical displacement of the eye, not side to side nystagmus when attempting upward or down ward gaze. As defined vertical nystagmus always indicates brainstem dysfunction”", "cop": 1, "opa": "Midbrain", "opb": "Labyrinth", "opc": "Vestibule", "opd": "Cochlea", "subject_name": "ENT", "topic_name": null, "id": "738c6881-0818-4f2d-89f7-6b0cc9cbafac", "choice_type": "single"} {"question": "Type I thyroplasty is for", "exp": "Type I Thyroplasty It is the most commonly used surgical procedure to correct unilateral vocal cord paralysis (a condition where the vocal cord of one side is paralysed). Procedure In this type of thyroplasty, a rectangular poion of the thyroid cailage is mobilized and pushed towards the medial side using a piece of silastic block of proper shape under local anesthesia. Earlier, the piece of the thyroid cailage was kept along with implant and the stitches were taken, but nowadays, the piece of the thyroid cailage is cut and removed to avoid complications. Ref : https://en.m.wikipedia.org/wiki/Thyroplasty", "cop": 1, "opa": "Vocal cord Medialization", "opb": "Vocal cord Lateralization", "opc": "Vocal cord shoening", "opd": "Vocal cord lengthening", "subject_name": "ENT", "topic_name": "All India exam", "id": "262c8cb5-0872-4186-a3ff-40341f8e367e", "choice_type": "single"} {"question": "Pendred's syndrome is due to a defect in", "exp": "Pendred disease is an autosomal recessive disorder leading to congenital bilateral sensorineural hearing loss and goitre with euthyroid or mild hypothyroidism. It has been linked to mutations in the PDS gene, which codes for the pendrin protein. The gene is located on the long arm of chromosome 7 (7q) Ref : Dhingra 7e pg 130.", "cop": 2, "opa": "Chromosome 7p", "opb": "Chromosome 7q", "opc": "Chromosome 8p", "opd": "Chromosome 8q", "subject_name": "ENT", "topic_name": "Ear", "id": "d5d671a8-5c04-433f-9fed-039ccf62efcf", "choice_type": "single"} {"question": "IOC for angiofibroma is", "exp": "NASOPHARYNGEAL FIBROMA (JUVENILE NASOPHARYNGEAL ANGIOFIBROMA) INVESTIGATIONS :- 1. Computed tomography (CT) scan of the head with contrast enhancement is now the investigation of choice. It has replaced conventional radiographs. It shows the extent of tumour, bony destruction or displacements. Anterior bowing of the posterior wall of maxillary sinus, often called antral sign or Holman-Miller sign, is pathognomic of angiofibroma. 2. Magnetic resonance imaging (MRI) is complementary to CT scans and shows any soft tissue extensions present intracranially in the infratemporal fossa or in the orbit. 3. Carotid angiography shows the extent of tumours, its vascularity and feeding vessels which mostly come from the external carotid system. In very large tumours or those with intracranial extension vessels may also come from internal carotid system. Embolization of vessels can be done at this time to decrease bleeding at operation. Feeders from only the external carotid system can be embolized. Resection of tumour should not be delayed beyond 24-48 h of embolization to avoid vascularization from the contralateral side. Ref:- Dhingra; pg num:-247,248", "cop": 2, "opa": "Plain X-ray", "opb": "CT scan", "opc": "MRI", "opd": "Angiography", "subject_name": "ENT", "topic_name": "Pharynx", "id": "689505ab-8f34-4ffe-a14a-9afd36ea7a65", "choice_type": "single"} {"question": "Ceruminous glands are modified", "exp": "Cailaginous pa of an external auditory meatusThe skin covering it is thick and has ceruminous glands (modified apocrine sweat glands), pilosebaceous glands and hair. Since hair is confined to cailaginous pa, furuncle is seen only is the outer third of the canal.Ref: PL Dhingra, Diseases of Ear, Nose & Throat, 7th edition, pg no. 3", "cop": 2, "opa": "Eccrine glands", "opb": "Apocrine glands", "opc": "Sebaceous glands", "opd": "Holocrine glands", "subject_name": "ENT", "topic_name": "Nose and paranasal sinuses", "id": "279389e8-281d-416c-8d60-8e90fbd7ffda", "choice_type": "single"} {"question": "Auditory fatigue is seen at", "exp": "Auditory fatigue occurs at 90dB, 4000Hz. 90dB for 8hour a day for 5weeks is the maximum safe limit. (Ref: Diseases of EAR, NOSE AND THROAT by PL DHINGRA - 6th Edition)", "cop": 1, "opa": "90 dB, 4000Hz", "opb": "110 dB, 4000Hz", "opc": "130 dB, 4000Hz", "opd": "150 dB, 4000Hz", "subject_name": "ENT", "topic_name": "Ear", "id": "29a34b1e-a2cb-45cf-b05c-ccd5aa4f1c3e", "choice_type": "single"} {"question": "charecters of tb otitis media is", "exp": "Tuberculous Otitis Media * Painless , odourless otorrhoea refractory to antibiotics * Multiple TM perforations and a large perforation * Middle ear mucosa pale (congest., around E.T.0.) * Pale granulations in mastoid & middle ear * Severe deafness with bony necrosis (caries) * Facial palsy & labyrinthitis * Tb: Anti-TB therapy and mastoidectomy red : hazarika 4th ed", "cop": 1, "opa": "painful ottorhea", "opb": "multiple tympaniic perforation", "opc": "pale granulation", "opd": "facial palsy", "subject_name": "ENT", "topic_name": "All India exam", "id": "8431cca2-8312-4519-8870-dba357c4b21e", "choice_type": "single"} {"question": "The sleep apnea syndrome is defined as", "exp": "Sleep apnea is a potentially serious sleep disorder in which breathing repeatedly stops and stas. If you snore loudly and feel tired even after a full night's sleep, you might have sleep apnea. The main types of sleep apnea are: Obstructive sleep apnea, the more common form that occurs when throat muscles relax Central sleep apnea, which occurs when your brain doesn't send proper signals to the muscles that control breathing Complex sleep apnea syndrome, also known as treatment-emergent central sleep apnea, which occurs when someone has both obstructive sleep apnea and central sleep apnea", "cop": 3, "opa": "30 or more episodes of apnea during 12hours", "opb": "28 or more episodes of apnea during 7hours", "opc": "30 or more episodes of apnea during 7hours", "opd": "30 or more episodes of apnea during 10hours", "subject_name": "ENT", "topic_name": "Miscellaneous ENT", "id": "acdcc3c5-21e8-4bf6-8dd0-f8b2ec74b469", "choice_type": "single"} {"question": "High frequency audiometry is used in", "exp": "In ototoxicity, there is high-frequency hearing loss. Note: Pure tone audiometry has only frequency of 125 to 8000 Hz. OTOTOXIC DRUGS: A. Aminoglycoside antibiotics * Streptomycin * Dihydrostreptomycin * Gentamicin * Tobramycin * Neomycin * Kanamycin * Amikacin * Netilmycin * Sisomycin B. Diuretics * Furosemide * Ethacrynic acid * Bumetanide C. Antimalarials * Quinine * Chloroquine * Hydroxychloroquine D. Cytotoxic drugs * Nitrogen mustard (Mechlorethamine) * Cisplatin * Carboplatin E. Analgesics * Salicylates * Indomethacin * Phenylbutazone * Ibuprofen F. Chemicals * Alcohol * Tobacco * Marijuana * Carbon monoxide poisoning G. Miscellaneous * Erythromycin * Ampicillin * Propranolol * Propylthiouracil * Deferoxamine Ref: Dhingra; 6th Edition; pg no 34", "cop": 2, "opa": "Otosclerosis", "opb": "Ototoxicity", "opc": "Non-organic hearing loss", "opd": "Meniere's disease", "subject_name": "ENT", "topic_name": "Ear", "id": "40c8a924-8132-4674-be69-92fe83d3e164", "choice_type": "single"} {"question": "The most frequent complication after resection of Carotid Body tumour is", "exp": "The vagal nerve or hypoglossal nerve damage associated with removal of larger tumours. Ref : ENT textbook by Dhingra 6th edition Pgno : 392,393", "cop": 3, "opa": "First-bite syndrome", "opb": "Hypoglossal nerve injury", "opc": "Superior laryngeal nerve injury", "opd": "Baroreceptor failure with fluctuations in blood pressure", "subject_name": "ENT", "topic_name": "All India exam", "id": "bea29cc5-95e3-48c8-a93f-fc0c6df13946", "choice_type": "single"} {"question": "Impedance audiometry is done using frequency probe of", "exp": "It consists of a probe which fits into the external acoustic canal and has 3 channels: To deliver a tone of 220Hz To pick up the reflected sound a microphone To bring about changes in air pressure in the canal from positive to normal and then negative Ref: Dhingra; 6th Edition; pg no 24", "cop": 1, "opa": "220 Hz", "opb": "555 Hz", "opc": "440 Hz", "opd": "1000 Hz", "subject_name": "ENT", "topic_name": "Ear", "id": "db705f42-90eb-4809-b1b3-495a0744780f", "choice_type": "single"} {"question": "Preferred imaging modality for choanal atresia is", "exp": "CT SCAN in choanal atresia CT SCAN in coronal and saggital projections provides a thorough evaluation of choanal atresia and adjescent structures. The axial views supply fundamental information including site of obstruction, composition of atretic plate and unilateral or bilateral involvement. Thus CT SCAN is the preferred imaging modality.", "cop": 2, "opa": "X ray", "opb": "CT SCAN", "opc": "MRI", "opd": "PET SCAN", "subject_name": "ENT", "topic_name": null, "id": "c3c2d535-15e0-40c0-a9f3-1a4d2e3fe2bf", "choice_type": "single"} {"question": "Common age for otosclerosis is", "exp": "Hearing loss usually stas between 20 to 30 years of age and is rare before 10 and after 40 years. (Ref: Textbook of diseases of ENT, PL Dhingra, 7th edition, pg no. 95)", "cop": 3, "opa": "5-10 years", "opb": "10-20 years", "opc": "20-30 years", "opd": "30-45 years", "subject_name": "ENT", "topic_name": "Ear", "id": "e97e400c-4c2f-4e97-b0af-e3c36d1c21be", "choice_type": "single"} {"question": "Nasopharyngeal Ca is caused by", "exp": "NASOPHARYNGEAL CANCER AETIOLOGY :- The exact aetiology is not known. The factors responsible are: 1. Genetic:- Chinese have a higher genetic susceptibility to nasopharyngeal cancer. Even after migration to other countries they continue to have higher incidence. 2. Viral:- Epstein-Barr (EB) virus is closely associated with nasopharyngeal cancer. Specific viral markers are being developed to screen people in high-incidence areas. EB virus has two impoant antigens: viral capsid antigen (VCA) and early antigen (EA). IgA antibodies of EA are highly specific for nasopharyngeal cancer but have sensitivity of only 70-80% while IgA antibodies of VCA are more sensitive but less specific. AgA antibodies against Both EA and VCA should be done for screening of patients for nasopharyngeal cancer. 3. Environmental:- Airpollution, smoking of tobacco and opium, nitrosamines from dry salted fish, smoke from burning of incense and wood have all been incriminated. Ref:- Dhingra; pg num:-250", "cop": 1, "opa": "EBV", "opb": "Adeno virus", "opc": "Parvo virus", "opd": "Papilloma virus", "subject_name": "ENT", "topic_name": "Pharynx", "id": "57db32ad-cb58-41f8-8e3d-17c0091dcc15", "choice_type": "single"} {"question": "CSF rhinorrhea \"immediate\" management is", "exp": "CSF rhinorrhea may be classified as: Traumatic (>90%) - Approximately 80% of all traumatic leaks occur in the setting of accidental trauma, and the remaining traumatic leaks occur after neurosurgical and rhinological procedures Nontraumatic (Nontraumatic etiologies include neoplasms and hydrocephalus High pressure flow- intracranial tumours & hydrocephalous Low pressure flow- congenital defects Most common site for leak is through cribform plate and ethmoidal air sinuses. Less common sites are through frontal and sphenoidal sinuses. Rarely, the leak can originate in the middle or posterior cranial fossa and can reach the nasal cavity by way of the middle ear and eustachian tube Diagnosis: Basic clinical tests - Rhinoscopy-visualisation of CSF leakage from paranasal sinuses - Tissue test-unlike nasal mucous ,CSFdoes not cause a tissue to stiffen - Filter paper test-sample of nasal discharge on a filter paper exhibits a light CSF border and a dark central area of blood 'double ring sign' or ' halo sign' (in cases of traumatic CSF leak where blood and CSF are mixed.) - Queckenstedt test-compression of jugular veins leads to increased CSF leakage d/t increase in 1CP Biochemical tests: - Concentrations of glucose & protein are higher in CSF than in nasal discharge. - 12-transferrin is the preferred biochemical marker of CSF. It helps in distinguishing CSF from other nasal secretions. - Beta-trace protein (11TP) is another chemical marker that could be used for the detection of CSF CSF tracers: Intrathecal fluorescein dye administration, radionuclide cisternography, CTcisternography Radiological studies: High-resolution CT provides detailed information about the bony skull base anatomy, and MR1 assesses soft tissues , including unrecognized tumors and coincidental meningoencephaloceles Treatment: Traumatic rhinorrhea often stops spontaneously Conservative treatment consists of 1-2 weeks trial of? - Strict bed rest - Head elevation - Stool softeners - Advising patient to avoid coughing, sneezing, nose blowing, and straining - Prophylactic antibiotics - Subarachnoid drainage through a lumbar catheter Surgical repair is generally advocated in patients with large fistulas especially in the presence of pneurnocephalous.", "cop": 2, "opa": "Plugging with petrolleum jelly plugs", "opb": "Wait & watch for 7 days + antibiotics", "opc": "Blow the nose repeatedly", "opd": "Surgery", "subject_name": "ENT", "topic_name": null, "id": "3f3ce522-1ba4-41d8-9d13-489e97b9f9c7", "choice_type": "single"} {"question": "Gradenigo&;s triad is caused due to the involvement of", "exp": "Gradenigo syndrome is the classical presentation and consists of a triad of (i) external rectus palsy (VIth nerve palsy),(ii) deep-seated ear or retro-orbital pain (Vth nerve involvement) and (iii) persistent ear dischargeDue to petrositis in CSOM.(Spread of infection from middle ear and mastoid to the petrous pa of the temporal bone is called petrositis.)Dhingra 6e pg: 79", "cop": 1, "opa": "Abducent nerve", "opb": "Facial nerve", "opc": "Optic nerve", "opd": "Occulomotor nerve", "subject_name": "ENT", "topic_name": "Ear", "id": "df7691e2-568d-47a9-b269-d5dc5464deda", "choice_type": "single"} {"question": "During normal conversation, sound heard at 1meter distance is", "exp": "At a distance of 1 m, intensity of Whisper = 30 dB Normal conversation = 60 dB Shout = 90 dB Discomfo of the ear = 120 dB Pain in the ear = 130 dB Ref: Dhingra 7e pg 21.", "cop": 1, "opa": "60dB", "opb": "80dB", "opc": "90dB", "opd": "120dB", "subject_name": "ENT", "topic_name": "Ear", "id": "b362cadb-72b6-4319-8cd3-3db39d25b8ed", "choice_type": "single"} {"question": "Muscular voice in females is treated by", "exp": "THYROPLASTY Isshiki divided thyroplasty procedures into four categories to produce functional alteration of vocal cords. (a) Type I:- It is medial displacement of vocal cord as is achieved in teflon paste injection. (b) Type II:- It is lateral displacement of vocal cord and is used to improve the airway (c) Type III:- It is used to shoen (relax) the vocal cord. Relaxation of vocal cord lowers the pitch. This procedure is done in mutational falsetto or in those who have undergone gender transformation from female to male. (d) Type IV:- This procedure is used to lengthen (tighten) the vocal cord and elevate the pitch. It conves male character of voice to female and has been used in gender transformation. It is also used when vocal cord is lax and bowing due to aging process or trauma. Ref:- Dhingra; pg num:-301,302", "cop": 4, "opa": "Thyroplasty type 1", "opb": "Thyroplasty type 2", "opc": "Thyroplasty type 3", "opd": "Thyroplasty type 4", "subject_name": "ENT", "topic_name": "Larynx", "id": "51691a5f-236f-4bf0-99f7-85053d3ac4bd", "choice_type": "single"} {"question": "Wood workers are associated sinus Ca", "exp": null, "cop": 1, "opa": "Adeno Ca", "opb": "Sq. cell Ca", "opc": "Anaplastic Ca", "opd": "Melanoma", "subject_name": "ENT", "topic_name": null, "id": "f2c63623-3aef-4d01-9101-95a04246c75f", "choice_type": "single"} {"question": "Infection of CNS spreads to inner ear through", "exp": "The scala tympani is closed by a secondary tympanic membrane. It is also connected with subarachnoid space through aqueduct of cochlea Reference: Dhingra 6th edition.", "cop": 1, "opa": "Cochlear aqueduct", "opb": "Endolymphatic sac", "opc": "Vestibular aqueduct", "opd": "Hyle fissure", "subject_name": "ENT", "topic_name": "Ear", "id": "0dfddde1-0368-421b-b97e-f42b21013b26", "choice_type": "single"} {"question": "Pseudosulcus in larynx is", "exp": "Laryngeal pseudosulcus refers to edema of the ventral surface of the true vocal fold that extends from the anterior commissure to the posterior larynx. Patients with pseudosulcus are nearly 2.5 times more likely to have pH-documented Larynopharyngela reflux.", "cop": 1, "opa": "Laryngopharangeal reflux", "opb": "Vocal abuse", "opc": "Chronic steroid use", "opd": "Tuberculosis", "subject_name": "ENT", "topic_name": "Larynx", "id": "432a6642-0d76-42d9-b589-f82a871866eb", "choice_type": "single"} {"question": "Commonest cause for grommet inseion is", "exp": "The main pathology in Serous otitis media is effusion, which can be managed by drainage. Inseion of grommet in infection can lead to foreign body granuloma. www.mydr.com.au Tympanostomy tube, also known as a grommet or myringotomy tube, is a small tube inseed into the eardrum in order to keep the middle ear aerated for a prolonged period of time, and to prevent the accumulation of fluid in the middleear. Re: Textbook of Ear, Nose and Throat, Dhingra, 6th Edition; Pg no: 396", "cop": 1, "opa": "Secretory otitis media", "opb": "Otosclerosis", "opc": "CSOM", "opd": "Cholesteatoma", "subject_name": "ENT", "topic_name": "Diagnostic and operative ENT", "id": "7b53450c-959a-42d2-b6c9-55370e149cd3", "choice_type": "single"} {"question": "Most common site of stapedial otosclerosis is", "exp": "(D) Fissula ante fenestram # STAPEDIAL OTOSCLEROSIS. Stapedial otosclerosis causing stapes fixation & conductive deafness is the most common variety.> Lesion starts just in front of the oval window in an area called 'fissula ante fenestram'. This is the site predilection (anterior focus).> Lesion may start behind the oval window (posterior focus), around the margin of the stapes footplate (circumferential), in the footplate but annular ligament being free (biscuit type). Sometimes, it may completely obliterate the oval window niche (obliteralivetype)", "cop": 4, "opa": "Round window", "opb": "Cochlear capsule", "opc": "Promontory", "opd": "Fissula ante fenestram", "subject_name": "ENT", "topic_name": "Miscellaneous (E.N.T.)", "id": "a2ffb552-0099-4d51-a33c-2c0934bc895f", "choice_type": "single"} {"question": "The following test is useful for diagnosis of", "exp": "Ans. (b) Meniere's disease.* The image shows performance of the caloric stimulation test.* Irrigation of the external meatus with water 7deg above and later 7deg below body temperature sets up convection currents of the endolymph in the semi-circular canals.* This causes nystagmus, and the duration of the nystagmus gives an index of the activity of the labyrinth. The nystagmus can be directly observed or recorded electrically (electronystagmography).* This test is particularly valuable in the diagnosis of Meniere's disease and acoustic neuroma. A reduced or absent nystagmus is found (canal paresis).", "cop": 2, "opa": "Unsafe CSOM", "opb": "Meniere's disease", "opc": "Gradenigo syndrome", "opd": "Cholesteatoma", "subject_name": "ENT", "topic_name": "Diagnostic & Operative ENT", "id": "feb8573f-f524-4eec-98d1-4bac1abe25dd", "choice_type": "single"} {"question": "Mouse Nibbled appearance of vocal cord seen in", "exp": "Earliest sign : Hyperaemia of UC &bposterior commissure area.\nMamillated arytenoids : Swollen arytenoids.\nMouse nibbud appearance : Multiple ulcer on vocal cord.\nTubon epiglottis : Swollen epiglottis of cellular infiitrations (pseudoedima).", "cop": 1, "opa": "TB", "opb": "Syphilis", "opc": "Cancer", "opd": "Papilloma", "subject_name": "ENT", "topic_name": null, "id": "c7912e04-854d-4a51-afee-6967b10b0979", "choice_type": "single"} {"question": "Stria vascularis are seen in", "exp": "Ans. (c) CochleaRef: Dhingra's ENT 5th ed. /12* Upon taking the section of cochlea, we see 3 compartments and 3 membranes:# Scala vestibuli: contains perilymph# Scala media: contains endolymph# Scala tympani: contains perilymph* 3 membranes are:# Reissner's membrane: separates scala vestibule from scala media.# Basilar membrane: separates scala tympani from scala media. Also supports organ of corti.# Stria vascularis: on the wall of scala media. Converts perilymph into endolymph.", "cop": 3, "opa": "Vestibule", "opb": "Utricle", "opc": "Cochlea", "opd": "Middle ear", "subject_name": "ENT", "topic_name": "Ear", "id": "eb14d543-6f48-4fab-be69-70dd04f395b9", "choice_type": "single"} {"question": "Ossicular ratio is =", "exp": null, "cop": 1, "opa": "1.3 : 1", "opb": "1.2 : 1", "opc": "1.1 : 1", "opd": "1:01", "subject_name": "ENT", "topic_name": null, "id": "6003d13a-874b-4d11-8d16-86b2b3de3289", "choice_type": "single"} {"question": "Turban epiglottis is seen in", "exp": "Turban epiglottis is due to oedema and infiltration of the epiglottis and is caused by laryngeal tuberculosis. Lupus: a form of tuberculosis, on the other hand, eats away and destroys the epiglottis. Pseudoedema of the epiglottis is known as \"turban epiglottis\". Ref: PL Dhingra 7th edition of Ear, Nose and Throat; Pg no 331", "cop": 1, "opa": "Tuberculosis", "opb": "Leprosy", "opc": "Laryngeal papilloma", "opd": "Epiglottitis", "subject_name": "ENT", "topic_name": "Larynx", "id": "69beeaad-ca09-4c1e-b28e-0a66156d64b7", "choice_type": "single"} {"question": "Female with stapedectomy done, has a conductive hearing loss at 60 dB. Diagnosis is", "exp": "1. Complete obstruction of ear canal: 30 dB 2. Perforation of tympanic membrane (It varies and is directly propoional to the size of perforation): 10-40 dB 3. Ossicular interruption with intact drum: 54 dB 4. Ossicular interruption with perforation: 38 dB 5. Malleus fixation: 10-25 dB 6. Closure of oval window: 60 dB Ref: Dhingra 7e pg 31.", "cop": 3, "opa": "Implant failure", "opb": "Fistula", "opc": "Closure of oval window", "opd": "Tympanic membrane perforation", "subject_name": "ENT", "topic_name": "Ear", "id": "ec6cbb6c-b396-47eb-89a2-66beb72bacd8", "choice_type": "single"} {"question": "Dohlmann&;s procedure is done in", "exp": "PHARYNGEAL POUCH Also called hypopharyngeal diveiculum or Zenker's diveiculum, it is a pulsion diveiculum where pharyngeal mucosa herniates through the Killian's dehiscence--a weak area between two pas of the inferior constrictor. TREATMENT :- 1. Excision of pouch and cricopharyngeal myotomy. This is done through cervical approach. 2. Dohlman's procedure. The paition wall between the oesophagus and the pouch is divided by diathermy through an endoscope. This is done in poor risk debilitated patients. 3. Endoscopic laser treatment. It is similar to Dohlman's procedure. Paition between the pouch and oesophagus is divided by CO2 laser using operating microscope. Ref:- Dhingra; pg num:-274,275", "cop": 1, "opa": "Zenkers diveiculum", "opb": "Barretts oesophagus", "opc": "Schatzki ring", "opd": "Achalasia cardia", "subject_name": "ENT", "topic_name": "Pharynx", "id": "cbfe43ce-9879-4504-a5cc-89828a373d47", "choice_type": "single"} {"question": "A 3 year old child with Bilateral hearing loss, On examination Bluish tympanic membrane", "exp": null, "cop": 2, "opa": "Acute serous atitis media", "opb": "Glue Ear", "opc": "Acute Necrotising Otitis media", "opd": "Cheonic serous Otitis media", "subject_name": "ENT", "topic_name": null, "id": "8fc66804-b835-4140-8cc9-7e3773bc53b3", "choice_type": "single"} {"question": "Most common benign lesion of larynx in the pediatric age group is", "exp": "Juvenile papillomatosis is the most common benign neoplasm of the larynx in children. It is viral in origin and is caused by human papilloma DNA virus type 6 and 11. It is presumed that affected children got the disease at bih from their mothers who had vaginal human papilloma virus disease.Patient, often a child, between the age of 3 and 5 years presents with hoarseness or aphonia with respiratory difficulty or even stridor.Dhingra 6e pg: 306", "cop": 3, "opa": "Granular cell tumor", "opb": "Chondroma", "opc": "Juvenile papillomatosis", "opd": "Solitary papilloma", "subject_name": "ENT", "topic_name": "Larynx", "id": "cb57f8b1-c8fd-47b2-804d-edd52b74e358", "choice_type": "single"} {"question": "The incision used in endomeatal approach to the ear is", "exp": "The tympanomeatal flap is raised to expose the middle ear in endomeatal approach. Rosen's incision is used in the endomeatal approach for stapedectomy. It is also used for exploratory tympanotomy. It requires the meatus and canal to be wide enough to work. It consists of two pas: (i) a small veical incision at 12 o'clock position near the annulus (ii) a curvilinear incision staing at 6 o'clock position to meet the first incision in the posterosuperior region of the canals, 5-7 mm away from the annulus Ref : Diseases of ENT by Dhingra 6th edition Pgno : 401", "cop": 2, "opa": "Lempe I incision", "opb": "Rosen's incision", "opc": "Lempe II incision", "opd": "Wilde's incision", "subject_name": "ENT", "topic_name": "Ear", "id": "d39d5dc6-3625-46af-a5dc-708aafde2954", "choice_type": "single"} {"question": "Identity the Instrument used in ENT surgery", "exp": null, "cop": 1, "opa": "Eve's Tonsillar snare", "opb": "Luc Nasal forceps", "opc": "Denis Browne Tonsil Holding forceps", "opd": "ST Clair Thompson curette", "subject_name": "ENT", "topic_name": null, "id": "7b1ef539-21ac-4c19-be06-f20d40c51903", "choice_type": "single"} {"question": "Hot potato voice seen in", "exp": null, "cop": 1, "opa": "Quinsy", "opb": "Retropharyngeal Abscess", "opc": "Ludwig's Angina", "opd": "Glottic carcinoma", "subject_name": "ENT", "topic_name": null, "id": "47129b6e-7965-43c9-b749-ea4db93e5658", "choice_type": "single"} {"question": "If a patient gets an attack of veigo by loud noise, he is having", "exp": "Tullio phenomenon - A condition in which loud sound produces veigo. It can occur in syphilis. (Ref: Diseases of EAR, NOSE AND THROAT by PL DHINGRA - 6th Edition)", "cop": 1, "opa": "Tullio phenomenon", "opb": "Dysplacusis", "opc": "Hyperacusis", "opd": "Paracusis", "subject_name": "ENT", "topic_name": "Ear", "id": "ba8f8179-0b22-4dc4-9492-1a5ffb184cb7", "choice_type": "single"} {"question": "During the ear examination, cough occurs due to stimulation of", "exp": "Arnold nerve. It is an auricular branch of jugular ganglion of the vagus. \nIt supplies the skin of posterior part of pinna on the medial surface of the concha, and posterior wall of the external auditory canal and tympanic membrane.\nThe auricular nerve is the afferent limb of the Ear-Cough or Arnold Reflex.", "cop": 4, "opa": "Hypoglossal", "opb": "Trochlear", "opc": "Trigeminal", "opd": "Vagus", "subject_name": "ENT", "topic_name": null, "id": "5c868612-910a-44c3-96a1-f64cf8bd472c", "choice_type": "single"} {"question": "The common site for CSF rhinorrhoea is", "exp": "The cribriform plate and air cells of the ethmoid sinus account for a maximum number of CSF leaks, i.e., through anterior cranial fossa. Other sites are frontal sinus, an area of sella turcica and sphenoid sinus. Rare sites of the leak are middle or posterior cranial fossa and CSF and reach the nasal cavity by way of the middle ear and Eustachian tubeTraumatic leak-Cribriform plate and ethmoidal air cellsSpontaneous (non-traumatic) leak -Cribriform plateRef: PL Dhingra, Diseases of Ear, Nose & Throat, 7th edition, pg no. 183 - 185", "cop": 4, "opa": "Sphenoid sinus", "opb": "Frontal sinus", "opc": "Petrous", "opd": "Cribriform plate", "subject_name": "ENT", "topic_name": "Nose and paranasal sinuses", "id": "2f535537-e195-49dc-a646-17ebb3aad26c", "choice_type": "single"} {"question": "Treatment of erythroplakia", "exp": null, "cop": 2, "opa": "Excision", "opb": "Excision and regular follow up", "opc": "Radiotherapy", "opd": "Steroid injection", "subject_name": "ENT", "topic_name": null, "id": "10d89bb9-982a-41c2-a978-2fb13dbb3d14", "choice_type": "single"} {"question": "Ideal treatment of rhinosporidiosis is", "exp": "Rhinosporodiosis * Rhinosporodiosis is a Fungal Granuloma. * Agent : Rhinosporidium seeberi. * Source : Contaminated water of ponds. Clinical features:- * Leafy polypoidal mass, pink to purple in color studded with while dots (Strawberry appearance). * It is highly vascular and bleeds on touch. * Patients complain of blood tinged nasal discharge/epistaxis Diagnosis:- * Biopsy shows several sporangia filled with spores. * Treatment - Complete excision of mass with diathermy knife and cauterization of base. * Recurrence is common after surgery. Ref:- Dhingra 5th edition pg num:- 174", "cop": 3, "opa": "Rifampicin", "opb": "Dapsone", "opc": "Excision with cautery at base", "opd": "Laser", "subject_name": "ENT", "topic_name": "Nose and paranasal sinuses", "id": "cb5a444e-eaa9-41c3-9eae-ad23028efb3a", "choice_type": "single"} {"question": "Ostamann fat pad is related to", "exp": "It is related laterally to the membranous pa of the eustachian tube.It helps to keep the tube closed and thus protect it from the reflux of nasopharyngeal secretions. Veical section of Eustachian tube- Ref: Dhingra 6th edition pg. no - 57", "cop": 3, "opa": "Ear lobule", "opb": "Buccal mucosa", "opc": "Eustachian tube", "opd": "Tip of nose", "subject_name": "ENT", "topic_name": "Ear", "id": "9491a89f-71e5-4586-9ed8-daeec544cda5", "choice_type": "single"} {"question": "Lymphatics of tip of the tongue drain into", "exp": null, "cop": 3, "opa": "Submadibular Lymph Nodes", "opb": "Jugulodigastric Lymph Nodes", "opc": "Submental Lymph Nodes", "opd": "Jugulo - Omohyoid Lymph Nodes", "subject_name": "ENT", "topic_name": null, "id": "86423e2f-e446-4f6f-9a51-dd262048e3c1", "choice_type": "single"} {"question": "Bryce sign seen in", "exp": "On compression of sxternal swelling of laryngoule cause escape of air through larynk causing hissing of sound is know as Bryce sign.", "cop": 1, "opa": "Laryngouell", "opb": "Post cricoids carcinoma", "opc": "Down's syndrome", "opd": "Acute tonsillitis", "subject_name": "ENT", "topic_name": null, "id": "782607cc-1e25-4169-80cf-1b972b0c71e8", "choice_type": "single"} {"question": "Ratio of tympanic members to oval window is", "exp": "According to some workers (Wever and Lawrence) out of a total of 90 mm2 area of human tympanic membrane, only 55 mm2 is functional and given the area of stapes footplate (3.2 mm2), the areal ratio is 17:1 and total transformer ratio (17x 1.3) is 22.1. Ref: Dhingra 7e pg 17.", "cop": 1, "opa": "17:01", "opb": "22:01", "opc": "50:01:00", "opd": "25:01:00", "subject_name": "ENT", "topic_name": "Ear", "id": "784a2c86-e247-459f-8b06-3b2e797ec81c", "choice_type": "single"} {"question": "The voice in a patient with bilateral abductor paralysis of larynx is", "exp": "As both the cords lie in median or paramedian position, the airway is inadequate causing dyspnoea and stridor but the voice is good. Dyspnoea and stridor become worse on exeion or during an attack of acute laryngitis. Ref: PL Dhingra Textbook of Ear, Nose and Throat, Edition 7, page - 339", "cop": 4, "opa": "Pubophonia", "opb": "Phonasthenia", "opc": "Dysphonia plicae ventricularis", "opd": "Normal or good voice", "subject_name": "ENT", "topic_name": "Larynx", "id": "6fb49274-0d7e-48e1-8b2d-b953d00e2b5e", "choice_type": "single"} {"question": "Hot potato voice is characteristic of", "exp": "Peritonsillar abscess shows muffled and thick speech often called as \" hot potato voice \". Ref: Textbook of diseases of ENT, PL Dhingra, 7th edition, pg no. 298", "cop": 1, "opa": "Pre tonsillar abscess", "opb": "Glottis carcinoma", "opc": "Subglottic carcinoma", "opd": "Supraglottic carcinoma", "subject_name": "ENT", "topic_name": "Larynx", "id": "37701e61-16cd-4a82-8f6b-69b345c07850", "choice_type": "single"} {"question": "A patient after having a fracture of middle cranial fossa complains of loss of tear secretion. This is due to the injury to", "exp": "Greater petrosal nerve arises from the geniculate ganglion of the facial nerve, enters the middle cranial fossa by emerging at the hiatus for the greater petrosal nerve on the anterior surface of the petrous temporal bone. It joins the deep petrosal nerve and forms the nerve of the pterygoid canal which reaches the pterygopalatine ganglion. The postganglionic parasympathetic fibres ultimately supply the lacrimal gland and the mucosal glands of pharynx, palate and nose. Ref:", "cop": 3, "opa": "Trigeminal nerve", "opb": "Stellate ganglion", "opc": "Greater petrosal nerve", "opd": "Ciliary ganglion", "subject_name": "ENT", "topic_name": "Miscellaneous ENT", "id": "a75e047e-3fd3-4aff-855f-a89c82457f6c", "choice_type": "single"} {"question": "Last taste sensation discovered is", "exp": "Umami a savory taste is one of the five basic tastes (together with sweetness, sourness, bitterness, and saltiness)People taste umami through receptors specific to glutamate. Glutamate is widely present in savory foods, such as meat broths and fermented products, and commonly added to some foods in the form of monosodium glutamate (MSG) since umami has its own receptors rather than arising out of a combination of the traditionally recognized taste receptors, scientists now consider umami to be a distinct taste. Ref: https://en.m.wikipedia.org/wiki/Umami", "cop": 1, "opa": "Umami", "opb": "Sweetness", "opc": "Bitter", "opd": "Sour", "subject_name": "ENT", "topic_name": "Oral cavity & Oesophagus", "id": "a433596f-c61f-4f88-a8d6-e3d8735d2bc7", "choice_type": "single"} {"question": "Passavant ridge is formed by", "exp": "Passavant ridge Near the superior margin of the pharynx, a few fibers of superior constrictor blend with a band of muscle fibers belonging to the palatopharyngeus muscle These fused fibers form a band or ring around the posterior wall and sidewalls of the nasopharyngeal isthmus When the soft palate is elevated this muscle band appears as a ridge is known as passavant ridge. Ref: PL Dhingra, Diseases of Ear, Nose & Throat, 7th edition, pg no. 271", "cop": 1, "opa": "Superior constrictor and palatopharyngeus", "opb": "Inferior constrictor and palatopharyngeus", "opc": "Superior constrictor and palatoglossus", "opd": "Inferior constrictor and palatoglossus", "subject_name": "ENT", "topic_name": "Pharynx", "id": "30e03b6a-1a0d-4eef-be93-7fc7add6c146", "choice_type": "single"} {"question": "Elastic cartilage is present in", "exp": "Elastic cartilage is present in – Epiglottis. It is a yellow, leaf-like elastic cartilage forming anterior wall of laryngeal inlet. It is attached to the body of hyoid bone by the hyoepiglottic ligament, which divides into suprahyoid and infrahyoid epiglottis.\nOther elastic cartilage structures are corniculate, cuneiform and tip of arytenoid near corniculate cartilage.\nThyroid, cricoid and most of the arytenoid cartilage are hyaline cartilages.", "cop": 4, "opa": "Thyroid cartilage", "opb": "Cricoid", "opc": "Arytenoid cartilage", "opd": "Epiglottis", "subject_name": "ENT", "topic_name": null, "id": "ec5f3df0-a576-406b-bd03-45fc7e42ec9a", "choice_type": "single"} {"question": "Gradcnigo's syndrome is characterized by a/e", "exp": "Ans. a (Conductive deafness). (Ref. PL Dhingra, ENT, 2nd ed., 78)GRADENIGO'S SYNDROME# Petrositis, a complication of otitis media classically presents with a triad of symptoms, known as Gradenigo's syndrome characterized by:- Abducent nerve (lateral rectus) palsy.- Deep seated ear or retro-orbital pain (Vth nerve involvement)- Persistent ear discharge# However, fever, headache, vomiting & sometimes neck stiffness occurs.# Towne's & Stenver's (StockholmC) view for petrous apex and internal auditory meatus point towards diagnosis whileHRCT of temporal bone is confirmatory & gives greater anatomical details. CTT1 MRIT2MRICEMRIAcute petrous apecitis with osteomyelitis (Gradenigo syndrome)Opacified air cells with bony breakdown; nonexpansileLowHighrim enhancement or enhancement of adjacent meninges# Rx these days is antibiotic therapy alone, but sometimes mastoidectomy may be required.", "cop": 1, "opa": "Conductive deafness", "opb": "Retroorbital pain", "opc": "Ear discharge", "opd": "Diplopia", "subject_name": "ENT", "topic_name": "CSOM and its Complications", "id": "3128be43-d75f-499a-8f21-b9f6988ffc27", "choice_type": "single"} {"question": "Most common site for mucoepidermoid carcinoma is", "exp": "Mucoepidermoid carcinoma is the most frequently diagnosed malignancy of the salivary gland. Among the major salivary glands, the parotid gland is most commonly involved.", "cop": 1, "opa": "Parotid gland", "opb": "Submandibular gland", "opc": "Sublingual gland", "opd": "Minor salivary gland", "subject_name": "ENT", "topic_name": null, "id": "57a9e87b-2301-4bde-8a2b-ef569fd5daa0", "choice_type": "single"} {"question": "In Galen's anastomosis, superior laryngeal nerve pierces inferior constrictor of pharynx and unites with", "exp": "GALEN'S ANASTOMOSIS: Superior laryngeal nerve ends by piercing inferior constrictor of pharynx and unites with ascending branch of recurrent laryngeal nerve. This is called as Galen's anastomosis & is purely sensory.", "cop": 3, "opa": "Superior laryngeal nerve of opposite side", "opb": "Descending branch of RLN", "opc": "Ascending branch of RLN", "opd": "Glossopharyngeal nerve", "subject_name": "ENT", "topic_name": "Anatomy of Larynx", "id": "9229fd10-2619-413d-b49b-f06698df03f9", "choice_type": "single"} {"question": "A lumpy feeling in throat relieved on taking food is attributed to", "exp": "Globus PharyngeusSymptom: a patient describes something stuck in the throat or a sensation of lump or tightness in the throat which is relieved by taking food or talkingRef: PL Dhingra, Diseases of Ear, Nose & Throat, 7th edition, pg no. 389", "cop": 1, "opa": "Globus pharyngeus", "opb": "Pharyngeal pouch", "opc": "Diveicular disease", "opd": "Exophageal atresia", "subject_name": "ENT", "topic_name": "Larynx", "id": "7900941f-b325-45ed-81ed-eb3284c0df11", "choice_type": "single"} {"question": "Facial nerve exits the skull through", "exp": "Facial nerve enters the temporal bone through - Internal acoustic meatus Facial nerve exits the skull through - Stylomastoid foramen. Ref: PL Dhingra, Diseases of Ear, Nose & Throat, 7th edition, pg no. 99", "cop": 1, "opa": "Stylomastoid foramen", "opb": "Internal acoustic meatus", "opc": "Foramen Lacerum", "opd": "Foramen Rotundum", "subject_name": "ENT", "topic_name": "Oral cavity & Oesophagus", "id": "1f45779a-010a-4ecb-a284-532a3aab3ae9", "choice_type": "single"} {"question": "For a case of pubero phonia,Thyroplasty used is", "exp": null, "cop": 3, "opa": "Type 1", "opb": "Type 2", "opc": "Type 3", "opd": "Type 4", "subject_name": "ENT", "topic_name": null, "id": "4ce3bed9-14d8-45dc-8c42-7bba7d2a5dc1", "choice_type": "single"} {"question": "4 year old male, recurrent URTI, has difficulty breathing, High arched palate, Failure to grow and impaired hearing, management is", "exp": null, "cop": 4, "opa": "Tonsillectomy", "opb": "Adenoidectomy", "opc": "Antibiotics and nasal decongestants", "opd": "Adenoidectomy with grommet insertion", "subject_name": "ENT", "topic_name": null, "id": "9e2392fa-8882-4dae-8849-4e268bbb7a27", "choice_type": "single"} {"question": "Most common cause of vocal cord palsy is", "exp": "Neuritis or surgical trauma (thyroidectomy) is the most impoant cause. Thyroid surgery is the most common cause when both recurrent and external laryngeal nerves of one side may be involved. Ref: PL Dhingra Textbook of Ear, Nose and Throat, Edition 6, page - 300", "cop": 1, "opa": "Surgical", "opb": "Inflammatory", "opc": "Trauma", "opd": "Malignancy", "subject_name": "ENT", "topic_name": "Larynx", "id": "6e47ef5a-0981-4726-a7e3-a64919db1112", "choice_type": "single"} {"question": "Crocodile tears is due to abnormal regeneration of", "exp": null, "cop": 1, "opa": "Facial Nerve", "opb": "Auriculotemporal Nerve", "opc": "Vagus Nerve", "opd": "Glossopharyngeal Nerve", "subject_name": "ENT", "topic_name": null, "id": "05f3ff5f-e5de-4e90-8021-ae32c179b07c", "choice_type": "single"} {"question": "The cause for contact ulcer of vocal cord", "exp": null, "cop": 1, "opa": "Voice abuse", "opb": "Smoking", "opc": "TB", "opd": "Valignancy", "subject_name": "ENT", "topic_name": null, "id": "ef3dc1dd-1ce6-4e6d-86de-ff8235133620", "choice_type": "single"} {"question": "The most common site of the leak in CSF rhinorrhea is", "exp": "CSF rhinorrhea- Commonest site- Lateral lamella of cribriform plate of the ethmoidOther sites- Roof of ethmoids, posterior wall of the frontal sinus, Sphenoid sinusThe commonest cause of CSF rhinorrhoea- Iatrogenic TraumaRef: Hazarika; 3rd ed; Pg 361", "cop": 3, "opa": "Sphenoid sinus", "opb": "Frontal sinus", "opc": "Cribriform plate", "opd": "Tegmen tympani", "subject_name": "ENT", "topic_name": "Nose and paranasal sinuses", "id": "d11b4b7d-1282-4daa-ac7d-3ed3cbcd3a21", "choice_type": "single"} {"question": "Most common cause of chronic laryngeal stenosis is", "exp": "The most common cause of chronic laryngeal stenosis is Trauma. Complications of laryngeal trauma Laryngeal stenosis which is supraglottic, glottic or subglottic. Perichondritis and laryngeal abscess. Vocal cord paralysis. Ref: Textbook of diseases of ENT, PL Dhingra, 7th edition, pg no. 326", "cop": 1, "opa": "Trauma", "opb": "Tuberculosis", "opc": "SLE", "opd": "Tumour", "subject_name": "ENT", "topic_name": "Larynx", "id": "00428210-6d4a-4af0-a64b-e05f0c29f684", "choice_type": "single"} {"question": "Rosen's incision is used for", "exp": "Rosen's incision is the most commonly used for stapedectomy. It requires the meatus and canal to be wide enough to work. It consists of two pas: A small veical incision at 12 o'clock position near the annulus and A curvilinear incision staing at 6 o'clock position to meet the first incision in the posterosuperior region of the canals, 5-7 mm away from the annulus. Tympanomeatal flap is raised to expose the middle ear in endomeatal approach. Rosen's incision is used in the endomeatal approach for stapedectomy. It is also used for exploratory tympanotomy Re: Textbook of Ear, Nose and Throat, Dhingra, 6th Edition", "cop": 3, "opa": "Septoplasty", "opb": "SMR", "opc": "Stapedectomy", "opd": "Tonsillectomy", "subject_name": "ENT", "topic_name": "Diagnostic and operative ENT", "id": "61839004-a26e-40dd-b12b-5569822e2781", "choice_type": "single"} {"question": "Injury to superior laryngeal nerve causes", "exp": "Paralysis of the superior laryngeal nerve causes paralysis of cricothyroid muscle and ipsilateral anaesthesia of the larynx above the vocal cord. Paralysis of cricothyroid can also occur when the external laryngeal nerve is involved in thyroid surgery, tumours, neuritis or diphtheria. Voice is weak and pitch cannot be raised with decreased ability to sing. Flapping of the paralyzed cord. As the tension of the cord is lost, it sags down during inspiration and bulges up during expiration. Ref: PL Dhingra Textbook of Ear, Nose and Throat, Edition 7, page - 339", "cop": 4, "opa": "Hoarseness", "opb": "Paralysis of vocal cords", "opc": "No effect", "opd": "Loss of timbre of voice", "subject_name": "ENT", "topic_name": "Larynx", "id": "5685ee23-5853-401f-9f89-509f86ff7ecb", "choice_type": "single"} {"question": "Sensory nerve supply of larynx below the level of vocal cord is", "exp": "NERVE SUPPLY OF LARYNX Motor:- All the muscles which move the vocal cord (abductors, adductors or tensors) are supplied by the recurrent laryngeal nerve except the cricothyroid muscle. The latter receives its innervation from the external laryngeal nerve-- a branch of superior laryngeal nerve. Sensory:- Above the vocal cords, larynx is supplied by internal laryngeal nerve--a branch of superior laryngeal, and below the vocal cords by recurrent laryngeal nerve. Ref:- Dhingra; pg num:-298", "cop": 3, "opa": "External branch of superior laryngeal nerve", "opb": "Internal branch of superior laryngeal nerve", "opc": "Recurrent laryngeal nerve", "opd": "Inferior pharyngeal nerve", "subject_name": "ENT", "topic_name": "Larynx", "id": "bc5fb7e2-c5d7-4ed2-ae58-8c69a83394db", "choice_type": "single"} {"question": "During normal conversation sound heard at a 1meter distance is", "exp": "Intensity: It is the strength of sound which determines its loudness. It is usually measured in decibels. At a distance of 1 m, intensity of Whisper = 30 dB Normal conversation = 60 dB Shout = 90 dB Discomfo of the ear = 120 dB Pain in the ear = 130 dB Ref : Diseases of ENT by Dhingra 6th edition Pgno : 19", "cop": 1, "opa": "60dB", "opb": "80dB", "opc": "90dB", "opd": "1200dB", "subject_name": "ENT", "topic_name": "Ear", "id": "f483c0f2-1f85-4e19-8b6f-c7c5ee7c9031", "choice_type": "single"} {"question": "Schwaz operation is done in", "exp": "Coical mastoidectomy, known as simple or complete mastoidectomy or Schwaz operation It is complete exenteration of all accessible mastoid air cells and conveing them into a single cavity. The posterior meatal wall is left intact and Middle ear structures are not disturbed. INDICATIONS 1. Acute coalescent mastoiditis. 2. Incompletely resolved acute otitis media with reservoir sign. 3. Masked mastoiditis. Re: Textbook of Ear, Nose and Throat, Dhingra, 6th Edition; Pg no: 401", "cop": 4, "opa": "CSOM", "opb": "Serous otitis media", "opc": "Otosclerosis", "opd": "Acute mastoiditis", "subject_name": "ENT", "topic_name": "Diagnostic and operative ENT", "id": "3ca19f98-7c24-4c6c-91c8-9ad4f99ddf26", "choice_type": "single"} {"question": "Rhinitis medicamentosa is due to", "exp": "Rhinitis medicamentosa is due to the prolonged use of topical nasal decongestants like oxymetazoline and xylometazolineTreatment- Stoppage of the decongestants, topical and systemic steroids.Ref: Hazarika; 3rd ed; Pg 298", "cop": 1, "opa": "Nasal decongestants", "opb": "Steroids", "opc": "Surgery", "opd": "Antihistaminics", "subject_name": "ENT", "topic_name": "Nose and paranasal sinuses", "id": "4acd79c7-cca0-466f-870e-44e082ca031e", "choice_type": "single"} {"question": "Acoustic neuroma causes", "exp": "The tumour almost arises from the Schwann cells of the vestibular, but rarely from the cochlear division of VIIIth cranial nerve. Acoustic neuroma causes progressive unilateral sensorineural hearing loss by pressure on cochlear nerve fibres. Ref: Textbook of diseases of ENT, PL Dhingra, 7th edition, pg no. 125", "cop": 2, "opa": "Cochlear deafness", "opb": "Retrocochlear deafness", "opc": "Conductive deafness", "opd": "Veigo", "subject_name": "ENT", "topic_name": "Ear", "id": "e5beb40b-7434-47f4-ac15-367e8ee7803c", "choice_type": "single"} {"question": "A patient with Pancoast's tumour develops loss of voice after radiation. It is due to", "exp": "The recurrent laryngeal nerve is involved in Pancoast's tumour. Pancoast's tumour is a tumour of the pulmonary apex. Right recurrent laryngeal nerve arises from the vagus at the level of the subclan aery, hooks around it and then ascends between the trachea and oesophagus. The left recurrent laryngeal nerve arises from the vagus in the mediastinum at the level of the arch of the aoa, loops around it and then ascends into the neck in the tracheo-oesophageal groove. Ref: PL Dhingra; Textbook of EAR, NOSE and THROAT; edition 6", "cop": 2, "opa": "Vocal cord infiltration with secondary", "opb": "Involvement of recurrent laryngeal nerve", "opc": "Irradiation to vocal cords", "opd": "Radiation stenosis of larynx", "subject_name": "ENT", "topic_name": "Larynx", "id": "c5ccb467-9bec-4224-885b-dd98504bf709", "choice_type": "single"} {"question": "Endolymph in the inner ear", "exp": "Endolymph fills the membranous labyrinth and rich in K ions and is secreted by stria vascularis and the dark cells present in the utricle and near the ampullated end of semicircular ducts. Reference Dhingra 6th edition pg-24.", "cop": 2, "opa": "Is a filterate of blood serum", "opb": "Is secreted by Stria vascularis", "opc": "Is secreted by Basilar membrane", "opd": "Is secreted by Hair cells", "subject_name": "ENT", "topic_name": "Ear", "id": "8bc62535-d61c-485e-b0fa-75c9e0722c7b", "choice_type": "single"} {"question": "Not a cardinal sign of chronic tonsillitis", "exp": null, "cop": 4, "opa": "Irwin Moore sign", "opb": "Flushing of Anterior pillars", "opc": "Upper deep cervical lymph nodes enlarged", "opd": "Dull Tympanic membrane", "subject_name": "ENT", "topic_name": null, "id": "35a84f4c-413b-4ded-9412-d02c96d51b25", "choice_type": "single"} {"question": "Tober Ayer test is positive in", "exp": "Tobey Ayer test is positive in Lateral sinus thrombophlebitis.Compression of Inferior Jugular Vein in the neck on the normal side causes a rise in CSF pressure but not on the affected side.Ref: Hazarika; 3rd ed; Pg 171", "cop": 1, "opa": "Lateral sinus thrombosis", "opb": "Petrositis", "opc": "Cerebral abscess", "opd": "Subarachnoid haemorrhage", "subject_name": "ENT", "topic_name": "Ear", "id": "6115052c-b29d-4070-9865-9b8db32b8514", "choice_type": "single"} {"question": "Mode of inheritance of otosclerosis is", "exp": "There is a marked hereditary tendency to otosclerosis in ceain families, and in nearly 50 percent of cases, a positive family history can be obtained. The pattern of inheritance is autosomal dominant with incomplete penetrance.Ref: PL Dhingra, Diseases of Ear, Nose & Throat, 7th edition, pg no. 95 - 98", "cop": 3, "opa": "Autosomal Recessive", "opb": "X linked Recessive", "opc": "Autosomal Dominant", "opd": "Y linked", "subject_name": "ENT", "topic_name": "Ear", "id": "8f778620-edd2-4aa6-80f0-68f446fef5b6", "choice_type": "single"} {"question": "Carhart's notch is seen in", "exp": "i.e. (Otosclerosis): (99-Dhingra 5th)* Carhart's notch - maximum at 2000 Hz or 2KHZ carharts notch disappears after successful stapedectomy in OTOSCLEROSIS* Schwartze sign - flamingo-pink bluish tympanic membrane due to hyperaemia of the promontory and affords good evidence that the otosclerotic process is an active phase in OTOSCLEROSISOTOSCLEROSIS (otospongiosis)- Commonest site -fissula ante fenestram**- Painless and progressive bilateral conductive deafness**- Paracusis willisii (hears better in noisy than quite surroundings)**- Tinnitus, vertigo, monotonous speech**", "cop": 1, "opa": "Otosclerosis", "opb": "Acoustic neuroma", "opc": "Meneir's disease", "opd": "CSOM", "subject_name": "ENT", "topic_name": "Ear", "id": "925873cf-2eca-478b-81fc-db7ae9446c29", "choice_type": "single"} {"question": "Pathognomic features of Maxillary sinusitis is", "exp": "Clinical features of maxillary sinusitis: Constitutional symptoms: fever, general malaise and body ache. Headache Pain: It is typically situated over the jaw but may be referred to gums or teeth. Occasionally, it is referred to the ipsilateral supraorbital margin. Tenderness: pressure or tapping over the anterior wall of the antrum produces pain. Redness and oedema of the cheek: lower eyelid may become puffy. Nasal discharge: Anterior rhinoscopy shows pus/mucopus in the middle meatus. Mucosa of the middle meatus and terminate may appear red and swollen. Postnatal discharge: Pus may be seen on the upper soft palate on posterior rhinoscopy or nasal endoscopy. (Ref: Textbook of Diseases of ENT, 7th edition, PL Dhingra, pg no. 214)", "cop": 1, "opa": "Mucopus in the middle meatus", "opb": "Inferior turbinate hyperophy", "opc": "Purulent nasal discharge", "opd": "Atrophic sinusitis", "subject_name": "ENT", "topic_name": "Nose and paranasal sinuses", "id": "93afe70e-3734-4aee-93d1-71914c22374d", "choice_type": "single"} {"question": "The voice is not affected in", "exp": "Unilateral injury to recurrent laryngeal nerve results in ipsilateral paralysis of all the intrinsic muscles except the cricothyroid. The vocal cord thus assumes a median or paramedian position and does not move laterally on deep inspiration. Unilateral recurrent laryngeal paralysis may pass undetected as about one-third of the patients are asymptomatic. Others have some change in voice but no problems of aspiration or airway obstruction. The voice in unilateral paralysis gradually improves due to compensation by the healthy cord which crosses the midline to meet the paralyzed one. Ref: PL Dhingra Textbook of Ear, Nose and Throat, Edition 7, page - 339", "cop": 1, "opa": "Unilateral abductor palsy", "opb": "Unilateral adductor palsy", "opc": "Paial abductor palsy", "opd": "Total adductor palsy", "subject_name": "ENT", "topic_name": "Larynx", "id": "b3889807-cc49-4332-8af4-957092100bef", "choice_type": "single"} {"question": "Not used in treatment of Rhinoscleroma", "exp": null, "cop": 3, "opa": "Stretomycin", "opb": "Rifampicin", "opc": "Chloramphenicol", "opd": "Tetracyline", "subject_name": "ENT", "topic_name": null, "id": "8ad31b83-d4bd-46f3-99dd-42b072323d8d", "choice_type": "single"} {"question": "Tonsil reach their maximum size by", "exp": "Tonsils reach maximum size by 6-7 years and then involuteClosest among the options is 5 yearsRef: Hazarika; 3rd ed", "cop": 3, "opa": "1 year", "opb": "3 years", "opc": "5 years", "opd": "12 years", "subject_name": "ENT", "topic_name": "Pharynx", "id": "88ea8130-5486-4517-9da1-a2271cacb622", "choice_type": "single"} {"question": "Unsafe eardrum perforation is", "exp": "Ans. b (Marginal) (Ref. Diseases of ENT PL Dhingra 3rd/pg, 91-99).Attic and posterosuperior marginal perforations are seen in dangerous type of CSOM and are often associated with cholesteatoma while central perforation is considered safe, as cholesteatoma is usually not asso with it.OTITIS MEDIA# Tubotympanic disease (central perforation) remains localized to mucosa and that too mostly to anteroinferior part of middle ear cavity.# While atticoantral type or marginal or unsafe otitis media involves posterosuperior part of middle ear cleft (attic, antrum, posterior tympanum and mastoid) and is associated with cholesteatoma, which due to its bone-eroding property causes risk of serious complications and hence called unsafe or dangerous type.Chronic suppurative otitis mediaChronic suppurative otitis media (CSOM) is classified into two types: tubotympanic disease, in which there is a perforation of the pars tensa; and atticoantral disease, in which a retraction pocket develops from the pars flaccida.CSOM of the tubotympanic type. CSOM of the tubotympanic type can result from trauma or infection. When perforated the tympanic membrane usually repairs itself, but occasionally the outer layer of the tympanic membrane fuses with inner mucosa and a chronic perforation results. With this type of disease the patient's main symptoms are of an intermittent or chronic mucoid discharge associated with a mild conductive hearing loss. It is rare for this type of disease to be associated with intracranial complications. A diagnosis is made on otoscopy and the tuning forks usually suggest a conductive hearing impairment. The first-line treatment is topical antibiotic and steroid drops, and on occasion microsuction. If medical treatment fails, the patient may request an operation to graft the tympanic membrane in order to give a dry ear. This operation is termed a myringoplasty (type I tympanoplasty). The edges of the perforation are freshened and a small piece of temporalis fascia is inserted under the tympanic membrane to graft the drum.CSOM of the atticotympanic type. CSOM of the atticoantral type is important because of the complications associated with it. Cholesteatoma is the alternative name and means a cyst or sac of squamous epithelium that is present in the attic part of the middle ear. The exact aetiology of cholesteatoma is not known, although poor eustachian tube function is implicated. A retraction pocket develops in the pars flaccida and, if the squamous epithelium cannot migrate out of this pocket, a cholesteatoma results. The expanding ball of skin causes a low-grade osteomyelitis which results in the release of fatty acids from the bone. This gives the discharge its characteristic faecal smell. Invariably the discharge is accompanied by hearing loss and mild discomfort. The infection spreads to the dura via emissary veins which connect the middle ear mucosa to the dura or by direct extension of the disease through the bone. Meningitis, extradural, subdural or intracerebral abscess, or a combination of these may occur. Diagnosis should be suspected on otoscopy. Pus, crusts, granulations or a whitish debris in the attic are hallmarks of the disease. Examination under the microscope, audiometry and, sometimes, HRCT are indicated. The treatment is surgical and follows the principle of exposing the disease, excising the disease and then exteriorising the affected area. Two commonly applied operations for this disease are an atticotomy and a modified radical mastoidectomy.", "cop": 2, "opa": "Central", "opb": "Marginal", "opc": "Paracentral", "opd": "Multiple central", "subject_name": "ENT", "topic_name": "CSOM and its Complications", "id": "06b01318-bc2e-49b2-828c-31c9c991f523", "choice_type": "single"} {"question": "The investigation of choice for vestibular schwannoma is", "exp": "MRI with gadolinium contrast. It is superior to CT scan and is the gold standard for diagnosis of acoustic neuroma.Intracanalicular tumour, of even a few millimetres, can be easily diagnosed by this method.Dhingra 6e pg: 113", "cop": 2, "opa": "Contrast enhanced CT scan", "opb": "Gadolinium enhanced MRI", "opc": "SPECT", "opd": "PET scan", "subject_name": "ENT", "topic_name": "Ear", "id": "84586ae3-2e32-4cc6-8563-11e46ebad27d", "choice_type": "single"} {"question": "Posterior epistaxis occurs from", "exp": "Ans) a or CRef: Dhingra 5th ed p l90, 191There is a confusion regarding the formation of this question. In this question whether the commonest siteOR the commonest cause of the epistaxis is to be answered is not clear.If the commonest site is asked answer will be Woodruffs plexus and if the commonest cause is asked it will be atherosclerosis Littles area is situated in the anterior inferior part of nasal septum, just above the vestibule.Four arteries - anterior ethmoidal, septal branch of superior labial, septal branch of sphenopalatine and the grater palatine, anastomose here to form a vascular plexus called Kiesselbach's plexus. Anterior epistaxis mostly occurs from this site.Woodruffs area is situated under the posterior end of inferior tur-binate where the sphenopalatine artery anastomoses with the posterior pharyngeal artery. Posterior epistaxis may occur in this area", "cop": 1, "opa": "Woodruffs plexus", "opb": "Kiesselbach's plexus", "opc": "Atherosclerosis", "opd": "Littles area", "subject_name": "ENT", "topic_name": "Nose and PNS", "id": "a299ee75-7b16-4bb3-a830-415ab64e300f", "choice_type": "single"} {"question": "Earliest sign seen in Acoustic neuroma is", "exp": "The earliest cranial nerve to be involved is the fifth cranial nerve causing reduced corneal sensitivity, numbness, anaesthesia of face. Ref: Textbook of diseases of ENT, PL Dhingra, 7th edition, pg no. 125", "cop": 3, "opa": "Facial weakness", "opb": "Unilateral deafness", "opc": "Reduced corneal reflex", "opd": "Cerebellar signs", "subject_name": "ENT", "topic_name": "Ear", "id": "c5586aea-0e9a-4a5f-9fa6-97ed4c84442c", "choice_type": "single"} {"question": "Best time for hearing assessment in an infant is", "exp": "It is recommended that all infants be screened for hearing loss by 1 month of age All infants who do not pass the hearing screening need to complete a diagnostic hearing evaluation by 3 months of age and All children identified with permanent hearing loss need to receive appropriate intervention by 6 months of age This plan is also called the 1-3-6 plan Ref: Logan Turner&;s pg no 382", "cop": 1, "opa": "1st month of life", "opb": "3-6 months", "opc": "6-9 months", "opd": "9-12 months", "subject_name": "ENT", "topic_name": "Ear", "id": "8acbda60-bce2-42b0-8edb-a5eaaf0a6418", "choice_type": "single"} {"question": "Most common complication of Sinusitis is", "exp": "Orbital cellulitis is an infection of the soft tissues of the orbit posterior to the orbital septum. Orbital cellulitis is caused most commonly in all age groups by ethmoid sinusitis. The organisms gain access to the orbit through thin bones of the orbital walls, venous channels, foramina, and dehiscence. Orbital cellulitis resulting from infection of the maxillary sinus secondary to dental infections can be caused by microorganisms indigenous to the mouth.", "cop": 1, "opa": "Orbital cellulitis", "opb": "Meningitis", "opc": "Brain abscess", "opd": "Septicemia", "subject_name": "ENT", "topic_name": "Nose and paranasal sinuses", "id": "585744b4-9b64-4148-afd2-1f2e5700592f", "choice_type": "single"} {"question": "Most common presentation of nasopharyngeal carcinoma.", "exp": "Painless cervical lymphadenopathyis the most common presenting feature of nasopharyngeal carcinoma. Presence ofunilateral serous otitis media in an adult should raise suspicion of nasopharyngeal growth. (Eustachian tube obstruction leads to serous otitis media which causes deafness). Commonest site of origin for nasopharyngeal carcinoma is thefossa of Rosenmuller. EXTRA EDGE: Nasopharyngeal cancer can causeTrotter's triadof Conductive deafness(eustachian tube blockage), Ipsilateral temporoparietal neuralgia (involvement ofCN V) and Palatal paralysis (CN X)", "cop": 1, "opa": "Neck lymph node", "opb": "Blockage of nose", "opc": "Loss of smell", "opd": "Blood tinged discharge", "subject_name": "ENT", "topic_name": "FMGE 2019", "id": "0647c744-1e1b-4304-803d-040dcd442824", "choice_type": "single"} {"question": "Passavant's ridge is formed by", "exp": "It is a mucosal ridge raised by fibres of palatopharyngeus. It encircles the posterior and lateral walls of the nasopharyngeal isthmus.\nSoft palate, during its contraction, makes firm contact with this ridge to cut off nasopharynx from the oropharynx during the deglutition or speech.", "cop": 1, "opa": "Superior constrictor and palatopharyngeus", "opb": "Inferior constrictor and palatopharyngeus", "opc": "Superior constrictor and palatoglossus", "opd": "Inferior constrictor and palatoglossus", "subject_name": "ENT", "topic_name": null, "id": "e2dd2590-9501-48df-91d1-441af3086a8d", "choice_type": "single"} {"question": "Anti gravity Aspiration done in", "exp": null, "cop": 2, "opa": "Quinsy", "opb": "TB abscess", "opc": "Parapharymgeal abscess", "opd": "Ludwig's Angina", "subject_name": "ENT", "topic_name": null, "id": "cfb53502-bca3-44af-a806-22d5b0de115d", "choice_type": "single"} {"question": "Cotton&;s grading is used for", "exp": "Subglottic stenosis is graded according to the Cotton-Myer classification system from one to four based on the severity of the blockage. Grade 1 - <50% obstruction Grade 2 - 51-70% obstruction Grade 3 - 71-99% obstruction Grade 4 - no detectable lumen Ref: Internet sources- Wikipedia", "cop": 1, "opa": "Subglottic stenosis", "opb": "Laryngeal carcinoma", "opc": "Superior nerve palsy", "opd": "Voice abuse", "subject_name": "ENT", "topic_name": "Larynx", "id": "e1860d01-efaf-4762-95aa-469c00b106c8", "choice_type": "single"} {"question": "Cold water is not used for ear cleaning because", "exp": "temperature for ear syringing - 370c because if you decrease or increase the temperature we can stimulate lateral semicircular canal in the middle ear and patient will have veigo and nystagmus (caloric stimulation )", "cop": 3, "opa": "It will make the wax hard", "opb": "Damage to tympanic membrane", "opc": "Caloric stimulation caused by cold water", "opd": "It will cause infection", "subject_name": "ENT", "topic_name": "FMGE 2018", "id": "af2dc32f-ef07-481e-89fb-6be45381b2f5", "choice_type": "single"} {"question": "Most common cause of chronic retropharyngeal abscess", "exp": "CHRONIC RETROPHARYNGEAL ABSCESS (PREVEEBRAL ABSCESS) AETIOLOGY :- It is tubercular in nature and is the result of (i) caries of cervical spine or (ii) tuberculous infection of retropharyngeal lymph nodes secondary to tuberculosis of deep cervical nodes. Ref:- Dhingra; pg num:-266,267", "cop": 3, "opa": "Infective foreign body", "opb": "Caries teeth", "opc": "Caries of cervical spine", "opd": "Suppuration of retropharyngeal lymph node", "subject_name": "ENT", "topic_name": "Pharynx", "id": "3f2f00c9-aa59-4fd0-bc58-f7e95bbfb1e6", "choice_type": "single"} {"question": "Multiple perforations of pars tensa is charecteristic of", "exp": "TUBERCULAR OTITIS MEDIA In most of the cases, infection is secondary to pulmonary tuberculosis; infection reaches the middle ear through eustachian tube. Disease is mostly seen in children and young adults. PATHOLOGY The process is slow and insidious. Tubercles appear in the submucosal layers of middle ear cleft and caseate. There is painless necrosis of tympanic membrane. Multiple perforations may form which coalesce to form a single large perforation. Middle ear and mastoid get filled with pale granulations. Caries of bone and ossicles may occur leading to complications. Mastoiditis, facial paralysis, postauricular fistula, osteomyelitis with the formation of bony sequestra and profound hearing loss are often seen in these cases. Ref : Diseases of ENT by Dhingra 6th edition Pgno : 74", "cop": 4, "opa": "Myringitis bullosa", "opb": "Serous otitis media", "opc": "Malignant otitis externa", "opd": "Tuberculous otitis media", "subject_name": "ENT", "topic_name": "Ear", "id": "99116506-d4ad-49e9-9e09-514516186275", "choice_type": "single"} {"question": "Gerlach tonsil in waldeyer's ring is", "exp": "Tubal tonsil is otherwise called Gerlach`s tonsil, seen towards the tubal openingWaldeyer&;s ring consists of - Adenoids, Palatine, Lingual, Tubal tonsils, and Lateral pharyngeal bandsRef: Hazarika; 3rd ed; Pg 436", "cop": 1, "opa": "Tubal tonsil", "opb": "Palatine tonsil", "opc": "Pharyngeal tonsil", "opd": "Lingual tonsil", "subject_name": "ENT", "topic_name": "Pharynx", "id": "2d80205c-05fd-4bdd-a65d-f7af2d48c8c1", "choice_type": "single"} {"question": "The most common cause of conductive deafness in children is", "exp": "SEROUS OTITIS MEDIA, SECRETORY OTITIS MEDIA, MUCOID OTITIS MEDIA, \"GLUE EAR\" This is an insidious condition characterized by accumulation of nonpurulent effusion in the middle ear cleft. The fluid is nearly sterile. The disease affects children of 5-8 years of age. Hearing loss. This is the presenting and sometimes the only symptom. It is insidious in onset and rarely exceeds 40 dB. Deafness may pass unnoticed by the parents and may be accidentally discovered during audiometric screening tests. Ref : Diseases of ENT by Dhingra 6th edition Pgno : 64", "cop": 3, "opa": "ASOM", "opb": "Wax", "opc": "Serous otitis media", "opd": "Otosclerosis", "subject_name": "ENT", "topic_name": "Ear", "id": "f10fe683-0482-41ec-8387-a275cadddc1b", "choice_type": "single"} {"question": "Laser used in laryngeal work is", "exp": "CO2 laser:- CO2 laser is used in laryngeal surgery to excise vocal nodules, polyps, cysts, granulomas or juvenile laryngeal papilloma. Also used in case of leukoplakia, T1 lesion of vocal cord or localized leisions of supraepiglottis and infraglottis. 1) CO2 laser has wavelength 10,400nm 2) It can cut pericisely (0.3 mm percision), coagulate bleeders and vaporise tissues 3) Besides laryngeal surgery it is used in oropharyngeal surgery to excise benign or malignant lesions and in plastic surgery. Ref:- Dhingra; pg num:-357", "cop": 4, "opa": "Argon", "opb": "Nd:Yag", "opc": "Holmium", "opd": "CO2", "subject_name": "ENT", "topic_name": "Larynx", "id": "60e45b1c-aec4-468a-b1a6-ae55f56c9487", "choice_type": "single"} {"question": "Most common malignant tumour of minor salivary glands", "exp": null, "cop": 2, "opa": "Pleomorphic odenoma", "opb": "Adenoid cystic carcinoma", "opc": "Squamous cell carcinoma", "opd": "Mucoepidermoid carcinoma", "subject_name": "ENT", "topic_name": null, "id": "93196ec3-ba54-4681-9e21-a2a3c59129f9", "choice_type": "single"} {"question": "First branch of the facial nerve is", "exp": "Greater petrosal nerve arises from geniculate ganglion Ref: Dhingra 6th edition pg. 90", "cop": 1, "opa": "Greater petrosal nerve", "opb": "Lesser petrosal nerve", "opc": "Chorda-tympanic nerve", "opd": "Nerve to the stapedius", "subject_name": "ENT", "topic_name": "Ear", "id": "f2b14599-4383-4c94-8667-e520193187d2", "choice_type": "single"} {"question": "Persistent bucconasal membrane result in", "exp": "Ans. b (Choanal atresia). (Ref. Diseases of ENT, PL Dhingra, 3rd ed., 78, 210, 510)# Persistant bucconasal membrane leads to choanal atresia.Choanal atresia# Is a autosomal recessive trait in which the posterior choanae unilaterally or bilaterally fail to develop properly.# Persistent bucconasal membrane is the basic etiology.# Occurs in 1 in 5,000 births; more common among girls (2:1)# Unilateral > bilateral atresia.# Because newborns are obligate nose breathers, bilateral atresia is immediately apparent as respiratory distress.# Characteristics is the history or failure to pass a rubber catheter or nasogastric tube into the pharynx.# Symptoms of choanal atresia include failure to thrive due to poor feeding and mucoid nasal discharge.# The presence of choanal atresia can be confirmed with visualization of retention material in the posterior part of the nose on a lateral radiograph with the patient in the supine position.# Anatomic characterization of the deformity with CT can be important for planning surgical procedures.# Surgical treatment methods for membranous atresia include puncture of the choanal membrane and placement of a stent for 6 weeks.# If bony atresia is present, the bony wall can be taken down transnasally with microsurgical techniques followed by placement of a stent.# When suboptimal resection for atresia or choanal stenosis occurs, transpalatal repair at 3-4 years age is advised.# The septum usually deviates to the affected side; however, more posterior examination shows atresia.# Transpalatal repair is being replaced by endoscopic techniques of repair of atresia in children and adults.", "cop": 2, "opa": "Cleft lip", "opb": "Choanal atresia", "opc": "Cleft palate", "opd": "T-O Fistula", "subject_name": "ENT", "topic_name": "Nose and PNS", "id": "392da5ca-e097-4dc4-a8e8-b55576c97d75", "choice_type": "single"} {"question": "Unilateral sensorineural hearing loss may occur in", "exp": "Mumps(epidemic parotitis) may result in a profound sensorineural hearing loss (90 dB or more), unilaterally (one ear) or bilaterally (both ears). Coronavirus and Rotavirus are associated with diarrhoea. (Ref: Diseases of EAR, NOSE AND THROAT by PL DHINGRA - 6th Edition, Microbiology - Anantanarayanan)", "cop": 2, "opa": "Coronavirus", "opb": "Mumps", "opc": "Peussis", "opd": "Rotavirus", "subject_name": "ENT", "topic_name": "Ear", "id": "d3b7b1fd-16aa-490b-828e-fbd6830ae4b8", "choice_type": "single"} {"question": "Primary receptor cells of hearing are", "exp": "Hair cells are impoant receptor cells of hearing. Inner hair cells are more impoant in the transmission of auditory impulses. Outer hair cells mainly receive efferent innervation from the olivary complex and are concerned with modulating the function of inner hair cells. (Ref: Diseases of EAR, NOSE and THROAT by PL Dhingra; 6th edition; page no.13)", "cop": 4, "opa": "Suppoing cells", "opb": "Tectorial membrance", "opc": "Tunnel of coi", "opd": "Hair cells", "subject_name": "ENT", "topic_name": "Ear", "id": "4c9d53ba-d814-4734-8632-d03d42589cf3", "choice_type": "single"} {"question": "Killian&;s incision is used for", "exp": "Killian&;s incision is used for septoplasty Killians incision in case of deted nasal septum, make a slightly curvilinear incision, 2-3mm above the caudal end of septal cailage on the concave side . Re: Textbook of Ear, Nose and Throat, Dhingra, 6th Edition; Pg no: 416", "cop": 1, "opa": "Septoplasty", "opb": "Myringotomy", "opc": "Proof puncture", "opd": "Modified radical mastoidectomy", "subject_name": "ENT", "topic_name": "Diagnostic and operative ENT", "id": "0ae83a6a-37d0-4f03-9093-d3dc72a20775", "choice_type": "single"} {"question": "Type of speech seen in nasopharyngeal carcinoma is", "exp": "Denasal speech (rhinolalia clausa) is seen in nasopharyngeal carcinoma. It is lack of nasal resonance for words which are resonated in the nasal cavity Rhinolalia apea (hypernasality- Seen when ceain words which have little nasal resonance are resonated through the nose) Muffled and thick speech, often called \"hot potato voice. (Ref: Diseases of Ear, Nose and Throat, P.L Dhingra, 7th edition, Pg no. 284)", "cop": 1, "opa": "Rhinolalia clausa", "opb": "Rhinolalia apea", "opc": "Hot potato voice", "opd": "Hoarse voice", "subject_name": "ENT", "topic_name": "Pharynx", "id": "906b4a52-e5e7-4991-a724-075097b15236", "choice_type": "single"} {"question": "Steeple sign is seen in case of", "exp": "X-ray finding in Acute LaryngotracheobronchitisIt is an inflammatory condition of the larynx, trachea and bronchi.Mostly, it is a viral infection (parainfluenza type I and II) affecting children between 6 months and 3 years of age. - Thumb sign is seen in Acute epiglottitisDhingra 6e pg: 290", "cop": 4, "opa": "Acute epiglottitis", "opb": "Diphtheria", "opc": "Acute tonsillitis", "opd": "Acute Laryngo tracheo bronchitis", "subject_name": "ENT", "topic_name": "Miscellaneous ENT", "id": "52d877d0-9c1c-4294-9162-efdcc9e3cf9a", "choice_type": "single"} {"question": "Modified radical neck dissection type II", "exp": null, "cop": 2, "opa": "Removal of level I - V lymph nodes with only spinal accessory nerve", "opb": "Removal of level I - V lymph nodes with sternocleidomastoid muscle", "opc": "Removal of level I - V lymph nodes with spinal accessory nerve and internal jugular vein", "opd": "Removal of level I - V lymph nodes with sternocleidomastoid muscle and internal jugular vein", "subject_name": "ENT", "topic_name": null, "id": "1df72097-53c9-41dc-b085-fe0ef119d3b1", "choice_type": "single"} {"question": "In sensorineural hearing loss, weber's test is lateralized to", "exp": "In Sendeafness deafness: Lateralised to better ear. In conductive deafness: Lateralised to worse ear. Ref: Dhingra; 6th Edition; pg no 22", "cop": 1, "opa": "Normal ear", "opb": "Defective ear", "opc": "Not lateralized", "opd": "May alternate", "subject_name": "ENT", "topic_name": "Ear", "id": "ed6279fd-3858-442f-af37-5e46d90f4f39", "choice_type": "single"} {"question": "Vocal cord is lined by", "exp": "MUCOUS MEMBRANE OF THE LARYNX It lines the larynx and is loosely attached except over the posterior surface of epiglottis, true vocal cords and corniculate and cuneiform cailages. Epithelium of the mucous membrane is ciliated columnar type except over the vocal cords and upper pa of the vestibule where it is stratified squamous type. Mucous glands are distributed all over the mucous lining and are paicularly numerous on the posterior surface of epiglottis, posterior pa of the aryepiglottic folds and in the saccules. There are no mucous glands in the vocal folds. Ref:- Dhingra; pg num:-285", "cop": 2, "opa": "Stratified columnar epithelium", "opb": "Stratified squamous epithelium", "opc": "Cuboidal epithelium", "opd": "Pseudociliated columnar epithelium", "subject_name": "ENT", "topic_name": "Larynx", "id": "06d144ac-9e97-4cdf-a0d3-5f86df371945", "choice_type": "single"} {"question": "Primary receptor cells of hearing is", "exp": "Hair cells: are impoant receptor cells of hearing and transduce sound energy into electrical energy.", "cop": 4, "opa": "Suppoing cell", "opb": "Tectorial membrane", "opc": "Tunnel of Coi", "opd": "Hair cell", "subject_name": "ENT", "topic_name": null, "id": "7414156f-aa5c-40a3-aaba-031e071135a8", "choice_type": "single"} {"question": "Most common extracranial complication of ASOM is", "exp": "Pus may break through mastoid coex leading to subperiosteal abscess which may even burst on surface leading to a discharging fistula It occurs as a sequence of mastoiditis Ref: Dhingra 7e pg 84.", "cop": 3, "opa": "Facial nerve paralysis", "opb": "Lateral sinus thrombosis", "opc": "Subperiosteal abscess", "opd": "Brain abscess", "subject_name": "ENT", "topic_name": "Ear", "id": "a447055c-444e-488b-862f-87cad48c69ac", "choice_type": "single"} {"question": "After tonsillectomy, secondary haemorrhage occurs", "exp": "After tonsillectomy primary haemorrhage occurs during surgery reactionary bleeding with in 24 hrs, secondary haemorrhage between 5-10 days Secondary haemorrhage is the result of sepsis and premature separation of the membrane Simple measures like removal of clots, application of adrenaline or hydrogen peroxide usually suffice. For profuse bleeding, electrocoagulation is done Re: Textbook of Ear, Nose and Throat, Dhingra, 6th Edition; Pg no: 430", "cop": 3, "opa": "Within 24 hours", "opb": "After 2 weeks", "opc": "5-10 post operative days", "opd": "After a month", "subject_name": "ENT", "topic_name": "Diagnostic and operative ENT", "id": "354668a5-888a-4588-bbf6-5990525272f3", "choice_type": "single"} {"question": "Most common nerve injured in ligation of inferior thyroid aery", "exp": "Recurrent laryngeal nerve may pass anterior, posterior or through the branches of the aery and care is taken to preserve it. RLN is injured near the ligament of Berry when cautery or aery forceps is used indiscriminately while cutting the ligament of Berry to free the thyroid lobe. Ref: Dhingra 7e pg 382.", "cop": 1, "opa": "Recurrent laryngeal nerve", "opb": "Facial nerve", "opc": "Mandibular nerve", "opd": "Audirory nerve", "subject_name": "ENT", "topic_name": "Diagnostic and operative ENT", "id": "519a9d2c-c1e9-4684-b3a5-59ce07c36571", "choice_type": "single"} {"question": "Ca Wheel sign is seen in", "exp": "In a stage of pre-suppuration of ASOM, there is congestion of pars tensa. A leash of blood vessels appears along with the handle of malleus and at the periphery of tympanic membrane impaing it a ca-wheel appearance. Later, whole tympanic membrane including pars flaccid becomes uniformly red. Ref: Textbook of diseases of ENT, PL Dhingra, 7th edition, pg no. 67", "cop": 1, "opa": "ASOM", "opb": "AOM", "opc": "OME", "opd": "CSOM", "subject_name": "ENT", "topic_name": "Ear", "id": "0af6471b-6d90-4eca-92c9-03b510abe19d", "choice_type": "single"} {"question": "Congenital larygeal stridor also know as", "exp": null, "cop": 2, "opa": "Laryngeal web", "opb": "Laryngomalocia", "opc": "Laryngeal atenosis", "opd": "Vocal cord palsy", "subject_name": "ENT", "topic_name": null, "id": "0a23a2b4-6d4c-4c8b-a3e3-2be96d49c72d", "choice_type": "single"} {"question": "Mucormycosis of paranasal sinus is most common in", "exp": "MUCORMYCOSIS: Caused by - rhizopus , rhizomucor , cunninghamella Moulds with non septate hyphae The higher prevalence of mucormycosis in India turned out to be statistically significant (p < 0.0001) in comparison with all other countries Predisposing factors - Organ transplant recipients - Long term desferroxamine therapy - Immunosuppression due to steroids or cytotoxic drugs Hematological malignancy - Diabetics - Chronic renal failure Five forms Rhinocerebral (most common site) Pulmonary mucormycosis (2' most common) Cutaneous Gastrointestinal Disseminated Diagnosis: Microscopy and biopsy show organisms that appears as broad ribbon like usually non septate hyphae which branch at right angles", "cop": 2, "opa": "HIV", "opb": "Diabetes", "opc": "Those on immuno suppressants", "opd": "Post surgical", "subject_name": "ENT", "topic_name": null, "id": "b65ca73f-bca9-45cc-9cb3-6a9934924ccb", "choice_type": "single"} {"question": "Stage II laryngeal carcinoma treated by", "exp": null, "cop": 1, "opa": "Radiation", "opb": "Surgery", "opc": "Chemo therapy", "opd": "Neo adjuvant chemotherapy and then surgery", "subject_name": "ENT", "topic_name": null, "id": "954ba0d2-9b6f-4799-8f4a-1345201cd386", "choice_type": "single"} {"question": "Retraction of tympanic membrane touching the promontory. It is", "exp": "Retraction of tympanic membrane touching the promontory is called atelectasis. When the tympanic membrane is thin and atrophic a segment of it or the entire membrane may collapse due to eustachian tube insufficiency. It may get plastered onto a promontory and also wrap around the ossicles. Stage Ill is also called as middle ear atelectasis. Tympanic membrane comes to lie on the promontory and ossicles. Middle ear space is totally or paially obliterated but middle ear mucosa is intact. The tympanic membrane can be lifted from the promontory with suction tip. It also balloons up when N2O is used during anaesthesia. The tympanic membrane is thin because it's collagenous middle layer had been absorbed due to prolonged retraction. In these cases, long process of the incus and stapes superstructure are absorbed. Placement of the ventilation tube helps to restore the position of the tympanic membrane. Ref: PL Dhingra; 7th edition of Ear, Nose and Throat; Pg no 79", "cop": 3, "opa": "Mild retraction", "opb": "Severe retraction", "opc": "Atelectasis", "opd": "Adhesive otitis", "subject_name": "ENT", "topic_name": "Ear", "id": "310e179c-f60d-419b-a059-5702c63d7625", "choice_type": "single"} {"question": "Following total thyrordectory the patient duclops respiratory stridor cause is", "exp": null, "cop": 1, "opa": "B/L recurrent laryngeal nerve palsy", "opb": "B/L complete palsy", "opc": "U/L recurent laryngeal nerve palsy", "opd": "U/L complete palsy", "subject_name": "ENT", "topic_name": null, "id": "b947abc7-8d7c-4588-beb2-20887c70c7c0", "choice_type": "single"} {"question": "Cochlear microphonic potentials exhibits", "exp": "i.e. (Produced by outer hair cells):COCHLEAR MICROPHONIC (CM) - when basilar membrane moves in response to sound stimulus, electrical resistance at the tip of hair cells changes allowing flow of K+ through hair cells and produces voltage fluctuations called cochlear microphonic. It is an alternating current (AC) potentialsElectrical Potentials of Cochlea and CN VIIIFROM COCHLEA1. Endocochear potentials - It is direct current (DC) potential recorded from scala media. It is +80 mV and is generated from the stria vascularis by Na+ / K+ ATPase pump and provides source of energy for cochlear transduction. It is present at rest and does not require sound stimulus. This potential provides a sort of \"battery\" to drive the current through hair cells when they move in response to a sound stimulus.2. (Cochlear microphonic (CM)3. Summating potentials (SP) - It is a DC potential and follows \"envelop\" of stimulating sound It is produced by hair cells. It may be negative or positive. SP has been used in diagnosis of Meniere's disease. It is superimposed on VIII nerve lesionsFROM NERVE FIBRERS (CN VIII)4. Compound action potential - It is an all or none response of auditory nerve fibresBoth CM and SP are receptor potentials as seen in other sensory end organs - The differ from action potentials in that(a) They are graded rather than all or none phenomenon(b) Have no latency(c) Are not propagated(d) Have no post response refractory period", "cop": 1, "opa": "Produced by outer hair cells", "opb": "Produced by inner hair cells", "opc": "Electrodes have to be inserted over round window", "opd": "Surface electrodes can not measure it", "subject_name": "ENT", "topic_name": "Ear", "id": "b2e9c1ab-efa9-488e-b1ff-09dc981c678c", "choice_type": "single"} {"question": "In infants most sensitive audiometric screening is", "exp": "BERA is the investigation of choice for congenital deafness in children.OAE (otoacoustic emissions) is the commonest screening test in neonates.Electrocochleography- Used to measure the electrical activity of the cochlea, commonly used to diagnose Meniere&;s disease. Ref: Hazarika; 3rd ed; Pg 64", "cop": 2, "opa": "Electrocochleography", "opb": "BERA", "opc": "Coical evoked response", "opd": "Tympanometry", "subject_name": "ENT", "topic_name": "Ear", "id": "c70605e4-6b9a-4246-bb3d-3a42b70f992e", "choice_type": "single"} {"question": "Presenting symptom of nasal myiasis is", "exp": "Clinical features of nasal myiasisInitial symptoms (3-4 days maggots):- Intense irritation, sneezing, headache, blood stained discharge, lacrimationLater:- Maggots may crawl out of the nose and there is a foul smellRef: PL Dhingra, Diseases of Ear, Nose & Throat, 7th edition, pg no. 181 - 182", "cop": 1, "opa": "Severe irritation with sneezing", "opb": "Maggots", "opc": "Nasal pain", "opd": "Impaired olfaction", "subject_name": "ENT", "topic_name": "Nose and paranasal sinuses", "id": "67efd388-c8b7-49b8-b011-3f12f5a81805", "choice_type": "single"} {"question": "Treatement of choice of vocal nodule is", "exp": "Ans. is 'b' i.e. Micro laryngoscopic removal Vocal Nodules (Also k/a singers nodule)Bilateral*Appear symmetrically on the free edge of vocal card at the junction of anterior one - third with the posterior 2/3* as this is the area of maximum vibration of the cord and thus subject to maximum trauma. They are seen in people or children who misuse their voice (Speak too much or speak in unnatural low tones)Symptoms:* Hoarseness of voice, vocal fatigue, pain in neck. t/t:-Small nodules and early cases can be tit conservatively by education of patient in proper use of voice.Large nodules or long standing nodules - Micro laryngoscopic surgery*.", "cop": 2, "opa": "Radial excision", "opb": "Microlaryngoscopic removal", "opc": "Cryotherapy", "opd": "Wait and watch", "subject_name": "ENT", "topic_name": "Larynx", "id": "48b8ddcb-c85b-4864-91fe-76f175d8e947", "choice_type": "single"} {"question": "Endolymph resembles", "exp": "PerilymphEndolymphBony labyrinthMembranous labyrinthFormed from CSFSecreted by stria vascularisResembles ECFResembles ICFNa+ richK+ richReabsorbed into subarachnoid spaceReabsorbed into endolymphatic sacRef: Hazarika; 3rd ed; Pg 24", "cop": 2, "opa": "CSF", "opb": "ICF", "opc": "ECF", "opd": "Plasma", "subject_name": "ENT", "topic_name": "Ear", "id": "602aef4c-0adf-4ad2-94e0-e584622364d5", "choice_type": "single"} {"question": "Acoustic neuroma commonly arises from", "exp": "Acoustic neuroma commonly arises from vestibular nerve and rarely from the cochlear division of the 8th nerve. Ref: Textbook of diseases of Ent, PL Dhingra, 7th edition, pg. 125", "cop": 1, "opa": "Superior division of vestibular nerve", "opb": "Inferior division of vestibular nerve", "opc": "Cochlear nerve", "opd": "VIIth nerve", "subject_name": "ENT", "topic_name": "Ear", "id": "8c58bcc6-36b5-40d9-ad4b-fcfd22f57eb0", "choice_type": "single"} {"question": "In jarjaway fracture of nasal bone, the fracture line is", "exp": "Jarjaway' fracture of nasal septum occurs due to blows from the front. Fracture line stas just above the anterior nasal spine and runs horizontally backwards just above the junction of septal cailage with the vomer. Ref: Textbook of ENT; PL Dhingra;7th edition, pg no.165", "cop": 4, "opa": "Oblique", "opb": "Comminuted", "opc": "Veical", "opd": "Horizontal", "subject_name": "ENT", "topic_name": "Nose and paranasal sinuses", "id": "5e4904dc-3023-4861-8cc6-5407e28199f4", "choice_type": "single"} {"question": "The most common site of perforation in the Tympanic membrane is(REPEAT QUESTION)", "exp": "In the stage of resolution, usually, a small perforation is seen in the anteroinferior quadrant which is commonest of all quadrants of pars tensa. (Ref: Textbook of diseases of ENT, PL Dhingra, pg no. 68)", "cop": 1, "opa": "Antero inferior", "opb": "Postero inferior", "opc": "Postero superior", "opd": "Antero superior", "subject_name": "ENT", "topic_name": "Ear", "id": "3a3e39d0-7dba-4207-af17-c81ce5495f02", "choice_type": "single"} {"question": "Carcinoma known for perineural invasion is", "exp": "ADENOID CYSTIC CARCINOMA (CYLINDROMA) It is a slow-growing tumour but infiltrates widely into the tissue planes and muscles. It also invades perineural spaces and lymphatics and thus causes pain and VIIth nerve paralysis. It can metastasize to lymph nodes. Local recurrences after surgical excision are common and can occur as late as 10-20 years after surgery. Distant metastases go to the lung, brain and bone. Treatment is radical parotidectomy with largest cuff of grossly normal tissue around the boundaries of the tumour. Radical neck is not done unless nodal metastases are present. Postoperative radiation is given if margins of the resected specimen are not free of tumour. 2nd most common Malignancy of salivary glands after mucoepidermoid carcinoma Advanced tumours may present with pain or nerve paralysis, as it has a propensity to invade peripheral nerves Histology of Adenoid cystic carcinoma Ref : Diseases of ENT by Dhingra 6th edition Pgno : 235", "cop": 4, "opa": "Pleomorphic adenoma", "opb": "Squamous cell carcinoma", "opc": "Mucoepidermoid carcinoma", "opd": "Adenoid cystic carcinoma", "subject_name": "ENT", "topic_name": "Nose and paranasal sinuses", "id": "10aa2514-bf01-4179-84c4-b21fafe6731a", "choice_type": "single"} {"question": "TESPAL is used in management of", "exp": "Tespal: (Trans nasal endoscopic sphenopalatine aery ligation) Indication: Epistaxis not responding to conventional conservative management. Posterior epistaxis Complications of TESPAL: 1. Palatal numbness 2. Sinusitis 3. Decreased lacrimation 4. Septal perforation 5. Inferior turbinate necrosis Ref otolaryngology dhingra 3e", "cop": 2, "opa": "CSOM", "opb": "Epistaxis", "opc": "Ulcerative tonsilitis", "opd": "Ca larynx", "subject_name": "ENT", "topic_name": "Ear", "id": "b70e152b-da87-4972-bb59-c563a44ff291", "choice_type": "single"} {"question": "The most common cause of recurrent epistaxis in a 15 year old female is", "exp": "Causes for recurrent epistaxis in the female child are haematological disorders like Haemophilia, Christmas disease, Vit. K deficiency, Scurvy Leukaemia, Aplastic anaemia, Thrombocytopenic and vascular purpura", "cop": 4, "opa": "Juvenile nasopharyngeal fibroma", "opb": "Rhinosporidiosis", "opc": "Foreign body", "opd": "Hematological disorder", "subject_name": "ENT", "topic_name": "Nose and paranasal sinuses", "id": "9095786d-a3f7-45b6-bbd3-93faa4783f5b", "choice_type": "single"} {"question": "Crypta Magna is the remnant of", "exp": "Medial surface of each tonsil has 15-20 crypts, the largest of which is called Intratonsillar cleft or crypta Magna (which represents the persistence of the ventral poion of the second pharyngeal pouch)'.Ref: PL Dhingra, Diseases of Ear, Nose & Throat, 7th edition, pg no. 291", "cop": 2, "opa": "First Pharyngeal pouch", "opb": "Second Pharyngeal pouch", "opc": "Third pharyngeal pouch", "opd": "Fouh pharyngeal pouch", "subject_name": "ENT", "topic_name": "Oral cavity & Oesophagus", "id": "f929aff2-b8c2-44f7-aebe-c02459ae88e5", "choice_type": "single"} {"question": "The only abductor of vocal cord is", "exp": "Ans. a (Posterior cricoarytenoid) (Ref. Diseases of ENT by PL Dhingra 2nd ed., 337)The posterior cricoarytenoid is the only true abductor of the vocal folds and is principally responsible for control of the glottic airway.ActionMusclesAbductor of Vocal cord-Posterior cricoarytenoidAdductor of Vocal cord-Lateral cricoarytenoid, Inter-arytenoid, Cricothyroid, ThyroarytenoidTensor of Vocal cord -Cricothyroid, VocalisOpening of laryngeal inlet -ThyroepiglotticusClosing of laryngeal inlet-Interarytenoid, AryepiglotticusAlso Know:Type of thyroplastyMeaningUseIMedialization of vocal cordCombined (RLN and superior laryngeal N) palsyIILateralization of vocal cordBilateral RLN paralysis to improve airwayIIIShortening of Vocal cordFemale -male voice (For androphonia)IVLengthening of Vocal cordMale -female voice. Also used when vocal cord is lax.", "cop": 1, "opa": "Posterior cricoarytenoid", "opb": "Lateral cricoarytenoid", "opc": "Interarytenoid", "opd": "Cricothyroid", "subject_name": "ENT", "topic_name": "Larynx", "id": "41522924-6fc3-4b1a-8998-04c257965619", "choice_type": "single"} {"question": "12 years old presents with fever, unilateral post auricular pain, mastoid bulging and displacing the pinna forward outwards with loss of bony trabeculae. This patient has a history of chronic persistent pus discharge from the same ear. Treatment of choice is", "exp": "The patient is presenting with features of the postauricular subperiosteal abscess. Treatment for this is antibiotics along with drainage of the abscess and coical mastoidectomyThe patients have developed this abscess as a complication of CSOM (History of chronic ear discharge) for which he requires mastoidectomyRef: PL Dhingra, Diseases of Ear, Nose & Throat, 7th edition, pg no. 86 - 87", "cop": 4, "opa": "Antibiotics only", "opb": "Incision and drainage", "opc": "Antibiotics and incision and drainage", "opd": "Mastoidectomy with incision and drainage and antibiotics", "subject_name": "ENT", "topic_name": "Nose and paranasal sinuses", "id": "175e7818-fd50-488e-9c16-47fa8a8bf3cb", "choice_type": "single"} {"question": "Suprastructure of stapes develops from", "exp": "Mesoderm of 2nd Branchial arch.", "cop": 3, "opa": "Ectoderm", "opb": "Endoderm", "opc": "Mesoderm", "opd": "Neuro Ectoderm", "subject_name": "ENT", "topic_name": null, "id": "7c2da9a9-d701-4252-833f-4d8dffc19f4d", "choice_type": "single"} {"question": "The water cane in the larynx (saccules) are present in", "exp": "Ventricle (sinus of larynx):- It is a deep elliptical space between vestibular and vocal folds, also extending a sho distance above and lateral to vestibular fold. The saccule is a diveiculum of mucous membrane which stas from the anterior pa of ventricular cavity and extends upwards between vestibular folds and lamina of thyroid cailage. When abnormally enlarged and distended, it may form a laryngocele--an air containing sac which may present in the neck. There are many mucous glands in the saccule, which help to lubricate the vocal cords. Ref:- Dhingra; pg num:-284", "cop": 1, "opa": "Laryngeal ventricles", "opb": "Reinke's space", "opc": "Paraglottic space", "opd": "Pyriform fossa", "subject_name": "ENT", "topic_name": "Larynx", "id": "30bb2c6d-fa2b-4bef-bebf-804b6091515c", "choice_type": "single"} {"question": "Treatment of choice of Carcinoma maxillary sinus with T3NOMO", "exp": null, "cop": 3, "opa": "Surgery", "opb": "Radiotherapy", "opc": "Surgery and Radiotherapy", "opd": "Surgery and chemotherapy", "subject_name": "ENT", "topic_name": null, "id": "fe018f8f-7a0c-4367-93f8-c822b8fd16e8", "choice_type": "single"} {"question": "A patient is suspected to have Vestibular Shwanomma. The investigation of choice for its diagnosis is", "exp": "(B) Gadolinium enhanced MRI # Neurological tests. Complete examination of cranial nerves, cerebellar functions, brainstem signs of pyramidal and sensory tracts should be done. Fundus is examined for blurring of disc margins or papilloedema.> Radiological tests, (a) Plain x-rays (transorbital, Stenver's, Towne's and submentovertical views) give positive findings in 80% of patients. However, small intracanalicular tumours are not detected.> CT scan. A tumour that projects even 0.5 cm into the posterior fossa can be detected by a CT scan. If combined with intrathecal air, even the intrameatal tumour can be detected.> CT scan has replaced earlier methods of pneumoencephalography and myodil meatography.> MRI with gadolinium contrast. It is superior to CT scan and is the gold standard for diagnosis of acoustic neuroma. Intracanalicular tumour, of even a few millimeters, can be easily diagnosed by this method.> Vertebral angiography. This is helpful to differentiate acoustic neuroma from other tumours of cerebellopontine angle when doubt exists.> Electric response audiometry. It is very useful in the diagnosis of retrocochlear lesions.> In the presence of VINth nerve tumour, a delay of >0.2 msec in wave V between two ears is significant.> CSF examination. Protein level is raised. Lumbar puncture is usually avoided.", "cop": 2, "opa": "Contrast enhanced CT scan", "opb": "Gadolinium enhanced MR1", "opc": "SPECT", "opd": "PET scan", "subject_name": "ENT", "topic_name": "Miscellaneous (E.N.T.)", "id": "3dfb2501-4a7d-4099-8f65-940711c2a1fc", "choice_type": "single"} {"question": "Commonest cause of acute otitis media in children is", "exp": "Ans. is 'b' i.e Streptococcus pneumoniae Aetiology of ASOM in children:S. pneumoniae (35%)H. influenzae (25%)M. Catarrhalis (15%)Aetiology in adultsH. influenza (25%)*S. pneumonia (21%)", "cop": 2, "opa": "H. inflenzae", "opb": "Strepto-pneumoniae", "opc": "Staph aureus", "opd": "Pseudomonas", "subject_name": "ENT", "topic_name": "Ear", "id": "c784815a-533c-4502-8e21-7aeafa1688e2", "choice_type": "single"} {"question": "Masculine voice in females is treated by", "exp": "The masculine voice in females is treated by Thyroplasty type 3. It is used to shoen (relax) the vocal cord. Relaxation of vocal cord lowers the pitch. This procedure is done in mutational falsetto or in those who have undergone gender transformation from female to male. Ref: PL Dhingra Textbook of Ear, Nose and Throat, Edition 6, page - 302", "cop": 3, "opa": "Thyroplasty type 1", "opb": "Thyroplasty type 2", "opc": "Thyroplasty type 3", "opd": "Thyroplasty type 4", "subject_name": "ENT", "topic_name": "Larynx", "id": "823672fa-e0e2-4ddc-afc5-cd3646bd0cb8", "choice_type": "single"} {"question": "CHARGE syndrome does not include", "exp": null, "cop": 1, "opa": "Renal abnormalities", "opb": "Coloboma of eye", "opc": "Heart abnormalities", "opd": "Ear abnormalities", "subject_name": "ENT", "topic_name": null, "id": "8ec3e80b-e6d8-4ba9-8bde-d593903c0982", "choice_type": "single"} {"question": "\"Rising sun\" appearance is seen in", "exp": "Rising sun appearance is seen when a glomus tumour arises from the floor of the middle ear in otoscopy. Sometimes, the tympanic membrane appears bluish and maybe bulging. Ref: Dhingra 7th edition pg. 121", "cop": 3, "opa": "ASOM", "opb": "CSOM", "opc": "Glomus tumor", "opd": "Acoustic neuroma", "subject_name": "ENT", "topic_name": "Ear", "id": "d9d09b4d-5e64-4378-be8f-ee990efe362e", "choice_type": "single"} {"question": "Frog face deformity of nose caused by", "exp": "EXTENSIONS OF NASOPHARYNGEAL FIBROMA Nasopharyngeal fibroma is a benign tumour but locally invasive and destroys the adjoining structures. It may extend into: 1. Nasal cavity causing nasal obstruction, epistaxis and nasal discharge. 2. Paranasal sinuses. Maxillary, sphenoid and ethmoid sinuses can all be invaded. 3. Pterygomaxillary fossa, infratemporal fossa and cheek. 4. Orbits giving rise to proptosis and \"frog-face deformity.\" It enters through the inferior orbital fissure and also destroys apex of the orbit. It can also enter the orbit through superior orbital fissure. 5. Cranial cavity. It can extend into: (a) Anterior cranial fossa through roof of ethmoids or cribriform plate. (b) Middle cranial fossa through erosion of floor of middle cranial fossa or indirectly by invading the sphenoid sinus and sella turcica. In the former case, tumour lies lateral to internal carotid aery and in the latter case medial to the aery. Ref:- Dhingra; pg num:-246", "cop": 4, "opa": "Antral polyp", "opb": "Rhinoscleroma", "opc": "Ethmoidal polyp", "opd": "Angiofibroma", "subject_name": "ENT", "topic_name": "Pharynx", "id": "6a800b3f-9f9f-4fe4-a5ac-90387db05c8b", "choice_type": "single"} {"question": "Superior sagittal sinus thrombosis causes", "exp": "(D) Otitis hydrocephalus> Adequacy of collateral circulation in the hand Extracranial Complications of Otitis Media: Petrositis# Intracranial Complications of Otitis Media> Extradural abscess> Lateral sinus thrombosis> Otitis hydrocephalus> Meningitis> Brain abscess: temporal lobe; cerebellum> Subdural abscess> Cortical thrombophlebitis# Lateral Sinus Thrombophlebitis (Sigmoid Sinus Thrombosis)> It is an inflammation of inner wall of lateral venous sinus with formation of a thrombus.> Thrombosis of the sigmoid sinus is always secondary to infection in the middle ear or otitis media, or in the mastoid process called the mastoiditis.> During operations on the mastoid process one should be careful about the sigmoid sinus, so that it is not exposed.> Spread of infection or thrombosis from the sigmoid and transverse sinuses to the superior sagittal sinus may cause impaired CSF drainage into the latter and may, therefore, lead to the development of hydrocephalus,> Such a hydrocephalus associated with sinus thrombosis following ear infection is known as otitis hydrocephalus.", "cop": 4, "opa": "Petrositis", "opb": "Meningitis", "opc": "Cortical thrombophlebitis", "opd": "Otitis hydrocephalus", "subject_name": "ENT", "topic_name": "Miscellaneous (E.N.T.)", "id": "40e9c28c-a604-44f2-bced-ec4d3f8e999c", "choice_type": "single"} {"question": "Positive head impulse test is suggestive of", "exp": "The HIT test is a bedside technique used to diagnose reduction in vestibular function in one ear versus the other. It describes the result of having an examiner abruptly accelerate and then decelerate the head, moving the head rapidly at high speed and then stopping it. Positive HIT is suggestive of injury to the peripheral vestibular nerve.", "cop": 2, "opa": "Injury to vestibular nuclei", "opb": "Injury to peripheral vestibular nerve", "opc": "Lesion in the brain stem", "opd": "Injury to Occulomotor nerve", "subject_name": "ENT", "topic_name": "Ear", "id": "1af209fc-d038-4fa6-9fcc-6efaf42fe8a5", "choice_type": "single"} {"question": "In acoustic neuroma, cranial nerve to be involved earliest is", "exp": "The earliest cranial nerve to be involved is V nerve causing reduced corneal sensitivity. Ref: Textbook of diseases of ENT, PL Dhingra, 7th edition, pg no. 125", "cop": 1, "opa": "5", "opb": "7", "opc": "9", "opd": "10", "subject_name": "ENT", "topic_name": "Ear", "id": "5739b6ea-01cd-4ae2-b509-5744e8bac9d9", "choice_type": "single"} {"question": "Glomus tumor invading the veical pa of carotid canal. It is", "exp": "Ans. is 'c' i.e., Type C2 Fisch classification The Fisch classification of glomus tumors is based on extension of the tumor to surrounding anatomic structures and is closely related to moality and morbidity. Type A :- Limited to middle ear cleft (glomus tympanicum). Type B :- Limited to tympanomastoid area with no involvement of infralabyrinthine compament. Type C :- Involving infralabrinthine compament extending upto petrous apex Type C1 :- Limited involvement of veical poion of carotid canal Type C2 :-Invading veical poion of carotid canal Type C3 :-Invasion of horizontal poion of carotid canal Type D Intracranial extension Type D1 Intracranial extension < 2 cm in diameter Type D2 :-Intracranial extension > 2 cm in diameter", "cop": 3, "opa": "Type B", "opb": "Type CI", "opc": "Type C2", "opd": "Type C3", "subject_name": "ENT", "topic_name": null, "id": "dcd8a784-47b3-4a05-ab4c-421698bbf999", "choice_type": "single"} {"question": "Test of detecting damage to cochlea is", "exp": "Bone conduction is a measure of cochlear function. In ABC test: Patient's bone conduction is compared with that of the examiner (presuming that the examiner has normal hearing). In conductive deafness, the patient and the examiner hear the fork for the same duration of time. In sensorineural deafness, the patient hears the fork for a shoer duration. Ref: Dhingra; 6th Edition; pg no 22", "cop": 4, "opa": "Caloric test", "opb": "Weber test", "opc": "Rinnie's test", "opd": "ABC test", "subject_name": "ENT", "topic_name": "Ear", "id": "36799227-84cd-41b9-abaa-407197db7a8f", "choice_type": "single"} {"question": "Most common pa of larynx involved in Tuberculosis is", "exp": "Tubercular laryngitisThe disease affects the posterior third of larynx more commonly than anterior paThe pas affected in descending order of frequency are:- i) Interarytenoid fold, ii) Ventricular band, iii) Vocal cords, iv) EpiglottisRef: PL Dhingra, Diseases of Ear, Nose & Throat, 7th edition, pg no. 329 - 330", "cop": 2, "opa": "Anterior", "opb": "Posterior", "opc": "Middle", "opd": "Anywhere", "subject_name": "ENT", "topic_name": "Larynx", "id": "b84fe3a2-69ff-4a57-806e-11050a3d7c30", "choice_type": "single"} {"question": "Most definitive diagnosis of sinusitis is", "exp": "Sinuscopy or Diagnostic nasal endoscopy is the most appropriate investigation.The classical feature is the presence of mucopus in the middle meatus or above the middle meatusA more appropriate investigation would be CT scan of the nose and PNS.Ref: Hazarika; 3rd ed; Pg 331", "cop": 3, "opa": "X-ray PNS", "opb": "Proof puncture", "opc": "Sinuscopy", "opd": "Transillumination test", "subject_name": "ENT", "topic_name": "Nose and paranasal sinuses", "id": "79202c53-4f02-49ab-ac97-475b1219e301", "choice_type": "single"} {"question": "The commonest cause of acute otitis media in children is", "exp": "The commonest cause of Acute otitis media is Streptococcus pneumoniae in all age groups.Other agents- H. Influenzae, Moraxella catarrhalis, and Pseudomonas.Ref: Hazarika; 3rd ed; Pg: 149", "cop": 2, "opa": "H influenza", "opb": "S pneumonia", "opc": "S aureus", "opd": "Pseudomonas", "subject_name": "ENT", "topic_name": "Ear", "id": "da12f59e-3afb-44a4-8a1e-1107246d6fd3", "choice_type": "single"} {"question": "First lymph node involved in maxillary carcinoma", "exp": "In paranasal sinus tumors\n\nLymphatic spread: Nodal metastases are uncommon and occur only in the late stages of disease. Submandibular and upper jugular nodes are enlarged. Maxillary and ethmoid sinuses drain primarily into retropharyngeal nodes, but these nodes are inaccessible to palpation.\nSystemic metastases are rare. May be seen in the lungs (most commonly) and occasionally in bone.\nIntracranial spread can occur through ethmoids, cribriform plate or foramen lacerum.", "cop": 2, "opa": "Submental", "opb": "Submandibular", "opc": "Clavicular", "opd": "Jugular", "subject_name": "ENT", "topic_name": null, "id": "50b3aa2a-3180-4f22-a3bd-96e1dcac3d58", "choice_type": "single"} {"question": "Most common congenital anomaly of larynx is", "exp": "Laryngomalacia (congenital laryngeal stridor): It is the most common congenital abnormality of the larynx. It is characterized by laryngeal excessive flaccidity of supraglottic region which is sucked in during inspiration producing stridor and sometimes cyanosis. Stridor is increased on crying but subsides on placing the child in the prone position. The cry is normal. This condition manifests at bih or soon after, and usually disappears by 2 years of age. Direct laryngoscopy shows elongated epiglottis, curled upon itself (omega-shaped O), floppy aryepiglottic folds and prominent arytenoids. A flexible laryngoscope is very useful to make the diagnosis. Laryngomalacia cannot be diagnosed in a paralyzed patient. Mostly, treatment is conservative. Tracheostomy may be required for some cases of severe respiratory obstruction. Supraglottoplasty is required in cases of severe laryngomalacia. Ref: PL Dhingra 7th edition of Ear, Nose and Throat, pg no. 333", "cop": 2, "opa": "Laryngeal web", "opb": "Laryngomalacia", "opc": "Laryngeal stenosis", "opd": "Vocal cord Palsy", "subject_name": "ENT", "topic_name": "Larynx", "id": "d75accae-d489-47e5-90e7-429b4c0c5bb2", "choice_type": "single"} {"question": "Pseudomembrane over tonsil and pharynx is caused by", "exp": "Pseudomembrane is formed in diphtheria, which is caused by Corynebacterium diphtheria, which is a gram-positive bacillus Ref: Harshmohan textbook of pathology; 7th edition", "cop": 1, "opa": "Gram positive bacili", "opb": "Gram negative bacili", "opc": "SS RNA virus", "opd": "Catalase negative cocci", "subject_name": "ENT", "topic_name": "Pharynx", "id": "746803cb-0401-4353-8475-8f0a76fbfce6", "choice_type": "single"} {"question": "Infection of submandibular space is seen in", "exp": "Ludwig's angina is an infection of submandibular space. AETIOLOGY 1. Dental infections. They account for 80% of the cases. Roots of premolars often lie above the attachment of mylohyoid and cause sublingual space infection while roots of the molar teeth extend up to or below the mylohyoid line and primarily cause submaxillary space infection 2. Submandibular sialadenitis, injuries of oral mucosa and fractures of the mandible account for other cases (Ref: Diseases of ear, nose and throat, P.L Dhingra, 7th edition, page 297)", "cop": 1, "opa": "Ludwig's angina", "opb": "Vincent's angina", "opc": "Prinzmetal's angina", "opd": "Unstable angina", "subject_name": "ENT", "topic_name": "Pharynx", "id": "5cde728a-1e7a-498c-885b-fe4e3bb14e78", "choice_type": "single"} {"question": "Stapedius muscle is", "exp": null, "cop": 2, "opa": "Unipennate muscle", "opb": "Asymmetric Bipennete", "opc": "Symmetric Bipennete", "opd": "Muscle of Neck region", "subject_name": "ENT", "topic_name": null, "id": "6b7d1e11-8571-417f-bc11-2f10cd4b904a", "choice_type": "single"} {"question": "Frog face deformity of nose is caused by", "exp": "It is due to the extension of nasopharyngeal fibroma into orbit through the inferior and superior orbital fissure and destroys apex of the orbit, giving rise to proptosis and frog face deformity. (Ref: Diseases of ear, nose and throat, PL Dhingra, 7th edition, Pg no. 279)", "cop": 2, "opa": "Rhinoscleroma", "opb": "Angiofibroma", "opc": "Antral polyp", "opd": "Ethmoidal polyp", "subject_name": "ENT", "topic_name": "Pharynx", "id": "eb49dd49-3116-45f0-afc1-5621b9eba340", "choice_type": "single"} {"question": "Adam's apple in Male is due to.", "exp": "Thyroid Cailage Largest cailage, hyaline in nature. It is V shaped and consists of right and left lamina which meet anteriorly in midline and form an angle (Adams angle) Adams angle: -- Male: 90 degree -- Female: 120 degree The outer surface of each lamina is marked by an oblique line which extends from superior thyroid tubercle to inferior thyroid tubercle. Oblique line gives attachment to: -- Thyrohyoid -- Sternothyroid -- Inferior constrictor muscle. Vocal cords are attached to middle of thyroid cailage.", "cop": 1, "opa": "Thyroid cailage", "opb": "Cricoid cailage", "opc": "Hyoid cailage", "opd": "Epiglottis cailage", "subject_name": "ENT", "topic_name": "FMGE 2019", "id": "97c164d8-059f-4e9d-8acf-1239eba5ff2c", "choice_type": "single"} {"question": "Presbycusis is", "exp": "Presbycusis is a sensorineural hearing loss associated with ageing. Manifests at the age of 65years (Ref: Diseases of EAR, NOSE AND THROAT by PL DHINGRA - 6th Edition)", "cop": 2, "opa": "Loss of accommodation power", "opb": "Hearing loss due to ageing", "opc": "Noise induced hearing loss", "opd": "Congenital deafness", "subject_name": "ENT", "topic_name": "Ear", "id": "60284368-e762-42f1-b701-c466f1958d65", "choice_type": "single"} {"question": "The glomus tumour, invasion to the Jugular bub, is diagnosed by", "exp": "(Jugular venography) (214-15-LT) (140-41-LT) (108-D 4,h) (97-Maqbool 11th)Diagnosis of Glomus tumours* Carotid angiography and vertebral angiography will demonstrate the blood supply of the tumour* Jugular venography may demonstrate the extent of tumour invasion into the jugular bulb.MRI- will demonstrate the extent of tumour invasion* CT-scan head - helps to distinguish glomus tympanicum from the glomus jugular tumour by identification of carotico-jugular spine which is eroded in later CT-scan also helps to differentiate it from the aberrant carotid artery, high or dehiscent jugular bulbMRI- MR angiography and venography further help to delineate invasion of jugular bulb and vein or compression of the carotid artery.* Four vessel angiography - useful to found out extent of tumor, compression of internal carotid artery.* Diagnostic biopsy is CONTRAINDICATEDTreatment - smaller tumour are excised. Larger lesions may be treated by radiotherapy followed by surgery", "cop": 4, "opa": "Carotid angiography", "opb": "Vertebral venous angiography", "opc": "X ray", "opd": "Jugular venography", "subject_name": "ENT", "topic_name": "Ear", "id": "b00a463d-7dd7-4150-8768-482a4ba60d27", "choice_type": "single"} {"question": "Cauliflower ear is", "exp": "It is due to collection of blood between the auricular cailage and its perichondrium. If haematoma gets infected, severe perichondritis may set in. Often it is the result of blunt trauma seen in boxers, wrestlers and rugby players. Extravasated blood may clot and then organize, resulting in 'Cauliflower ear'. (Reference: Diseased of Ear, Nose and Throat, PL Dhingra,7th edition, pg no.52)", "cop": 2, "opa": "Keloid", "opb": "Perichondritis in Boxers", "opc": "Squamous cell carcinoma", "opd": "Anaplastic cell carcinoma", "subject_name": "ENT", "topic_name": "Ear", "id": "87c058d6-9e61-4a30-92be-47aec31e6d80", "choice_type": "single"} {"question": "Carha's notch in audiometry is seen in", "exp": "In some cases of otosclerosis, there is a dip in bone conduction curve. It is different at different frequencies but maximum at 2000Hz and is called Carha's notch. (5db at 500Hz, 10db at 1000Hz, 15db at 2000Hz and 5db at 4000Hz) Ref: Diseases of EAR, NOSE and THROAT by PL Dhingra; 7th Edition; Page no.96", "cop": 4, "opa": "Ossicular discontinuity", "opb": "Haemotympanum", "opc": "Otomycosis", "opd": "Otosclerosis", "subject_name": "ENT", "topic_name": "Ear", "id": "1c656b08-9542-4895-8025-df83830a52a3", "choice_type": "single"} {"question": "Drug treatment for laryngeal stenosis is", "exp": "Mitomycin-C is an antineoplastic antibiotic that acts as an alkylating agent by inhibiting DNA and protein synthesis. ... Topical application of mitomycin-C (0.4 mg/mL) was used as an adjuvant treatment in the endoscopic laser management of laryngeal and tracheal stenosis Ref: Internet sources", "cop": 2, "opa": "Adriamycin", "opb": "Mitomycin C", "opc": "Cyclophosphamide", "opd": "Doxorubicin", "subject_name": "ENT", "topic_name": "Larynx", "id": "78287b9c-d2ff-40bd-aa9f-afa70e8d805d", "choice_type": "single"} {"question": "TURBHAN epiglottis is pathognomic of", "exp": "Appears as turban like thickening of epiglottis .", "cop": 1, "opa": "Sacroidosis", "opb": "Wegener's gronulomatosis", "opc": "Histoplasmosis", "opd": "Malignancy", "subject_name": "ENT", "topic_name": null, "id": "22ac19ca-5b47-4ffe-83ba-873c5a1cdac5", "choice_type": "single"} {"question": "Metastasis of carcinoma buccal mucosa goes to", "exp": "The spread of carcinoma of buccal mucosa \n\nBuccal carcinoma spread locally and to regional lymph nodes.\n\n1) Local spread: - From its site of origin, the lesion may spread deeply involving submucosa → muscle → Subcutaneous fat → skin. Involvement of buccinator muscle or anterior masseter causes trismus.\n2) Lymphatic spread: - Nodal involvement occurs in about 50% of cases. Submandibular and later the upper jugular nodes may get involved. Upper jugular nodes may also be involved, directly skipping the submandibular group.", "cop": 1, "opa": "Regional lymph node", "opb": "Liver", "opc": "Heart", "opd": "Brain", "subject_name": "ENT", "topic_name": null, "id": "5798f366-98fc-456c-b425-e0e88379d364", "choice_type": "single"} {"question": "Key nob appearance is seen in", "exp": "PHONASTHENIA It is weakness of voice due to fatigue of phonatory muscles. Thyroarytenoid and interarytenoids or both may be affected. It is seen in abuse or misuse of voice or following laryngitis. Patient complains of easy fatiguability of voice. Clinical examination:- Indirect laryngoscopy shows three characteristic findings: 1. Elliptical space between the cords in weakness of thyroarytenoid. 2. Triangular gap near the posterior commissure in weakness of interarytenoid. 3. Key-hole appearance of glottis when both thyroarytenoid and interarytenoids are involved. Treatment:- Treatment is voice rest and vocal hygiene, emphasizing on periods of voice rest after excessive use of voice. Ref:- Dhingra; pg num:-314", "cop": 3, "opa": "Functional aphonia", "opb": "Puberphonia", "opc": "Phonasthenia", "opd": "Vocal cord paralysis", "subject_name": "ENT", "topic_name": "Larynx", "id": "ee7b19ed-bc5d-42b3-8a86-9096f252a8df", "choice_type": "single"} {"question": "Shape of septal cartilage of nose is", "exp": "Ans. is 'c' i.e., Quadrilateral * The septal cartilage of the nasal septum is a quadrilateral cartilage.\"Cartilage of the septum, also known as the quandragular cartilage because it is roughly quadrilateral in shape - separates the nostrils\".Medial nasal wall or Nasal septum* Nasal septum is the osseo - cartilaginous partition between the two halves of the nasal cavity.* Nasal septum consists of1) Columellar septum It is formed by columella containing the medial crura of alar cartilages united together by fibrous tissue.2) Membranous septum It consists of double layer of skin with no bony or cartilaginous support.3) Septum properThis forms the major portion of nasal spetum and consists of osteocartilagenous framework which is coverd by mucous membrane. Its constituents arei) Osseous part# The vomer# Perpendicular plate of ethmoid# Nasal spine of frontal bone# Rostrum and crest of sphenoid# Nasal crest of nasal bone# Nasal crest of palatine bone# Nasal crest of maxillary bone ii) Cartilaginous part# Septal (Quadrilateral or Quadrangular) cartilage.", "cop": 3, "opa": "Diamond shaped", "opb": "Triangular", "opc": "Quadrilateral", "opd": "Pentagonal", "subject_name": "ENT", "topic_name": "Nose and PNS", "id": "0b9d6ad0-0337-450b-b138-32f50bced397", "choice_type": "single"} {"question": "Diameter of head mirror", "exp": null, "cop": 2, "opa": "3 inches", "opb": "3.5 inches", "opc": "2.5 inches", "opd": "9 mm", "subject_name": "ENT", "topic_name": null, "id": "15ec2b3d-9257-446f-8ac9-87493c6eecac", "choice_type": "single"} {"question": "Paranasal sinuses are ventilated during", "exp": null, "cop": 2, "opa": "Inspiration", "opb": "Expiration", "opc": "Mid inspiration", "opd": "deep inspiration", "subject_name": "ENT", "topic_name": null, "id": "cb43f64b-d2eb-473a-86f6-13ed7af7d768", "choice_type": "single"} {"question": "Sensory nerve supply of middle ear cavity is provided by", "exp": "The nerve supply of middle ear is derived from tympanic plexus which lies over the promontory.\nThe inferior ganglion of the glossopharyngeal nerve gives off the tympanic nerve which enters the middle ear through the tympanic canaliculus and takes part in formation of the tympanic plexus on the medial wall of middle ear.\nThis distributes it fibres to the middle ear, and also to the auditory tube, aditus ad atrum mastoideum (aditus to mastoid antrum).", "cop": 2, "opa": "Facial", "opb": "Glossopharyngeal", "opc": "Vagus", "opd": "Trigeminal", "subject_name": "ENT", "topic_name": null, "id": "1fa69acf-b884-45ad-ba5c-148c71f7e5b6", "choice_type": "single"} {"question": "Cochlear aqueduct", "exp": "THE INTERNAL EAR The internal ear or the labyrinth is an impoant organ of hearing and balance. It consists of a bony and a membranous labyrinth. The membranous labyrinth is filled with a clear fluid called endolymph while the space between membranous and bony labyrinths is filled with perilymph. BONY LABYRINTH It consists of three pas: the vestibule, the semicircular canals and the cochlea. Cochlea: The bony cochlea is a coiled tube making 2.5 to 2.75 turns round a central pyramid of bone called modiolus. The base of modiolus is directed towards internal acoustic meatus and transmits vessels and nerves to the cochlea. Around the modiolus and winding spirally like the thread of a screw, is a thin plate of bone called osseous spiral lamina. It divides the bony cochlea incompletely and gives attachment to the basilar membrane. The bony bulge in the medial wall of middle ear, the promontory, is due to the basal coil of the cochlea. The bony cochlea contains three compaments: (a) Scala vestibuli, (b) Scala tympani, (c) Scala media or the membranous cochlea. The scala vestibuli and scala tympani are filled with peri- lymph and communicate with each other at the apex of cochlea through an opening called helicotrema. Scala vestibuli is closed by the footplate of stapes which separates it from the air-filled middle ear. The scala tympani is closed by secondary tympanic membrane; it is also connected with the subarachnoid space through the aqueduct of cochlea. Ref:- Dhingra; pg num:-09,10", "cop": 3, "opa": "Contains endoylymph", "opb": "Connects cochlea with vestibule", "opc": "Connects internal ear with subarachnoid space", "opd": "Same as S media", "subject_name": "ENT", "topic_name": "Ear", "id": "a38ab09e-f067-4032-b5ac-9e7912c40ec5", "choice_type": "single"} {"question": "Paracusis Willisii is a symptom frequently present in", "exp": "(B) Otosclerosis # SYMPTOMS OF OTOSCLEROSIS:> Hearing loss This is the presenting symptom and usually starts in twenties. It is painless and progressive with insidious onset.> Often it is bilateral conductive type. Paracusiss willisii Tinnitus Vertigo Speech Chronic conductive hearing loss (CHL) is the finding in almost all cases of otosclerosis> This usually will begin in one ear but will eventually affect both ears with a variable course.> On audiometry, the hearing loss is characteristically low frequency, with higher frequencies being affected later> Sensorineural hearing loss has also been noted in patients with otosclerosisthis is usually a high frequency loss, and usually manifests late in the disease.", "cop": 2, "opa": "Advanced Meniere's disease", "opb": "Otosclerosis", "opc": "Acoustic neuroma", "opd": "Circumscribed labyrinthitis", "subject_name": "ENT", "topic_name": "Miscellaneous (E.N.T.)", "id": "828c885f-cfa7-4fc4-9c41-735186455616", "choice_type": "single"} {"question": "Positional veigo is most commonly seen due to pathology in", "exp": "Positional veigo is commonly due to pathology in the Posterior Semicircular Canal.BPPV is the commonest cause of peripheral veigoRef: Hazarika; 3rd ed; Pg 105", "cop": 4, "opa": "Lateral semicircular canal", "opb": "Superior semicircular canal", "opc": "Inferior semicircular canal", "opd": "Posterior semicircular canal", "subject_name": "ENT", "topic_name": "Ear", "id": "82101f07-ab8a-464d-a17e-6815feaefe97", "choice_type": "single"} {"question": "The most common site of sinusitis in children(( REPEAT QUESTION)", "exp": "Acute ethmoid sinusitis Aetiology Acute ethmoiditis is often associated with infection of other sinuses. Ethmoid sinuses are more often involved in infants and young children.It opens into middle meatus. (Ref: Diseases of ENT, PL Dhingra, 7th edition, of no. 216)", "cop": 2, "opa": "Inferior meatus", "opb": "Middle meatus", "opc": "Superior meatus", "opd": "Spheno-ethmoidal recess", "subject_name": "ENT", "topic_name": "Nose and paranasal sinuses", "id": "0d45e835-edf0-46d0-914d-47f9aa73cf4b", "choice_type": "single"} {"question": "Holman Millar sign is seen in", "exp": "Anterior bowing of the posterior wall of maxillary sinus often called antral sign or Holman-Miller sign, which is pathognomic of angiofibroma Ref:(Diseases of Ear, Nose and Throat, P.L Dhingra, 7th edition, page 281)", "cop": 1, "opa": "Nasopharyngeal angiofibroma", "opb": "Cranipharyngioma", "opc": "Nasopharyngeal carcinoma", "opd": "Laryngeal carcinoma", "subject_name": "ENT", "topic_name": "Pharynx", "id": "0031da48-42f7-43df-9271-67f66788e3e4", "choice_type": "single"} {"question": "Lighthouse sign is seen in", "exp": "Lighthouse sign is seen in Stage of suppuration of AOMStages of AOM-1. Stage of hyperemia/Congestion- Cawheel appearance of TM2. Stage of exudation/Pre suppuration3. Stage of suppuration- Lighthouse sign (Pinpoint perforation)4. Stage of resolution/complicationsRef: Hazarika; 3rd ed; Pg 149", "cop": 1, "opa": "Stage of suppuration", "opb": "Stage of hyperaemia", "opc": "Stage of resolution", "opd": "Stage of pre-suppuration", "subject_name": "ENT", "topic_name": "Ear", "id": "da613fe7-6d35-4298-8e73-35f34a645d2e", "choice_type": "single"} {"question": "Sphenopalatine foramen is 1 cm behind", "exp": null, "cop": 3, "opa": "Superior Turbinate", "opb": "Middle Turbinate", "opc": "Inferior Turbinate", "opd": "Tonsil", "subject_name": "ENT", "topic_name": null, "id": "3ff2d048-e172-4416-b9b9-fd66360ce294", "choice_type": "single"} {"question": "Apple jelly nodule in Nasal septum", "exp": null, "cop": 3, "opa": "Syphilis", "opb": "Leprosy", "opc": "Lupus vulgaris", "opd": "Wegener's Granulomatosis", "subject_name": "ENT", "topic_name": null, "id": "5466afa8-02a3-480b-8269-f6a34433e11c", "choice_type": "single"} {"question": "Most Common site of perilymph fistula is", "exp": "In this condition perilymph leaks into the middle ear. Footplate of stapes is fixed on oval window. It can occur as a complication of stapedectomy, or ear surgery when stapes is dislocated, and also from sudden pressure change in the middle ear. (Ref: Diseases of EAR, NOSE AND THROAT by PL DHINGRA - 6th Edition)", "cop": 2, "opa": "Round window", "opb": "Oval window", "opc": "Otic capsule", "opd": "Attic", "subject_name": "ENT", "topic_name": "Ear", "id": "6065b031-6189-4720-a3e4-96803a5c2347", "choice_type": "single"} {"question": "Commonest causative organism for ASOM in 2 years child is", "exp": "ACUTE SUPPURATIVE OTITIS MEDIA It is an acute inflammation of middle ear by pyogenic organ isms. Here, middle ear implies middle ear cleft, i.e. eustachian tube, middle ear, attic, aditus, antrum and mastoid air cells. ROUTES OF INFECTION 1. Via eustachian tube: It is the most common route. Infection travels the lumen of the tube or along subepithelial peritubal lymphatics. Eustachian tube in infants and young children is shoer, wider and more horizontal and thus may account for higher incidence of infections in this age group. Breast or bottle feeding in a young infant in horizontal position may force fluids through the tube into the middle ear and hence the need to keep the infant propped up with head a little higher. Swimming and diving can also force water through the tube into the middle ear. 2. Via external ear: Traumatic perforations of tympanic membrane due to any cause open a route to middle ear infection. 3. Blood-borne: This is an uncommon route. AETIOLOGY : It is more common especially in infants and children of lower socioeconomic group. Bacteriology: Most common organisms in infants and young children are Streptococcus pneumoniae (30%), Haemophilus influenzae (20%) and Moraxella catarrhalis (12%). Other organisms include Streptococcus pyogenes, Staphylococcus aureus and sometimes Pseudomonas aeruginosa. Ref:- Dhingra; pg num:-62", "cop": 1, "opa": "Pneumococcus", "opb": "Streptococcus", "opc": "Staphylococcus", "opd": "H. influenzae", "subject_name": "ENT", "topic_name": "Ear", "id": "2e8082b3-2786-4159-93df-5d2783a2a45b", "choice_type": "single"} {"question": "Most common nerve damaged in CSOM is(REPEAT QUESTION)", "exp": "Facial paralysis occurs as a complication of both acute and chronic otitis media. (Ref: Textbook of diseases of ENT, PL Dhingra, 7th edition, pg no. 88)", "cop": 2, "opa": "III", "opb": "VII", "opc": "II", "opd": "VI", "subject_name": "ENT", "topic_name": "Ear", "id": "e3838de1-a4fc-49bd-a2e6-633f2e691a3a", "choice_type": "single"} {"question": "The Lombard test is used in the diagnosis of", "exp": "Tests for Functional (Non-organic) Hearing loss:1. Stenger`s test2. Chimani Moos test3. Teal test4. Erhards test5. Lombard`s test6. Acoustic reflex threshold7. BERA8. Bekesy audiometryRef: Hazarika; 3rd ed; Pg 72", "cop": 3, "opa": "Conductive hearing loss", "opb": "Sensorineural hearing loss", "opc": "Non-organic hearing loss", "opd": "Mixed hearing loss", "subject_name": "ENT", "topic_name": "Ear", "id": "1da4a8e8-6c26-4406-833e-572f329093ee", "choice_type": "single"} {"question": "Pain sensation from ethmoid sinus is carried by", "exp": null, "cop": 2, "opa": "Ethmoid nerve", "opb": "Nasociliary nerve", "opc": "Maxillary nerve", "opd": "Greater petrosal nerve", "subject_name": "ENT", "topic_name": null, "id": "cbea466c-9cb1-4ba6-828d-52721dbbdd0c", "choice_type": "single"} {"question": "Nerve supply of the tympanic membrane is by", "exp": "The auriculotemporal nerve supplies the anterior half of the lateral surface. Auricular branch of vagus nerve supplies posterior half of lateral surface. Tympanic branch of CN 9 supply medial surface. Reference: Dhingra 6th edition", "cop": 1, "opa": "Auriculotemporal", "opb": "Lesser occipital", "opc": "Greater occipital", "opd": "Parasympathetic ganglion", "subject_name": "ENT", "topic_name": "Ear", "id": "5bcd37cf-fee8-42ac-b929-defa7bf918b3", "choice_type": "single"} {"question": "Hectic picket type of fever with rigor is important feature in", "exp": "(B) (Lateral sinus thrombosis) (84- Dhingra 6th)LATERAL SINUS THROMBOSIS* Inflammation of inner wall of lateral venous sinus with formation of an intrasinus thrombosis* Complicatin of - Acute coalescent mastoiditis- Masked mastoiditis- CSOM- Cholesteatoma* Clinical features1. Hectic picket. Fence type of fever with rigor. This is due to septicaemia2. Headache in early stage due to perisinus abscess3. Progressive anaemia and emaciation4. Griesinger's sign- due to thrombosis of mastoid emissary vein.5. Papilloedema6. Tobey-Ayer test - record CSF pressure7. Crowe Beck test = pressure on jugular vein of healthy side produces engorgement of retinal vein(seen in ophthalmoscopy) and supraorbital vein, Engorgement of vein subsides on release of pressure8. Tendemess along jugular veinCT & MRI shows- \"Delta sign\"TREATMENT(1) Intravenous antibacterial therapy(2) Mastoidectomy and exposure of sinus(3) Ligation of internal jugular vein(4) Anticoagulant therapy.", "cop": 2, "opa": "Cavernous sinus thrombosis", "opb": "Lateral sinus thrombosis", "opc": "Ethmoid sinusitis", "opd": "Sphenoid sinusitis", "subject_name": "ENT", "topic_name": "Ear", "id": "9ee50257-c6d7-4fcc-8139-1056cab0c663", "choice_type": "single"} {"question": "Most common frequency of Tuning fork used in ENT", "exp": "Tuning fork test can be performed using tuning fork of different frequencies such as:\n. • MC used/ ideal for routine clinical practice is of 512 Hz.\n• Tuning fork of lower frequencies produce sense of bone vibration while those of higher frequencies have a shorter decay time and are thus not routinely preferred.\n• Tests done with tuning fork:\n■ Rinne’s test\n■ Weber's test\n■ Absolute bone conduction (ABC) test\n■ Bing test\n■ Gelle’s test\n■ Lewis test", "cop": 2, "opa": "256 Hz", "opb": "512 Hz", "opc": "1024 Hz", "opd": "2048 Hz", "subject_name": "ENT", "topic_name": null, "id": "01e62356-13e5-42c7-bcce-1bff60c7c93a", "choice_type": "single"} {"question": "Pierre Robbin syndrome does not include", "exp": null, "cop": 3, "opa": "Hearing loss", "opb": "Respiratory distress", "opc": "Coloboma Iris", "opd": "Mandibular dysplasia", "subject_name": "ENT", "topic_name": null, "id": "83e839b0-31f0-4cab-bce8-ef2510561e6b", "choice_type": "single"} {"question": "Graft for myringoplasty is taken from", "exp": "Graft material used for myringoplasty are:Temporalis fascia (most common)Tragal cailagePerichondrium from the tragusVeinRef: PL Dhingra, Diseases of Ear, Nose & Throat, 7th edition, pg no. 463", "cop": 1, "opa": "Temporalis fascia", "opb": "Iliacus fascia", "opc": "Colles fascia", "opd": "Iliotibial band", "subject_name": "ENT", "topic_name": "Ear", "id": "f489bcb4-305e-447d-b93c-2d17f65d6ecd", "choice_type": "single"} {"question": "Surfers ear is", "exp": "Exostoses are multiple and bilateral, often presenting as smooth, sessile, bony swellings in the deeper pa of the meatus near the tympanic membrane. They arise from the compact bone. Exostosis is often seen in persons exposed to the entry of cold water in the meatus as in divers and swimmers. Hence called a surfers ear. Reference: Diseased of Ear, Nose and Throat, PL Dhingra, 7th edition, pg no.119", "cop": 1, "opa": "Exostosis", "opb": "Otosclerosis", "opc": "Otitis externa", "opd": "Squamous cell Carcinoma", "subject_name": "ENT", "topic_name": "Ear", "id": "c0515b43-1a11-46cc-9e95-fe22edb84ab1", "choice_type": "single"} {"question": "Patient with anotia, ideal hearing aid", "exp": "Bone-anchored hearing aids can be used in People who have chronic inflammation or infection of the ear canal and cannot wear the standard \"in the ear\" air conduction hearing aids Children with malformed or absent outer ear and ear canals as in microtia or canal atresia Single-sideddeafness. Ref: Dhingra 7e pg 137.", "cop": 2, "opa": "In canal", "opb": "Bone anchored hearing aid", "opc": "Vestibular", "opd": "Transcutaneous", "subject_name": "ENT", "topic_name": "Ear", "id": "845e0d30-8116-4db1-bb17-c94bbc7e581d", "choice_type": "single"} {"question": "Tone decay test is used for", "exp": "Answer- D. Sensory neural deafnessThreshold tone decay test is used for retrocochlear type of SNHL.", "cop": 4, "opa": "Meinneirs disease", "opb": "Otosclerosis", "opc": "Cochlear deafness", "opd": "Sensory neural deafness", "subject_name": "ENT", "topic_name": null, "id": "dc8fcc8c-1019-44b4-a4bc-dcdd34ae9bfe", "choice_type": "single"} {"question": "Most common site for contact ulcer in larynx is", "exp": null, "cop": 1, "opa": "Arytenoids", "opb": "Corniculate", "opc": "Anterior 1/3 rd of vocal cord", "opd": "Cricoid", "subject_name": "ENT", "topic_name": null, "id": "f47de07e-6e98-41b6-ad78-d81adf7dbbb3", "choice_type": "single"} {"question": "Ramu, 15 yrs of age presents with haemorrhage 5 hrs after tonsillectomy. Treatment of choice is", "exp": "Ans. is 'd' i.e Reopen immediately Post tonsillectomy haemorrhages can be of two types :Primary or reactionary- occurs with in a few hrs of the operation (by definition within 24 hrs)Secondary haemorrhage- usually seen between 5th to 10th post - op dayIt is due to infection.*MxPrimary - return to the operation theatre where the bleeding vessel is ligated under anesthesia.Secondary - Patient should be readmitted to hospital;An appropriate dose of morphine for adults or heroin for children is injectedA course of systemic antibiotics.Bleeding vessel is searched and ligated if the conservative methods fail which is not common.Remember:MC vessel responsible for post tonsillectomy haemorrhage*- Palatine vein {also called as external palatine vein)", "cop": 4, "opa": "External gauze packing", "opb": "Antibiotics & mouth wash", "opc": "Irrigation with saline", "opd": "Reopen immediately", "subject_name": "ENT", "topic_name": "Ear", "id": "ecaae7ce-199c-4370-b258-db301154f392", "choice_type": "single"} {"question": "Most common cause of SNHL", "exp": "Presbycusis - Most common cause of SNHL. A/W physiological aging process in the ear Bilateral slopping curve Usually manifests at the age of 65 years, (old age) but may do so early if there is a hereditary predisposition, chronic noise exposure or generalized vascular disease.", "cop": 2, "opa": "Labyrinthitis", "opb": "Presbycusis", "opc": "Meniere's disease", "opd": "Vestibular Schwannoma", "subject_name": "ENT", "topic_name": "FMGE 2018", "id": "0e1178c8-f6ce-4d8c-a274-244532fe3c72", "choice_type": "single"} {"question": "Definition of sleep apnoea", "exp": null, "cop": 2, "opa": ">10 apnoeas during 7 hours of sleep", "opb": ">30 apnoeas during 7 hours of sleep", "opc": ">10 apnoeas during 2 hours of sleep", "opd": ">30 apnoeas during 2 hours of sleep", "subject_name": "ENT", "topic_name": null, "id": "112594bc-90bf-445d-a26f-4444845aa069", "choice_type": "single"} {"question": "The collaural fistula is an abnormality of", "exp": "The collaural fistula is a 1st branchial cleft anomaly which arises from the failure of fusion of the ventral pa of the 1st cleftIts upper pa opens into the floor of the external auditory canalIts lower pa opens in the neck between the angle of the mandible and sternocleidomastoid muscleRef: PL Dhingra, Diseases of Ear, Nose & Throat, 7th edition, pg no. 53", "cop": 2, "opa": "1st branchial arch", "opb": "1st branchial cleft", "opc": "2nd branchial arch", "opd": "2nd branchial cleft", "subject_name": "ENT", "topic_name": "Ear", "id": "e6d05c2e-4b3e-4f86-b3eb-2e66e01722db", "choice_type": "single"} {"question": "Submandibular nodes are classified as", "exp": "Level 1A =Submental Node Level1B = Submandibular nodes, lie between the anterior and posterior bellies of digastric muscle and the lower border of mandible. Level II=Upper Jugular Nodes Level III=Middle Jugular Nodes Ref: Textbook of ENT; Dhingra; 6th edition; Page no: 387; Table 76 ref img", "cop": 2, "opa": "Level 1 A neck nodes", "opb": "Level 1B neck nodes", "opc": "Level II neck nodes", "opd": "Level III neck nodes", "subject_name": "ENT", "topic_name": "Miscellaneous ENT", "id": "a6fd4530-5c9e-4ebb-8baf-9fff2694d6ec", "choice_type": "single"} {"question": "Most common organism cultured in CSOM is", "exp": "Pus culture in both aerobic and anaerobic CSOM may show multiple organisms. Common aerobic organisms are Pseudomonas aeruginosa Proteus Escherichia coli Staphylococccus aureus While anaerobes include Bacteroides fragilis Anaerobic streptococcal (Ref: Textbook of diseases of ENT, PL Dhingra, 7th edition, pg no. 76)", "cop": 4, "opa": "Staphylococcus aureus", "opb": "Staphylococcus epidermidis", "opc": "Staphylococcus pneumonia", "opd": "Pseudomonas aeruginosa", "subject_name": "ENT", "topic_name": "Ear", "id": "b373cb86-86e5-4cee-92cb-6e7bea4680a1", "choice_type": "single"} {"question": "Nasopharyngeal Carcinoma is caused by", "exp": "AETIOLOGY The exact aetiology is not known. The factors responsible are: 1. Genetic. Chinese have a higher genetic susceptibility 2. Viral. Epstein-Barr (EB) virus is closely associated with nasopharyngeal cancer. EB virus has two impoant antigens: viral capsid antigen (VCA) and early antigen (EA). IgA antibodies of EA are highly specific for nasopharyngeal cancer but have a sensitivity of only 70-80% while IgA antibodies of VCA are more sensitive but less specific. 3. Environmental. Air pollution, smoking of tobacco and opium, nitrosamines from dry salted fish, smoke from the burning of incense and would have all been incriminated. (Ref: Diseases of Ear, Nose and Throat, P.L Dhingra, 7th edition, page 283)", "cop": 1, "opa": "EBV", "opb": "Papilloma virus", "opc": "Parvo virus", "opd": "Adeno virus", "subject_name": "ENT", "topic_name": "Pharynx", "id": "233c0864-e80b-41d2-9fe0-9a0d7baa607c", "choice_type": "single"} {"question": "Maximum hearing loss is caused by", "exp": "1. Complete obstruction of ear canal: 30 dB 2. Perforation of tympanic membrane. (It varies and is directly propoional to the size of perforation): 10-40 dB 3. Ossicular interruption with intact drum: 54 dB 4. Ossicular interruption with perforation: 38 dB 5. Malleus fixation: 10-25 dB 6. Closure of oval window: 60 dB Ref: Dhingra 7e pg 31.", "cop": 3, "opa": "Ossicular chain damage", "opb": "Tympanic membrane perforation", "opc": "Obliteration of oval window", "opd": "Blockage of ear canal", "subject_name": "ENT", "topic_name": "Ear", "id": "b3abd6e6-0d87-45b0-b9ec-49ec6560e7a7", "choice_type": "single"} {"question": "Frontal Sinus can be best visualized by", "exp": "A. Caldwell's view -frontal sinuses are seen best B. Water's view-maxillary sinus seen best C. Towne's view-usually taken for acoustic neuroma and apical petrositis D. Schuller's view-to visualise the key area of the mastoid (attic, aditus, antrum) (Ref: Textbook of ENT, Dhingra; 7th edition, of no. 493, 494)", "cop": 1, "opa": "Caldwell's view", "opb": "Water's view", "opc": "Towne's view", "opd": "Schuller's view", "subject_name": "ENT", "topic_name": "Nose and paranasal sinuses", "id": "a64cd25b-f1de-4833-bf43-2703d24e6702", "choice_type": "single"} {"question": "Pleomorphic adenoma is", "exp": null, "cop": 4, "opa": "More common in males", "opb": "Endothelial origin", "opc": "Most common site is parotid", "opd": "More common in females", "subject_name": "ENT", "topic_name": null, "id": "110e8193-b795-4a9f-9c38-c2481b9b7e10", "choice_type": "single"} {"question": "Choanal atresia is due to the persistence of", "exp": "Choanal atresia \nChoana, also called posterior nares, is an opening in the posterior part of each nasal cavity, through which the nasal cavity communicates with nasopharynx.\nChoanal atresia is a congenital anomaly characterized by closure of one or both posterior nasal cavies (i.e., choana). It is due to the persistence of bucconasal membrane.\nApproximately 60- 70% of cases are unilateral and are more common on the right side.\nBilateral cases occur in 20-30% of patients and 50% of these patients are associated with other congenital anomaly, i.e.,\nCHARGE syndrome\n→ Coloboma, Heart defects, Choanal atresia, Retarded growth, Genitourinary abnormalies and Ear defects.\nDiagnosis - Unilateral lesions go unnoticed until the child presents with persistent unilateral nasal discharge.\nThere is the absence of air bubbles in nasal discharge.\nA simple test for unilateral choanal atresia is to have the child attempt nose blowing with opposite nostril occluded by external pressure.\nFailure to detect any air movement is suspicious for complete obstruction.\nBilateral choanal atresia presents as respiratory distress in newborn and requires support immediately after birth.", "cop": 1, "opa": "Bucconasal membrane", "opb": "Oropharyngeal membrane", "opc": "Laryngotracheal fold", "opd": "Tracheoesophageal fold", "subject_name": "ENT", "topic_name": null, "id": "fe2e1548-36ce-49a6-b6b5-19254bbf113e", "choice_type": "single"} {"question": "Parasympathetic secretomotor fibers to paranasal sinus", "exp": null, "cop": 2, "opa": "Otic ganglion", "opb": "Pterygopalatine ganglion", "opc": "Ciliary ganglion", "opd": "Spiral ganglion", "subject_name": "ENT", "topic_name": null, "id": "6e24ad2c-cb2b-4734-bb70-9cccb3b0573e", "choice_type": "single"} {"question": "Not an indication for Tracheostomy", "exp": null, "cop": 2, "opa": "Flial chest", "opb": "Respiratory distress in carcinoma larynx", "opc": "Foreign body aspiration", "opd": "Tetanus", "subject_name": "ENT", "topic_name": null, "id": "78cb34c4-5aa2-4523-9c82-fb880cc872c9", "choice_type": "single"} {"question": "The lymphatic drainage of pyriform fossa is to", "exp": "Lymphatic drainage of Hypopharynx 1) Pyriform sinus drains into upper jugular chain and then to deep cervical group of lymph nodes. 2) Postcricoid region drains into parapharyngeal and paratracheal group of lymph nodes. 3) Posterior pharyngeal wall drains into parapharyngeal lymph nodes and finally to deep cervical lymph nodes. Ref:- Tuli; pg num:- 231, 232", "cop": 4, "opa": "Prelaryngeal node", "opb": "Parapharyngeal nodes", "opc": "Mediastinal nodes", "opd": "Upper deep cervical nodes", "subject_name": "ENT", "topic_name": "Pharynx", "id": "ea703677-71f4-46a1-9754-64cda87d09bb", "choice_type": "single"} {"question": "Epley's manoeuvre is used in treatment of", "exp": "Ans. d (BPPV) (Ref. PL Dhingra ENT, 4th ed, 58)Epley Maneuver for Rx of BPPV: The patient is taken through 4 moves, starting in the sitting position with the head turned at a 45-degree angle toward the affected side. (1) The patient is placed into the Dix-Hallpike position (supine with the affected ear down) until the vertigo and nystagmus subside.(2) The patient's head is then turned to the opposite side, causing the affected ear to be up and the unaffected ear to be down.(3) The whole body and head is then turned away from the affected side to a lateral decubitus position, with the head in a face down position.(4) The last step is to bring the patient back to a sitting position with the head turned toward the unaffected shoulder.BPPV (BENIGN PAROXYSMAL POSITIONAL VERTIGO)It is characterized by vertigo when head is placed in a certain critical position.# Cause:- Disorder of posterior semicircular canal.- Otoconial debris, consisting of calcium carbonate, is released from the degenerating macula of utricle and floats freely in endolymph. When it settles on the cupula of posterior semicircular canal in critical head position, it causes displacement of cupula and vertigo.# H/o: Deafness, Vomiting, Tinnitus# Positional vertigo is precipitated by a recumbent head position, either to the right or to the left.# Benign paroxysmal positional (or positioning) vertigo (BPPV) of the posterior semicircular canal is particularly common. Although the condition may be due to head trauma, usually no precipitating factors are identified. It generally abates spontaneously after weeks or months. The vertigo and accompanying nystagmus have a distinct pattern of latency, fatigability, and habituation that differs from the less common central positional vertigo due to lesions in and around the fourth ventricle. Moreover, the pattern of nystagmus in posterior canal BPPV is distinctive.# When supine, with the head turned to the side of the offending ear (bad ear down), the lower eye displays a large- amplitude torsional nystagmus, and the upper eye has a lesser degree of torsion combined with upbeating nystagmus. If the eyes are directed to the upper ear, the vertical nystagmus in the upper eye increases in amplitude. Mild dysequilibrium when upright may also be present.FeaturesBPPVCentralLatency3-40 sNone: immediate vertigo and nystagmusFatigabilityYesNoHabituationYesNoIntensity of vertigoSevereMildReproducibilityVariableGood# Treatment:a) Epley's Maneuverb) Intra-labyrinthine Streptomycin / Gentamycin.# BPPV is caused by dislodged otoconia that move with changes in head position. Single applications of the Epley procedure or the Semont maneuver have been reported to relieve symptoms in the majority of patients. Repeated treatment sessions may be necessary before symptoms resolve. Sign or symptomPeripheral (Labyrinth)Central (Brainstem or Cerebellum)1. Direction of associated nystagmusUnidirectional; fast phase oppositeBidirectional or unidirectional2. Purely horizontal nystagmus without torsional componentUncommonCommon3. Vertical/purely torsional nystagmusNever presentMay be present4. Visual fixationInhibits nystagmus and vertigoNo inhibition5. Severity of vertigoMarkedOften mild6. Direction of spinToward fast phaseVariable7. Direction of fallToward slow phaseVariable8. Duration of symptomsFinite (minutes, days, weeks) but recurrentMay be chronic9. Tinnitus and/or deafnessOften presentUsually absent10. Associated CNS abnormalitiesNoneExtremely common (e.g., diplopia, hiccups, cranial neuropathies, dysarthria)11. Common causesBPPV, infection (labyrinthitis),Meniere's, neuronitis, ischemia, trauma, toxinVascularAdditional Educational Points:", "cop": 4, "opa": "Infective labyrinthitis", "opb": "Tinnitus", "opc": "Cervical spondylosis", "opd": "BPPV (Benign primary positional vertigo)", "subject_name": "ENT", "topic_name": "Assessment of Vestibular Function", "id": "40fe0800-6fff-438f-bdfd-31afc1f8d151", "choice_type": "single"} {"question": "Commando operation is used as treatment for", "exp": null, "cop": 4, "opa": "Colon cancer", "opb": "Carcinoma esophagus", "opc": "Parotid gland tumour", "opd": "Carcinoma of oral cavity", "subject_name": "ENT", "topic_name": null, "id": "aed8cb44-f751-4d7b-96e2-0a7ef69063c0", "choice_type": "single"} {"question": "Graveyard of ENT surgeon", "exp": "Tonsilolingual sulcus is seat of carcinoma usually missed by ENT doctor in OPD to check.", "cop": 3, "opa": "Pyriform Fossa", "opb": "Bucco Labial sulcus", "opc": "Tonsilolingual sulcus", "opd": "Peritonsillar space", "subject_name": "ENT", "topic_name": null, "id": "66006504-ed9a-4abf-a097-a589a06a69e3", "choice_type": "single"} {"question": "In right external auditory canal, warm water (44 deg,c) pressure applied for 40 seconds then", "exp": "(Right sided nystagmus): (92. P. Hazarika 3rd/ edition)* Cold water induces nystagmus to opposite side and warm water to the same side (mneumonic- COWS: cold- opposite, warm - same)CALORIC TEST (Filzgerald - Hallpike test)* Patient lies supine with head tited 30deg forward so that horizontal canal is vertical. Ear are irrigated for 40 second alternately with water at 30degc and at 44degc (ie 7deg below and above normal body temperature) and eyes observed for appearance of nystagmus till its end point* We can find -* Canal paresis or dead labyrinth* Directional preponderance (ie nytagmus is more in one particular direction than in the other)* Both canal paresis and directional preponderance", "cop": 1, "opa": "Right side nystagmus", "opb": "Left side nystagmus", "opc": "Circular nystagmus", "opd": "No nystagmus", "subject_name": "ENT", "topic_name": "Ear", "id": "e105b604-9b14-4e91-aee5-aa39c20b1ef9", "choice_type": "single"} {"question": "Endolymphatic decompression is done in", "exp": "Decompression of endolymphatic sac is done as a conservative procedure as a pa of the surgical treatment of Meniere's disease. Ref: Diseases of EAR, NOSE and THROAT by PL Dhingra 7th Edition page no.115", "cop": 3, "opa": "Tinnitus", "opb": "Acoustic neuroma", "opc": "Meniere's disease", "opd": "Endolymphatic fistula", "subject_name": "ENT", "topic_name": "Ear", "id": "51b67144-c11a-46e1-8c98-df28453bd298", "choice_type": "single"} {"question": "A 20 year old patient presents with difficulty in breathing and impaired hearing on examination enlargement of upper deep cervical lymph nodes, most likely diagnosis is", "exp": null, "cop": 3, "opa": "Angiofibroma", "opb": "Hodgkin's Lymphoma", "opc": "Nasapharyngeal carcinoma", "opd": "Carcinoma tongue", "subject_name": "ENT", "topic_name": null, "id": "cb4e05b8-d01a-48e1-a510-dcbfd4cb1485", "choice_type": "single"} {"question": "Bell's palsy is paralysis of", "exp": "Bells palsy is due to the lesion of LMN VII nerve which is idiopathic in nature. Ref: Dhingra 6th edition pg. 94", "cop": 4, "opa": "UMN V nerve", "opb": "UMN VII nerve", "opc": "LMN V nerve", "opd": "LMN VII nerve", "subject_name": "ENT", "topic_name": "Ear", "id": "046a5616-4112-4694-ac32-c580b5862bb6", "choice_type": "single"} {"question": "Cricothyroid joint is a", "exp": "Cricothyroid joint is a synol joint. Each is formed by inferior cornu of thyroid cailage with a facet on cricoid cailage. Cricoid cailage rotates at these joints on a transverse axis which passes transversely through these joints.", "cop": 4, "opa": "Hinge joint", "opb": "Hyaline joint", "opc": "Atavistic joint", "opd": "Synol joint", "subject_name": "ENT", "topic_name": "Anatomy of Larynx", "id": "4ce5b6e8-62eb-4185-84b2-fdf961c097e5", "choice_type": "single"} {"question": "Laryngocole arise as herniation of lorry regal mucosa through following membrane", "exp": null, "cop": 1, "opa": "Thyrohyoid", "opb": "Cricothyroid", "opc": "Cricotracheal", "opd": "Cricosternal", "subject_name": "ENT", "topic_name": null, "id": "5117cfc3-8e2e-4af8-bfdf-3fd565c4fa44", "choice_type": "single"} {"question": "Gillette&;s space is", "exp": "Retropharyngeal space is also called Space of Gillette.Also called danger space of neck as it extends from skull base to superior mediastinum and infections can lead to mediastinitis.Contains Glands of Henle and Rouviere`s nodes.Ref: Hazarika; 3rd ed; Pg 492", "cop": 1, "opa": "Retropharyngeal space", "opb": "Peritonsillar space", "opc": "Parapharygeal space", "opd": "Prelaryngeal space", "subject_name": "ENT", "topic_name": "Pharynx", "id": "79e308f8-024f-459b-bff5-d67ec4bd4d43", "choice_type": "single"} {"question": "Treatment of choice for acoustic neuroma is", "exp": "Acoustic neuroma is surgically removed through translabyrinthine, middle cranial fossa, suboccipital or combined translabyrinthine- suboccipital approach. Ref: Textbook of diseases of ENT, PL Dhingra, 7th edition, pg no. 127", "cop": 4, "opa": "Steroids", "opb": "Radiotherapy", "opc": "Anti-neoplastic drugs", "opd": "Surgery", "subject_name": "ENT", "topic_name": "Ear", "id": "60d1188c-b8b1-4d11-a454-ae0ceebabe82", "choice_type": "single"} {"question": "Danger space of pharynx is bounded by", "exp": "Ref:- Dhingra; pg num:-268", "cop": 3, "opa": "Buccopharyngeal fascia anteriorly and alar fascia posteriorly", "opb": "Preveebral fascia anteriorly and veebral body posteriorly", "opc": "Alar fascia anteriorly and preveebral fascia posteriorly", "opd": "Tonsils anteriorly and superior constrictor muscle posteriorly", "subject_name": "ENT", "topic_name": "Pharynx", "id": "6f70dccb-915e-477f-a6b1-9b8d167f9299", "choice_type": "single"} {"question": "In fracture maxilla, most common nerve involved is", "exp": "Clinical Features of maxillary fractures: Ecchymosis of lid, conjunctiva and sclera Enophthalmos with inferior displacement of the eyeball. This becomes apparent when edema subsides. Diplopia, which may be due to displacement of the eyeball or entrapment of inferior rectus and inferior oblique muscles. Hypoaesthesia or anesthesia of cheek and upper lip, if infraorbital nerve is involved", "cop": 1, "opa": "Infra orbital nerve", "opb": "Supraorbital nerve", "opc": "Trochlear nerve", "opd": "Mandibular nerve", "subject_name": "ENT", "topic_name": null, "id": "189edbe2-d32b-4328-bd41-60ad6b7c1faa", "choice_type": "single"} {"question": "During normal conversation sound heard at 1meter distance is", "exp": "At a distance of 1 m , intensity of Whisper - 30 dB Normal conversation - 60 dB Shout - 90 dB Discomfo of the ear - 120 dB Pain in the ear - 130 dB Ref: Dhingra 7e pg 21.", "cop": 1, "opa": "60 dB", "opb": "80 dB", "opc": "90 dB", "opd": "120 dB", "subject_name": "ENT", "topic_name": "Ear", "id": "6b54cc85-9c17-40ee-acb4-df2c5e0c17e5", "choice_type": "single"} {"question": "Pain sensations from the ethmoidal sinus are carried by", "exp": "nasociliary nerve branch of ophthalmic division of trigeminal nerve carries pain sensation from ethmoid sinus. Ref Dhingra 6/e,p 188", "cop": 3, "opa": "Frontal Nerve", "opb": "Lacrimal Nerve", "opc": "Nasocilliary Nerve", "opd": "Intraorbital Nerve", "subject_name": "ENT", "topic_name": "Nose and paranasal sinuses", "id": "cb45ec27-c241-4d44-9a43-fc654150a641", "choice_type": "single"} {"question": "In Stylalgia most common nerve affected is", "exp": "The patient complains of pain in tonsillar fossa and upper neck which radiates to the ipsilateral ear. It gets aggravated on swallowing. Ref: Textbook of ENT, Dhingra; 6th edition; Pg no: 272", "cop": 1, "opa": "Glossopharyngeal nerve", "opb": "Abducent nerve", "opc": "Auditory nerve", "opd": "Greater petrosal nerve", "subject_name": "ENT", "topic_name": "Miscellaneous ENT", "id": "8bcb4c3a-0f55-49f4-847a-b0da7672c37c", "choice_type": "single"} {"question": "Function of Larynx is A/E", "exp": "(Immunity enhancement): (303-4- Dhingra 5th edition; 577-P Hazarika 3rd)Function of the Larynx1. Primary function is to protect lower respiratory tractProtection of tracheobronchial tree is fascilitated by'Three - tier' sphincter action- Laryngeal elevation - Mylohyoid- Laryngeal tilting - Stylopharyngeous- Cricopharyngeal sphincter relaxes- Cough reflex- Cessation of respirationLarynx is aptly, called the watch-dog of lungs as it immediately barks' at the entry of any foreign intruder2. Phonation - is also an important function of the lung which facilitates communication3. Respiration: Reflex adjustments of the glottis apparatus plays a role in the mechanism of respiration Larynx regulates flow of air into the lungs.Vocal cords abduct during inspiration and adduct during expiration4. Fixation of the chest - To increase intrathoracic pressure when larynx is closed, chest wall gets fixed and various thoracic and abdominal muscles can then act best. This function is important in digging, pulling and climbing. Coughing, vomiting, defaecation, micturition and childbirth (Straining) also require a fixed thoracic cage against a closed glottis.", "cop": 4, "opa": "Phonation", "opb": "Protect lower respiratory tract from infection", "opc": "Prevent food regurgitation", "opd": "Immunity enhancement", "subject_name": "ENT", "topic_name": "Larynx", "id": "eac3a814-de6e-4725-be72-41f59063ce09", "choice_type": "single"} {"question": "Woodruff's plexus is seen at", "exp": "Woodruff's Plexus - Site of posterior epistaxis in adults. Plexus of veins situated inferior to posterior end of inferior turbinate Ref: Dhingra 7e pg 197.", "cop": 3, "opa": "Anteroinferior pa of superior turbinate", "opb": "Middle turbinate", "opc": "Posterior pa of inferior turbinate", "opd": "Anterior pa of inferior turbinate", "subject_name": "ENT", "topic_name": "Nose and paranasal sinuses", "id": "1ffb53b3-8965-410d-9813-b8b662ee7cdb", "choice_type": "single"} {"question": "Trotter's triad is seen in", "exp": "Nasopharyngeal cancer can cause conductive deafness (eustachian tube blockage), ipsilateral temporoparietal neuralgia (involvement of CN V) and palatal paralysis (CN X)-- collectively called Trotter's triad. (Ref: Diseases of Ear, Nose and Throat, PL Dhingra, 7th edition, page 285)", "cop": 1, "opa": "Nasopharyngeal carcinoma", "opb": "Maxillary sinusitis", "opc": "Maxillary carcinoma", "opd": "Angiofibroma", "subject_name": "ENT", "topic_name": "Pharynx", "id": "dcc67eae-9707-487d-9657-b14f4d112fee", "choice_type": "single"} {"question": "Crypts are seen in", "exp": "Ans. (a) TonsilsRef: Dhingra's ENT 6th ed.l 257-58* Crypts are inward folding of the mucosa which are found in tonsils and they increase the surface area.* Crypts are also seen in small intestine.", "cop": 1, "opa": "Tonsils", "opb": "Adenoids", "opc": "Lymph nodes", "opd": "Parotids", "subject_name": "ENT", "topic_name": "Oral Cavity", "id": "9f0abff7-27d2-44b0-98be-a6754516bc0f", "choice_type": "single"} {"question": "Treatment of choice for nasopharyngeal carcinoma is", "exp": "Treatment of nasopharyngeal carcinomaIrradiation is the treatment of choice (external radiotherapy)Radical neck dissection is required for persistent nodes when the primary has been controlled and in post-radiation cervical metastasisSystemic chemotherapy is used as palliation for distant metastases or radiation failureFor advanced stages (stage III & IV), the cure rate can be doubled when chemotherapy is combined with radiotherapyRef: PL Dhingra, Diseases of Ear, Nose & Throat, 7th edition, pg no. 279 - 286", "cop": 2, "opa": "Surgery", "opb": "Radiotherapy", "opc": "Surgery & radiotherapy", "opd": "Chemotherapy", "subject_name": "ENT", "topic_name": "Pharynx", "id": "0ae6fe4f-c4fb-472d-9008-cc0c08f73efd", "choice_type": "single"} {"question": "Tear drop sign is seen in", "exp": "FRACTURES OF ORBITAL FLOOR Zygomatic and Le Fo II maxillary fractures are always accompanied by fractures of orbital floor. Isolated fractures of orbital floor, when a large blunt object strikes the globes, are called \"blow out fractures.\" Orbital contents may herniate into the antrum. CLINICAL FEATURES 1. Ecchymosis of lid, conjunctiva and sclera. 2. Enophthalmos with inferior displacement of the eyeball. This becomes apparent when oedema subsides. 3. Diplopia, which may be due to displacement of the eyeball or entrapment of inferior rectus and inferior oblique muscles. 4. Hypoaesthesia or anaesthesia of cheek and upper lip, if infraorbital nerve is involved. DIAGNOSIS Waters' view shows a convex opacity bulging into the antrum from above (tear-drop opacity). CT scans may confirm the Diagnosis. Entrapment of inferior rectus and inferior oblique muscles is diagnosed by asking the patient to look up and down, or by the traction test. The latter is performed by grasping the globe and passively rotating it to check for restriction of its movements. TREATMENT Indications for surgery include enophthalmos and persistent diplopia due to entrapment of muscle. Orbital floor fractures can be satisfactorily reduced by a finger passed into the antrum through a transantral approach. A pack can be kept in the antrum to suppo the fragments. Infraorbital approach, through a skin crease of the lower lid, can also be used either alone or in combination with transantral approach. Badly comminuted fractures of orbital floor can be repaired by a bone graft from the iliac crest, nasal septum or the anterior wall of the antrum. Silicon or teflon sheets have also been used to reconstruct the orbital floor but autogenous grafts are preferable. Ref : Diseases of ENT by Dhingra 6th edition Pgno : 184", "cop": 1, "opa": "Orbital floor fracture", "opb": "Zygomatic bone fracture", "opc": "Mandibular fracture", "opd": "Hyoid bone fracture", "subject_name": "ENT", "topic_name": "Nose and paranasal sinuses", "id": "4677be85-ea2b-4ea6-ac2f-774aad7b6811", "choice_type": "single"} {"question": "A minor symptom of sinusitis is", "exp": "The clinical symptoms of acute sinusitis have been classified into major and minor. Major Facial pain or pressure Purulent nasal discharge Fever Nasal congestion Nasal obstruction Hyposmia or Anosmia Facial congestion or fullness Minor Headache Cough Fatigue Halitosis Dental pain Ear pain or pressure", "cop": 4, "opa": "Nasal blockage", "opb": "Facial congestion", "opc": "Nasal congestion", "opd": "Halitosis", "subject_name": "ENT", "topic_name": null, "id": "8a2f7069-2710-476c-aae5-d34200949e6f", "choice_type": "single"} {"question": "The earliest symptom of glomus tumor is", "exp": "Ans. A Pulsatile tinnitus \"The two most common presenting symptoms of paraganglioma of temporal bone (Glomus tumor) deg are conductive hearing loss and pulsatile tinnnus\" Hearing loss is conductive and slowly progressive. Tinnitus is pulsatile and of swishing character\", synchronous with pulse\", and can be temporarily stopped by carotid pressure\", Thus, both pulsatile tinnitus and deafness are seen in glomus tumor.", "cop": 1, "opa": "Pulsatile tinnitus", "opb": "Hearing loss", "opc": "Poly growth", "opd": "Blood stained otorrhoea", "subject_name": "ENT", "topic_name": null, "id": "6641d304-a7f9-434b-bb68-038bd76ffc02", "choice_type": "single"} {"question": "Treatment of choice in bilateral ethmoidal polyp is", "exp": "Ref : Dhingra 7e pg 195.", "cop": 2, "opa": "Ethmoidectomy", "opb": "FESS", "opc": "Endoscopic removal", "opd": "Chemotherapy and Radiotherapy", "subject_name": "ENT", "topic_name": "Nose and paranasal sinuses", "id": "d86a8e79-7af1-409a-9c6e-a0484503e8d4", "choice_type": "single"} {"question": "The danger space in front of prevertebral fascia extends till", "exp": null, "cop": 3, "opa": "T4 vertebrae", "opb": "T2 vertebrae", "opc": "Diaphragm", "opd": "Coccyx", "subject_name": "ENT", "topic_name": null, "id": "7114cb34-cc32-4d8b-9083-1a8d1315cc54", "choice_type": "single"} {"question": "Epley manoeuvre", "exp": "This manoeuvre repositions the otoconial debris from the posterior semicircular canal back into utricle. (Ref: Diseases of EAR, NOSE AND THROAT by PL DHINGRA - 6th Edition)", "cop": 1, "opa": "Positional veigo", "opb": "Otosclerosis", "opc": "ASOM", "opd": "CSOM", "subject_name": "ENT", "topic_name": "Ear", "id": "33c7b95a-2aae-4dab-bbb6-8b75833508b1", "choice_type": "single"} {"question": "Most comon cause of obstruction in atrophic rhinitis is", "exp": null, "cop": 1, "opa": "Excessive formation of crust", "opb": "Polyp", "opc": "Synechiae", "opd": "Hypertrophy of turbinate", "subject_name": "ENT", "topic_name": null, "id": "1abcfdf1-5537-479e-862a-07c21389b522", "choice_type": "single"} {"question": "Ranula is a", "exp": "Ranula The term \"ranula\" is used to describe a diffuse swelling in the floor of the mouth caused by either a mucous extravasation or, less commonly, a mucous retention cyst derived from the major sublingual or submandibular salivary glands. It is a cystic translucent lesion seen in the floor of the mouth on one side of the frenulum and pushing the tongue tip. Treatment It is complete surgical excision if small, or marsupialization, if large. Often it is not possible to excise the ranula completely because of its thin wall or ramifications in various tissue planes. (Ref:Textbook of diseases of ENT, PL Dhingra, 7th edition, pg no. 252)", "cop": 1, "opa": "Mucous cyst", "opb": "Tumour", "opc": "Submandibular gland swelling", "opd": "Swelling on dorsum of tongue", "subject_name": "ENT", "topic_name": "Oral cavity & Oesophagus", "id": "777acfe3-82c2-44e6-a7b5-32acf2723a4f", "choice_type": "single"} {"question": "Pseudosulcus in larynx", "exp": "Ans. c. Laryngopharyngeal reflux Pseudosulcus vocalic or laryngeal pseudosulcus is a variable finding in patients with laryngopharyngeal reflux. This sulcus is believed to arise from reflux induced swelling in the subglottic area and is located between the true vocal cords and the suglottic swelling. Laryngeal sulcus Laryngeal Pseudosulcus (Pseudosulcus Vocalis) This sulcus is believed to arise from reflux induced swelling in the subglottic area secondary to laryngotracheal reflux Located between the true vocal cords and the subglottic swelling Laryngeal True sulcus (Sulcus Vergeture) True sulcus is related to scarring of the vocal folds in the phonatory striking zone Located within the true vocal folds at the site of the adherence of vocal fold epithelium to the vocal ligament", "cop": 3, "opa": "Vocal abuse", "opb": "Tuberculosis", "opc": "Laryngopharyngeal reflux", "opd": "Chronic steroid use", "subject_name": "ENT", "topic_name": null, "id": "f3375602-e5f2-47b3-a192-6a76b29d5dbe", "choice_type": "single"} {"question": "Rinne&;s test is negative in", "exp": "Rinne test: In this test air conduction of the ear is compared with its bone conduction. A vibrating tuning fork is placed on the patient's mastoid and when he stops hearing, it is brought beside the meatus. If he still hears, AC is more than BC. Alternatively, the patient is asked to compare the loudness of sound heard through air and bone conduction. Rinne test is called positive when AC is longer or louder than BC. It is seen in normal persons or those having sensorineural deafness. A negative Rinne (BC > AC) is seen in conductive deafness. A negative Rinne indicates a minimum air-bone gap of 15-20 dB. Ref:- Dhingra; pg num:-22", "cop": 2, "opa": "Acoustic neuroma", "opb": "Tympanosclerosis", "opc": "Meniere's disease", "opd": "Sensorineural deafness", "subject_name": "ENT", "topic_name": "Ear", "id": "d0b865df-5575-4604-8c7c-00056e10674e", "choice_type": "single"} {"question": "Most common cause of stridor in children is", "exp": "The most common cause of stridor in children is Laryngomalacia. Laryngomalacia is due to excessive flaccidity of supraglottic larynx which is sucked during inspiration. It manifests at bih or soon after and usually disappear by 2 years of age. Re: Textbook of Ear, Nose and Throat, Dhingra, 7th Edition; Pg no: 333", "cop": 1, "opa": "Laryngomalacia", "opb": "Congenital laryngeal paralysis", "opc": "Foreign body in larynx", "opd": "Congenital laryngeal tumors", "subject_name": "ENT", "topic_name": "Larynx", "id": "302d7558-1b28-4930-a385-c81e0f5f8dac", "choice_type": "single"} {"question": "Topical mitomycin C is used to aid the following treatment", "exp": "Mitomycin-C is an antineoplastic antibiotic that acts as an alkylating agent by inhibiting DNA and protein synthesis. ... Topical application of mitomycin C (0.4 mg/mL) was used as an adjuvant treatment in the endoscopic laser management of laryngeal and tracheal stenosis Ref: Internet sources.", "cop": 2, "opa": "Endoscopic treatment of angiofibroma", "opb": "Treatment of laryngotracheal stenosis", "opc": "Skull base osteomyelitis", "opd": "Sturge weber syndrome", "subject_name": "ENT", "topic_name": "Larynx", "id": "cd437baf-a648-415e-b825-39b7261fb99f", "choice_type": "single"} {"question": "Surfer's ear is", "exp": null, "cop": 1, "opa": "Exostosis", "opb": "Otosclerosis", "opc": "Otitis externa", "opd": "Squamous cell Ca", "subject_name": "ENT", "topic_name": null, "id": "c9c32ee9-9652-44c4-af23-1edb4014b31a", "choice_type": "single"} {"question": "CSF is similar to", "exp": "Perilymph resembles extracellular fluid and rich in Na ions. It communicates with CSF through aqueduct of the cochlea which opens into scala tympani near the round window. Reference: Dhingra 6th edition.", "cop": 2, "opa": "Endolymph", "opb": "Perilymph", "opc": "Coilymph", "opd": "Urine", "subject_name": "ENT", "topic_name": "Ear", "id": "11b5d24f-ad31-4559-8620-c7554e5498f1", "choice_type": "single"} {"question": "Halo sign and handkerchief test are positive in", "exp": "Ans. is 'a' i.e., CSF Rhinorrhoea Tissue test (Handker chief test) and holo sign are for CSF Rhinorrhoea. Detection of CSF LeakBiochemical testsConcentrations of Glucose are higher in CSF than in nasal discharge. Glucose value > 30-40 mg% and protein value < 100 mg % (max 200 mg %) suppo a diagnosis of CSF leak.Presence of p, transferrin is the most definitive test for detection of CSF and P2 transferrin assay is the test of choice when a confirmatory test is needed, because of high sensitivity as well as specificity.(3-trace protein (prostaglandin D synthase) is also used, however it is nonspecific as it is also present in human testes, hea and seroma.Basic clinical testsTissue test (Handker chief test) : - Unlike nasal mucous, CSF does not cause a tissue to stiffen.Filter paper test : - Sample of nasal discharge on a filter paper exhibits a light CSF border and a dark central area of blood, i.e., double ring sign or halo sign.Queckensted test : - Compression of the jugular vein leas to increased CSF leak due to increase in ICP.Rhinoscopy : - Visualization of CSF leak from paranasal sinus.CSF tracersIntrathecal fluorescein dye adminstration, radionuclide cisternography, CT cisternography.", "cop": 1, "opa": "CSF Rhinorrhoea", "opb": "Deted nasal septum", "opc": "Nasal Myiasis", "opd": "Choanal atresia", "subject_name": "ENT", "topic_name": null, "id": "4cd477df-3e5d-463b-b8e3-7d4ddf46e1ac", "choice_type": "single"} {"question": "End tracheostomy is performed in patients undergoing surgery for", "exp": "Answer: a) Laryngectomy (DHINGRA 6th ED, P-316)TRACHEOSTOMY* Emergency tracheostomy* Elective or tranquil tracheostomy* Permanent tracheostomy* Percutaneous dilatational tracheostomy* Mini tracheostomy (cricothyroidotomy)Emergency tracheostomy* It is employed when airway obstruction is complete or almost complete and there is an urgent need to establish the airway.* Intubation or laryngotomy are either not possible or feasible in such cases.Elective tracheostomy (tranquil, orderly or routine tracheostomy)* Almost all operative surgical facilities are available, endotracheal tube can be put and local or general anaesthesia can be given.* Elective tracheostomy is often temporary and is closed when indication is over.* It is of two types:> Therapeutic, to relieve respiratory obstruction, remove tracheobronchial secretions or give assisted ventilation.> Prophylactic, to guard against anticipated respiratory obstruction or aspiration of blood or pharyngeal secretions such as in extensive surgery of tongue, floor of mouth, mandibular resection or laryngofissure.Permanent tracheostomy* This may be required for cases of bilateral abductor paralysis or laryngeal stenosis.* In laryngectomy or laryngopharyngectomy, lower tracheal stump is brought to surface and stitched to the skin.High tracheostomy* Above the level of thyroid isthmus (isthmus lies against II, III, IV tracheal rings)* It violates the I tracheal ring* Can cause perichondritis of the cricoid cartilage and subglottic stenosis* Always avoided* Only indication: carcinoma larynxMid tracheostomy* Preferred one* Through II or III rings (isthmus either divided or retracted)Low tracheostomy* Below the level of isthmus", "cop": 1, "opa": "Laryngectomy", "opb": "Laryngofissure surgery", "opc": "Oropharyngeal growth", "opd": "Obstructive sleep apnea with stridor", "subject_name": "ENT", "topic_name": "Larynx", "id": "895f4932-6852-4dba-9106-0a31e8b63545", "choice_type": "single"} {"question": "Citelli's abscess is", "exp": "Ans. is 'b' i.e., Formed over occipital bone Abscesses related to mastoid infection* Acute mastoiditis can be complicated by following abscesses:-1) Postauricular abscess:- This is the commonest subperiosteal abscess associated with acute mastoiditis. It occurs lateral to the cortex of mastoid in MacEwen's triangle.2) Bezold abscess: - Pus passes through mastoid tip(UP06) and presents as upper neck swelling. The abscess may:-i) Lies deep to stemocleidomastoid(DNB 07, A1IMS 92)ii) Follow the posterior belly of diagastric and presents swelling between mastoid tip and angle of jaw.iii) Be present in posterior triangleiv) Reach the parapharyngeal pouchv) Track down along the carotid vessels.3) Zygomatic abscess: - Infection of zygomatic air cells.4) Meatal (Luc's) abscess: - Pus passes between the antrum and external osseous meatus.5) Citelli's abscess : - Abscess is formed behind the mastoid more towards occipital bone. Some consider it as abscess of diagastric triangle.6) Parapharyngeal and retropharyngeal abscess.", "cop": 2, "opa": "Drained through external meatus", "opb": "Formed over occipital bone", "opc": "Presents as upper neck swelling", "opd": "Retroorbital abscess", "subject_name": "ENT", "topic_name": "Ear", "id": "f7bc917e-82a0-44f3-b3e5-3192efbe3a2b", "choice_type": "single"} {"question": "Schuller's view and law's view is for", "exp": "Law's view It is lateral oblique view of mastoid. Patient lies in such a way that sagittal plane of the skull is parallel to the film and X-ray beam is projected 15deg cephalocaudal Structures seen are: 1. External auditory canal superimposed on internal auditory canal 2. Mastoid air cells 3. Tegmen 4. Lateral sinus plate 5. Temporo mandibular joint. (Key area of the mastoid, i.e. attic, aditus and antrum are not well seen) Schuller's view Similar to Law's view but cephalocaudal beam makes an angle of 30deg to the sagittal. Structures seen are: 1. External canal superimposed on internal canal 2. Mastoid air cells 3. Tegmen 4. Lateral sinus plate 5. Condyle of mandible 6. Sino-dural angle 7. Antrum and upper pa of attic (i.e. key area). (This view is taken to see the extent of pneumatisation, destruction of intercellular septa as in mastoiditis, location of sinus plate and tegmen, cholesteatoma and longitudinal fracture of 6, petrous pyramid)", "cop": 2, "opa": "Sphenoid sinus", "opb": "Mastoid air cells", "opc": "Foramen ovale and spinosum", "opd": "Carotid canal", "subject_name": "ENT", "topic_name": null, "id": "c16d680b-2651-4e11-8515-ca9d804c4674", "choice_type": "single"} {"question": "Acute tonsillitis is caused by", "exp": "B. i.e. (Beta hemolytic streptococcus) (240 - Dhingra)ACUTE TONSILLITIS* Haemolytic streptococcus is the most commonly infecting organism others causes may be - staphylococcus pneumococci or H. influenzae* Penicillin is the drug of choice*** Complications include - chronic tonsillitis, peritonsillar abscess, Parapharyngeal abscess cervical abscess,Acute otitis media, Rheumatic fever, acute glomerulonephritis, SABE* Tonsil is supplied by1. Tonsillar branch offacial artery*2. Ascending pharyngeal artery*3. Ascending palatine*4. Dorsal lingual*5. Descending palatine*TREATMENTS* Bedrest, giving plenty of fluids, analgesics* Penicilline is the drug of choice *** Erythromycin and ampicilline may be needed for resistant cases", "cop": 2, "opa": "H. influenza", "opb": "Beta-hemolytic streptococcus", "opc": "Staphylococcus aureus", "opd": "Pneumococcus", "subject_name": "ENT", "topic_name": "Pharynx", "id": "70a275dd-36bf-477b-ab5b-8f9b8a066071", "choice_type": "single"} {"question": "In infant most sensitive audiometric screening is", "exp": "Used as a screening procedure for infants. Other uses of ABR: To determine the threshold of hearing in infants; also in children and adults who do not cooperate and in malingerers. To diagnose retrocochlear pathology paicularly acoustic neuroma. To diagnose brainstem pathology, e.g. multiple sclerosis or pontine tumours. To monitor CN VIII intraoperatively in surgery of acoustic neuromas to preserve the function of the cochlea. Ref: Dhingra; 6th Edition; pg no 27", "cop": 2, "opa": "Electrocochleography", "opb": "BERA", "opc": "Coical evoked response", "opd": "Tympanometry", "subject_name": "ENT", "topic_name": "Ear", "id": "4a608aed-e462-483f-bd69-648952d92ad5", "choice_type": "single"} {"question": "Androphonia is treated by", "exp": null, "cop": 4, "opa": "Type 1 thyroplasty", "opb": "Type 2 thyroplasty", "opc": "Type 3 thyroplasty", "opd": "Type 4 thyroplasty", "subject_name": "ENT", "topic_name": null, "id": "16e45539-ae17-4a87-a748-5c9a6e343416", "choice_type": "single"} {"question": "Tonsillar fossa is bounded anteriorly by", "exp": "Tonsilar fossa is bounded by Anterior - Palatoglossal fold containing palatoglossal muscle Posterior - Palatopharyngeal fold containing palato pharyngeal muscle Apex - Soft palate, where both arches meet Base - Dorsal surface of posterior one-third of tongue. (Ref: Anatomy of head, neck and brain. Vishram Singh. P 227 )", "cop": 4, "opa": "Pharyngobasilar fascia", "opb": "Palatopharyngeal fold", "opc": "Buccopharyngeal fascia", "opd": "Palatoglossal fold", "subject_name": "ENT", "topic_name": "Pharynx", "id": "b90b1e07-21e0-4760-a2dc-8edd5fddac7d", "choice_type": "single"} {"question": "A 10 years boy presented with sensory neural deafness not benefited with hearing aid, Next treatment is", "exp": "A. i.e. (Cochlear implant) (125- Maqbool 11th) (121- Dhingra 4th)COCHLEAR IMPLANT - is indicated in patients with bilateral severe sensorineural deafness who do not respond to amplified sound stimuli* Cochlear implants are more useful in post lingually deaf patients i.e. who lost their hearing after acquisition of language* Stapedectomy with prosthesis replacement is the treatment of choice for otosclerosis*** Pathway for extension of the infection from meninges to labyrinth - Cochlear aqueduct*** **Cochlear implants electrodes are most commonly placed at cochlea to directly stimulate the fibre of VIII. nerve", "cop": 1, "opa": "Cochlear implant", "opb": "Fenestromy", "opc": "Stapedectomy", "opd": "Stapes fixation", "subject_name": "ENT", "topic_name": "Ear", "id": "a0a269b2-39d3-4952-ac4a-b94612a51731", "choice_type": "single"} {"question": "The most frequent site of branchial cyst is at", "exp": "The branchial cyst is common in the second decade of life but can occur at any age with equal frequency in both sexes. Cyst presents as a swelling in the upper pa of the neck anterior to the sternocleidomastoid muscle. Mass is smooth, round, fluctuant, non-tender and not transilluminate. A painful increase in size at the time of upper respiratory infection can occur. Anomalies of the second branchial arch are the most common. A brachial cyst maybe associated with sinus or fistula. A second arch brachial sinus has an external opening at junction of lower and middle of the anterior border of sternomastoid and may exude mucoid discharge. It may have an internal opening in the tonsillar fossa. Treatment involves surgical excision along with its tract of the present. (Ref: Textbook of diseases of ENT, PL Dhingra, 7th edition, pg no. 446,447)", "cop": 2, "opa": "Upper third of posterior border of sternocleidomastoid", "opb": "Upper third of anteromedial border of sternocleidomastoid", "opc": "Supraclavicular fossa", "opd": "Lower third of anterior border of sternocleidomastoid", "subject_name": "ENT", "topic_name": "Miscellaneous ENT", "id": "49e8a39c-b398-41e4-9e9c-cad9491c5625", "choice_type": "single"} {"question": "Surface area of tympanic membrance", "exp": "The surface area of tympanic membrane is 90 mm2. Only 55 mm2 is functional. Ref: PL Dhingra 6th edition of Ear, Nose and Throat; pg no 14", "cop": 4, "opa": "55mm2", "opb": "70mm2", "opc": "80mm2", "opd": "90mm2", "subject_name": "ENT", "topic_name": "Ear", "id": "24a52c08-b205-423e-a6f6-1e5633148213", "choice_type": "single"} {"question": "Tubercular laryngitis affects primarily", "exp": "The disease affects posterior pa of larynx more than anterior. Pas affected are (i) interarytenoid fold (ii) ventricular bands (iii) vocal cords and (iv) epiglottis, in that order. Tubercle bacilli, carried by sputum from the bronchi, settle and penetrate the intact laryngeal mucosa paicularly in the interarytenoid region (bronchogenic spread). This leads to the formation of submucosal tubercles which may later caseate and ulcerate. Ref: PL Dhingra 6th edition of Ear, Nose and Throat; Pg no 293", "cop": 2, "opa": "Anterior commissure", "opb": "Posterior commissure of larynx", "opc": "Anywhere within the larynx", "opd": "Superior surface of larynx", "subject_name": "ENT", "topic_name": "Larynx", "id": "6bac3f16-7999-4b84-bdde-4edb35066422", "choice_type": "single"} {"question": "Rinne's test is negative in", "exp": "A negative Rinne test is seen in conductive deafness - when bone conduction is greater than air conduction. Chronic suppurative otitis media is a disease of the middle ear and hence causes conductive deafness. (Ref: Diseases of EAR, NOSE AND THROAT by PL DHINGRA - 6th Edition. Page no. 22)", "cop": 2, "opa": "Presbycusis", "opb": "CSOM", "opc": "Labyrinthitis", "opd": "Meniere's disease", "subject_name": "ENT", "topic_name": "Ear", "id": "af52469e-331f-4605-a929-b7926910b7fa", "choice_type": "single"} {"question": "\"Steeple sign\" seen in", "exp": null, "cop": 1, "opa": "Croup", "opb": "Acule epiglottitis", "opc": "Laryngmalacia", "opd": "Quinry", "subject_name": "ENT", "topic_name": null, "id": "61e0cc18-9d27-4e75-9582-772d84f92bfc", "choice_type": "single"} {"question": "Mucoperichondrial flap in septoplasty is made on", "exp": "In cases of deted septum, make a slightly curvilinear incision, 2-3 mm above the caudal end of septal cailage on the concave side (Killian's incision). In case of caudal dislocation, a transfixion or hemitransfixion (Freer's) incision is made. The latter is septocolumellar incision between caudal end of septal cailage and columella. Raise mucoperichondrial or mucoperiosteal flap on one side only. Separate septal cailage from the vomer and ethmoid plate and raise mucoperiosteal flap on the opposite side of septum. Ref: Dhingra 7e pg 473.", "cop": 2, "opa": "Alar cailage", "opb": "Septal cailage", "opc": "Maxillary spine", "opd": "Sphenoid spine", "subject_name": "ENT", "topic_name": "Diagnostic and operative ENT", "id": "16701430-077b-4a9c-ab52-44f947224b13", "choice_type": "single"} {"question": "Merciful anosmia is seen in", "exp": "Merciful anosmia is seen in atrophic rhinitis(Ozaena). There is foul smell from the nose, but the patient himself is unaware of smell due to marked anosmia due to degenerative changes Ref: Dhingra 7e pg 173.", "cop": 2, "opa": "Nasal polyp", "opb": "Atrophic rhinitis", "opc": "Rhinosporidiosis", "opd": "Rhinoscleroma", "subject_name": "ENT", "topic_name": "Nose and paranasal sinuses", "id": "668eedaf-9165-4f2e-b86e-e9973eb8727e", "choice_type": "single"} {"question": "Steeple sign is seen in", "exp": "Acute laryngotracheobronchitis or croup is a viral infection caused by parainfluenza type 1, 2, and sometimes 3. The critical area involved is subglottic larynx producing oedema with stridor and respiratory distress. X-ray (PA view) larynx shows typical \"steeple sign\" but X-rays are avoided as any manipulation may precipitate acute obstruction. Ref: PL Dhingra 7th edition of Ear, Nose and Throat; Pg no 328", "cop": 1, "opa": "Croup", "opb": "Acute epiglottitis", "opc": "Laryngomalacia", "opd": "Quinsy", "subject_name": "ENT", "topic_name": "Larynx", "id": "9ff1b39e-3523-4c43-aeb4-2e8190f093bd", "choice_type": "single"} {"question": "The valve of Hasner is", "exp": "Nasolacrimal duct (NLD) opens into the Inferior meatus through the Hasner`s valve.The direction of NLD--downward, backward, and laterally from the lacrimal sac to the nose, Length- 1.8 cmSphenoid sinus and posterior ethmoids open into the sphenoethmoidal recess (Superior meatus)Frontal, Maxillary and Anterior ethmoids open into the middle meatus.Ref: Hazarika; 3rd ed; Pg 240", "cop": 1, "opa": "Opening of nasolacrimal duct", "opb": "Sphenoidal sinus opening", "opc": "Frontal sinus opening", "opd": "Ethmoidal sinus opening", "subject_name": "ENT", "topic_name": "Nose and paranasal sinuses", "id": "7ff9749d-b4c2-4ec3-a41b-3271b7835e96", "choice_type": "single"} {"question": "Columellar tympanoplasty is", "exp": "Types of Tympanoplasty Wullstein classified tympanoplasty into five types . Type I Defect is perforation of tympanic membrane which is repaired with a graft. It is also called myringoplasty. Type II Defect is perforation of tympanic membrane with erosion of malleus. Graft is placed on the incus or remnant of malleus. Type III Malleus and incus are absent. Graft is placed directly on the stapes head. It is also called Myringostapediopexy or Columella tympanoplasty. Type IV Only the footplate of stapes is present. It is exposed to the external ear, and graft is placed between the oval and round windows. A narrow middle ear (cavum minor) is thus created to have an air pocket around the round window. A mucosa-lined space extends from the eustachian tube to the round window. Sound waves in this case act directly on the footplate while the round window has been shielded. Type V Stapes footplate is fixed but round window is functioning. In such cases, another window is created on horizontal semicircular canal and covered with a graft. Also called fenestration operation. Re: Textbook of Ear, Nose and Throat, Dhingra, 6th Edition", "cop": 3, "opa": "Type I", "opb": "Type II", "opc": "Type III", "opd": "Type IV", "subject_name": "ENT", "topic_name": "Diagnostic and operative ENT", "id": "99a27d0f-8070-4685-9ec5-b381c20a2858", "choice_type": "single"} {"question": "Rhinoscleroma is caused by", "exp": "Ans. (b) BacteriaRef. Dhingra's ENT 6th ed./156-57B RHINOSCLEROMA* It is a chronic granulomatous disease caused by gram negative coccobacillus- Klebsiella Rhinoscleromatis.* It has 3 stages:# Catarrhal or atrophic stage: characterized by foul smelling purulent nasal discharge and crusting (causing obstruction). This stage resembles atrophic rhinitis.# Granulomatous stage: in this stage there are painless and non-ulcerative granulomatous nodules in nasal mucosa. There is also subdermal infiltration of lower part of external nose and upper lip giving a 'woody'feel.# Cicatricial stage: the last stage which causes adverse conditions like stenosis of nares, distortion of upper lip, adhesions in the nose, nasopharynx and oropharynx. There may be subglottic stenosis with respiratory distress.# Rhinoscleroma causes woody infiltration of nose, so it is also known as Woody Nose.# Characteristic cells upon biopsy are: Miculicz's cell and Russel Bodies.# Treatment: Streptomycin (lg/day) and tetracycline (2g/day) are given together for 4-6 weeks.", "cop": 2, "opa": "Virus", "opb": "Bacteria", "opc": "Fungus", "opd": "Anaerobes", "subject_name": "ENT", "topic_name": "Nose and PNS", "id": "50e7658a-8c24-46c3-a9c1-575f25b378ec", "choice_type": "single"} {"question": "Mucocele most commonly occurs", "exp": "Mucoceles are dilated, mucous-containing sacs that are lined by mucous membranes and result from continuous secretions into an obstructed sinus. The frontal sinus is most commonly affected, whereas the sphenoid sinus is rarely involved. Sinus expansion with thinning and remodeling of the sinus walls occurs. When the frontal sinus is involved, as it expands downward and outward proptosis can occur. Ref : ENT textbook by Dhingra 6th edition Pgno : 198", "cop": 1, "opa": "Frontal sinus", "opb": "Ethmoid sinus", "opc": "Maxillary sinus", "opd": "Sphenoid sinus", "subject_name": "ENT", "topic_name": "All India exam", "id": "753851d5-f472-4dd3-a02a-84785fa336e3", "choice_type": "single"} {"question": "Gradenigo's triad is caused due to involvement of", "exp": "Gradenigo Syndrome (GS) is classically described as a clinicaltriad of otitis media, facial pain, and abducens palsy that in the past most commonly developed from infection in the petrous temporal bone (i.e., petrous apicitis). Ref otolaryngology dhingra 3e p278", "cop": 2, "opa": "Abducens nerve", "opb": "Facial nerve", "opc": "Optic nerve", "opd": "Vestibulocochlear nerve", "subject_name": "ENT", "topic_name": "Ear", "id": "754303cc-039d-4a3c-b3dc-d101a01cfacd", "choice_type": "single"} {"question": "In sesorineural hearing loss, weber's test is lateralized to", "exp": "Ans. is 'a' i.e., Normal ear Test Normal Conductive deafness SN deafness Rinne AC > BC (Rinne positive) BC > AC (Rinne AC > BC (Rinne Weber Not lateralized negative) Lateralized to negative) Lateralized to ABC Same as examiner's poorer ear Same as better ear Reduced Schwabach Equal examiner's Lengthened Shoened", "cop": 1, "opa": "Normal ear", "opb": "Defective ear", "opc": "Not lateralized", "opd": "May alternate", "subject_name": "ENT", "topic_name": null, "id": "eaee73c0-f879-44e7-abe6-83434017cefa", "choice_type": "single"} {"question": "Sinus of Morgagni is present in", "exp": "It is a space between the base of the skull and upper free border of superior constrictor muscle. Through it enters the 1. Eustachian tube 2. Levator veli palatine 3. Tensor Veli palatine 4. Ascending palatine aery (facial aery). (Ref: Diseases of ENT, PL Dhingra, 7th edition, pg no. 271)", "cop": 2, "opa": "Oropharynx", "opb": "Nasopharynx", "opc": "Hypopharynx", "opd": "Laryngopharynx", "subject_name": "ENT", "topic_name": "Pharynx", "id": "f7099219-e0f7-4a1d-ae4b-f6011146d42c", "choice_type": "single"} {"question": "Type of tympanic membrane perforation commonly seen in tubotympanic CSOM is", "exp": "Ref: Diseases of ENT by Dhingra 6th edition Pgno : 69-74 Differences between tubotympanic and atticoantral type of CSOM Tubotympanic or safe type Atticoantral or unsafe type Discharge Profuse, mucoid, odourless Scanty, purulent, foul smelling Perforation Central Attic or marginal Granulations Uncommon Common Polyp Pale Red and fleshy Cholesteatoma Absent Present Complications Rare Common Audiogram Mild to moderate conductive deafness Conductive or mixed deafness", "cop": 1, "opa": "Central", "opb": "Antero-superior", "opc": "Postero-superior", "opd": "Postero-inferior", "subject_name": "ENT", "topic_name": "Ear", "id": "13b1399d-c48f-4da9-91f7-706106b493af", "choice_type": "single"} {"question": "Lyre sign is seen in", "exp": "Carotid body tumour arises from the chemoreceptor cells in the carotid body, hence also called chemodectoma. Mostly presents after 40 years. It is a very slow-growing tumour and the history of mass in the neck may extend into several years. It presents as a painless swelling which is pulsatile.MRI angiography shows splaying of internal and external carotid aeries (Lyre's sign).Dhingra 6e pg: 392", "cop": 2, "opa": "Thyroglossal fistula", "opb": "Carotid body tumor", "opc": "Carcinoma maxilla", "opd": "Abdominal aoic aneurysm", "subject_name": "ENT", "topic_name": "Miscellaneous ENT", "id": "e71ab1f0-e572-40f6-aac7-a9ecc3e065ea", "choice_type": "single"} {"question": "Galens anastomosis between SLN and RLN is", "exp": "Galens anastomosis is purely sensory. Branch of internal branch of superior laryngeal nerve communicating with the recurrent laryngeal nerve in the wall of the laryngopharynx supplying sensory fibers to the latter. Ref:- TB of ENT, Hazarika; pg num:- 623", "cop": 3, "opa": "Pure motor", "opb": "Secretomotor", "opc": "Pure sensory", "opd": "Mixed", "subject_name": "ENT", "topic_name": "Larynx", "id": "29d158f7-b2ec-48d5-a12e-2bcdfcabeee0", "choice_type": "single"} {"question": "Mastoid tip is involved in", "exp": "Bezold abscess can occur following acute coalescent mastoiditis when pus breaks through the thin medial side of the tip of the mastoid and present as a swelling in the upper pa of the neck. (Ref: Textbook of diseases of ENT, PL Dhingra, 7th edition, pg no. 87)", "cop": 1, "opa": "Bezold abscess", "opb": "Luc abscess", "opc": "Citelli abscess", "opd": "Parapharyngeal abscess", "subject_name": "ENT", "topic_name": "Ear", "id": "28508037-0ac6-4993-835b-9ffc47e3edda", "choice_type": "single"} {"question": "Tonsils reach their maximum size by", "exp": "Tonsils are most active from 4 to 10 years of age. Involution begins after pubey resulting in a decrease of B-cell production and a relative increase in the ratio of T to B cells. (Ref: Diseases of Ear, Nose and Throat, PL Dhingra; 7th edition ,pg no. 292)", "cop": 4, "opa": "1 year", "opb": "3 years", "opc": "5 years", "opd": "12 years", "subject_name": "ENT", "topic_name": "Pharynx", "id": "3fa90d79-a694-4a0b-adfb-bd8ee2faf4e9", "choice_type": "single"} {"question": "Stage IV a with thyroid cartilage invasion in larynegeal carcinoma is treated with", "exp": null, "cop": 4, "opa": "Radiotherapy", "opb": "Hemilaryngectomy", "opc": "Total laryngectomy", "opd": "Total laryngectomy with radiotherapy", "subject_name": "ENT", "topic_name": null, "id": "9f1e4d5d-094f-48b3-bf5d-aa6e694f9ec1", "choice_type": "single"} {"question": "Mikulicz and Russell bodies are characteristic of", "exp": "Biopsy shows infiltration of submucosa with plasma cells, lymphocytes, eosinophils, mikulicz cells and Russell bodies.\nThe latter two are diagnostic features of the disease RHINOSCLEROMA. Mikulicz cells are large foam cells with a central nucleus and vacuolated cytoplasm containing causative bacilli. Russell bodies are homogenous eosinophilic inclusion bodies found in the plasma cells. They occur due to the accumulation of immunoglobulins secreted by the plasma cells.", "cop": 3, "opa": "Lethal midline granuloma", "opb": "Plasma cell disorder", "opc": "Rhinoscleroma", "opd": "Rhinosporidiosis", "subject_name": "ENT", "topic_name": null, "id": "063db332-f818-4a03-af9a-82727f0c9dd8", "choice_type": "single"} {"question": "Epley's maneuver is used in the treatment of", "exp": "Epley`s maneuver is used in the treatment of Positional Veigo (BPPV)Dix Hallpike`s test is the diagnostic testOther exercises are: Semont`s maneuver, Brandt Darrof exercisesSurgeries: Singular neurectomy, Posterior canal occlusion, Labyrinthectomy, Vestibular nerve sectionRef: Hazarika; 3rd edition; Page no: 108", "cop": 1, "opa": "Positional veigo", "opb": "Otosclerosis", "opc": "ASOM", "opd": "CSOM", "subject_name": "ENT", "topic_name": "Ear", "id": "43cfb3cb-6fb9-41da-8ccf-e3b23f828394", "choice_type": "single"} {"question": "A pure tone audiogram with a dip at 2000Hz is characteristic of", "exp": "In otosclerosis, there is a dip in bone conduction curve. It is different at different frequencies but maximum at 2000Hz and is called Carha's notch. Ref: Diseases of EAR, NOSE and THROAT by PL Dhingra; 7th Edition; Page no.96", "cop": 3, "opa": "Presbycusis", "opb": "Ototoxicity", "opc": "Otosclerosis", "opd": "Noise induced hearing loss", "subject_name": "ENT", "topic_name": "Ear", "id": "4467e3e8-b887-4138-9e1c-2fd03b9ca26d", "choice_type": "single"} {"question": "Otoconia is related to", "exp": "Otoconia are bio-crystals which couple mechanic forces to the sensory hair cells in the utricle and saccule, a process essential for us to sense linear acceleration and gravity for the purpose of maintaining bodily balance. In fish, structurally similar bio-crystals called otoliths mediate balance Ref : Internet sources.", "cop": 2, "opa": "Hearing", "opb": "Balance", "opc": "Smell", "opd": "Taste", "subject_name": "ENT", "topic_name": "Ear", "id": "6a97b712-564d-45cb-8ac6-556f97abb63f", "choice_type": "single"} {"question": "Bryce&;s sign is seen in", "exp": "Bryce's sign: It is seen in laryngocele. When the swelling is pressed, a gurgling sound is produced. Ref: Textbook of diseases of ENT, PL Dhingra, 7th edition, pg no. 507", "cop": 1, "opa": "Laryngocele", "opb": "Post cricoid carcinoma", "opc": "Angiofibroma", "opd": "Chronic tonsillitis", "subject_name": "ENT", "topic_name": "Larynx", "id": "1e75d3e2-6a8c-40a6-876a-21607f085cdb", "choice_type": "single"} {"question": "Bezold abscess is located in", "exp": "Bezold abscess: It can occur following acute coalescent mastoiditis when pus breaks through the thin medial side of the tip of the mastoid and presents as a swelling in the upper pa of the neck. The abscess may (i) Lie deep to sternocleidomastoid pushing the muscle outwards. (ii) Follows the posterior belly of digastric and present as swelling between the tip of the mastoid and angle o the jaw, (iii) Present in the upper pa of the posterior triangle, (iv) Reaches the parapharyngeal space or (v) Tracks down along the carotid vessels. (Ref: Textbook of diseases of ENT, PL Dhingra, 7th edition, pg no. 87)", "cop": 2, "opa": "Submandibular region", "opb": "Sternocleidomastioid muscle", "opc": "Digastric triangle", "opd": "Infratemporal region", "subject_name": "ENT", "topic_name": "Ear", "id": "6f30bb76-cd59-413b-abbc-33fd8c87df30", "choice_type": "single"} {"question": "Vincent's angina is also known as", "exp": "(A) Trench mouth # Acute necrotizing ulcerative gingivitis:> It is also known as \"Plaut-Vincent angina\", \"Vincent's stomatitis\", \"Vincent's angina\", \"Vincent's infection\", \"acute ulcerative gingivitis\" or \"trench mouth.\"> Causes: Causative organisms include anaerobes such as Bacteroides and Fusobacterium as well as spirochetes (Borrelia and Treponema spp.).", "cop": 1, "opa": "Trench mouth", "opb": "Dry mouth", "opc": "Foot mouth disease", "opd": "Red mouth", "subject_name": "ENT", "topic_name": "Miscellaneous (E.N.T.)", "id": "fc3e0dfb-4e01-4bc8-81d5-2822a1c869cb", "choice_type": "single"} {"question": "Caldwell luc approach opens", "exp": "Caldwell luc approach opens maxillary sinus. Ref- Dhingra, 6th edition, Page 412.", "cop": 1, "opa": "Maxillary sinus", "opb": "Frontal sinus", "opc": "Sphenoid sinus", "opd": "Ethmoid sinus", "subject_name": "ENT", "topic_name": "Diagnostic and operative ENT", "id": "3f4696bf-f6a8-48db-8ba4-a071193fef4a", "choice_type": "single"} {"question": "Cadaveric position of vocal cords is seen in", "exp": "Position of cord Location from midline Defect Median midline RLN palsy Paramedian 1.5 mm RLN palsy Intermediate 3.5 mm Both RLN and superior laryngeal nerve palsy Gentle abduction 7.5mm ADDUCTOR PALSY", "cop": 1, "opa": "Bilateral recurrent and superior laryngeal nerve palsy", "opb": "Bilateral recurrent laryngeal nerve palsy", "opc": "Unilateral superior laryngeal nerve palsy", "opd": "Bilateral superior laryngeal nerve palsy", "subject_name": "ENT", "topic_name": null, "id": "6b37a74f-7515-4fb5-a11a-fd0059984dd3", "choice_type": "single"} {"question": "The most common site of osteoma is", "exp": "The commonest site of osteoma in the PNS is frontal sinus.Commonest benign tumor of PNS is an osteomaMost common benign tumor of the nose is capillary hemangioma (nasal septum)Gardner`s syndrome- Colonic polyposis, multiple soft tissue swellings, and facial osteomas(Autosomal Dominant hereditary defect of connective tissue)Ref: Hazarika; 3rd ed; Pg 370", "cop": 3, "opa": "Maxillary sinus", "opb": "Ethmoidal sinus", "opc": "Frontal sinus", "opd": "Sphenoidal sinus", "subject_name": "ENT", "topic_name": "Nose and paranasal sinuses", "id": "7c294c9c-a64c-4c90-aa12-7cb4057f5492", "choice_type": "single"} {"question": "In Jarjaway fracture of nasal bone, the fracture line runs", "exp": "Ans. c (Horizontal) (Ref. PL Dhingra ENT 4th/p. 141)Hint: Jar a horizontally jaway!!! & vertically CHEVAL !!!CauseMeans?JARJAWAY FRACTURE of nasal septumResults from blow from the frontIt starts just above the anterior nasal spine and runs horizontally backwards just above the junction of the septal cartilage with the vomer.CHEVALLETE FRACTURE of septal cartilageResults due to blow from belowIt runs vertically from the anterior nasal spine upwards to the junction of bony and cartilaginous dorsum of the nose.# Nasal bone is the most common fracture of the face.# Nasal bone fracture is best diagnosed b\\ palpation. not by x-ray.# In Chevallet fracture of nasal bone. the mechanism of trauma is Blow from below.", "cop": 3, "opa": "Vertical", "opb": "Spiral", "opc": "Horizontal", "opd": "In any direction", "subject_name": "ENT", "topic_name": "Nose and PNS", "id": "fad01eb5-176c-4c7f-b462-c0e48583521a", "choice_type": "single"} {"question": "An elderly diabetic with excruciating pain in ear, appearance of granulation in meatus, skull base infection with facial paralysis should be treated with", "exp": "All clinical features are suggestive of malignant otitis externa, an inflammatory condition of the external ear. Malignant otitis externa (also k/a Necrotizing external otitis) Malignant otitis externa is an inflammatory condition of the external ear usually spreading deep to cause osteomyelitis of temporal bone and base of skull. It occurs primarily in immunocompromised persons, especially older persons with diabetes mellitus, and is often initiated by self-inflicted or iatrogenic trauma to the external auditory canal [May also be seen in pts. who received radiotherapy to skull base I The most frequent pathogen is Pseudomonas aeruginosa. Others may be S. aureus, Staphylococcus epidermidis, Aspergillus, Actinomyces, and some gram-negative bacterial. One of the hallmarkof malignant otitis externs is granulation tissue in the external auditory canal, especially at the hone-cailage junction. As the infection spreads to the temporal bone, it may extend into the cranium and result in cranial nerve palsies (commonly the facial nerve) Cranial nerve involvement indicates poor prognosis. Death is usually due to intracranial complications such as sigmoid sinus thrombosis. It has high moality rate due to which the name 'malignant' is used for this disease. Treatment Includes correction of immunosuppression (when possible), local treatment of the auditory canal, long-term systemic antibiotic therapy, and in selected patients, surgery. - In all cases, the external ear canal is cleansed and a biopsy specimen of the granulation tissue sent for culture. - IV antibiotics is directed against the offending organism. - For Pseudomonas aeruginosa, the most common pathogen, the regimen involves an antipseudomonal penicillin or cephalosporin (3rd generation-piperacillin or ceftazidime) with an aminoglycoside. A fluoroquinolone antibiotic can be used in place of the aminoglycoside. Ear drops containing antipseudomonal antibiotic e.g. ciproflaxacin plus a glucocoticoid is also used. - Early cases can be managed with oral and otic fluoroquinolones only. - Extensive surgical debridement once an impoant pa of the treatment is now rarely needed.", "cop": 1, "opa": "Penicillin", "opb": "Ciprofloxacin", "opc": "2nd generation ciphalosporin", "opd": "Erythromycin", "subject_name": "ENT", "topic_name": null, "id": "2f39ca11-d54f-459e-aee2-7ad761bdc720", "choice_type": "single"} {"question": "Triad of Meniere's disease is", "exp": "A. i.e. (Fluctuating hearing loss, vertgo, tinnitus) (100 Dhingra 4th) (107-8 - Maqbool 11th)Cardinal symptoms of Meniere's disease1. Episodic vertigo2. Fluctuating hearing loss (Sensory neural)3. Tinnitus4. Sense of fullness or pressure in the involved ear** Recruitment phenomenon is seen in - Meniere's diseaseTREATMENTSMedical Surgical1. Reassurance, vitamins (Nicotinic acid and A and D)2. Bed rest. Diuretic (acetazolamide)3. Vestibular sedative (Dimanhydrinate, promethazine theoclate prochlorperazine, Diazepam)4. Vasodilators** (Betahistadine hydrochloride)(i) Inhalation of carbogen (5% CO2 with 95% O2)(ii) Histamine drip5. Intratympanic gentamicin therapy1. Labyrinthectomy2. Intermittent low pressure pulse therapy(Meniett device therapy)* LERMOYEZ'S SYNDROME - is a variant of Meniere syndrome in which hearing loss and tinnitus occurs first, followed by vertigo that appears suddenly and with it hearing and tinnitus improve", "cop": 1, "opa": "Fluctuating hearing loss, vertigo, tinnitus", "opb": "Conductive deafness, vertigo tinnitus", "opc": "Vertigo, tinnitus, sense of fullness", "opd": "Vertigo, tinnitus, cataract", "subject_name": "ENT", "topic_name": "Ear", "id": "1f7277eb-e79e-4fb9-a910-b06578b68d7f", "choice_type": "single"} {"question": "Recruitment phenomenon is seen in", "exp": "Recruitment test which is a special audiometry test is positive in Meniere's disease. Ref: Diseases of EAR, NOSE and THROAT by PL Dhingra; 7th Edition; page no.113", "cop": 2, "opa": "Otosclerosis", "opb": "Meniere's disease", "opc": "Acoustic schwannoma", "opd": "Otitis media with effusion", "subject_name": "ENT", "topic_name": "Ear", "id": "3b322102-b0b0-48a0-addd-c1f98c4a69a7", "choice_type": "single"} {"question": "CSF Rhinorrhea occurs d/t", "exp": "Commonest area of # causing CSF Rhinorrhea → cribriform plate of ethmoid bone as it is extremely thin.\nOther possible areas of # causing CSF Rhinorrhea Frontal sinus (Posterior wall) or of the anterior cranial fossa.\nDifference between CSF Rhinorrhea & allergic or vasomotor rhinitis CSF contains glucose which can be demonstrated by chemical tests.\nCSF when collected into a test tube and allowed to stand, it remains clear in contradistinction to nasal discharge that leaves a sediment because of mucus and other proteins.\nNasal discharge stiffens the handkerchief and CSF dries soft in it.\nTreatment: Prophylactic antibiotics & X-ray * If more than 2 wks then correct it surgically.", "cop": 3, "opa": "Roof of orbit", "opb": "Frontal sinus", "opc": "Cribriform plate of ethmoid bone", "opd": "Sphenoid bone", "subject_name": "ENT", "topic_name": null, "id": "d2639fbf-9bcb-4d81-8092-37625b3baa06", "choice_type": "single"} {"question": "Laryngocele arises as a herniation of laryngeal mucosa through the following membrane", "exp": "Laryngocele arises as a herniation of laryngeal mucosa through the Thyrohyoid membrane. It is dilatation of laryngeal saccule and extends between the thyroid cailage and the ventricle. It may be internal, external or combined. Treatment is endoscopic or external excision. Ref: PL Dhingra 7th edition of Ear, Nose and Throat, pg no. 345", "cop": 1, "opa": "Thyrohyoid", "opb": "Cricothyroid", "opc": "Cricotracheal", "opd": "Crisosternal", "subject_name": "ENT", "topic_name": "Larynx", "id": "53901703-6ab7-4ebb-adbc-29b62be09d00", "choice_type": "single"} {"question": "Patient presents with high fever, signs of raised ICT and a past history of chronic otitis media likely diagnosis is", "exp": "Brain abscess develops through four stages. 1. Stage of invasion (initial encephalitis). It often passes unnoticed as symptoms are slight. Patient may have head-ache, low-grade fever, malaise and drowsiness. 2. Stage of localization (latent abscess). There are no symptoms during this stage. Nature tries to localize the pus by formation of a capsule. The stage may last for several weeks. 3. Stage of enlargement (manifest abscess). Abscess begins to enlarge. A zone of oedema appears round the abscess and is responsible for aggravation of symptoms. Clinical features at this stage are due to: (a) Raised intracranial tension. (b) Disturbance of function in the cerebrum or cerebellum, causing focal symptoms and signs. 4. Stage of termination (rupture of abscess). An expanding abscess in the white matter of brain ruptures into the ventricle or subarachnoid space resulting in fatal meningitis. Ref: Dhingra 7e pg 90.", "cop": 1, "opa": "Brain abscess", "opb": "Pyogenic meningitis", "opc": "Acute subarachnoid hemorrhage", "opd": "Acute osteomyelitis of skull bone", "subject_name": "ENT", "topic_name": "Ear", "id": "b04ccafa-74f3-4da3-9865-d8139b829e12", "choice_type": "single"} {"question": "Most common presentation in nasopharyngeal carcinoma is with", "exp": "This may be the only manifestation of nasopharyngeal cancer. A lump of nodes is found between the angle of the jaw and the mastoid and some nodes along with the spinal accessory in the posterior triangle of the neck. Nodal metastases are seen in 75% of the patients when first seen, about half of them with bilateral nodes (Ref: Diseases of Ear, Nose and Throat, P.L Dhingra, 7th edition, page 284)", "cop": 4, "opa": "Epistaxis", "opb": "Hoarseness of voice", "opc": "Nasal stuffiness", "opd": "Cervical lymphadenopathy", "subject_name": "ENT", "topic_name": "Pharynx", "id": "cd036b9b-46bf-40f9-8715-1250ebb3b0ee", "choice_type": "single"} {"question": "Antrum of Highmore is", "exp": "Antrum of Highmore is maxillary sinus.\nIt is the largest of paranasal sinuses.", "cop": 1, "opa": "Maxillary", "opb": "Ethmoid", "opc": "Sphenoid", "opd": "Frontal", "subject_name": "ENT", "topic_name": null, "id": "8dff35b0-7d96-4535-b372-dbfd72e4b193", "choice_type": "single"} {"question": "Fracture mandible occurs most commonly in", "exp": "Condylar fractures are most common fractures among the fractures of mandible. They are followed by in frequency of fractures of CABS Condylar process Angle Body Symphysis Caused by indirect trauma to the chin Ref: Dhingra 7e pg 207", "cop": 3, "opa": "Body", "opb": "Angle", "opc": "Condylar process", "opd": "Coronoid process", "subject_name": "ENT", "topic_name": "Miscellaneous ENT", "id": "fba69703-e166-490b-b874-f768f62638b8", "choice_type": "single"} {"question": "Tone decay test is done for", "exp": "THRESHOLD TONE DECAY TEST It is a measure of nerve fatigue and is used to detect retrocochlear lesions. Normally, a person can hear a tone continuously for 60 s. In nerve fatigue, he stops hearing earlier. The threshold tone decay test is simple and is performed in the following manner: A tone of 4000 Hz is presented at 5 dB above the patient's threshold of hearing, continuously for a period of 60 s. If patient stops hearing earlier, intensity is increased by another 5 dB. The procedure is continued till patient can hear the tone continuously for 60 s, or no level exists above the threshold where tone is audible for full 60 s. The result is expressed as number of dB of decay. A decay more than 25 dB is diagnostic of a retrocochlear lesion. Ref:- Dhingra; pg num:-26", "cop": 1, "opa": "Neural deafness", "opb": "Cochlear deafness", "opc": "Middle ear problem", "opd": "Otosclerosis", "subject_name": "ENT", "topic_name": "Ear", "id": "18a7f2e8-8e4e-4706-8ae5-f9184b9e4616", "choice_type": "single"} {"question": "Most common congenital dysplasia of Ear is", "exp": null, "cop": 4, "opa": "Michel aplasia", "opb": "Mondini's dysplasia", "opc": "Alexander's dysplasia", "opd": "Scheibe's dysplasia", "subject_name": "ENT", "topic_name": null, "id": "0e8a51d7-4054-44f6-8c40-aec842909426", "choice_type": "single"} {"question": "Sinusitis is LEAST involved in", "exp": "(Sphenoid) (181- Dhingra 4th)* Most common sinus involved is - Maxillary* Least common sinus involved in - Sphenoid* Order of sequence - Maxillary >ethmoid > frontal > sphenoid", "cop": 4, "opa": "Maxillary", "opb": "Ethmoid", "opc": "Frontal", "opd": "Sphenoid", "subject_name": "ENT", "topic_name": "Nose and PNS", "id": "abb09c59-f915-4046-a230-c369a02e56b9", "choice_type": "single"} {"question": "Brown's sign is seen In", "exp": "Brown's sign, also called pulsation sign, is seen in glomus tumour. When pressure is raised in the ear canal with a Siegel's speculum, tumour first pulsates vigorously and then blanches; reverse occurs when pressure is released.", "cop": 1, "opa": "Glomus tumour", "opb": "Meniere's disease", "opc": "Acoustic neuroma", "opd": "Otosclerosis", "subject_name": "ENT", "topic_name": "Ear", "id": "5310f118-549b-4120-ae64-aaa6bb13f189", "choice_type": "single"} {"question": "Fordyce&;s (Spots) Granules in oral cavity arise from", "exp": "Fordyce spots They are aberrant sebaceous glands present under the buccal or labial mucosa and shine through it as yellowish or yellow-brown spots. They are seen with equal frequency in both males and females and are considered normal. Ref:- Dhingra; pg num:-220", "cop": 1, "opa": "Sebaceous glands", "opb": "Mucous glands", "opc": "Minor salivary glands", "opd": "Taste buds", "subject_name": "ENT", "topic_name": "Oral cavity & Oesophagus", "id": "b92d27c0-33aa-4c42-a791-c791d1088ae2", "choice_type": "single"} {"question": "Horizontal semicircular canal responds to", "exp": "The three canals lie at right angles to each other but the one which lies at right angles to the axis of rotation is stimulated the most.Thus horizontal canal will respond maximum to the rotation on the veical axis. (Ref: Diseases of EAR, NOSE and THROAT by PL Dhingra; 6th edition; page no.17)", "cop": 2, "opa": "Horizontal acceleration", "opb": "Rotational acceleration", "opc": "Gravity", "opd": "Antero-posterior acceleration", "subject_name": "ENT", "topic_name": "Ear", "id": "9e307cf2-906c-4880-9c66-434b8ab8685f", "choice_type": "single"} {"question": "In Ramsay Hunt syndrome, most commonly involved nerve is", "exp": "Ramsay Hunt syndrome affects V, VII and VIII cranial nerves and among them, VII is most commonly affected. Ref: Dhingra 6th edition pg. 96", "cop": 2, "opa": "V", "opb": "VII", "opc": "VIII", "opd": "IX", "subject_name": "ENT", "topic_name": "Ear", "id": "9ed679cf-84d1-4050-b4f7-9100a433698e", "choice_type": "single"} {"question": "Boyce sign is seen in", "exp": "Zenkers diveiculum - A Zenker&;s diveiculum, also pharyngeal pouch, is a diveiculum of the mucosa of the human pharynx, just above the cricopharyngeal muscle. It is a pseudo diveiculum. Boyce sign:- Boyce's sign, a gurgling sound due to noise of splashing fluid accumulated in a large Zenker's diveiculum, can be heard on palpation. Although small diveicula might not cause symptoms, larger diveicula are usually symptomatic and surgery could be warranted. Ref:- Dhingra; pg num:-275", "cop": 2, "opa": "Epiglottis", "opb": "Zenkers diveiculum", "opc": "Plummer-Vinson syndrome", "opd": "Barretts esophagus", "subject_name": "ENT", "topic_name": "Pharynx", "id": "dec43560-0b7f-4542-bb98-c9154495ea44", "choice_type": "single"} {"question": "The length of Eustachian tube is", "exp": "Eustachian tube, also called auditory or pharyngotympanic tube, connects nasopharynx with the tympanic cavity. In an adult, it is about 36 mm long and runs downwards, forwards and medially from its tympanic end, forming an angle of 45deg with the horizontal. It is divided into two pas: bony, which is posterolateral, forms one-third (12 mm) of the total length and f ibrocailaginous, which is anteromedial, forms two-thirds (24 mm). The two pas meet at isthmus which is the narrowest pa of the tube . Reference-Dhingra 6/e, p57.", "cop": 3, "opa": "16 mm", "opb": "24 mm", "opc": "36 mm", "opd": "40 mm", "subject_name": "ENT", "topic_name": "Ear", "id": "c4d01fd4-428e-4be7-997e-d82953b2d8d3", "choice_type": "single"} {"question": "Dysphagia lusoria is due to", "exp": "The MC anomaly is right subclan aery arising from the descending aoa and travels behind the esophagus to complete its course to the right upper extremity, may cause significant posterior compression of the esophagus Ref : ENT textbook by Dhingra 6th edition Pgno : 447", "cop": 4, "opa": "Esophageal diveiculum", "opb": "Aneurysm of aoa", "opc": "Esophageal web", "opd": "Compression by aberrant blood vessel", "subject_name": "ENT", "topic_name": "All India exam", "id": "403543b5-327e-4ff4-8bec-323b19df47b5", "choice_type": "single"} {"question": "Ostmann fat pad is related to", "exp": "*There are small fat bodies located inferomedial to Eustachian tube. These are called Ostmann fat pads*They are impoant in normal closure of Eustachian tube and preventing transmission of nasopharyngeal pressure to the middle ear*These fat pads are absent in patulous tube syndromeRef: PL Dhingra, Diseases of Ear, Nose & Throat, 7th edition, pg no. 62", "cop": 3, "opa": "Ear lobule", "opb": "Buccal mucosa", "opc": "Eustachian tube", "opd": "Tip of nose", "subject_name": "ENT", "topic_name": "Nose and paranasal sinuses", "id": "fd14fc7c-4061-4d84-9af8-6ce18c31a517", "choice_type": "single"} {"question": "Alkaline Nasal Douche does not contain", "exp": null, "cop": 4, "opa": "Sodium chloride", "opb": "Sodium bicarbonate", "opc": "Sodium biborate", "opd": "Trisodium citrate", "subject_name": "ENT", "topic_name": null, "id": "490f7c5e-df84-47a7-9002-3ad4eb269c05", "choice_type": "single"} {"question": "Best management for inhaled Foreign body in an infant", "exp": "Tracheal and bronchial foreign bodies can be removed by bronchoscopy with full preparation and under general anaesthesia. Emergency removal of these foreign bodies is not indicated unless there is airway obstruction or they are of the vegetable nature (e.g. seeds) and likely to swell up. Methods to remove the tracheobronchial foreign body Conventional rigid bronchoscopy. Rigid bronchoscopy with telescopic aid. Bronchoscopy with C-arm fluoroscopy. Use of Dormia basket or Fogay's balloon for rounded objects. Tracheostomy first and then bronchoscopy through the tracheostomy. Thoracotomy and bronchotomy for peripheral foreign bodies. Flexible fibreoptic bronchoscopy in selected adult patients. Ref: PL Dhingra; Textbook of EAR, NOSE and THROAT; edition 7; page no 367", "cop": 3, "opa": "Tracheostomy", "opb": "Steroids", "opc": "Bronchoscopy", "opd": "IPPPV", "subject_name": "ENT", "topic_name": "Larynx", "id": "a68b4225-2611-4bef-ae41-4a6bdb71e832", "choice_type": "single"} {"question": "The prerequisite for endoscopic sinus surgery is", "exp": "Endoscopic surgery of inflammatory diseases of paranasal sinuses (sinusitis or polyp) requires a very detailed preoperative knowledge of the individual anatomical conditions and pathological changesCT scan is used best to visualize sinus areasCT scan provides an excellent definition of paranasal sinuses and is a prerequisite for endoscopic surgeryRef: PL Dhingra, Diseases of Ear, Nose & Throat, 7th edition, pg no. 220 - 221", "cop": 2, "opa": "MRI of paranasal sinuses", "opb": "CT of paranasal sinuses", "opc": "Mucocilliary clearing testing", "opd": "Acoustic tests", "subject_name": "ENT", "topic_name": "Nose and paranasal sinuses", "id": "14d35beb-8efb-47af-b98c-f1f31ce5765f", "choice_type": "single"} {"question": "The most common site of perforation in tympanic membrane is", "exp": "Perforations. They may be central, attic or marginal and are associated with chronic otitis media The most common site of Tympanic membrane perforation is antero-inferior quadrant. Ref : Diseases of ENT by Dhingra 6th edition Pgno : 56,71", "cop": 1, "opa": "Antero inferior", "opb": "Postero inferior", "opc": "Postero superior", "opd": "Antero superior", "subject_name": "ENT", "topic_name": "Ear", "id": "6264db2a-a94d-4806-a0d7-23cba0aeb0f7", "choice_type": "single"} {"question": "Epiglottis is derived from", "exp": null, "cop": 4, "opa": "1st arch", "opb": "2nd arch", "opc": "3rd arch", "opd": "4th arch", "subject_name": "ENT", "topic_name": null, "id": "74cd8224-3dab-4844-b70b-7781df166b02", "choice_type": "single"} {"question": "Sago grain appearance is seen in", "exp": "Myringitis bullosa It is a painful condition characterized by the formation of haemorrhagic blebs on the tympanic membrane and deep meatus. It is probably caused by a virus or Mycoplasma pneumonia. Ref: Dhingra, ENT, 6th edition, pg no.55", "cop": 1, "opa": "Healed myringitis bullosa", "opb": "Otomycosis", "opc": "Malignant otitis externa", "opd": "Keratosis obturans", "subject_name": "ENT", "topic_name": "Ear", "id": "58a51911-2b4a-4327-a38a-36aa1d1bf696", "choice_type": "single"} {"question": "Commonest cause of epistaxis in children is", "exp": "Commonest cause of epistaxis in children is trauma in Little&;s area", "cop": 1, "opa": "Trauma", "opb": "Foreign body", "opc": "Nasal diphtheria", "opd": "Enlarged adenosis", "subject_name": "ENT", "topic_name": "Nose and paranasal sinuses", "id": "ef0804b7-d767-435a-8110-f61d6c6f43b6", "choice_type": "single"} {"question": "Moure's sign can be detected in", "exp": "Moure`s sign- Absence of Laryngeal click or crepitusClassical sign in post-cricoid malignancy.Bocca`s sign - is also an absence of post-cricoid click or crackle in cricoid malignancy.Ref: Hazarika; 3rd ed; Pg 534", "cop": 3, "opa": "Nasopharyngeal carcinoma", "opb": "Oropharyngeal carcinoma", "opc": "Postcricoid carcinoma", "opd": "Supraglottic carcinoma", "subject_name": "ENT", "topic_name": "Larynx", "id": "86a83376-6be9-4ac9-9d63-8f7c206bb5a6", "choice_type": "single"} {"question": "Phelps sign is seen in", "exp": "Phelps sign is the loss of crust of bone between the carotid canal and jugular bulb in glomus jugulare as seen in CT scan. Ref: Dhingra 7th edition pg.122", "cop": 1, "opa": "Glomus jugulare", "opb": "Vestibular Schwannoma", "opc": "Menniers disease", "opd": "Neurofibromatosis", "subject_name": "ENT", "topic_name": "Ear", "id": "c23425ee-427c-4d5c-a7b5-b2a00d1f19fe", "choice_type": "single"} {"question": "In facial nerve injury, loss of lacrimation is due to involvement of", "exp": "Secretomotor fibres to lacrimal gland leave at geniculate ganglion greater superficial petrosal nerve Ref: Dhingra 6th edition Pg. 97", "cop": 3, "opa": "Chorda tympani nerve", "opb": "Buccal nerve", "opc": "Greater superficial petrosal nerve", "opd": "Deep petrosal nerve", "subject_name": "ENT", "topic_name": "Ear", "id": "1772962b-340d-4aa7-b74c-cad69b77824b", "choice_type": "single"} {"question": "Familial tendency is not seen in", "exp": null, "cop": 4, "opa": "Sarcomas", "opb": "Stomach carcinoma", "opc": "Colon carcinoma", "opd": "Laryngeal carcinoma", "subject_name": "ENT", "topic_name": null, "id": "f6d0dbc1-656c-4275-a256-f30332591447", "choice_type": "single"} {"question": "The glomus tumour invasion of jugular bulb is diagnosed by", "exp": "Jugular venography must be done to delineate multiple tumours, feeding vessels, embolization and involvement of jugular bulb, carotid aery or intradural extension. Ref: Dhingra 6th edition pg. 110", "cop": 4, "opa": "Carotid angiography", "opb": "Veebral venous venography", "opc": "X-ray", "opd": "Jugular venography", "subject_name": "ENT", "topic_name": "Ear", "id": "ad911821-adf1-4277-bbae-0a6fc6b77075", "choice_type": "single"} {"question": "Glue ear is", "exp": "Otitis media with effusion/ Serous OM/Glue ear/Secretory OM is a painless condition following accumulation of non-inflammatory exudate in the middle ear. Commonly seen in children.OME is the commonest cause of conductive deafness in children, and due to adenoid hyperophyIn adults, Nasopharyngeal carcinoma is to be ruled out.Treatment of choice- Myringotomy with grommet inseion.Ref: Hazarika; 3rd ed; Pg 146", "cop": 2, "opa": "Painful", "opb": "Painless", "opc": "Naf is useful", "opd": "Radical mastoidectomy is required", "subject_name": "ENT", "topic_name": "Ear", "id": "5833ffb5-8112-4e54-8b39-162b94685797", "choice_type": "single"} {"question": "Multiple Perforation and Pale granulation of Tympanic membrane is seen in", "exp": null, "cop": 1, "opa": "TB Otitis media", "opb": "Syphilis Otitis media", "opc": "Pseudomonas aerugenosa otitis media", "opd": "Herpes Otitis media", "subject_name": "ENT", "topic_name": null, "id": "3df69fda-495c-408c-a4a7-426efd791e2b", "choice_type": "single"} {"question": "Crocodile tears is due to", "exp": "Crocodile tears or gustatory lacrimation is due to faulty regeneration of parasympathetic nerve fibres. Ref: Dhingra 6th edition pg. 98", "cop": 4, "opa": "Cross innervation of facial nerve fibers", "opb": "Cross innervation of trigeminal nerve fibers", "opc": "Improper regeneration of trigeminal nerve", "opd": "Improper regeneration of facial nerve", "subject_name": "ENT", "topic_name": "Ear", "id": "0c7fb546-675a-4932-8d27-3b36ab2039d4", "choice_type": "single"} {"question": "In right middle ear pathology, weber test will be", "exp": "Weber`s test lateralizes to the affected ear in a conductive hearing loss and to the normal ear in a sensorineural hearing loss. In case of a middle ear pathology, there is conductive hearing loss and hence weber`s test lateralizes to the affected ear.Weber`s is the most sensitive test, lateralizes even at 5 dB of hearing difference between the two ears.Ref: Hazarika; 3rd ed; Pg 53", "cop": 3, "opa": "Normal", "opb": "Centralised", "opc": "Deted to right", "opd": "Deted to left", "subject_name": "ENT", "topic_name": "Ear", "id": "40a26a04-2f6a-4dad-ac46-2d977394f16b", "choice_type": "single"} {"question": "C shaped curve on tympanometry is seen in", "exp": "Type C tympanometry Maximum compliance occurs with negative pressure in excess of 100mm H2O Seen in retracted tympanic membrane and may show some fluid in middle ear. Ref: Dhingra 7e pg 27.", "cop": 4, "opa": "Serous otitis media", "opb": "Otosclerosis", "opc": "TM perforation", "opd": "Retracted tympanic membrane", "subject_name": "ENT", "topic_name": "Ear", "id": "539db3b7-1a9d-4f5a-a1ae-34d1bcec8553", "choice_type": "single"} {"question": "Meyer cotton Grading system used for", "exp": null, "cop": 2, "opa": "Laryngocoele", "opb": "Subglottic stenosis", "opc": "Laryngomolocia", "opd": "TB larynx", "subject_name": "ENT", "topic_name": null, "id": "b6de38b7-31bf-4014-961a-d8d729356cf0", "choice_type": "single"} {"question": "Bovine cough is charactstic of", "exp": "(C) Laryngeal paralysis # Clinical Features: As all the muscles of larynx on one side are paralysed, vocal cord will lie in the cadaveric position i.e. 3.5 mm from the midline. The healthy cord is unable to approximate the paralysed cord, thus causing glottic incompetence. This results in hoarseness of voice and aspiration of liquids through the glottis. Cough is ineffective due to air waste.", "cop": 3, "opa": "Acute epiglottis", "opb": "Foreign body", "opc": "Laryngeal paralysis", "opd": "Chronic bronchitis", "subject_name": "ENT", "topic_name": "Miscellaneous (E.N.T.)", "id": "7d59f215-dcb1-4d7d-9169-13404b3f401b", "choice_type": "single"} {"question": "Tripod fracture is seen in", "exp": "Tripod fracture is seen in zygoma fractures as it involves three sites:Zygomatico-maxillary, Zygomatico-frontal, Zygomatico-temporal(arch)Presentation: Flattening of malar prominence, Step deformity, infra orbital paraesthesia, epistaxis, trismus.Ref: Hazarika; 3rd ed; Pg 359", "cop": 4, "opa": "Mandible", "opb": "Maxilla", "opc": "Nasal bone", "opd": "Zygoma", "subject_name": "ENT", "topic_name": "Nose and paranasal sinuses", "id": "f4ceb2b3-3c3b-4742-b080-8d0b4bc3faad", "choice_type": "single"} {"question": "Most common salivary gland to develop stones", "exp": null, "cop": 1, "opa": "Submandibular", "opb": "Sublingual", "opc": "Parotid", "opd": "Lacrimal", "subject_name": "ENT", "topic_name": null, "id": "0fa2a8b8-9fd4-4436-8357-228cdc2cff68", "choice_type": "single"} {"question": "Tapir Nose is Complication of", "exp": null, "cop": 3, "opa": "Atropic Rhinitis", "opb": "Rhinosporidiosis", "opc": "Rhinoscleroma", "opd": "Syphilis", "subject_name": "ENT", "topic_name": null, "id": "08c77273-8068-424b-9cde-efeb2d4436fd", "choice_type": "single"} {"question": "10 year old boy developed hoarseness of voice following on attach of diphtheria, No other significant OLE or VR is paralysed, treatment of choice is", "exp": "It is right recurent Laryageal nerve palsy, hense only hoarsness is seen,so wait for recovery.", "cop": 4, "opa": "Get foam infection of right VC", "opb": "Fat injection of rigth VC", "opc": "Thyroplasty Type 1", "opd": "Walt for spontaneous recovery", "subject_name": "ENT", "topic_name": null, "id": "8f254bbd-19c7-49b9-b37a-3bd5aac3c71d", "choice_type": "single"} {"question": "Pneumocephalus is seen in fracture of", "exp": "Pneumocephalus is the presence of air or gas within the cranial cavity.Pneumocephalus is seen in fracture of frontal sinus (Posterior table); may also be seen in ethmoid fracturesRef: Hazarika; 3rd ed; Pg 355", "cop": 2, "opa": "Maxillary sinus", "opb": "Frontal sinus", "opc": "Sphenoid sinus", "opd": "Mastoid antrum", "subject_name": "ENT", "topic_name": "Nose and paranasal sinuses", "id": "8cde41cb-8264-47dc-9028-17665a39f499", "choice_type": "single"} {"question": "Static equilibrium is due to", "exp": "Macula is the peripheral receptor of utricle and saccule; Static equilibrium is controlled by utricle and saccule Semicircular canals respond to angular acceleration and deceleration Cristae is located in the ampullary ends of the semicircular canals Ref: PL Dhingra 6th edition of Ear, Nose and Throat; pg no 18", "cop": 1, "opa": "Macula", "opb": "Cupula", "opc": "End organ of coi", "opd": "Cristae ampulla", "subject_name": "ENT", "topic_name": "Ear", "id": "35235f44-8d26-479f-9902-132d6dd9768e", "choice_type": "single"} {"question": "Maxillary sinus achieves maximum size at", "exp": "Maxillary sinus is the first sinus to develop at bih. It is completely developed by 9 years of agei.e approximately at secondary dentition Ref Maqbool 11/e,p148.", "cop": 3, "opa": "bih", "opb": "Primary dentition", "opc": "At secondary dentition", "opd": "At pubey", "subject_name": "ENT", "topic_name": "Nose and paranasal sinuses", "id": "efa4ab49-5a52-4458-880f-c2feadb68a5a", "choice_type": "single"} {"question": "Glottic chink, in cadaveric positions of vocal cords is", "exp": "Glottic chink It is the distance between the vocal cords. In cadaveric position--the vocal cords are 3.5 mm away from midline, so the distance between them i.e glottic chink is 7 mm. Similary in full abduction it is about 19 mm. Ref:- Mohan Bansal; pg num:- 374", "cop": 3, "opa": "3 mm", "opb": "3.5 mm", "opc": "7 mm", "opd": "19 mm", "subject_name": "ENT", "topic_name": "Larynx", "id": "8c21c41c-1564-43a7-81ef-c0f4071691e3", "choice_type": "single"} {"question": "The voice in a patient with Bilateral abductor paralysis of the larynx is", "exp": "The vocal cord vibrates during speech to produce speech. These vibrations occur due to subglottic airflow when the cord is in adduction (Median & paramedian).\nDuring normal phonation the vocal cords are in median position and during strong whisper vocal cords are in paramedian position.\nSo, for proper voice, the vocal cords should be in median or paramedian position.\nSame occurs in bilateral recurrent laryngeal nerve paralysis, where the vocal cords are in median or paramedian position → Voice is less likely to be impaired.\n\nNow, the question comes to the mind immediately : - Why the voice is affected in unilateral paralysis, in which the cord is in median or paramedian position. The answer is : -\n\nIn unilateral vocal cord paralysis, one cord is paralysed and vibrates abnormally or does not vibrate at all, therefore cannot coordinate with the other side (normal) vocal cord. This results in voice problems.\nIn bilateral paralysis, both vocal cords vibrate abnormally but coordinate with each other, therefore voice is less affected.", "cop": 4, "opa": "Puberphonia", "opb": "Phonasthenia", "opc": "Dysphonia plicae ventricularis", "opd": "Normal or good voice", "subject_name": "ENT", "topic_name": null, "id": "1ae5220e-c8bb-4aee-80bb-8d7e914e6f9c", "choice_type": "single"} {"question": "The most common site of osteoma is( REPEAT QUESTION)", "exp": "Most osteomas are asymptomatic and are found incidentally when imaging the sinuses either for sinonasal symptoms or for unrelated complaints. Osteomas are frequently seen elsewhere in the head and neck paicularly the mandible and outer table of the skull vault. There is a paicular frequency distribution within the paranasal sinuses: frontal sinuses: 80%; ethmoid air cells: ~15%; maxillary sinuses: ~5 %; sphenoid sinus: rare. Osteomas are most commonly seen in the frontal sinus followed in turn by those of ethmoid and maxillary. They may remain asymptomatic, being discovered incidentally on X rays. Treatment is indicated when they become symptomatic, causing obstruction to the sinus sodium, formation of a mucocele, pressure symptoms due to their growth in the orbit, nose or cranium. (Ref: Textbook of Diseases of ENT, PL Dhingra, 7th edition, pg no. 231)", "cop": 3, "opa": "Maxillary sinus", "opb": "Ethmoidal sinus", "opc": "Frontal sinus", "opd": "Sphenoid sinus", "subject_name": "ENT", "topic_name": "Nose and paranasal sinuses", "id": "e3fdfcda-450d-44fa-bbdf-244abb70e4c5", "choice_type": "single"} {"question": "Rhinoscleroma occurs due to", "exp": "Rhinoscleroma\n\nThe causative organism is Klebsiella rhinoscleromatis Frisch bacillus, which can be cultured from the biopsy material.\nThe disease is endemic in several parts of the world.\nIn India, it is seen more open in northern than in the southern parts.\nBiopsy shows infiltration of submucosa with plasma cells, lymphocytes, eosinophils, Mikulicz cells & Russell bodies.\nThere two are diagnostic features of the disease.\nThe disease starts in the nose & extends to nasopharynx, oropharynx, larynx, trachea & bronchi.\nMode of infection is unknown.\nBoth sexes of any age may be affected.", "cop": 2, "opa": "Auto immune cause", "opb": "Klebsiella rhinoscleromatis infection", "opc": "Inflammatory cause", "opd": "Mycotic infection", "subject_name": "ENT", "topic_name": null, "id": "299849b7-e793-4ae2-b9c9-ae6c20240177", "choice_type": "single"} {"question": "Osteomeatal complex (OMC) connects", "exp": "Ans. a (Nasal cavity with maxillary sinus). (Ref. ENT secrets 3rd ed., 100-107)Osteomeatal unit/complex (OMC) confluence# OMC is the normal aerated channel which provide airflow & mucociliary clearance for sinuses.# Anterior ethmoidal air cells, maxillary & frontal sinuses drain into it.# It is the cross road for drainage of most paranasal sinuses.# It is the key area for pathogenesis of chronic sinus diseases.# The concept that obstruction of OMC results in ethmoid, frontal & maxillary sinusitis is fundamental to FESS# Well identified on coronal CT with patient prone & head hyperextended (Coronal CT).# Thus, components of OMC include:# Middle meatus# Maxillary sinus ostium & infundibulum# Anterior & middle ethmoidal air cells ostia & ethmoidal infundibulum# Frontal recess/frontonasal duct# Bulla ethmoidalis# Hiatus semilunaris# Uncinate process# Sphenoethmoid recess & superior meatus (included by some authors)# Uncinate process is thin, curved lamina of bone from the lateral side of ethmoidal labyrinth that forms a portion of lateral nasal wall. It projects downwards & backwards & ranges from 1 to 4 mm in height and 14 to 22 mm in length. Superior edge of uncinate process is free and forms medial boundary of hiatus semilunaris.# Ethmoidal infundibulum is a trough shaped air space that is below bulla & above & lateral to uncinate process. It receives drain from anterior & middle ethmoid air cells & frontal & maxillary sinuses. The infund-ibulum may be narrowed if there is Concha bullosa &/or large Haller's cell (pneumatized middle turbinate).# Hiatus semilunaris is curvilinear opening of lateral wall that is above uncinate process & below the bulla ethmoidalis. This opening separates uncinate process from ethmoidal bulla & serves as connection between infundibulum & middle meatus.# The middle ethmoidal air cell produces a round swelling called bulla ethmoidalis on lateral wall of middle meatus.# Onodi cells are most posterior ethmoidal cells that surround optic canal.", "cop": 1, "opa": "Nasal cavity with maxillary sinus", "opb": "Nasal cavity with sphenoid sinus", "opc": "The two nasal cavities", "opd": "Ethmoidal sinus with ethmoidal bulla", "subject_name": "ENT", "topic_name": "Nose and PNS", "id": "be3a80ac-fbf4-45d0-bd90-18828dd778b6", "choice_type": "single"} {"question": "Carcinoma of maxillary sinus stage III {T3 No Mo}, Treatment of choice is", "exp": "T3N0M0: Tumor invading the skin of the cheek, anterior ethmoid, pterygoid muscles and orbit with no nodal involvement and no distant metastasis. In most cases, a combination of surgery and radiotherapy is required. However, depending on histopathological nature, therapy may differ. (Ref: Textbook of diseases of ENT, PL Dhingra, 7th edition, pg no. 234)", "cop": 2, "opa": "Radiotherapy", "opb": "Surgery and Radiotherapy", "opc": "Chemotherapy", "opd": "Chemotherapy and Surgery", "subject_name": "ENT", "topic_name": "Nose and paranasal sinuses", "id": "1c38353b-f3bd-4fd0-a585-5b2edbfcfad3", "choice_type": "single"} {"question": "Best view for superior orbital fissure", "exp": null, "cop": 3, "opa": "Water's view", "opb": "Basal view", "opc": "Caldwell view", "opd": "Lateral view", "subject_name": "ENT", "topic_name": null, "id": "2ae87f99-db11-421b-bd2a-7e6b60ac9302", "choice_type": "single"} {"question": "Commando operation describes", "exp": null, "cop": 3, "opa": "Maxillectomy, Neck Dissection, Omohyoid muscle removal", "opb": "Mandibulectomy, Neck Dissection, Omohyoid muscle removal", "opc": "Mandibulectomy, Neck dissection, Oropharyngeal resection", "opd": "Ombined maxillectomy and mandibulectomy", "subject_name": "ENT", "topic_name": null, "id": "baf7a6c5-1cc1-44f5-9ba0-60f510fac8d2", "choice_type": "single"} {"question": "Middle Ear Function is", "exp": "Middle ear functions: Conduct the sound from outer ear to inner ear Middle ear muscles provide protection from loud sound It serves as an acoustic transformer to match the impedance of air to the much higher impedance of cochlear fluids Couples sound preferably to only one window of cochlea, thus producing differential pressure between windows, necessary for cochlear fluid movement", "cop": 1, "opa": "Impendence matching", "opb": "Frequency discrimination", "opc": "Intensity Discrimination", "opd": "Speech Localization", "subject_name": "ENT", "topic_name": "DNB 2018", "id": "5e8c9179-5a28-45f9-82a3-d99f4c49f335", "choice_type": "single"} {"question": "Commonest benign tumor of the esophagus", "exp": "BENIGN NEOPLASMS OF OESOPHAGUS Benign neoplasms are rare compared to malignant ones. Leiomyoma is the most common and accounts for two- thirds of all the benign neoplasms. It arises from the smooth muscle and grows in the wall of oesophagus. Dysphagia is produced when tumour exceeds the diameter of 5 cm. Barium swallow shows an ovoid filling defect. Endoscopy reveals a submucosal swelling. Biopsy should not be taken. Treatment is enucleation of the tumour by thoracotomy. Mucosal polyps, lipomas, fibromas and haemangiomas are other benign tumours. They are often pedunculated and present in the oesophageal lumen. Endoscopic removal is avoided because of the danger of oesophageal perforation. Treatment is surgical excision by oesophagotomy Ref : ENT textbook by Dhingra 6th edition Pgno : 345", "cop": 2, "opa": "Papilloma", "opb": "Leiomyoma", "opc": "Adenoma", "opd": "Hemangioma", "subject_name": "ENT", "topic_name": "All India exam", "id": "0c8cf30c-4e37-4185-b8f8-b55706952bd3", "choice_type": "single"} {"question": "Incisura terminalis is between", "exp": "There is no cartilage between tragus and crus of the helix and an incision made in this area will not cut through the cartilage and is used for the endaural approach in surgery of external auditory canal and mastoid.", "cop": 1, "opa": "Tragus and crux of helix", "opb": "Ear lobule and antihelix", "opc": "Antihelix and external auditory meatus", "opd": "Tragus and ear lobule", "subject_name": "ENT", "topic_name": null, "id": "0970f35e-649c-4ebf-8d2d-ffbebd0fc922", "choice_type": "single"} {"question": "According to European laryngeal Society, Subligamental cordectomy is classified as", "exp": "Subligamental cordectomy is classified as Type II Excision of vocal cord after splitting the larynx (cordectomy laryngofissure) Ref: PL Dhingra; Textbook of EAR, NOSE and THROAT; edition 6; page no 310", "cop": 2, "opa": "Type I", "opb": "Type II", "opc": "Type III", "opd": "Type IV", "subject_name": "ENT", "topic_name": "Larynx", "id": "911974d3-9ff4-4b66-a4df-39f59bd8c7dc", "choice_type": "single"} {"question": "For myringoplasty, Graft material of choice is", "exp": "Low metabolic rate, no need for a separate incision, same thickness as that of tympanic membrane ours the use of temporalis fascia. Other graft materials also include perichondrium, dura and cadaveric tympanic membrane. Re: Textbook of Ear, Nose and Throat, Dhingra, 6th Edition", "cop": 3, "opa": "Peritoneum", "opb": "Palmar fascia", "opc": "Temporalis fascia", "opd": "Fascia lata", "subject_name": "ENT", "topic_name": "Diagnostic and operative ENT", "id": "ca69c564-4381-4fc6-a468-e79263297c97", "choice_type": "single"} {"question": "Nasopharyngeal angiofibroma is most commonly seen in", "exp": "Ans. (c) Young malesRef: Dhingra's ENT 5th ed.l 261\"Nasopharyngeal angiofibroma is seen almost exclusively in ADOLESCENT males in the age group of 10-20 years.\"ABOUT NASOPHARYNGEAL ANGIOFIBROMA* It is a vascular tumor with lots of fibrous tissue without muscular coat. Hence it causes severe bleeding because of inability to contract.* Most common benign tumour of nasopharynx.* Arises from the posterior part of nasal cavity close to superior margin of sphenopalatine foramen.* Frog face deformity- if tumor Extends to orbit and causes proptosis* Antral/Holman-Millar sign -- Anterior bowing of posterior wall of maxillary sinus is pathognomonic.* Investigation of choice is C.E.C.T, scan* Treatment of choice is surgical excision.", "cop": 3, "opa": "Elderly males", "opb": "Females", "opc": "Young males", "opd": "Infants", "subject_name": "ENT", "topic_name": "Tumors of Nose and PNS", "id": "832f8c1b-ebea-4dd0-a309-74d09428ed18", "choice_type": "single"} {"question": "Palpatory thud, an audible snap is seen in", "exp": "A foreign body in trachea may move up and down the trachea between the carnia and the undersurface of vocal cords causing \"audible slap\" and \"palpatory thud&;.Symptoms and signs of foreign bodies at different levelsSite of foreign bodiesSymptoms and signsLarynxChoking, Cyanosis, Complete obstruction leading to death* Paial obstruction: stridor, hoarseness, cough, respiratory difficultyTrachea* Choking, stridor, wheeze, cough, palpatory thud, audible slap* Cough, wheeze and diminished air entry to lung forms a \"triad\"Bronchi Respiratory distress with swelling of the foreign body* Lung collapse, emphysema, pneumonitis, bronchiectasis or lung abscess are late featuresRef: Hazarika; 3rd ed; Pg 651", "cop": 1, "opa": "Tracheal foreign body", "opb": "Bronchial foreign body", "opc": "Laryngeal foreign body", "opd": "Foreign body in ear", "subject_name": "ENT", "topic_name": "Larynx", "id": "4123ffcc-6830-456b-8b4e-669fac15fae4", "choice_type": "single"} {"question": "Commonest cause of Eustachian tube disease is", "exp": "Adenoids cause eustachian tube dysfunction by a) Mechanical obstruction to tubal opening b) Acting as a reservoir for pathogenic organisms c) In case of allergy, mast cells of the adenoid tissue release inflammatory mediators which cause tubal blockage (Ref: Textbook of diseases if ENT, PL Dhingra, 7th edition, pg no. 64)", "cop": 1, "opa": "Adenoids", "opb": "Sinusitis", "opc": "Otitis media", "opd": "Pharyngitis", "subject_name": "ENT", "topic_name": "Ear", "id": "a54b0f7d-b468-4c9f-8acb-7a0fcdb4992c", "choice_type": "single"} {"question": "Diameter of head mirror in ENT is", "exp": "Head mirror: It is a concave mirror used to reflect light from the Bull's eye lamp onto the pa being examined. It has a focal length of approximately 25 cm. The examiner sees through the hole in the centre of the mirror. Diameter of the mirror is 89 mm (3 1/2 \") and that of the central hole is 19 mm (3/4\") Ref: Dhingra 7e pg 427.", "cop": 3, "opa": "20 cm", "opb": "22 cm", "opc": "10 cm", "opd": "26 cm", "subject_name": "ENT", "topic_name": "Ear", "id": "390376da-ee0a-48ed-8fa1-4e176ab361b8", "choice_type": "single"} {"question": "Tensors of vocal cord are", "exp": "Intrinsic muscles acting on vocal cords: Abductors: Posterior cricoarytenoid Adductors: Lateral cricoarytenoid, lnterarytenoid (transverse arytenoid) Thyroarytenoid (external pa) Tensors: Cricothyroid, Vocalis (internal pa of thyroarytenoid) Intrinsic muscles acting on laryngeal inlet: Openers of laryngeal inlet: Thyroepiglottic (pa of thyroarytenoid) Closers of laryingeal inlet: Interarytenoid (oblique pa), Aryepiglottic (posterior oblique pa of interarytenoids)", "cop": 4, "opa": "Posterior cricothyroid, internal interarytenoid", "opb": "Lateral cricothyroid, internal interarytenoid 1", "opc": "Thyroarytenoid, internal interarytenoid", "opd": "Cricothyroid and internal thyroarytenoid", "subject_name": "ENT", "topic_name": null, "id": "7f9752d0-d5b1-4190-8bae-ac9240a67bd5", "choice_type": "single"} {"question": "Treatment of choice for vocal nodule is", "exp": "Treatment of vocal nodules Early cases of vocal nodules can be treated conservatively by educating the patient about the proper use of voice. With this treatment, many nodules in children disappear completely. Surgery is required for large nodules or nodules of long standing in adults. They are excised with precision under an operating microscope either with cold instruments or laser avoiding any trauma to the underlying vocal ligament. Speech therapy and re-education in voice production are essential to prevent their recurrence. Ref: Textbook of diseases of ENT, PL Dhingra, 7th edition, pg no. 343", "cop": 2, "opa": "Radical excision", "opb": "Microlaryngoscopic removal", "opc": "Cryotherapy", "opd": "Wait and watch", "subject_name": "ENT", "topic_name": "Larynx", "id": "8a334eb2-2f8d-4220-8530-a0c9eb456276", "choice_type": "single"} {"question": "Structure injured during excision of Ranula", "exp": null, "cop": 3, "opa": "parotid duct", "opb": "Facial artery", "opc": "Submandibular duct", "opd": "Lingual nerve", "subject_name": "ENT", "topic_name": null, "id": "f2ce82b4-7c87-423c-8559-9995b4c28d79", "choice_type": "single"} {"question": "In normal adult wave V of ABR is generated from", "exp": "Wave I - Distal pa of CN VIII Wave II - Proximal pa of CN VIII near the brainstem Wave III - Cochlear nucleus Wave IV - Superior olivary complex Wave V - Lateral lemniscus Waves VI and VII - Inferior colliculus Note: Auditory brainstem response (ABR): Also called BAER or BAEP (brainstem auditory evoked response or potential) or BERA (brainstem evoked response audiometry) is to elicit brainstem responses to auditory stimulation by clicks or tone bursts. Ref: Dhingra; 6th Edition; pg no 27", "cop": 3, "opa": "Cochlear nucleus", "opb": "Superior olivary complex", "opc": "Lateral leminiscus", "opd": "Inferior colliculus", "subject_name": "ENT", "topic_name": "Ear", "id": "1c41ffba-2830-4590-956b-c4a714a4b50c", "choice_type": "single"} {"question": "Aery of epistaxis is the name given to", "exp": "The \"aery of epistaxis\" is the sphenopalatine branch of the internal maxillary aery. This is called so because this vessel supplies the major poion of the nasal cavity. It enters the nasal cavity at the posterior end of the middle turbinate to supply the lateral nasal wall, it also gives off a septal branch which supplies the nasal septum. Ref: Diseases of ENT by Dhingra 6th edition Pgno : 179,180", "cop": 1, "opa": "Sphenopalatine aery", "opb": "Superior labial aery", "opc": "Anterior ethmoidal aery", "opd": "Mulbery turbinates", "subject_name": "ENT", "topic_name": "Ear", "id": "c825281b-dc73-4298-97b3-e6a90720c83f", "choice_type": "single"} {"question": "During maxillary wash sudden death is due to", "exp": "Air embolism: It is rare but may prove fatal. This complication can be prevented by avoiding insufflation of air into the antrum after lavage. Ref : Dhingra 7e pg 466.", "cop": 1, "opa": "Air embolism", "opb": "Maxillary aery thrombus", "opc": "Bleeding", "opd": "Meningitis", "subject_name": "ENT", "topic_name": "Ear", "id": "7415b121-de47-400a-a0e0-2a7947fda189", "choice_type": "single"} {"question": "Treatment of choice for the following disease is", "exp": "The treatment of the established case is myringotomy and aspiration of the fluid. Performed under general anesthesia and with the help of the operating microscope. TREATMENT OF GLUE EAR: If the condition persists despite treatment of any underlying sinusitis or if no cause can be found, the patient should undergo an examination of the postnasal space with biopsies of any suspicious areas. Myringotomies should be carried out. If there is thick or serous fluid present. Grommets should be inseed even if there is a postnasal tumour present. Where the drum is collapsed or atrophic grommets may be inseed anteriorly, but if the atelectasis is longstanding they are unlikely to be beneficial, and the patient would be better served by having a hearing aid. If the condition persists after the grommets have come out. long-term ventilation tubes such as a T - tube or titanium should be used. Hence the best answer here would be choice A Ref: Diseases of ENT by Dhingra 6th edition Pgno : 64", "cop": 1, "opa": "Myringotomy with ventilation tube inseion", "opb": "Myringotomy with cold knife", "opc": "Myringotomy with diode laser", "opd": "Conservative treatment", "subject_name": "ENT", "topic_name": "Ear", "id": "35d27b8c-c2a8-4331-8be0-191c49faf578", "choice_type": "single"} {"question": "Most common site of perforation in tympanic membrane is", "exp": "In the stage of resolution, usually, a small perforation is seen in the anteroinferior quadrant which is commonest of all quadrants of pars tensa. (Ref: Textbook of diseases of ENT, PL Dhingra, pg no. 68)", "cop": 1, "opa": "Anteroinferior", "opb": "Posteroinferior", "opc": "Anterosuperior", "opd": "Posterosuperior", "subject_name": "ENT", "topic_name": "Ear", "id": "cba73beb-77b6-493d-821d-ecb7c29ca0a2", "choice_type": "single"} {"question": "Commonest site of Ivory osteoma is", "exp": "Osteoma: They are most commonly seen in the frontal sinus followed in turn by those of ethmoid and maxillary. They may remain asymptomatic, being discovered incidentally on X-rays. Ref: Textbook of ENT, Dhingra; 6th edition; Pg no: 205 ref img", "cop": 1, "opa": "Frontal-Ethmodial region", "opb": "Mandible", "opc": "Maxilla", "opd": "Spheroid", "subject_name": "ENT", "topic_name": "Miscellaneous ENT", "id": "fd93e0ca-6efd-4bb6-b211-d804c7760bdb", "choice_type": "single"} {"question": "In Adenoids hyperophy, treatment is", "exp": "When symptoms are not marked, breathing exercises, decongestant nasal drops, and antihistaminics. When symptoms are marked adenoidectomy is done. (Ref: Diseases of Ear, Nose and Throat, PL Dhingra; 7th edition, pg no. 276)", "cop": 1, "opa": "Nasal decongestants", "opb": "Antibiotics", "opc": "B-blockers", "opd": "B2-agonists", "subject_name": "ENT", "topic_name": "Pharynx", "id": "ac47df4c-9a66-44f9-b67b-16953833820d", "choice_type": "single"} {"question": "Most common sinus involved in acute sinusitis", "exp": "The alveolar process of the maxillae holds the upper teeth and is referred to as the maxillary arch.\nEach maxilla attaches laterally to the zygomatic bones (cheekbones).\nEach maxilla assists in forming the boundaries of three cavities: the roof of the mouth. the floor and lateral wall of the nasal cavity.", "cop": 2, "opa": "Ethmoid", "opb": "Maxillary", "opc": "Sphenoid", "opd": "Frontal", "subject_name": "ENT", "topic_name": null, "id": "840bed6d-dfc6-4fec-aab8-bbcc3c364d3a", "choice_type": "single"} {"question": "Not seen in Ludwig&;s angina", "exp": "Ludwig angina is a type of bacterial infection that occurs in the floor of the mouth, under the tongue. It often develops after an infection of the roots of the teeth (such as tooth abscess) or a mouth injury. This condition is uncommon in children. CLinical features: There is marked difficulty in swallowing (odynophagia) with varying degrees of trismus. When infection is localized to the sublingual space, structures in the floor of mouth are swollen and tongue seems to be pushed up and back. When infection spreads to submaxillary space, submental and submandibular regions become swollen and tender, and impa woody-hard feel. Usually, there is cellulitis of the tissues rather than frank abscess. Tongue is progressively pushed upwards and backwards threatening the airway. Laryngeal oedema may appear. Complications: 1.Spread of infection to parapharyngeal and retropharyngeal spaces and thence to the mediastinum. 2. Airway obstruction due to laryngeal oedema, or swelling and pushing back of the tongue. 3.Septicaemia. 4.Aspiration pneumonia. Ref - otolaryngology Dhingra 6E p263", "cop": 2, "opa": "Glottal edema, may require tracheostomy", "opb": "Aphthous ulcer in pharynx", "opc": "It is caused by anaerobic organisms", "opd": "Cellulitis in the floor of mouth", "subject_name": "ENT", "topic_name": "Ear", "id": "7f14e3d0-9bc0-4d5c-b61d-5fe4ecfddf59", "choice_type": "single"} {"question": "Queckenstedt test is done for", "exp": "Drainage of CSF in some cases may often be elicited on endoscopy by having the patient perform a Valsalva maneuver or by compressing both jugular veins (Queckenstedt-Stookeytest) Ref : internet sources", "cop": 2, "opa": "Glomus tumor", "opb": "CSF rhinorrhea", "opc": "Otosclerosis", "opd": "Acoustic neuroma", "subject_name": "ENT", "topic_name": "Ear", "id": "9176dea7-4ad9-4a97-aa94-ed5ffa9b0995", "choice_type": "single"} {"question": "Sensory Nerve supply of Anterior 2/3rd tongue", "exp": null, "cop": 1, "opa": "Lingual Nerve", "opb": "Maxillary Nerve", "opc": "Glossop haryngeal Nerve", "opd": "Chorda typmani", "subject_name": "ENT", "topic_name": null, "id": "cbe362d4-d3a3-414e-9894-54824c1a5ebd", "choice_type": "single"} {"question": "A patient presents with enophthalmos after a trauma to face by blunt object. There is no fever or extraocular muscle palsy. Diagnosis is", "exp": "Due to a trauma to face by a blunt object, an increased pressure may rupture the floor of the orbit and extrude the fat content of orbit, resulting in enophthalmos. Thus it is due to a &;blow-out fracture&; or fracture of orbital floor Ref: Diseases of nose throat and ear; Logan Turner; 10th edition; pg no 27", "cop": 3, "opa": "Fracture maxilla", "opb": "Fracture zygoma", "opc": "Blowout fracture", "opd": "Fracture ethmoid", "subject_name": "ENT", "topic_name": "Nose and paranasal sinuses", "id": "27b9292d-8c00-45e1-bdb6-ca862b577b3b", "choice_type": "single"} {"question": "Furstenberg's test is seen positive in", "exp": null, "cop": 3, "opa": "Nasal Glioma", "opb": "Nasalabial cyst", "opc": "Encephalocoele", "opd": "Nasal Bone fracture", "subject_name": "ENT", "topic_name": null, "id": "f6d3f8e0-d495-4a35-a184-157fdd0ced0b", "choice_type": "single"} {"question": "Drug of choice in laryngeal stenosis is", "exp": "Mitomycin-C is an antineoplastic antibiotic that acts as an alkylating agent by inhibiting DNA and protein synthesis. ... Topical application of mitomycin C(0.4 mg/mL) was used as an adjuvant treatment in the endoscopic laser management of laryngeal and tracheal stenosis Ref: Internet sources", "cop": 4, "opa": "Cyclophosphamide", "opb": "Doxorubicin", "opc": "Adriamycin", "opd": "Mitomycin C", "subject_name": "ENT", "topic_name": "Larynx", "id": "19976300-f70c-4a50-bc97-1818f0caabba", "choice_type": "single"} {"question": "Cause of unilateral secretory otitis media in an adult is", "exp": "Benign and malignant tumours of nasopharynx should always be excluded in unilateral serous otitis media in an adult. Nasopharyngeal carcinoma Clinical features Nasal: Nasal obstruction, nasal discharge, denasal speech and epistaxis. Otalgic: Due to obstruction of the eustachian tube, there is a conductive type of hearing loss, serous or suppoive otitis media. Tinnitus and dizziness may occur. Presence of unilateral serous otitis media in an adult should raise suspicion of nasopharyngeal growth. Ophthalmoneurologic: Squint and Diplopia(CN 6), Ophthalmoplegia( CN 3,4 and 6) facial pain and reduction of corneal reflex(CN 5), exophthalmos and blindness(CN 2).Involvement of CN 9,10,11 may occur constituting jugular foramen syndrome. Involvement of the cervical sympathetic chain causes horner's syndrome. Cervical nodal metastasis. (Ref: Textbook of diseases of ENT, PL Dhingra, 7th edition, pg no. 284,285)", "cop": 2, "opa": "CSOM", "opb": "Nasopharyngeal carcinoma", "opc": "Mastoiditis", "opd": "Foreign body of external ear", "subject_name": "ENT", "topic_name": "Ear", "id": "98b81962-df0a-4a8e-9747-68b5da00e4ce", "choice_type": "single"} {"question": "The earliest symptom of acoustic nerve tumour is", "exp": "Cochleovestibular symptoms are the earliest symptoms when a tumour is still intracanalicular and are caused by pressure on cochlear or vestibular nerve fibres or on the internal auditory aery.Progressive unilateral sensorineural hearing loss, often accompanied by tinnitus, is the presenting symptom in the majority of cases. There is marked difficulty in understanding speech, out of propoion to the pure tone hearing loss. This feature is characteristic of acoustic neuroma. Vestibular symptoms are imbalance or unsteadiness. True veigo is seldom seen. Ref : Diseases of ENT by Dhingra Pgno : 112", "cop": 2, "opa": "Tinnitus", "opb": "Hearing loss", "opc": "Veigo", "opd": "Otorrhoea", "subject_name": "ENT", "topic_name": "Ear", "id": "68f62a33-799d-4922-aa7f-1a4896715453", "choice_type": "single"} {"question": "Fistula test stimulates", "exp": "Fistula test: It induces nystagmus by producing pressure changes in the external canal which are then transmitted to labyrinth Normally the test is negative as the pressure changes in the external canal cannot be transmitted to labyrinth It is positive in the presence of the fistula which is attached to the lateral semicircular canal Ref: Dhingra 7e pg 43, internet sources.", "cop": 1, "opa": "Lateral semicircular canal", "opb": "Posterior semicircular canal", "opc": "Anterior semicircular canal", "opd": "Cochlea", "subject_name": "ENT", "topic_name": "Ear", "id": "5a604399-7285-49c1-89f1-ead81d4913ca", "choice_type": "single"} {"question": "Atrophic dry nasal mucosa, extensive encrustations with 'woody' hard external nose; is suggestive of", "exp": "Ans. b (Rhinoscleroma). (Ref. ENT PL Dhingra, 3rd ed., 194)In the granulomatous stage, painless and non-ulcerative nodules are formed in nasal mucosa, resulting in formation of so-called Woody Nose/ Hebry's Nose.RHINOSCLEROMA# Chronic granulomatous disease.# Caused by: a gram-negative bacterium Klebsiella rhinoscleromatis/Frisch bacillus- Atrophic stage--atrophic rhinitis--foul-smelling purulent nasal discharge & crusting.- Granulomatous stage--nodules in nasal mucosa, 'Woody' hard external nose & upper lip.- Cicatricial stage--stenosis of nares, distortion of upper parts in nose, nasopharynx & oropharynx.# It is common between 20 and 40y age and is found almost exclusively in lower socioeconomic class.# Mikulicz cells & Russel bodies are seen.- Mikulicz cells are large foam cells containing causative bacilli.# Rx: Streptomycin, tetracyclines, steroids, surgery.", "cop": 2, "opa": "Rhinosporidiosis", "opb": "Rhinoscleroma", "opc": "Atrophic rhinitis", "opd": "Carcinoma of nose.", "subject_name": "ENT", "topic_name": "Nose and PNS", "id": "d0b673c4-3b39-41b4-abe4-32ab9cd3093f", "choice_type": "single"} {"question": "Positional veigo is", "exp": "The disease is caused by the disorder of posterior semicircular canal. Otoconial debris when settles in the cupula of the posterior semicircular canal in a critical head position causes displacement of cupula and veigo. (Ref: Diseases of EAR, NOSE AND THROAT by PL DHINGRA - 6th Edition)", "cop": 4, "opa": "Lateral", "opb": "Superior", "opc": "Inferior", "opd": "Posterior", "subject_name": "ENT", "topic_name": "Ear", "id": "79c0a675-0af0-4bdc-8fac-5fcd7ea1aef3", "choice_type": "single"} {"question": "The most common site of the leak in CSF rhinorrhea is( REPEAT QUESTION)", "exp": "Leakage of CSF is most commonly seen in the anterior cranial fossa at the level of the cribriform plate as it is extremely thin. (Ref: Diseases of nose ear and throat; Logan Turner; pg 28)", "cop": 3, "opa": "Sphenoid sinus", "opb": "Frontal sinus", "opc": "Cribriform plate", "opd": "Tegmen tympani", "subject_name": "ENT", "topic_name": "Nose and paranasal sinuses", "id": "650cac84-f9ad-441e-89a9-9550c56d3916", "choice_type": "single"} {"question": "Rhinosporidiosis is caused by", "exp": "Rhinosporidiosis is a chronic granulomatous infection of the mucous membrane.\nThe ecological agent is Rhinosporidium seeberi.\nRhinosporidium seeberi is an aquatic bacterium (not a fungus). Infection usually results from a local trauma inoculation with the organism.\nIt is seen in India, Pakistan and Sri Lanka. In India, most of the cases are seen in Southern states.\nInfection of the nose and nasopharynx is observed in 70% of persons with rhinosporidiosis; infection of the palpebral conjunctiva or associated structures (including lacrimal apparatus) is observed in 15% of cases.\nOther structures of the mouth and upper airway may be sites of disease. Disease of the skin, ear, genitals and rectum has also been described.\nRhinosporidiosis is an infection that typically limited to the mucosal epithelium.\nThe disease progress with local replication of R. seeberi and associated hyperplastic growth of host tissue and a localized immune response.\nClinical features of Rhinosporidiosis\nRhinosporidiosis presents as soft leafy polypoidal mass (soft polyp), which is pink to purple in colour studded with white dots, i.e., strawberry appearance. This appearance results from sporangia, which is visible as grey or yellow spots in the vascular polypoid masses.\nBecause the polyps are vascular and friable, they bleed easily upon manipulation. \nTreatment - The treatment of choice is surgical excision.\nComplete excision of mass is done with diathermy knife and cauterization of the base. Dapsone is being tried for treating rhinosporidiosis but with limited success.", "cop": 2, "opa": "HPV", "opb": "R. seeberi", "opc": "Klebsiella rhinoscleromatis", "opd": "EBV", "subject_name": "ENT", "topic_name": null, "id": "6ff71660-c8c1-4262-983a-785a38090f6c", "choice_type": "single"} {"question": "Schwaz sign is seen in", "exp": "In otosclerosis sometimes, a reddish hue may be seen on the promontory through the tympanic membrane. This is called Schwaze sign. This is indicative of active focus with increased vascularity. Ref: Diseases of EAR, NOSE and THROAT by PL Dhingra; 7th Edition page no.96", "cop": 2, "opa": "Glomus Jugulare", "opb": "Otosclerosis", "opc": "Meniere's diseases", "opd": "Acoustic neuroma", "subject_name": "ENT", "topic_name": "Ear", "id": "9efe4d98-2532-45a2-8ce4-944d7e8497c5", "choice_type": "single"} {"question": "Destruction of right labyrinth causes nystagmus to", "exp": "In sudden vestibular failure, the patient is found to have first degree, second degree and third-degree nystagmus to unaffected side., a so-called paralytic nystagmus (Ref: Diseases of EAR - Harold Rudman, Tony Wright - 6th Edition)", "cop": 2, "opa": "Right side", "opb": "Left side", "opc": "Rotatory nystagmus", "opd": "No nystagmus", "subject_name": "ENT", "topic_name": "Ear", "id": "007cd80d-f01d-4700-af13-043fc2c10d47", "choice_type": "single"} {"question": "Teardrop sign is seen in", "exp": "In 'blowout fractures' or fractures of orbital floor, x-ray shows a convex opacity bulging into the antrum from above. This occurs because of the herniation of orbital contents into the antrum. This sign is known as teardrop sign or teardrop opacity. Ref: Textbook of diseases of ENT; PL Dhingra, 7th edition, pg no. 206", "cop": 4, "opa": "Fracture of zygomatic arch", "opb": "Fracture of maxilla", "opc": "Fracture of mandible", "opd": "Blow out fracture", "subject_name": "ENT", "topic_name": "Nose and paranasal sinuses", "id": "ec25e74a-b687-4af2-971a-416a57da3b9a", "choice_type": "single"} {"question": "Impedance audiometry is for pathology of", "exp": "It is done for the pathology of the middle ear. It is a test for conductive deafness. It is an objective test, widely used in clinical practice and is paicularly useful in children. It consists of: Tympanometry Acoustic reflex measurements. Ref: Dhingra; 6th Edition; pg no 24", "cop": 2, "opa": "External ear", "opb": "Middle ear", "opc": "Mastoid air cell", "opd": "Inner ear", "subject_name": "ENT", "topic_name": "Ear", "id": "dde4921e-5f2a-4753-a27c-317401c4aced", "choice_type": "single"} {"question": "The function of stria vascularis is", "exp": "Cochlear duct: Also called membranous cochlea or the scala media. It is a blind coiled tube. It appears triangular on cross-section and its three walls areformed by:(a) the basilar membrane, which suppos the organ of Coi(b) the Reissner's membrane, which separates it from the scala vestibule; and(c) the stria vascularis, which contains vascular epithelium and is concerned with the secretion of endolymphRef: Dhingra; 6th edition; Page no: 10", "cop": 4, "opa": "To produce perilymph", "opb": "To absorb perilymph", "opc": "To maintain the electronic milieu of endolymph", "opd": "To produce endolymph", "subject_name": "ENT", "topic_name": "Ear", "id": "302757bf-dfdd-4aa7-9c5d-be5760e87356", "choice_type": "single"} {"question": "Gerlach tonsil in Waldeyer’s ring is", "exp": "The tubal tonsil is one of the four main tonsil groups comprising Waldeyer's tonsillar ring, which also includes the palatine tonsils, the lingual tonsils, and the pharyngeal tonsils.", "cop": 1, "opa": "Tubal tonsil", "opb": "Palatine tonsil", "opc": "Pharyngeal tonsil", "opd": "Lingual tonsil", "subject_name": "ENT", "topic_name": null, "id": "85aa2fd5-b80e-4d25-a5bc-2ef47825d9d7", "choice_type": "single"} {"question": "Rollover phenomenon in audiometry is seen in", "exp": "Roll over phenomenon:- it is seen in retro cochlear hearing loss . With increase in intensity above paicular level phonetically balanced score (PB score) falls rather than maintain a plateau as in cochlear type of sensineural hearing loss .", "cop": 2, "opa": "Cochlear lesions", "opb": "Retro-cochlear lesions", "opc": "Lesion in inferior colliculus", "opd": "Lesion in medial geniculate body", "subject_name": "ENT", "topic_name": "Ear", "id": "316ef201-16fd-464d-adcf-501b00d1fe7a", "choice_type": "single"} {"question": "Gelle's phenomenon is absent in", "exp": "Gelle's test compares the intensity of bone conducted tuning fork sound without and with raising pressure on the tympanic membrane with Siegel's speculum. Normally, Gelle's test is positive because the intensity of hearing decreases when air pressure is raised in the external auditory canal. In ossicular fixation or ossicular discontinuity, increased air pressure makes no change in the sound intensity(Gelle's negative) Ref: Diseases of EAR, NOSE and Throat by PL Dhingra; 7th Edition; Page no.508", "cop": 4, "opa": "CSOM", "opb": "Tympanosclerosis", "opc": "Meniere's disease", "opd": "Otosclerosis", "subject_name": "ENT", "topic_name": "Ear", "id": "88547d57-3695-469a-951e-c97fd0b45aa9", "choice_type": "single"} {"question": "Not included in Gredenigo syndrome", "exp": null, "cop": 3, "opa": "Retro-orbital pain", "opb": "Ear discharge", "opc": "Nystagmus", "opd": "Diplopia", "subject_name": "ENT", "topic_name": null, "id": "2e019c34-b2d4-4d16-a500-0e037028b97a", "choice_type": "single"} {"question": "CT view in paranasal polyp is", "exp": "Both coronal and axial view are used, but coronal views are best to study paranasal sinus polypsRef: PL Dhingra, Diseases of Ear, Nose & Throat, 7th edition, pg no. 217 - 218", "cop": 1, "opa": "Coronal", "opb": "Axial", "opc": "Sagittal", "opd": "3D", "subject_name": "ENT", "topic_name": "Nose and paranasal sinuses", "id": "2fa0ec8c-74f4-4225-a67d-41bc4da3ba25", "choice_type": "single"} {"question": "Treatment of Puberphonia is", "exp": "Normally, childhood voice has a higher pitch. When the larynx matures at pubey, vocal cords lengthen and the voice changes to one of lower pitch. This is a feature exclusive to males. Failure of this change leads to persistence of childhood high-pitched voice and is called puberophonia. Type III Thyroplasty- It is used to shoen (relax) the vocal cord. Relaxation of vocal cord lowers the pitch. This procedure is done in mutational falsetto or in those who have undergone gender transformation from female to male. Ref: PL Dhingra Textbook of Ear, Nose and Throat, Edition 6, page - 302", "cop": 3, "opa": "Thyroplasty type I", "opb": "Thyroplasty type II", "opc": "Thyroplasty type III", "opd": "Thyroplasty type IV", "subject_name": "ENT", "topic_name": "Larynx", "id": "79a164e2-8756-4c2d-a275-36e2b33bd4fd", "choice_type": "single"} {"question": "Moure&;s sign can be detected in", "exp": "Moure's sign: in normal persons, there is a click felt when the larynx is moved from side to side over the veebral column. The sign is absent in patients with post-cricoid carcinoma (Moure's sign positive). Ref: Internet sources", "cop": 3, "opa": "Nasopharyngeal carcinoma", "opb": "Oropharyngeal carcinoma", "opc": "Postcricoid carcinoma", "opd": "Supraglottic carcinoma", "subject_name": "ENT", "topic_name": "Larynx", "id": "49d51a8b-f675-431f-892b-22495b04c748", "choice_type": "single"} {"question": "Fungus causing otomycosis most commonly is", "exp": "Otomycosis Otomycosis is a fungal infection of the ear canal that often occurs due to Aspergillus niger, A. fumigatus( niger > fumigatus) or Candida albicans. It is seen in hot and humid climate of tropical and subtropical countries. Secondary fungal growth is also seen in patients using topical antibiotics for treatment of otitis externa or middle ear suppuration. clinical features: include intense itching, discomfo or pain in the ear, watery discharge with a musty odour and ear blockage. The fungal mass may appear white, brown or black and has been likened to a wet piece of filter paper. Examined with an otoscope: A. niger appears as black- headed filamentous growth, A. fumigatus as pale blue or green and Candida as white or creamy deposit. Meatal skin appears sodden, red and oedematous. Ref:- Dhingra; pg num:-52", "cop": 2, "opa": "Aspergillus fumigatus", "opb": "Aspergillus niger", "opc": "Candida", "opd": "Mucor", "subject_name": "ENT", "topic_name": "Ear", "id": "4197a391-d22b-4e69-9822-ee2b9bfaff8f", "choice_type": "single"} {"question": "Most common site of mandibular fracture is", "exp": "Mandibular fractures are typically the result of trauma. This can include a fall onto the chin or a hit from the side. Rarely they may be due to osteonecrosis or tumors in the bone. The most common area of fracture is at the condyle (36%), body (21%), angle (20%) and symphysis (14%).", "cop": 2, "opa": "Angle of mandible", "opb": "Condylar process", "opc": "Coronoid process", "opd": "Ramus", "subject_name": "ENT", "topic_name": null, "id": "6393e62e-1d09-4dd1-a1c7-8c983942038d", "choice_type": "single"} {"question": "Darwin tubercle is seen in", "exp": "Darwin's tubercle (or auricular tubercle) is a congenital ear condition which often presents as a thickening on the helix at the junction of the upper and middle thirds The features present in approximately 10.4% of the population. This acuminate nodule represents the point of the mammalian ear. Ref: PL Dhingra, Diseases of Ear, Nose & Throat, 7th edition, pg no. 3", "cop": 2, "opa": "Tragus", "opb": "Helix", "opc": "Antihelix", "opd": "Lobule", "subject_name": "ENT", "topic_name": "Ear", "id": "08192e23-4fed-46e2-b82f-3c5ea50d41dd", "choice_type": "single"} {"question": "Woodruff plexus is seen at", "exp": "It is a plexus of veins situated inferior to posterior end of inferior turbinate . It is a site of posterior epistaxis in adults . ( Ref : Diseases of ENT , PL Dhingra , 6th edition , Pg no . 176 )", "cop": 3, "opa": "Anteroinferior pa of superior turbinate", "opb": "Middle turbinate", "opc": "Posterior pa of inferior turbinate", "opd": "Anterior pa of inferior turbinate", "subject_name": "ENT", "topic_name": "Nose and paranasal sinuses", "id": "c86b5622-655a-49af-ad59-b986f4e283d9", "choice_type": "single"} {"question": "Thyroplasty Type I is done for vocal cord.........", "exp": "Ans: a (Medialisation) Ref: Dhingra, 3rded, p. 364 & 4thed, p. 278Laryngeal frame work surgery - Isshiki's classificationType I thyroplastyType II thyroplastyType III ThyroplastyType IV thyroplasty- Medial displacement- Lateral displacement- Shortening (relaxation)- Elongation (tensioning) Indications: Type III thyroplastyIndications: Type IV thyroplasty1) Adductor spasmodic dysphonia2) Mutational falsetto3) Gender transformation (female to male)1) Androphonia2) Post surgical defects3) Gender transformation (male to female)", "cop": 1, "opa": "Medialisation", "opb": "Lateralisation", "opc": "Shortening", "opd": "Lengthening", "subject_name": "ENT", "topic_name": "Voice and Speech Disorders", "id": "e9109a57-b4f3-48db-94c4-8ff6104fe06a", "choice_type": "single"} {"question": "Most common bacterial cause of Rhinitis is", "exp": "Secondary Invaders Streptococcus haemolyticus Pneumococcus Staphylococcus Haemophilus influenza Klebsiella Ref : Dhingra 7e pg 171.", "cop": 1, "opa": "Haemophilus influenza", "opb": "E.coli", "opc": "Pasturella multocida", "opd": "Cornybacterium diphtheria", "subject_name": "ENT", "topic_name": "Nose and paranasal sinuses", "id": "2e651b0b-a756-47f0-8d55-075fe952b569", "choice_type": "single"} {"question": "Unilateral middle ear serous effusion in an adult, most common cause is", "exp": "\"In adults presenting with a unilateral middle ear effusion, the possibility of a nasopharyngeal carcinoma should be considered\"\"A high incidence of NPC (Nasopharyngeal carcinoma) in South East Asia and southern China correlates with the high incidence of OME (Otitis Media with Effusion) in adults in these regions\"Ref: PL Dhingra, Diseases of Ear, Nose & Throat, 7th edition, pg no. 279 - 286", "cop": 3, "opa": "TB", "opb": "Foreign body", "opc": "Nasopharyngeal carcinoma", "opd": "CSOM", "subject_name": "ENT", "topic_name": "Pharynx", "id": "d3969d1f-b15b-4e75-80de-cc6ede44ff09", "choice_type": "single"} {"question": "Most common site for laryngeal cailagenous tumors is", "exp": "Laryngeal cailaginous tumors- Cricoid cailage is the most common site of laryngeal cailaginous tumor. Ref:- Essentials of ENT Mohan Bansal; pg num:- 372", "cop": 1, "opa": "Cricoid cailage", "opb": "Thyroid cailage", "opc": "Arytenoid cailage", "opd": "Corniculate cailage", "subject_name": "ENT", "topic_name": "Larynx", "id": "725e2773-629a-40bc-b562-9688f30c1c4a", "choice_type": "single"} {"question": "Pitch discrimination is best between", "exp": "Pitch is a subjective sensation produced by frequency of the sound Normal person can hear frequencies of 20-20,000 Hz Audiometry testing is done for only 125 - 8000 Hz Ref: Dhingra 7e pg 21,22.", "cop": 2, "opa": "0-100 Hz", "opb": "100-1000 Hz", "opc": "1000-4000 Hz", "opd": "20-20,000 Hz", "subject_name": "ENT", "topic_name": "Ear", "id": "342a8c4f-240a-4297-aff8-105e43a4dfc3", "choice_type": "single"} {"question": "Paranasal sinuses presents at bih are", "exp": "Paranasal sinuses develop as outpouchings from the mucous membrane of a lateral wall of the nose. The growth of sinuses continues during childhood and into early adult life. Radiologically maxillary sinuses can be identified at 4-5 months, ethmoids at 1 year, sphenoid at 4 years and frontals at 6 years. Reference: Diseases of ear, nose and throat; PL Dhingra; 6th edition; Pg no.189", "cop": 2, "opa": "Frontal and maxillary", "opb": "Ethmoid and maxillary", "opc": "Frontal and ethmoid", "opd": "Sphenoid and ethmoid", "subject_name": "ENT", "topic_name": "Nose and paranasal sinuses", "id": "38fac735-9e58-4d5e-8c6a-5fd9e22550c8", "choice_type": "single"} {"question": "Woodruff's plexus is located at", "exp": "O Woodruff's plexus (also called nasopharyngeal plexus) is a confluence of vessels on the lateral wall of nose and is often implicated in 'posterior' bleeds. O It is a venous plexus situated posterior to posterior end of Middle and inferior turbinate. In recent histological studies it is found to consist of thin walled vessels with very little muscle or fibrous tissue. O It is the most common site of posterior epistaxis in adults. A more recent study with endoscopic photography and anatomical micro-dissectionconfirmed that the plexus does indeed exist but showedit to be a venous plexus.", "cop": 3, "opa": "Anterosuperior pa of nose", "opb": "Anteroinferior pa of nose", "opc": "Posteroinferior pa of nose", "opd": "Posterosuperior pa of nose", "subject_name": "ENT", "topic_name": "Epistaxis", "id": "16a8da2f-e50a-473b-b534-d477d514fe0d", "choice_type": "single"} {"question": "Lymphatic drainage of oropharynx is mainly through", "exp": "Lymphatics from oropharynx drain into the upper jugular chain, paicularly the jugulodigastric ( tonsilar) node. Some pas drain to the posterior cervical group also. The base of the tongue may drain bilaterally. Lymphatic drainage of the nasopharynx - upper deep cervical nodes. (Ref: Diseases of Ear, Nose and Throat, PL Dhingra; 7th edition, pg no. 271)", "cop": 3, "opa": "Superficial cervical lymph nodes", "opb": "Submandibular nodes", "opc": "Jugulodigastric nodes", "opd": "Jugulo-omohyoid nodes", "subject_name": "ENT", "topic_name": "Pharynx", "id": "56ac3a38-2a21-4481-9167-085469c91a8d", "choice_type": "single"} {"question": "Surgery to widen the cailaginous pa of EAC", "exp": "Meatoplasty- Operation in which crescent of conchal cailage is excised to widen the meatus. Myringoplasty is repair of a perforation of the tympanic membrane (the pars tensa). (TYMPANUM=Middle ear) Tympanoplasty is ossicular reconstruction with myringoplasty. Otoplasty is Reconstruction of Pinna", "cop": 2, "opa": "Myringoplasty", "opb": "Meastoplasty", "opc": "Otoplasty", "opd": "Tympanoplasty", "subject_name": "ENT", "topic_name": "NEET Jan 2020", "id": "a2c8aa5d-9869-412a-bf92-eb9643452a42", "choice_type": "single"} {"question": "Le Fort's fracture does NOT involve", "exp": "(D) Mandible # FRACTURES OF MAXILLA: They are classified into 3 types.> Le Fort I (transverse) fracture runs ab and parallel to the palate. It cross lower part of nasal septum, maxillary antra and the pterygoid plates.> Le Fort II (pyramidal) fracture through the root of nose, lacrimal bone, floor of orbit, upper part of maxillary sinus and pterygoid plates.> This fracture has some features common with the zygomatic fractures.> Le Fort III (craniofacial dysfunction): There is complete separation of facial bones from the cranial bones. The fracture line passes through root of nose, ethmofrontal junction, superior orbital fissure, lateral wall of orbit, frontozygomatic and temporozygomatic sutures and the upper part of pterygoid plates.> Mandibular fractures are not a part of Le Fort classification.", "cop": 4, "opa": "Zygoma", "opb": "Maxilla", "opc": "Nasal bones", "opd": "Mandible", "subject_name": "ENT", "topic_name": "Miscellaneous (E.N.T.)", "id": "56299b04-0017-427a-b771-357759de0b44", "choice_type": "single"} {"question": "Laryngitis sicca is associated with", "exp": "It is characterized by atrophy of laryngeal mucosa and crust formation. This condition is often seen in women and is associated with atrophic rhinitis and pharyngitis. Common symptoms include hoarseness of voice which temporarily improves on coughing and removal of crusts. Dry irritating cough and sometimes dyspnoea is due to obstructing crusts. Examination shows atrophic mucosa covered with foulsmelling crusts. When crusts have been expelled, the mucosa may show excoriation and bleeding. Crusting may also be seen in the trachea. Treatment is the elimination of the causative factor and humidification. Laryngeal sprays with glucose in glycerine or oil of pine are comfoing and help to loosen the crusts. Associated nasal and pharyngeal conditions will require attention. Expectorants containing ammonium chloride or iodides also help to loosen the crusts. Ref: PL Dhingra 7th edition of Ear, Nose and Throat; Pg no 331", "cop": 3, "opa": "Rhinosporidium", "opb": "Mycobacterium Leprae", "opc": "Klebsiella ozaenae", "opd": "Klebsiella rhinoscleromatosis", "subject_name": "ENT", "topic_name": "Larynx", "id": "9897edb8-dc81-461f-ab7f-93499da8552b", "choice_type": "single"} {"question": "A patients hears better in noisy surrounding known as", "exp": "A. i.e. (Paracusis) (87- Dhingra 4th) (120 - Maqbool 11th)* Paracusis willisii - an otosclerotic (otospongiosis patients hears better in noisy than quite surroundings* In cochlear lesions patients do not hear at conversational intensity but get irritated by loud sound - RECRUITMENT* Sociacusis - is the hearing impairment because of recreative places like pop music, snow mobiles and carraces etcHyperacusis (Phonophobia) - sounds appears as uncomfortably loud to the patients cause -(i) Stapedius muscle paralysis as after suprastaedial facial nerve paralysis(ii) Congenital syphilis (Hennebert's sign**)PRESBYACUSIS (Senile deafness) - Sensorineural hearing loss associated with physiological aging process in the ear is called presbycusis.* Young man whose voice has not broken is called Puberphonia* White noise - consist of a random mixture of all frequencies", "cop": 1, "opa": "Paracusis", "opb": "Diplacusis", "opc": "Hyperacusis", "opd": "Presbycusis", "subject_name": "ENT", "topic_name": "Ear", "id": "73b71bb9-e560-4451-befb-725871430f9d", "choice_type": "single"} {"question": "Most common nerve damaged in CSOM is", "exp": "The facial nerve is most commonly damaged in CSOM because it traverses through the middle ear.Ref: Hazarika; 3rd ed; Pg 164", "cop": 4, "opa": "II", "opb": "III", "opc": "VI", "opd": "VII", "subject_name": "ENT", "topic_name": "Ear", "id": "d48b2d14-c686-4fb0-b313-09612ab5e36d", "choice_type": "single"} {"question": "Orbital apex syndrome involves", "exp": "Orbital apex syndrome is a complication of a paranasal sinus infection. Superior orbital fissure syndrome along with the involvement of maxillary division of trigeminal nerve and optic nerve is known as orbital apex syndrome. In superior orbital fissure syndrome, there is progressive paralysis of abducent, oculomotor and trochlear nerves. Ref: Diseases of ENT by Dhingra 6th edition Pgno : 200", "cop": 3, "opa": "Ophthalmic division of trigeminal nerve", "opb": "Olfactory nerve", "opc": "Maxillary division of trigeminal nerve", "opd": "Mandibular division of trigeminal nerve", "subject_name": "ENT", "topic_name": "Ear", "id": "2342bf0e-179a-421a-80fe-42327394a32d", "choice_type": "single"} {"question": "WEBER'S test in otitis media will be", "exp": "Otitis media causes conductive hearing lossWeber's testLateralized to poorer ear-conductive hearing lossLateralized to better ear-SNHLNo lateralization - NormalRef: PL Dhingra, Diseases of Ear, Nose & Throat, 7th edition, pg no. 24", "cop": 2, "opa": "Not lateralized", "opb": "Lateralised to poorer ear", "opc": "Lateralised to better ear", "opd": "Inconclusive", "subject_name": "ENT", "topic_name": "Ear", "id": "f64063cf-a772-4d8b-886a-2515ff8250eb", "choice_type": "single"} {"question": "Causative organism for malignant otitis externa is", "exp": "Malignant/necrotizing otitis externa is a disorder involving inflammation and damage of the bones and cailage at the base of the skull. Malignant otitis externa is caused by the spread of an outer ear infection (otitis externa, also called swimmer's ear). It is an uncommon complication of both acute swimmer's ear and chronic swimmer's ear. Risks for this condition include: Chemotherapy Diabetes Weakened immune system External otitis is often caused by difficult-to-treat bacteria such as pseudomonas. The infection spreads from the floor of the ear canal to the nearby tissues and into the bones at the base of the skull. The infection and inflammation may damage or destroy the bones. The infection may spread more and affect the cranial nerves, brain, or other pas of the body.", "cop": 4, "opa": "Hemophilus", "opb": "Staphylococcus", "opc": "Streptococcus", "opd": "Pseudomonas", "subject_name": "ENT", "topic_name": null, "id": "f88c43d3-6f11-4679-a4df-ff531fc7ddb8", "choice_type": "single"} {"question": "Not a contra indication for Tonsillectomy", "exp": null, "cop": 3, "opa": "polio epidemic", "opb": "Hemophilia", "opc": "cleft lip", "opd": "Acute infection of Tonsils", "subject_name": "ENT", "topic_name": null, "id": "138d40ae-3ff1-4eaa-849f-66c8da13739f", "choice_type": "single"} {"question": "Type 'C' tympanogram is seen in", "exp": null, "cop": 4, "opa": "Serous Otitis media", "opb": "Otosclerosis", "opc": "Ossiullor discontinuity", "opd": "Eustachian Tube blockade", "subject_name": "ENT", "topic_name": null, "id": "e91fb6c6-27b5-44f5-878d-8874a1fa2bf7", "choice_type": "single"} {"question": "Most common nerve to be damaged in CSOM is", "exp": "Facial paralysis occurs as a complication of both acute and chronic otitis media. (Ref: Textbook of diseases of ENT, PL Dhingra, 7th edition, pg no. 88)", "cop": 2, "opa": "III", "opb": "VII", "opc": "V", "opd": "VI", "subject_name": "ENT", "topic_name": "Ear", "id": "0c26e9bf-4bce-41f6-a684-af6c50df6b81", "choice_type": "single"} {"question": "Pneumatocele is seen in fracture of", "exp": "Pneumatocele is a rare complication following head trauma. The pathophysiology is access of air into the cranium through a fracture involving air sinuses or middle ear roof or depressed fracture of the skull resulting in a craniodental fistula. Frontal bone fracture gives direct access to the cranium.", "cop": 2, "opa": "Maxillary sinus", "opb": "Frontal sinus", "opc": "Ethmoid sinus", "opd": "Sphenoid sinus", "subject_name": "ENT", "topic_name": "Nose and paranasal sinuses", "id": "bf84dd00-278e-41b9-bddd-56c6e263d104", "choice_type": "single"} {"question": "A patient has carcinoma of right tongue on its lateral border of anterior 2/3rd, with lymph node of size 4 cm in level 3 on left side of the neck, stage of disease is", "exp": "Carcinoma Oral Tongue: Ref:- Dhingra; pg num:-228", "cop": 3, "opa": "N0", "opb": "N1", "opc": "N2", "opd": "N3", "subject_name": "ENT", "topic_name": "Oral cavity & Oesophagus", "id": "c830d78d-6957-4839-bb62-15e00b85ea4e", "choice_type": "single"} {"question": "Most common type of hearing loss in TB otitis media", "exp": "Ans. (a) Conductive hearing lossRef: Dhingra's ENT 5th ed. / 83Clinical features of Tubercular otitis media:# Painless foul smelling ear discharge: very characteristic finding.# Perforation: multiple perforations usually 2-3 on pars tensa is classical sign of disease.# Hearing loss: There is a severe hearing loss , out of proportion to symptoms . Mostly conductive, it may be SNHL if labyrinth is involved.# Facial paralysis* In the presence of secondary pyogenic infection, tubercular otitis media may be indistinguishable from chronic suppurative otitis media. Culture of ear discharge for tubercle bacilli, histopathological examination of granulations and other evidence of tuberculosis in the body help to confirm the diagnosis.Treatment# Systemic antitubercular therapy as being carried for primary focus.# Local treatment in the form of aural toilet and control of secondary pyogenic infection. Mastoid surgery is indicated for complications. Healing is delayed in tuberculous cases. Wound break-down and fistula formation are common.", "cop": 1, "opa": "Conductive hearing loss", "opb": "SNHL", "opc": "Presbycusis", "opd": "Hyperacusis", "subject_name": "ENT", "topic_name": "Disorders of Middle Ear (Otitis Media)", "id": "6433db01-bec0-4b1d-a0db-b6d420661f39", "choice_type": "single"} {"question": "Lines of Sebileau pass through", "exp": "Lederman's classification of carcinoma of the maxillary sinus is using two lines of Sebileau, one passing through the floor of maxillary antrum the other through the roof of antrum or floor of orbit dividing area into Supra structure, Mesostructure and Intra structure. (Ref: Textbook of diseases of ENT, PL Dhingra, 7th edition, pg no. 233)", "cop": 1, "opa": "Floor of orbit and maxillary antrum", "opb": "Floor of nasal cavity and maxillary antrum", "opc": "Floor of orbit and nasal cavity", "opd": "Floor of orbit and roof of mouth", "subject_name": "ENT", "topic_name": "Nose and paranasal sinuses", "id": "ff1b23c1-07db-41fd-847e-aa48c66733f4", "choice_type": "single"} {"question": "Cadaveric position of vocal cords is", "exp": "As all the laryngeal muscles are paralyzed, both cords lie in cadaveric position. There is also total anaesthesia of the larynx. This is the neutral position of the cricoarytenoid joint. It is about 3.5 mm from the midline. The healthy cord is unable to approximate the paralyzed cord, thus causing glottic incompetence. This results in hoarseness of voice and aspiration of liquids through the glottis. Cough is ineffective due to air waste. Ref: PL Dhingra Textbook of Ear, Nose and Throat, Edition 7, page - 338", "cop": 3, "opa": "Midline", "opb": "1.5 mm from midline", "opc": "3.5 mm from midline", "opd": "7.5 mm from midline", "subject_name": "ENT", "topic_name": "Larynx", "id": "0751d79f-ffd7-4fe1-bfe3-69ea3d58bcee", "choice_type": "single"} {"question": "Lymphatics of Tonsils drain into", "exp": null, "cop": 1, "opa": "Jugulodiagastric Lymph node", "opb": "Jugulo - omohyoid Lymph node", "opc": "Sub mandibular Lymph node", "opd": "Submental Lymph node", "subject_name": "ENT", "topic_name": null, "id": "6081dc63-1322-4f07-ac25-52c1291b3afc", "choice_type": "single"} {"question": "Commonest cause for acute otitis media in children is", "exp": "Most common organisms in infants and young children are Streptococcus pneumonia (30 c/o ), Haemophilus influenza(20%) and Moraxella catarrhalis(12%). Other organisms include Streptococcus pyogenes, Staphylococcus aureus and sometimes Pseudomonas aeruginosa. (Ref: Textbook of diseases of ENT, PL Dhingra, 7th edition, pg no. 67)", "cop": 2, "opa": "H influenza", "opb": "S pneumonia", "opc": "S aureus", "opd": "Pseudomonas", "subject_name": "ENT", "topic_name": "Ear", "id": "0d5cfc3d-c902-47f5-b50d-396144783470", "choice_type": "single"} {"question": "A children presents with recurrent upper respiratory tract infection with mouth breathing and impairment of hearing. Treatment is", "exp": "(Adenoidectomy + Grommet insertion) (228- Dhingra 4th)* Recurrent URTI + mouth breathing and impairment of hearing reflects - diagnosis is ADENOIDSClinical featuresAetiology - Physiological enlargement* Recurrent attack of rhinitis, sinusitis or chronic tonsillitis* URTI** Nasal symptoms Aural Symptoms General Symptoms* Nasal obstruction is the commonest symptoms. This leads to mouth breathing*** Nasal discharge* Sinusitis - chronic maxillary sinusitis* Epistaxis* Voice change - toneless(i) Tubal obstruction 1/t conductive hearing loss**(ii) Recurrent attacks of Acute otitis media(iii) CSOM(iv) Serous otitis media**(1) Adenoid facies(2) Pulmonary hypertension and cor pulmonale* Adenoidectomy + Grommet insertion is treatment of choice", "cop": 3, "opa": "Tonsillectomy", "opb": "Grommet insertion", "opc": "Adenoidectomy + Grommet insertion", "opd": "Tympanoplasty", "subject_name": "ENT", "topic_name": "Miscellaneous (E.N.T.)", "id": "84968554-468e-4c91-9ebe-beedf1e7a218", "choice_type": "single"} {"question": "Most common source of hemorrhage during Tonsillectomy", "exp": "M/c cause of hemorrhage during tonsillectomy is external palatine vein / paratonsillar vein.\nM/c cause of hemorrhage of arterial origin during tonsillectomy is tonsillar branch of facial artery.", "cop": 1, "opa": "Injury to external palatine vein", "opb": "Injury to tonsillar branch of facial artery", "opc": "Injury to tonsillar branch of Ascending pharyngeal artery", "opd": "Injury to tonsillar branch of Lingual artery", "subject_name": "ENT", "topic_name": null, "id": "5c8a56e6-617b-41dc-b6fa-a1e3f517db81", "choice_type": "single"} {"question": "Red line in pure tone audiometry is for", "exp": "Red line: Right ear Blue line: Left ear Continuous line: Air Conduction Dotted line: Bone conduction Ref: Dhingra; 6th Edition; pg no 30", "cop": 3, "opa": "Bone conduction", "opb": "Air conduction", "opc": "Right ear", "opd": "Left ear", "subject_name": "ENT", "topic_name": "Ear", "id": "49b04bc1-1f2e-40eb-8cd3-63434df5e68c", "choice_type": "single"} {"question": "Tragal sign is elicited in", "exp": null, "cop": 2, "opa": "Otomycosis", "opb": "Furuncle of EAC", "opc": "Malignant otitis externa", "opd": "Diffuse otitis externa", "subject_name": "ENT", "topic_name": null, "id": "2c02e583-26fe-4bc9-bbae-b347fd70601f", "choice_type": "single"} {"question": "Fluctuating deafness is seen in", "exp": "Hearing loss usually accompanies veigo or may precede it. Hearing improves after the attack and may be normal during the periods of remission.This fluctuating character of hearing loss is quite characteristic of Meniere's disease. Ref: Diseases of EAR, NOSE and THROAT by PL Dhingra; 7th Edition; page no.111,112", "cop": 1, "opa": "Meniere's disease", "opb": "Otosclerosis", "opc": "CSOM", "opd": "ASOM", "subject_name": "ENT", "topic_name": "Ear", "id": "9269470d-c6e1-4105-a265-8b2a8e0ca50c", "choice_type": "single"} {"question": "The most common site for squamous cell carcinoma in oral cavity is", "exp": "Site distribution showed that the most common location of the tumors was 1. Border of the tongue (37%) 2. Alveolarmucosaand gingiva (20%) and 3. Floor of the mouth and ventral tongue (19%). Carcinoma tongue - Carcinoma involving anterior two-thirds of the tongue is commonly seen in men in the age group of 50-70 years. It may also occur in the younger age group and in females. It may also develop on a pre-existing leukoplakia, long-standing dental ulcer or syphilitic glossitis. The vast majority are squamous cell type. Site: Most common site is middle of the lateral border or the ventral aspect of the tongue. Uncommonly, the tip or the dorsum may be involved. (Ref: Textbook of diseases of ENT, PL Dhingra, 7th edition, pg no. 255)", "cop": 1, "opa": "Tongue", "opb": "Floor of mouth", "opc": "Upper and lower alveolus", "opd": "Buccal mucosa", "subject_name": "ENT", "topic_name": "Oral cavity & Oesophagus", "id": "3f0bd943-f1cf-4709-b4e3-2b1127e91310", "choice_type": "single"} {"question": "The complication of trauma to the danger area of the face among the following is", "exp": "The area of the upper lip and the lower pa of the nose is the danger area of the face. It is due to that this area is the common site of infectionThis area is drained by facial vein which communicates with the cavernous sinus through the superior ophthalmic vein and pterygoid venous plexus through the emissary veinIn case of any infection of this area, it may spread to the cavernous sinus causing infection and/or thrombosisRef: PL Dhingra, Diseases of Ear, Nose & Throat, 7th edition, pg no. 225 - 226", "cop": 1, "opa": "Cavernous sinus infection", "opb": "Meningitis", "opc": "Visual loss", "opd": "Loss of memory", "subject_name": "ENT", "topic_name": "Nose and paranasal sinuses", "id": "cef13577-6be9-497c-80c9-1057d1d11d6a", "choice_type": "single"} {"question": "Citelli angle is", "exp": "Citelli angle is also called as sinodural angle.Citelli&;s angle (Sinodural angle)- is an angle between the sigmoid sinus and middle fossa dural plate. Ref- Dhingraa textbook for ENT 6th edition.", "cop": 3, "opa": "Solid angle", "opb": "CP angle", "opc": "Sinodural angle", "opd": "Pa of macevan triangle", "subject_name": "ENT", "topic_name": "Ear", "id": "198ef052-811f-41fb-a238-af0d407be83f", "choice_type": "single"} {"question": "Occipitomental x ray view of paranasal sinuses with open mouth is Known as", "exp": "ref : hazarika 4th ed", "cop": 4, "opa": "water's view", "opb": "Caldwell view", "opc": "Towne's view", "opd": "Pierre's view", "subject_name": "ENT", "topic_name": "All India exam", "id": "34e74e88-53ad-4e0f-b29a-3d48ac3eb636", "choice_type": "single"} {"question": "Vestibular evoked myogenic potential detects lesion of", "exp": "Vestibular evoked myogenic potentials: It is to study function of otolith organs - sacule and utricle These are supplied by inferior vestibular nerve and superior vestibular nerve respectively These potentials can be picked up from either sternocleidomastoid or ocular muscles Ref : Dhingra 7e pg 46.", "cop": 3, "opa": "Cochlear Nerve", "opb": "Facial Nerve", "opc": "Inferior Vestibular Nerve", "opd": "Inflammatory Myopathy", "subject_name": "ENT", "topic_name": "Ear", "id": "d1e59a27-3a64-4a1a-bc4c-b331e3f65f45", "choice_type": "single"} {"question": "Angular movement is sensed by", "exp": "Semicircular canals respond to Angular acceleration and deceleration Utricle and Saccule maintain static equilibrium The cochlea is the auditory poion of the inner ear Ref: PL Dhingra 6th edition of Ear, Nose and Throat; Pg no 17", "cop": 4, "opa": "Cochlea", "opb": "Saccule", "opc": "Utricle", "opd": "Semicircular canal", "subject_name": "ENT", "topic_name": "Ear", "id": "c3bda01a-eeca-4ba1-9741-13309456078e", "choice_type": "single"} {"question": "Tripoid fracture is seen in", "exp": "Fracture of zygoma is otherwise called tripod fracture.Here zygoma is separated at its three processes; Fracture line passes through zygomaticofrontal suture, orbital floor, infraorbital margin and foramen, the anterior wall of maxillary sinus and zygomaticotemporal suture. (Ref: Textbook of ENT; Dhingra; 7th edition; Page no 205)", "cop": 4, "opa": "Mandible", "opb": "Maxilla", "opc": "Nasal bone", "opd": "Zygoma", "subject_name": "ENT", "topic_name": "Nose and paranasal sinuses", "id": "04a83926-5a0e-4d26-8578-c9230415efd3", "choice_type": "single"} {"question": "The dotted line has clinical significance in case of", "exp": "The above line is OHNGREN'S line - extending from medial canthus of the eye to the angle of mandible Maxillary carcinoma arising anteroinferior to this line has a better prognosis than posterosuperior carcinoma Ohngren's classification. An imaginary plane is drawn, extending between medial canthus of eye and the angle of mandible . Growths situated above this plane (suprastructural) have a poorer prognosis than those below it (intrastructural). Ref : Diseases of ENT by Dhingra 6th edition Pgno : 207", "cop": 2, "opa": "Maxillary sinusitis", "opb": "Maxillary carcinoma", "opc": "Mandibular fracture", "opd": "Maxillary fracture", "subject_name": "ENT", "topic_name": "Nose and paranasal sinuses", "id": "728d266c-a36b-49fe-bb5c-8b65c6ce7297", "choice_type": "single"} {"question": "Endolymphatic hydrops is seen in", "exp": "Normally endolymph is carried by the endolymphatic duct to the sac where it is absorbed. Defective absorption is by the sac maybe responsible for raised endolymph pressure. Experimental obstruction of the endolymphatic sac and its duct produces endolymphatic hydrops in Meniere's disease. Ref: Diseases of EAR, NOSE and THROAT by PL Dhingra; 7th Edition; page no.111", "cop": 1, "opa": "Meniere's disease", "opb": "Otosclerosis", "opc": "Acoustic neuroma", "opd": "Glomus tumor", "subject_name": "ENT", "topic_name": "Ear", "id": "c288746a-20ee-4a55-8025-806fa34f99aa", "choice_type": "single"} {"question": "Heamorrhagic external otitis media is caused by", "exp": "Otitis externa haemorrhagica It is characterized by for mation of haemorrhagic bullae on the tympanic membrane and deep meatus. It is probably viral in origin and may be seen in influenza epidemics. The condition causes severe pain in the ear and blood-stained discharge when the bullae rupture. Treatment: with analgesics is directed to give relief from pain. Antibiotics are given for secondary infection of the ear canal, or middle ear if the bulla has ruptured into the middle ear. Ref:- Dhingra; pg num:-52", "cop": 1, "opa": "Influenza", "opb": "Proteus", "opc": "Streptococcus", "opd": "Staphylococcus", "subject_name": "ENT", "topic_name": "Ear", "id": "e7ad538e-9273-4e0b-87b2-bab411a372ca", "choice_type": "single"} {"question": "Mode of trauma in petrous bone fracture", "exp": "(Longitudinal) (97-Dhingra 4th) (698- Bailey & Love 25th)* The otic capsule is the hardest bone in the body. If trauma to the head is severe. Temporal bone fractures may occur. These tends to be either longitudinal (80%) or Transverse (20%)* Transverse fracture usually involve the labyrinth and lead to a sensorineural hearing loss that is permanent* Fracture of temporal bone may be longitudinal, transverse or mixed* Facial palsy is seen in more often in transverse fracture (50%)", "cop": 1, "opa": "Longitudinal", "opb": "Transverse", "opc": "Mixed", "opd": "Any complete fracture", "subject_name": "ENT", "topic_name": "Ear", "id": "9a2d1b42-37ee-4971-9b08-fcc6a770387b", "choice_type": "single"} {"question": "Epley's manoeuvre is used in", "exp": "Epley's manoeuvre : This is to reposition the otoconial debris from the posterior semicircular canal back into utricle. Done in benign paroxysmal positional veigo Ref : Dhingra 7e pg 47.", "cop": 1, "opa": "Positional veigo", "opb": "Otosclerosis", "opc": "ASOM", "opd": "CSOM", "subject_name": "ENT", "topic_name": "Ear", "id": "6bc26193-1ed6-46d5-b52e-b883ca8ccff2", "choice_type": "single"} {"question": "The most common cause of fungal sinusitis is( REPEAT QUESTION)", "exp": "Fungal infections of the paranasal sinuses are uncommon and usually occur in individuals who are immunocompromised. Aspergillus fumigatus is most commonly involved in allergic fungal sinusitis and sinus mycetoma. (Ref: Textbook of diseases of ENT, PL DHINGRA, 7th edition, pg no. 220)", "cop": 1, "opa": "A fumigates", "opb": "A niger", "opc": "A flavus", "opd": "Candida", "subject_name": "ENT", "topic_name": "Nose and paranasal sinuses", "id": "044cd6eb-063f-400d-9311-ace020023523", "choice_type": "single"} {"question": "Tripod fracture is the name given for", "exp": "Zygoma fracture is also known as tripod fracture. Clinical features of zygoma fracture Considerable swelling over zygomatic arch is common and makes clinical diagnosis more difficult. Flattening of malar prominence. Step-deformity of infraorbital margin. Anaesthesia in the distribution of infraorbital nerve. Trismus, due to depression of zygoma on the underlying coronoid process. Oblique palpebral fissure, due to the displacement of lateral palpebral ligament. Restricted ocular movement, due to entrapment of inferior rectus muscle. It may cause diplopia. Periorbital emphysema, due to escape of air from the maxillary sinus on nose-blowing. The mucosa of the maxillary sinus may be lacerated and cause epistaxis on that side. Fracture of the zygoma may or may not be painful to palpation and running a finger along the zygomatic arch may give a feel of a depressed fracture or a small dimple. The cheek may appear flattened compared to symmetry with the opposite side. This may be obvious immediately following trauma or several days later once swelling has subsided.", "cop": 1, "opa": "Zygomatic fracture", "opb": "Maxillary fracture", "opc": "Mandibular fracture", "opd": "Temporal fracture", "subject_name": "ENT", "topic_name": null, "id": "7521c6ec-4921-42de-a95d-4d7d99c43071", "choice_type": "single"} {"question": "Swelling between tonsillar area and superior constrictor muscle is known as", "exp": "Ans. a (Quinsy). (Ref. Disease of ENT by PL Dhingra 3rd /pg. 318)PERITONSILLAR ABSCESS (QUINSY)# It is collection of pus between capsule of tonsils and superior constrictor muscle.# It is characterized by hot potato voice and odynophagia and halitosis.# While draining it should be opened at the point of maximum bulge above upper pole of tonsil or just lateral to point of junction of anterior pillar, with a line drawn through the base of uvula.# Tonsils are removed 5-6 weeks following an attack of Quinsy.6# Quinsy tonsillectomy vs. Interval tonsillectomy- Quinsy tonsillectomy can be a treatment option in pediatric patients to young to withstand bedside aspiration or * I&D for recurrent PTA.- Quinsy tonsillectomy can be surgically easier than interval tonsillectomy as fibrosis has not had time to set into themtonsillar capsule.- Interval tonsillectomy can be considered after successful abscess drainage, usually from recurrent PTA after 4-6 weeks.", "cop": 1, "opa": "Quinsy", "opb": "Dental abscess", "opc": "Parapharyngeal abscess", "opd": "Retropharyngeal abscess", "subject_name": "ENT", "topic_name": "Pharynx", "id": "8090f50f-81a6-451d-a66b-63a596e00d12", "choice_type": "single"} {"question": "A 17 years male presented with swelling in the cheek with recurrent epistaxis. Most likely diagnosis is", "exp": "A. i.e. (Angiofibroma) (230 - Dhingra 4th) (296 - maqbool 11th)NASOPHARYNGEAL ANGIOFIBROMA* Benign but locally invasive lesion of the nasophagrynx* Exclusively in males 10 and 25 years of age* Lesions arise from the ventral periosteum of the skull as a result of hormonal (imbalance or persistence of embryonic tissue* Gradually increasing nasal obstruction and recurrent attacks of epistaxis are the common presenting symptoms* Probing or palpation of the nasopharynx should not be done (Biopsy is contraindicated)*** ** CT scan of head with contrast enhancement is now the investigation of choice* **Surgical excision - Transpalatine + sublabial (Sardanas approach) is now the treatment of choice* Concomitant RT and cisplatin, followed by adjuvant chemotherapy is currently considered the best treatment of naso pharyngeal carcinoma.", "cop": 1, "opa": "Angiofibroma", "opb": "Carcinoma nasopharynx", "opc": "Rhabdomyosarcoma", "opd": "Ethmyoid polyp", "subject_name": "ENT", "topic_name": "Nose and PNS", "id": "d20181aa-c211-4260-8bf2-94421a4ba74f", "choice_type": "single"} {"question": "Trismus is commonly seen in", "exp": "Quinsy (Peritonsillar abscess) is a collection of pus in the peritonsillar space which lies between the capsule of tonsil and the superior constrictor muscle. Clinical Features: (a) Severe pain in the throat. Usually unilateral.(b) Odynophagia. It is so marked that the patient cannot even swallow his own saliva which dribbles from the angle of his mouth. The patient is usually dehydrated. (c) Muffled and thick speech, often called \"hot potato voice.\" (d) Foul breath due to sepsis in the oral cavity and poor hygiene. (e) An ipsilateral earache. This is referred pain CN IX which supplies both the tonsil and the ear. (f) Trismus due to spasm of pterygoid muscles which are in close proximity to the superior constrictor. (Ref: Textbook of diseases of ENT, PL Dhingra, 7th edition, pg no. 298)", "cop": 2, "opa": "Ludwig's angina", "opb": "Quinsy", "opc": "Retropharyngeal abscess", "opd": "Parapharyngeal abscess", "subject_name": "ENT", "topic_name": "Oral cavity & Oesophagus", "id": "a3536251-155b-4a65-82fd-be5b8ef78b0b", "choice_type": "single"} {"question": "Palatine Tonsils are lined by", "exp": "Palatine tonsil Palatine tonsils are two in number. Each tonsil is an ovoid mass of lymphoid tissue situated in the lateral wall of oropharynx between the anterior and posterior pillars. Actual size of the tonsil is bigger than the one that appears from its surface as pas of tonsil extend upwards into the soft palate, downwards into the base of tongue and anteriorly into palatoglossal arch. A tonsil presents two surfaces--a medial and a lateral, and two poles--an upper and a lower. Medial surface of the tonsil is covered by nonkeratinizing stratified squamous epithelium. Ref:- Dhingra; pg num:-257", "cop": 1, "opa": "Non keratinized Stratified squamous epithelium", "opb": "Ciliated columnar epithelium", "opc": "Cuboidal epithelium", "opd": "Transitional epithelium", "subject_name": "ENT", "topic_name": "Pharynx", "id": "33a215ab-9d5d-416a-9a78-9bcbee033625", "choice_type": "single"} {"question": "Stensons duct of parotid opens opposite to", "exp": null, "cop": 4, "opa": "Upper second premolar", "opb": "Lower second premolar", "opc": "Lower second molar", "opd": "Upper second molar", "subject_name": "ENT", "topic_name": null, "id": "f288675e-6409-4a24-b78a-15d2d54197e2", "choice_type": "single"} {"question": "Strawberry appearance is seen in", "exp": "Rhinosporidiosis caused by rhinosporidium seeberi, mostly affects nose and nasopharynx, other sites such as lip, conjunctiva, epiglottis, larynx, trachea, bronchi, skin, vulva and vagina. In the nose, the disease is presented as leafy, polypoidal mass, pink to purple in colour and attached to the nasal septum or lateral wall. \nThe mass is very vascular and bleeds too easily to touch. Its surface is studded with white dots representing the sporangia of fungus. hence giving it a \"strawberry \" appearance.", "cop": 2, "opa": "Lupus vulgaris", "opb": "Rhinosporidiosis", "opc": "Rhinoscleroma", "opd": "Angiofibroma", "subject_name": "ENT", "topic_name": null, "id": "e1f5f525-57ab-494e-a8fd-fd7256f505b1", "choice_type": "single"} {"question": "Commonest site of otosclerosis is", "exp": "The most common type of otosclerosis is stapedial otosclerosis. Stapedial otosclerosis causes stapes fixation and conductive deafness is the most common variety. Here the lesion mainly stas just in front of the oval window in an area called \"fissula ante fenestram\". This is the site of predilection( anterior focus). The lesion may sta behind the oval window(posterior focus), around the margin of the stapes footplate( circumferential), in the footplate but annular ligament being free (biscuit type). Sometimes, it may completely obliterate the oval window. ( obliterate type) (Ref: Diseases of EAR, NOSE and THROAT by PL Dhingra; 7th Edition; Page no.95.)", "cop": 2, "opa": "Round window", "opb": "Oval window", "opc": "Utricle", "opd": "Ossicles", "subject_name": "ENT", "topic_name": "Ear", "id": "ebe18dea-7154-4821-a803-aa05f9f03a4d", "choice_type": "single"} {"question": "The bifurcation of trachea occurs most commonly at", "exp": "\"The level of bifurcation of the trachea varies from the lower third of the fouh to the middle of the seventh thoracic veebrae. The average position is at the level of the lower third of the fifth thoracic veebra.Ref: PL Dhingra, Diseases of Ear, Nose & Throat, 7th edition, pg no. 319", "cop": 2, "opa": "T3", "opb": "T5", "opc": "T7", "opd": "T9", "subject_name": "ENT", "topic_name": "Oral cavity & Oesophagus", "id": "7e15ce26-233b-466f-b652-66fbe5c3db97", "choice_type": "single"} {"question": "Most common fungus causing otomycosis is", "exp": "Otomycosis is a fungal infection of the ear canal most commonly caused by Candida albicans, Aspergillus niger and Aspergillus fumigatus. (Ref: Textbook of diseases of ENT, PL Dhingra, 7th edition, of no. 55)", "cop": 2, "opa": "Histoplasma", "opb": "Candida", "opc": "Mucor", "opd": "Penicillin", "subject_name": "ENT", "topic_name": "Ear", "id": "a3238eda-32e8-4418-a7eb-af9d79d8171c", "choice_type": "single"} {"question": "The effective diameter of the tympanic membrane is", "exp": "Effective diameter or vibratory area of tympanic membrane is 45mm2; Area of tympanic membrane is 90mm2 Stapes footplate area is 3.2mm2 Ref: PL Dhingra 6th edition of Ear, Nose and Throat; pg no 15", "cop": 4, "opa": "25 mm2", "opb": "30 mm2", "opc": "40 mm2", "opd": "45 mm2", "subject_name": "ENT", "topic_name": "Ear", "id": "36f13e76-bb4b-49a0-9a99-1f9dd33fb85a", "choice_type": "single"} {"question": "Risk of Aspiration is high in", "exp": null, "cop": 4, "opa": "U/L recurrent Laryngeal nerve palsy", "opb": "B/L recurrent Laryngeal nerve palsy", "opc": "Adductor palsy", "opd": "B/L complete palsy", "subject_name": "ENT", "topic_name": null, "id": "9ab791cb-727c-4dff-884e-80fb1afca2b3", "choice_type": "single"} {"question": "Luschka's tonsil also know as", "exp": "Lushka's Tonsil - Adenoids.\nGerlach's Tonsil  - Tubal Tonsil.\nFaucial Tonsil - palatine  Tonsil.", "cop": 1, "opa": "Adenoids", "opb": "Palatine Tonsil", "opc": "Lingual Tonsil", "opd": "Gerlach's Tonsil", "subject_name": "ENT", "topic_name": null, "id": "8064df70-036e-4429-a36b-118aa3cd9a5f", "choice_type": "single"} {"question": "Skull base osteomyelitis is", "exp": null, "cop": 3, "opa": "Otomycosis", "opb": "Tumor of the Ear", "opc": "Malignant Otitis Externa", "opd": "Chronic serous Otitis Media", "subject_name": "ENT", "topic_name": null, "id": "3fc4b161-4a2a-4de0-be63-75343681ffd7", "choice_type": "single"} {"question": "A child presenting with recurrent respiratory tract infections, mouth breathing and decreased hearing. Treatment of choice is", "exp": "(Adenoidectomy) (228-385- Dhingra 4th, 367- Logan & Turner 10th)* Recurrent RTI, mouth breathing and decreased hearing suggest that child is suffering from adenoids,* Treatment of choice for adenoids is - AdenoidectomyADENOIDS* Adenoids (or nasopharyngeal tonsil / is situated at junction of the roof and posterior wall of the nasopharynx* Adenoids are present at birth, shows physiological enlargement upto the age of 6 years and then tends to atrophy at puberty and almost completely disappears by age of 20 years* Nasal obstruction is the commonest symptoms. This leads to mouth breathing* **. It also interferes with feeding or sucking. As respiration and feeding can not take place simultaneously such a child fails to thrive**** Nasal symptoms - Nasal obstruction, Nasal discharge, chronic sinusitis, Epistaxis, voice change* Aural symptoms - Tubal obstruction, Recurrent attack of acute DM, CSOM, Serous ottitis mediaChronic nasal obstruction and mouth breathing leads to characteristic facial appearance called Adenoid facies the child has an elongatedface with dull expression, open mouth, prominent and crowded upper teeth and hitched up upper lip* Hard palate is highly arched** as moulding action of the tongue or palate is loss* Diagnosis by examination of post nasal space is possible in some children. Soft tissue lateral radiograph will reveal size of adenoids as also the extent to which nasopharyngeal air space which has been compromised.CT scan has no role in diagnosisIndications of Adenoidectomy* Adenoid hypertrophy causing snoring, mouth breathing, sleep apnoea syndrome or speech abnormalities (i.e. rhinolalia)* Recurrent rhinosinusitis* Chronic secretory otitis media a/w adenoid hyperplasia* Recurrent ear discharge in benign CSOM a/w adenoitis / adenoid hyperplasia* Dental malocclusion (Adenoidectomy does't correct dental abnormalities but will prevent its recurrence after orthodontic treatment)", "cop": 2, "opa": "Tonsillectomy", "opb": "Adenoidectomy", "opc": "Gromet insertion", "opd": "Myringotomy", "subject_name": "ENT", "topic_name": "Pharynx", "id": "97395896-defa-4222-9e20-538ffc8b3c1a", "choice_type": "single"} {"question": "Posterior epistaxis is commonly seen in", "exp": "Posterior epistaxis is commonly seen in elderly hypeensive patients. Often due to aeriosclerosis, rarely spontaneously Ref: Dhingra 7e pg 199.", "cop": 3, "opa": "Children with ethmoidal polyps", "opb": "Foreign body of the nose", "opc": "Hypeension", "opd": "Nose picking", "subject_name": "ENT", "topic_name": "Nose and paranasal sinuses", "id": "15e13a37-9f1f-4739-b402-11039ac6f0c2", "choice_type": "single"} {"question": "Hyperacusis in Bell's palsy is due to the paralysis of the following muscle", "exp": "Hyperacusis is noise intolerance due to stapedial paralysis in bells palsy. Ref: Dhingra 6th edition Pg.95", "cop": 4, "opa": "Tensor tympani", "opb": "Levator palatii", "opc": "Tensor veli palatii", "opd": "Stapedius", "subject_name": "ENT", "topic_name": "Ear", "id": "4d41f933-3ce1-4987-a1cf-5d3b5e0d8dfe", "choice_type": "single"} {"question": "A diabetic patients presents with black necrotic mass filling the nasal cavity. Most likely fungal infection is", "exp": "(Mucormycosis) (151 -Dhingra 4th)Mucormycosis - common in uncontrolled DM or taking immuno compressive drugs* Invade arteries an causes endothelial damage and thrombosis* Typical finding - presence of a black necrotic mass** filling the nasal cavity and eroding the septum and hard palate* Treatment - Amphotericin - B & Surgical debridement** filling the nasal cavity and eroding the septum and hard palate* Treatment - Amphotericin - B & surgical debridement* Commonest fungeal infection in nose - Aspergillosis* Rhinosporidiosis - is a fungal granuloma- caused by Rhinosporidium seeberi**", "cop": 3, "opa": "Rhinosporidiosis", "opb": "Aspergillosis", "opc": "Mucormycosis", "opd": "Candidiasis", "subject_name": "ENT", "topic_name": "Nose and PNS", "id": "93037433-75bd-4d5a-8ea4-e4a5aa4c8571", "choice_type": "single"} {"question": "Identity the lesion of oral cavity", "exp": null, "cop": 2, "opa": "Erythroplakia", "opb": "Leukoplakia", "opc": "Submucous fibrosis", "opd": "Lichen planus", "subject_name": "ENT", "topic_name": null, "id": "fd81fc06-dabb-4341-9da0-8d78a6966ebb", "choice_type": "single"} {"question": "Fordyce spots are from", "exp": null, "cop": 1, "opa": "Sabaceous glands", "opb": "Sweat glands", "opc": "Endocrine glands", "opd": "Mammary glands", "subject_name": "ENT", "topic_name": null, "id": "2e0e49c0-f177-41e6-bc88-237b476edaa7", "choice_type": "single"} {"question": "Maxillary sinus epithelium is", "exp": "The sinus is lined with mucoperiosteum, with cilia that beat down the ostia. This membrane is referred to as the Schneiderian membrane, which is histologically a bilaminar membrane with ciliated columnar epithelium cells on the internal side and periosteum. Paranasal sinuses are lined by the mucous membrane which is continuous with that of the nasal cavity through the ostia of sinuses. It is thinner and less vascular compared to that of the nasal cavity. Histologically, it is ciliated columnar epithelium with goblet cells which secretes mucus. Cilia is more marked near ostia of the sinuses and help in drainage of mucus into the nasal cavity. (Ref: Diseases of ENT, PL. Dhingra, 7t edition, pg no. 188)", "cop": 4, "opa": "Squamous", "opb": "Non ciliated columnar", "opc": "Keratinized squamous", "opd": "Ciliated columnar", "subject_name": "ENT", "topic_name": "Nose and paranasal sinuses", "id": "b1f42236-8c74-4762-ac20-1a0c68c2c60c", "choice_type": "single"} {"question": "Investigation of choice for choanal atresia", "exp": null, "cop": 4, "opa": "X - ray", "opb": "MRI", "opc": "USG", "opd": "HRCT", "subject_name": "ENT", "topic_name": null, "id": "aeeac182-2e14-48a8-902f-1cafbdd9d244", "choice_type": "single"} {"question": "An antrochoanal polyp is associated most commonly with", "exp": "Antrochoanal polyp arises from the mucosa of the maxillary antrum near its accessory ostium, comes out of it and grows in the choana and nasal cavity. \nHypertrophied middle turbinate, one of the important differential diagnosis, which is differentiated by its pink appearance and hard feel of the bone on probe testing. \nAntrochoanal polyps are most commonly associated with middle meatus.", "cop": 1, "opa": "Middle meatus", "opb": "Sphenoethmoidal recess", "opc": "Inferior meatus", "opd": "Superior meatus", "subject_name": "ENT", "topic_name": null, "id": "a2561636-cc08-4d61-a52d-fbc4e9857b6d", "choice_type": "single"} {"question": "'T' staging of Glottic cancer with fixed vocal cords is", "exp": null, "cop": 3, "opa": "T1", "opb": "T2", "opc": "T3", "opd": "T4", "subject_name": "ENT", "topic_name": null, "id": "bfe9570e-548c-4c57-a00d-e0839d4de096", "choice_type": "single"} {"question": "Subjective test of hearing is", "exp": "Uses of Pure Tone Audiogram (a) It is a measure of threshold of hearing by air and bone conduction and thus the degree and type of hearing loss. (b) A record can be kept for future reference. (c) Audiogram is essential for prescription of hearing aid. (d) Helps to find degree of handicap for medicolegal purposes. (e) Helps to predict speech reception threshold. Ref : Dhingra 7e pg 25.", "cop": 1, "opa": "Pure tone audiometry", "opb": "Otoacoustic emission", "opc": "BERA", "opd": "Impedance audiometry", "subject_name": "ENT", "topic_name": "Ear", "id": "30357485-22d1-4103-994c-5d4cd087726f", "choice_type": "single"} {"question": "Muller's manoeuver is", "exp": "Muller's manoeuvre: Used to find the level and degree of obstruction in sleep-disordered breathing. It is performed while using flexible nasopharyngoscope. First, the examiner sees the upper airways at rest and then during the time when the patient makes a maximal inspiratory effo with nose and mouth closed. The base of tongue, lateral pharyngeal wall and palate are examined for collapsibility and then rated from 0 (minimal collapse) to 4+ (complete collapse). Ref: Textbook of ENT, Dhingra; 6th edition; Pg no: 449", "cop": 4, "opa": "Forceful expiration against closed glottis", "opb": "Forceful inspiration against closed glottis", "opc": "Forceful expiration against open glottis", "opd": "Reverse of Valsalva's maneuver", "subject_name": "ENT", "topic_name": "Miscellaneous ENT", "id": "a232a659-f439-4b65-9a67-1fe501a9d2d6", "choice_type": "single"} {"question": "\"Luc's abscess\" is", "exp": "Luc's abscess - Under periosteum of the roof of bony external auditory canal Ref : ENT textbook by Dhingra 6th edition Pgno : 51-53", "cop": 2, "opa": "Abscess over temporal region", "opb": "Abscess involving external auditory canal", "opc": "Abscess over mastoid", "opd": "Abscess along sternomastoid muscle", "subject_name": "ENT", "topic_name": "All India exam", "id": "0630fe65-2b63-4109-83ef-dc169044f1c0", "choice_type": "single"} {"question": "SADE classification is for", "exp": "Sade classification: Retraction of Pars Tensa.\nTos classification: Retraction of Pars Flaccida", "cop": 1, "opa": "Rectraction of pars Tensa of Tympanic membrane", "opb": "Rectraction of Pars Flaccida of Tympanic membrane", "opc": "Tympanosclerosis", "opd": "Perforation of Tympanic membrane", "subject_name": "ENT", "topic_name": null, "id": "0c3200d3-2e38-4003-9e1f-4ecec13a37f4", "choice_type": "single"} {"question": "Not a Boundary of Jackson's Triangle used in Tracheotomy", "exp": null, "cop": 3, "opa": "Suprasternal notch", "opb": "Lower edge thyroid cartilage", "opc": "Outer border of Sterno cleitomastoid", "opd": "Inner border of Sterno cleitomastoid", "subject_name": "ENT", "topic_name": null, "id": "caa19ec3-e13d-4dd3-ba37-3f9435e48b36", "choice_type": "single"} {"question": "Fitzgerald&;s caloric test uses temperature at", "exp": "TESTS OF VESTIBULAR FUNCTION Fitzgerald-Hallpike test (bithermal caloric test): In this test, patient lies supine with head tilted 30deg forward so that horizontal canal is veical. Ears are irrigated for 40 s alternately with water at 30degC and at 44degC (i.e. 7deg below and above normal body temperature) and eyes observed for appearance of nystagmus till its end point. Time taken from the sta of irrigation to the end point of nystagmus is recorded and chaed on a calorigram. Ref:- Dhingra; pg num:-43", "cop": 1, "opa": "30degCand44degC", "opb": "33degCand21degC", "opc": "34degCand41degC", "opd": "37degCand41degC", "subject_name": "ENT", "topic_name": "Ear", "id": "42495df4-61af-4cfa-820b-e13ad183150f", "choice_type": "single"} {"question": "Quinsy is also known as", "exp": "The medical term for quinsy is peritonsillar abscess. It's an uncommon complication of tonsillitis that can happen when a throat abscess develops after infection spreads from the tonsils. If the abscess grows large enough, it can block the airways, causing breathing difficulties, so it's impoant it's treated early. What are the symptoms of Quinsy? The main symptom is a sore throat that gets gradually worse, usually only on one side. Other symptoms include: Fever (temperature of 38degC or above) difficulty or pain on swallowing 'hot potato' voice, sounding as though you're speaking with a - hot potato in your mouth difficulty opening your mouth bad breath earache headache swelling on the affected side dehydration because of difficulty swallowing", "cop": 1, "opa": "Peritosillar abscess", "opb": "Retropharyngeal abscess", "opc": "Parapharyngeal abscess", "opd": "Paraepiglottic abscess", "subject_name": "ENT", "topic_name": null, "id": "41491f6f-24c4-470e-9089-00e8e0a12fb0", "choice_type": "single"} {"question": "In the right middle ear pathology, weber's test will be", "exp": "Right middle ear pathology means right conductive deafness. In conductive deafness, Weber is lateralized to the worse ear. (in question right ear) In SN deafness, Weber is lateralized to the better ear. Ref: Dhingra; 6th Edition; pg no 22", "cop": 3, "opa": "Normal", "opb": "Centralized", "opc": "Lateralised to right side", "opd": "Lateralised to left side", "subject_name": "ENT", "topic_name": "Ear", "id": "616ed637-b345-4cb6-b1a3-34ccdd16e76e", "choice_type": "single"} {"question": "Wagner and Grossman theory is related to", "exp": "Wagner and Grossman hypothesis which states that cricothyroid muscle which receives innervation from superior laryngeal nerve keeps the cord in paramedian position due to its adductor function. Ref: PL Dhingra Textbook of Ear, Nose and Throat, Edition 7, page - 338", "cop": 2, "opa": "Palatal palsy", "opb": "Vocal cord palsy", "opc": "Facial palsy", "opd": "Hypoglossal palsy", "subject_name": "ENT", "topic_name": "Larynx", "id": "b2e07a96-b50b-409e-b7ae-be5a00ccd623", "choice_type": "single"} {"question": "Cause of myringosclerosis is", "exp": "Grommet inseion is preferred if myringotomy and aspiration combined with medical measures have not helped and fluid recurs, a grommet is inseed to provide continued aeration of the middle ear. It is left in place for weeks or months or till it is spontaneously extruded. Ref - PL DHINGRA diseases of ear nose and throat, 6th edition, page no 65", "cop": 2, "opa": "Genetic", "opb": "Grommet inseion", "opc": "Otosclerosis", "opd": "Idiopathic", "subject_name": "ENT", "topic_name": "Ear", "id": "87eabf9f-a980-4585-9f41-33f58bb1f178", "choice_type": "single"} {"question": "Treatment of choice in nasopharyngeal carcinoma", "exp": "NASOPHARYNGEAL CANCER TREATMENT :- 1. Radiotherapy:- It is the treatment of choice for nasopharyngeal cancer. Stage I and II are treated by radiotherapy alone while stage III and IV require concomitant radiation and chemotherapy or radiation followed by chemotherapy. External beam radiation of 6000-7000 cGy can be delivered by linear accelerator to the primary and both sides of neck. More advanced techniques of radiotherapy such as three-dimensional conformal radiotherapy and intensity modulated radiotherapy (IM) are now being used more and more. They allow higher dose delivery to the tumour with reduced damage to the adjacent normal structures such as spinal cord, brainstem and parotid glands. IM has also been used for recurrent disease where conventional radiotherapy produces more serious side effects such as transverse myelitis. Ref:- Dhingra; pg num:-252", "cop": 3, "opa": "Surgery", "opb": "Chemotherapy", "opc": "Radiotherapy", "opd": "Surgery and radiotherapy", "subject_name": "ENT", "topic_name": "Pharynx", "id": "eff2b5c6-e77e-41fd-906a-02a55b8ae91c", "choice_type": "single"} {"question": "Thumb printing sign is seen in", "exp": "(Acute epiglottitis): (309-Dhingra 5th; 599-P-Hazarika 3rd/edition)ACUTE EPIGLOTTITIS(Syn - Acute supraglottic laryngitis)* Caused by Hemophilus influenza type BClinical FeaturesSymptomsSignsGeneral - usually starts as URTISigns* High grade fever* General - fever, toxic appearance and flushed skin, lethargic* Dribbling saliva* On auscultation decreased air entry Local* Sore throad* Dysphagia/odynophagia* Muffled (Hot potato) voice* F B sensation in the throat* Breathing difficulty -Inspiratory stridor is variably present and may be associated with expiratory rottles ORAL* Pharynx is usually congested and pooling of saliva* Red and edematous epiglottis popping up \"Cherry red epiglottis\"* Stridor is inspiratory and increases on supine position. Patient tends it sit up leaning forward supporting on upper limbs (Tripod sign) which relieves stridor to some extent* Inspiratory stritor if severe causes intermittent retraction, active accessory respiratory muscles and perioral cyanosis Differences between acute epiglottis and acute laryngo-tracheo-bronchitis in children Acute epiglottisAcute laryngo-tracheo-bronchitis (or group)* Causative organismHaemophilus influenza type BParainfluenza virus type I and II* Age2-7 years3 months to 3 years* PathologySupraglottic larynxSubglottic area* Prodromal symptomsAbsentPresent* OnsetSuddenSlow* FeverHighLow grade or no fever* Patient's lookToxicNon-toxic* CoughUsually absentPresent, (Barking seal-like)* StridorPresent and may be markedPresent* OdynophagiaPresent with drooling of secretionsUsually absent* RadiologyThumb sign on lateral viewSteeple sign on anteroposterior view of neck* TreatmentHumidified oxygen, third generation cephalosporin (ceftriaxone) or amoxicillinHumidified 02 tent, steroids", "cop": 1, "opa": "Acute epiglotitis", "opb": "Acute largnotracheobronchitis", "opc": "Acute Laryngitis", "opd": "Carcinoma vocal cords", "subject_name": "ENT", "topic_name": "Larynx", "id": "ac0fc69c-df6c-4bfc-a58a-d3917f637906", "choice_type": "single"} {"question": "In CSOM commonest operation done is", "exp": "In CSOM, commonest operation done is modified radical mastoidectomy. It is a modification of a radical mastoidectomy whereas much of the hearing mechanism is preserved. INDICATIONS Cholesteatoma confined to the attic and antrum. Localised chronic otitis media. Ref: Textbook of diseases of ENT, PL Dhingra, 7th edition, pg no. 461", "cop": 1, "opa": "Modified radical mastoidectomy", "opb": "Radical mastoidectomy", "opc": "Simple mastoidectomy", "opd": "Tympanoplasty", "subject_name": "ENT", "topic_name": "Ear", "id": "3cbdd520-906d-4441-8cff-c66a216feff0", "choice_type": "single"} {"question": "Sclerosing otitis media is diagnosed by", "exp": "Otoscope is used to view any of the middle ear pathology like granulations, sclerosis, ossicular chain, adhesions. Ref: Dhingra 7e pg 78.", "cop": 4, "opa": "Impedence audiometry", "opb": "Pure tone audiometry", "opc": "X-ray", "opd": "Otoscopy", "subject_name": "ENT", "topic_name": "Ear", "id": "e52acbdd-442f-4747-ab70-11a63de21323", "choice_type": "single"} {"question": "Mouse nibbled appearance of vocal cord is seen in", "exp": "TUBERCULOSIS OF LARYNX AETIOLOGY :- It is almost always secondary to pulmonary tuberculosis, mostly affecting males in middle age group. Tubercle bacilli reach the larynx by bronchogenic or haematogenous routes. PATHOLOGY:- Disease affects posterior pa of larynx more than anterior. Pas affected are: (i) interarytenoid fold, (ii) ventricular bands, (iii) vocal cords and (iv) epiglottis, in that order. Tubercle bacilli, carried by sputum from the bronchi, settle and penetrate the intact laryngeal mucosa paicularly in the interarytenoid region (bronchogenic spread). This leads to formation of submucosal tubercles which may later caseate and ulcerate. Laryngeal mucosa appears red and swollen due to cellular infiltration (pseudoedema). Stages of perichondritis and cailage necrosis are not commonly seen these days. SYMPTOMS AND SIGNS :- They would greatly depend on the stage of tuberculosis. Weakness of voice is the earliest symptom followed by hoarseness. Ulceration in the larynx gives rise to severe pain which may radiate to the ears. Swallowing is painful with marked dysphagia in later stages. LARYNGEAL EXAMINATION:- 1. Hyperaemia of the vocal cord in its whole extent or confined to posterior pa with impairment of adduction is the first sign. 2. Swelling in the interarytenoid region giving a mamillated appearance. 3. Ulceration of vocal cord giving mouse-nibbled appearance. 4. Superficial ragged ulceration on the arytenoids and interarytenoid region. 5. Granulation tissue in interarytenoid region or vocal process of arytenoid. 6. Pseudoedema of the epiglottis \"turban epiglottis.\" 7. Swelling of ventricular bands and aryepiglottic folds. 8. Marked pallor of surrounding mucosa. Ref:- Dhingra; pg num:-293", "cop": 1, "opa": "TB larynx", "opb": "Vocal nodules", "opc": "Vocal cord palsy", "opd": "Larynx Ca", "subject_name": "ENT", "topic_name": "Larynx", "id": "962c1c94-6ca0-43b4-b9a2-8971d2abbe6d", "choice_type": "single"} {"question": "Treatment of choice of Bell's palsy is", "exp": "Prednisolone is the drug of choice. Steroids prevent synkinesis and crocodile tears and shoen recovery time of facial paralysis. Ref: Dhingra 6th edition pg. 95", "cop": 2, "opa": "Surgical decompression", "opb": "Coicosteroids", "opc": "Electrical stimulation", "opd": "Antiviral drugs", "subject_name": "ENT", "topic_name": "Ear", "id": "a7f7d6bc-10cd-4184-a3e4-c5d4f340d584", "choice_type": "single"} {"question": "The earliest sign of tuberculosis of vocal cords is", "exp": "The weakness of voice is the earliest symptom followed by hoarseness of voice. Ulceration of larynx produces severe pain, which may radiate to ears. Pain during swallowing is seen in later stages. On examination, hyperemia of the vocal cord in its whole extent or confined to posterior pa with impairment of adduction is the first sign. Note the hyperaemia Ref : www.ncbi.nlm.gov", "cop": 2, "opa": "Mouse nibbled appearance", "opb": "Hyperemia of the free margin", "opc": "Turban epiglotitis", "opd": "Ulceration of vocal cord", "subject_name": "ENT", "topic_name": "Larynx", "id": "8ca24a19-4277-4b09-94d0-82861ade74ff", "choice_type": "single"} {"question": "The Transorbital view is carried out for", "exp": "Transorbital view: This view is taken with occiput on the film with an orbitomeatal line at right angles to the film. X-ray beam passes through the orbit. Structures seen are:(a) Internal auditory canal.(b) Cochlea.(c) Labyrinth.(d) Entire petrous pyramid projected through the orbit. Ref: Textbook of ENT, Dhingra; 6th edition Ref img Transorbital X-ray of patient 1 after the paial explantation of the cochlear implant due to the recurrence of FIPT. The intracochlear pa of the electrode is still in situ.", "cop": 4, "opa": "Petrous apex", "opb": "Mastoid process", "opc": "Bilateral mastoid pathology", "opd": "Internal auditory meatus", "subject_name": "ENT", "topic_name": "Miscellaneous ENT", "id": "28ba54ce-82c2-41c6-8908-4f1c65b41178", "choice_type": "single"} {"question": "Laryngocele arises as herniation of laryngeal mucosa through the following membrane", "exp": "Laryngocele Laryngocele is an air-filled cystic swelling which occurs due to dilatation of saccule. Saccule is a diveiculum arising from anterior pa of ventricle/sinus of larynx. External laryngocele is one in which distended saccule herniates through the thyrohyoid membrane and presents as a reducible swelling in the neck, which increases in size on coughing or performing Valsalva. Ref:- Mohan Bansal; pg num:-487", "cop": 4, "opa": "Cricotracheal", "opb": "Crisosternal", "opc": "Cricothyroid", "opd": "Thyrohyoid", "subject_name": "ENT", "topic_name": "Larynx", "id": "f0e5ec76-466b-4755-9ec5-3e807da7c922", "choice_type": "single"}